의학교육에서 에드가 데일의 배움의 피라미드: 신화의 확장(Med Educ, 2019)

Edgar Dale's Pyramid of Learning in medical education: Further expansion of the myth

Ken Masters




도입

1 | INTRODUCTION


에드가 데일의 학습 피라미드

1.1 | Edgar Dale's learning pyramid


교육 문헌에서, 학생들에게 학습 보존율을 보여주는, 그리고 보통 Edgar Dale (또는 NTL [National Training Laboratory])에 귀속되는 학습의 피라미드는 흔히 나타난다.1

In educational literature, a Pyramid of Learning, giving student learning retention rates and usually attributed to Edgar Dale (or the National Training Laboratories [NTL]), is a common feature.1


저자가 실시한 2013년 문헌 검토 결과 피라미드와 그 통계는 천문학, 생화학, 화학, 일반교육, 공학, 국제정치, 도서관학, 관리, 물리학, 가금류 과학 및 수의학 분야의 다양한 교육 문헌에 걸쳐 인용되었다. 또한 피라미드는 세계보건기구(WHO) 문서와 같은 비학문 문헌에서도 인용되었다.

A 2013 literature review conducted by the author1 showed that the Pyramid and its statistics had been cited across a diverse range of educational literature in astronomy, biochemistry, chemistry, general education, engineering, international politics, library science, management, physics, poultry science and veterinary science. In addition, the Pyramid had been cited in nonacademic literature, such as World Health Organisation (WHO) documentation.


피라미드의 가장 큰 문제는 피라미드가 전혀 증거에 기초하지 않는다는 것이다.

The chief problem with the Pyramid is that it is based on no evidence at all.


피라미드의 역사

1.2 | History of the Pyramid


피라미드의 역사를 정확히 추적하기는 어렵지만 몇 가지 포인터가 존재한다. 사물을 기억하는 가장 좋은 방법은 (듣거나 읽는 것이 아니라) 이미지를 보는 것이라는 주장은 적어도 170년 전에 제시되었다.2 그러나 이 저자가 교수법에 근거하여 학생 유지의 비율을 설명할 수 있는 가장 이른 언급은 1906년 찰스 로드스3의 글이다. 그는 "우리는 듣는 것의 10분의 1을 기억하고, 보는 것의 10분의 5을 기억하고, 우리가 하는 일의 10분의 9을 기억한다."고 주장한다. 도로들은 이 문장을 인용부호로 표시하지만, 출처를 알려주지 않기 때문에, 인용부호는 그것이 일반적인 표현이라는 것을 나타낼 수 있다.

It is difficult to trace exactly the history of the Pyramid, but a few pointers exist. The argument that the best way to remember things is to see images (rather than hearing or reading) was presented at least 170 years ago.2 The earliest reference that this author can find ascribing proportions of student retention based upon methods of teaching, however, is a 1906 article by Charles Roads,3 who argues that, “We remember one tenth of what we hear, five tenths of what we see, seven tenths of what we say, [and] nine tenths of what we do.” Roads places this sentence in quotation marks, but does not give any source, so the quotation marks possibly indicate that it is a common expression.


1913년부터 Socony Vacuum 정유회사가 백분율로 차트를 만든 20세기 중반까지 숫자4의 다양성이 있었다.5 이후, NTL은 그 수치와 함께 자체 차트를 제작했고, NTL에 따르면, 그 결과에 대한 원래의 연구는 없었지만, NTL은 그 수치를 가지고 "학습 피라미드"를 NTL에 귀속시켰다. 피라미드의 토대가 되는 original research는 지금까지 발견된 적이 없다.6

From 1913, there were variations of the numbers4 until the mid20th century, when the SoconyVacuum Oil Company produced a chart with the percentages.5 Later, “during the 1950s and 1960s” the NTL produced their own chart with their figures and “the Learning Pyramid became attributed to NTL,”6 although, according to the NTL, no original research on which to base the Pyramid has ever been found.6


미국 교육자인 에드거 데일이 피라미드와 그 수치와 어떻게 연관되었는지는 불확실하다. NTL에 따르면, "1954년 뉴욕에서 Edgar Dale Dryden Press가 출판한 "Audio-Visual Methods in Teaching"이라는 책의 43페이지에 약간 다른 숫자를 가진 유사한 피라미드가 나타났다."6 불행하게도, 다양한 판에서 데일의 Audio-Visual Methods를 자세히 읽어본 결과, 그러한 피라미드도 없고, 그런 수치도 나타나지 않았다. 이 텍스트에서 학습 피라미드에 가장 가까운 대상은 "경험의 원뿔"이다. 이 원뿔은 "각자가 제공할 수 있는 상대적 수준의 명료성에 따라 다양한 유형의 교재들을 분류한다"고 한다. 이는 모델일 뿐이며 학습 유지 비율도 없다. 사실, 그것은 학습 보존을 전혀 언급하지 않는다.

It is uncertain how Edgar Dale, an American Educator, became associated with the Pyramid and its figures. According to the NTL, “a similar pyramid with slightly different numbers appeared on page 43 of a book called Audio-Visual Methods in Teaching, published by Edgar Dale Dryden Press in New York, in 1954.”6 Unfortunately, a close reading of Dale's Audiovisual Methods in Teaching in its various editions7-9 has shown no such Pyramid and no such figures. In those texts, the object closest to a Learning Pyramid is a “Cone of Experience.” This Cone “classifies various types of instructional materials according to the relative degree of concreteness that each can provide.”9 It is a model only, and has no percentages of learning retention; in fact, it does not reference learning retention at all.


피라미드와 피라미드의 관련 비율은 완전히 신화적이라고 해도 무방하다.

it is safe to say that the Pyramid and its associated percentages are entirely mythical.


의학교육에서 학습 피라미드

1.3 | The Learning Pyramid in medical education


이 연구의 목적은 이러한 일이 일어나고 있는지 여부를 결정하고 질문에 답하는 것이다. 5년 만에 무엇이 달라졌는가?

The aim of this study is to determine whether or not this is happening and to answer the question: What has changed in 5 years?


방법

2 | METHODS


전체적인 방법은 이전 검토에서 수행된 검색을 반복하는 것이었다.

The overall method was to repeat the search that had been performed in the previous review.


결과

3 | RESULTS


전반

3.1 | Overall


(see Figure 1).


문헌과 출처

3.2 | Articles and their sources


표 1은 발견된 기사의 요약을 보여준다. 예상대로 에드거 데일과 NTL은 이 수치들의 주요 출처로 언급되고 있다. 그러나 뜻밖에도 이제는 구체적으로 피라미드를 반박하는 새로운 출처조차 피라미드를 지지하는 근거로 등장한다.

Table 1 shows a summary of the articles found. As expected, Edgar Dale and the NTL are cited as the main sources of the figures. Unexpectedly, however, new sources that specifically refute the Pyramid now also appear as supporting the Pyramid.




백분율

3.3 | The percentages


대부분의 저자들은 백분율을 인용한다(표 2 참조).

Most of the authors quote percentages (see Table 2).




증가하는 인용

3.4 | Increased citing


발견된 많은 수의 기사(그림 1)와 기사의 날짜(표 1)는 피라미드를 인용하는 경향이 증가하는 것을 나타내는 것으로 보인다. 실제로 2013년 리뷰의 의학 기사와 이 리뷰의 기사의 발행 일자를 도표로 나타낸다면, 그림 2에 도표가 표시되어 있다.

The high number of articles found (Figure 1) and the dates of the articles (Table 1) appear to indicate an increasing trend in citing the Pyramid. Indeed, if one plots the publication dates of the medical articles in the 2013 review and the articles in this review, one has the chart depicted in Figure 2.


마지막으로, 일부 연구자들은 피라미드가 심각한 한계를 가지고 있다는 것을 알고 있고, 심지어 피라미드의 신화적 지위에 대해서도 알고 있으면서도, 그들은 여전히 그것을 인용하고 있다.

Finally, even though some researchers are aware that the Pyramid has severe limitations, and they may even be aware of its mythical status, they are still citing it.



고찰

4 | DISCUSSION


조사 결과: 

  • 피라미드를 인용하는 추세가 지속되고 심지어 증가했다고 한다. 

  • 피라미드의 출처 수가 증가했다는 것. 

  • 피라미드를 반박refutation하는 몇 가지 근거가 반대로 피라미드를 지지confirmation하는 것처럼 인용된다는 것, 

  • 피라미드에 이의를 제기할 때에도 피라미드를 지지하는 것으로 인용되고 있다는 것이다.


It has found: 

  • that the trend of citing the Pyramid has continued, and even increased; 

  • that the number of sources of the Pyramid has increased; 

  • that several refutations of the Pyramid are cited as confirmations of the Pyramid; and, 

  • finally, that even when the sources dispute the Pyramid, they are still being cited in support of the Pyramid.


인용 트렌드

4.1 | Citation trend


첫 번째 연구 질문은 다음과 같았다. 2013년 검토 이후 피라미드에 대한 참조의 수는 사용량이 감소, 일정 또는 증가하는 추세를 나타내는가?

The first research question was the following: Do the number of references to the Pyramid since that 2013 review indicate a trend of decreasing, constant or increasing usage?


유행이 급격히 증가하고 있다. 이것은 부분적으로 의학 교육 간행물의 증가의 결과일 것이지만, 피라미드가 점점 더 많이 인용되고 있다는 것은 부인할 수 없는 사실이다.

the trend is increasing dramatically. This will, in part, be a result of increased medical education publication overall, but the fact remains that the Pyramid is being increasingly cited.


피라미드의 출처

4.2 | Sources of the Pyramid


두 번째 연구 질문은 다음과 같았다. 피라미드의 출처가 2013년 이전과 동일한가, 아니면 새로운 출처가 나타났는가? 요컨대 피라미드의 출처의 수가 증가했다.

The second research question was: Are the sources for the Pyramid the same as before 2013, or have new sources emerged? In short: the number of sources of the Pyramid has increased.


더 걱정스러운 것은 피라미드를 반박하는 여러 논문들이 현재 피라미드를 지지하는 증거로 인용되고 있다는 사실이다. 예를 들어, 표 1은 피라미드 인용의 흐름을 가로막는 마스터스 20131보다는, 현재 피라미드를 지지하는 최소 3개의 논문이 인용하였기 때문에, 실제로 이 문제에 기여했음을 보여준다.24-26

More worrying is the fact that several papers that are refutations of the Pyramid are now being cited as evidence supporting the Pyramid. For example, rather than Masters 20131 stemming the tide of Pyramid citations, Table 1 shows that it has actually contributed to the problem, as it has now been cited by at least three papers in support of the Pyramid.24-26


왜 반박 출처를 거론하는지 알 수 없다. 연구자들이 초록만 읽고 있거나, 피라미드를 찾아 기사에서 찾은 다음, 그 주위에 있는 어떤 글도 읽지 않는 것인지도 모른다.

It is difficult to know why the citing of refuting sources is happening. It may be that researchers are reading the abstracts only, or that they search for the Pyramid, find it in an article, and then do not read any of the text around it.


4.3 | 신뢰성이 없음에도 불구하고 숫자 인용

4.3 | Citing of the figures in spite of their unreliability


또 다른 걱정스러운 경향은 피라미드가 믿을 수 없다는 사실을 연구자들이 인정함에도 불구하고 여전히 연구자들에 의해 인용되고 있다는 사실이다. 아마도 여기서 문제는 학문적 작업에 있어서 너무 공손한 표현일 것이다: 피라미드를 반박하는 작가들은 그것이 쓰레기라고 구체적으로 말하지 않았고, 그들은 그것을 할 필요가 있다.

Another worrying trend is the fact that the Pyramid is still being cited by researchers in spite of the fact that those researchers acknowledge that it is not to be trusted. Perhaps the problem here is the too polite phrasing in academic work: authors who refute the Pyramid have not stated specifically that it is rubbish, and they need to do that.


2013년 평론에서 너무 공손한 언어가 그 예다. 나는 논문의 본문에 "배움의 피라미드는, 그 퍼센티지를 가진, 신빙성이 떨어지며, 어떤 의학 교육 문헌에서도 받아들여서는 안 된다"라고 꽤 노골적으로 썼다.

An example is the too polite language of the 2013 review. In the body of the paper, I wrote quite plainly: “The Pyramid of Learning, with its percentages, is discredited, and should not be accepted in any medical education literature.”


이것은 메네 외 연구진 33이 그 검토에서 피라미드를 인용하고 "이 피라미드 건설에 대한 과학적 원천은 논쟁의 여지가 있다"고 말하는 이유를 설명할 수 있을 것이다. 텐 케이트와 더닝은 그것을 "흔히 인용되었지만 잘 확립되지 않은 교수법의 계층구조"라고 묘사하고 있다.17 그리고 더 최근에는 "정확한 비율들이 광범위하게 논의되고 있다"고 언급하면서도, 그 수치를 인용하고 있다.

This might explain why Menet et al33 cite the Pyramid from that review and say that “scientific sources for the construction of this pyramid are debatable.” Ten Cate and Durning describe it as an “often cited, but not well founded hierarchy of teaching methods…”17 and, more recently, an editorial in the Journal of Vascular Surgery34 quotes the figures, in spite of noting that “these exact percentages are widely debated.”


따라서, 이제는 논문의 초록에서조차 예의 바른 학문적 발언speak을 중지할 때가 되었다. 결과적으로, 모호함이 없도록 정보를 명확하게 말하는 것이 가장 좋다.

it is time to put a stop to polite academic speak, especially in the article's Abstract. As a result, it is best to state the information clearly so that there is no ambiguity,


그리고, 이것이 (초록에서 반복한) 진술이다: 피라미드는 쓰레기고, 피라미드의 숫자(통계)도 쓰레기다. 그리고 그것들은 에드거 데일에서 나온 것이 아니다. NTL이 원래의 연구에 대한 세부사항을 제공할 수 있을 때까지, 그들의 버전은 또한 쓰레기처럼 취급되어야 한다.

And so, this is the statement (to be repeated in the Abstract): The Pyramid is rubbish, the statistics are rubbish, and they do not come from Edgar Dale. Until the NTL can provide details about the original research, their version must also be treated as rubbish.


4.4 | 낮은 수준의 연구논문 및 의학교육에 미치는 영향

4.4 | Poor research writing and its implications for medical education


연구자들이 1차 출처를 따르지 않고 기사를 제대로 읽지 않는 등 문제의 상당 부분이 잘못된 참고 기술에서 비롯된다는 것은 그 결과에서 명백하다.

It is apparent from the Results that much of the problem stems from poor referencing techniques: researchers’ not following through to primary sources and not reading articles properly.


의학 교육 연구원들은 순수 의학에서 기대할 수 있는 정확성, 근면성, 증거의 동일한 기준을 고수해야 한다. Edgar Dale이 원래 말했던 "경험의 원뿔"는 버려져서는 안 된다. 왜냐하면 그것은 "각자가 제공할 수 있는 상대적 수준의 concreteness에 따른 다양한 유형의 교육 자료"의 분류 체계로서 유용하기 때문이다. 그리고 전문적인 경험과 관찰을 바탕으로 의학 교육에서 하나의 대상으로 받아들여진다.

medical education researchers should hold themselves to the same standards of accuracy, diligence and evidence that one would expect in pure medicine.35-38 Edgar Dale's original Cone of Experience should not be dismissed out of hand, for it is useful as a classification system of “various types of instructional materials according to the relative degree of concreteness that each can provide”9 and is acceptable in medical education as an object based on professional experience and observation.35


4.5 | 피라미드 신화는 왜 지속되는가?

4.5 | Why does the Pyramid myth persist?


오히려 이 논문의 범위를 벗어나기는 하지만, 왜 아직도 신화가 인용되고 있는가를 되돌아보는 것이 유용할 수도 있다. 우리는 이미 연구자들이 그들이 인용하고 있는 출처를 읽지 않거나, 기껏해야 그들을 잠깐 노려보는 문제가 있다는 것을 보았다. 그러나 그 이면에는 능동적인 학습이 수동적인 학습보다 더 가치 있다고 느낄 가능성이 높고, 다소 직관적인 느낌이 있다. 위의 2-4를 인용한 19세기 및 20세기 초반의 출처는 이러한 느낌이 한동안 존재해왔으며, (특히 역사적으로 disadvantaged 그룹에 대해) 능동적 학습에 대한 증거가 있음을 나타낸다.39-42 이러한 영향은 강의실 배치가 능동적인 학습을 위해 특별히 설계된 경우 가장 두드러진다.43 따라서, 이것들이 피라미드의 타당성을 뒷받침하기 위해 결합되었을 가능성이 매우 높다. 가장 결정적으로 피라미드에 존재하는 활동의 깔끔한 숫자 대괄호numerical bracket의 매력을 무시할 수 없다.

Although it is rather beyond the scope of this paper, it might be useful to reflect on why the myth is still being cited. We have already seen that there is the problem of researchers’ not reading the sources they are citing or, at most, glancing briefly at them. Behind that, however, is the likely, and somewhat intuitive, feeling that active learning is more valuable than passive. The 19th-century and early 20th-century sources cited above2-4 indicate that this feeling has existed for some time and there is current evidence in favour of active learning methods, especially for historically disadvantaged groups.39-42 This impact is most noticeable if the classroom arrangement is specifically designed for active learning.43 So, it is very likely that these have combined to support the validity of the Pyramid. Most crucially, one cannot ignore the appeal of the neat numerical brackets of activities that exist in the Pyramid.


그러나 교육 분야에서 일하는 사람이라면 누구나 경종을 울려야 하는 것이 바로 이러한 깔끔한 수치이다. 나는 이 수치들을 어린 교육생으로 처음 보았고 나의 즉각적인 반응은 "그래? 모든 문화, 모든 연령, 모든 주제, 항상 일관되게? 그게 어떻게 막연히 가능한가?" 나는 아직도 어떻게 교육학자가 브라질에서 수학을 공부하는 10세 소년의 학습 보유율이 일본의 제2외국어로서 영문학 공부를 하는 60세 여성의 그것과 동일할 수 있고, 또한 24세 아이비리그 공학 학생과도 동일할 수 있다고 믿을 수 있는지 당혹스럽다. 자신의 개인적 경험을 통해 스스로에게 다음과 같이 물어보십시오. 단순히 남을 가르쳐보면, 배운 모든 것의 90%를 정말로 기억하는가? 이 피라미드를 바탕으로, 세계의 모든 선생님들은 그들이 가르친 모든 것의 90%를 기억한다. 그것은 확실히 "직관"을 환상으로 멀리까지 확장시키는 것이다.

But it is surely those very neat numerical brackets that must set off alarm bells for anybody who works in education. I first saw these figures as a young education student and my immediate reaction was “Really? Across all cultures, all ages, all subjects, consistently for all time? How is that even vaguely possible?” I am still rather perplexed at how an educationist could believe that the learning retention rates for a 10-year-old boy studying mathematics in Brazil could be identical to those of a 60-year-old woman studying English Literature as a second language in Japan, and could also be identical to a 24-year-old Ivy League Engineering student. From your own personal experience, ask yourself: Do you really remember 90% of all that you have learned simply because you taught it? Based on this Pyramid, every teacher in the world remembers 90% of everything they have ever taught. That is surely stretching “intuition” far into fantasy.


피라미드 신화의 더 큰 맥락은 물론 교육에서 몇몇 다른 신화의 시작과 지속이다. 아마도 가장 주제적인 신화는 "학습 스타일"에 관한 것일 것이다. 학습스타일을 매우 간략하게 요약하자면, (복잡한 범위의 이론적 기초와 접근으로) 학생들이 다른 유형의 미디어를 선호하면 학습 보존과 효과의 차이를 초래한다고 주장한다. 학습스타일은 교육에 광범위하게 사용되고 있다. 학습 스타일은 그것의 사용에 반대하거나, (적어도) 신중하고 신중한 사용에 대해 논쟁하는 몇 가지 상세한 연구에도 불구하고 계속 사용된다. 피라미드와 학습 양식 모두에서, 신화를 진압하는 가장 좋은 방법은 이론의 기초가 되는 경험적 증거를 요구하는 것으로 보인다. 경험적 증거가 없거나, 경험적 증거가 오해된 것일 수도 있다.50

The larger context of the Pyramid myth is, of course, the beginning and persistence of several other myths in education. Perhaps the most topical myth concerns that of “learning styles,” which, in very brief summary, argues that students’ preference for different types of media leads to differences in learning retention and effectiveness, with a complex range of theoretical underpinnings and approaches, and is used extensively in education.44-46 The use of learning styles continues, in spite of several detailed studies arguing against their use or for (at best) circumspect and cautious usage.44,45,47-49 In both the Pyramid and learning styles, it appears that the best way to squash the myth is to call for the empirical evidence underlying the theory. It may be that there is no empirical evidence, or that the empirical evidence has been misunderstood.50


4.6 | 본 연구의 한계

4.6 | Limitations of this study


4.7 | 포스트스크립트

4.7 | Postscript


위에 인용된 수치 외에도 2018년도는 피라미드를 인용한 기사가 더 늘었다는 것을 알 수 있다.

In addition to the figures cited above, it is apparent that 2018 shows a further increase in the number of articles citing the Pyramid.


5 | CONCLUSIONS








 2019 Oct 1. doi: 10.1111/medu.13813. [Epub ahead of print]

Edgar Dale's Pyramid of Learning in medical education: Further expansion of the myth.

Author information

1
Medical Education and Informatics, Sultan Qaboos University, Muscat, Oman.

Abstract

INTRODUCTION:

A mythical Pyramid of Learning, usually attributed to Edgar Dale (or the National Training Laboratories [NTL]) and giving student learning retention rates, has been cited in a wide range of educational literature. A 2013 literature review indicated that medical education literature similarly cites this Pyramid. It was hoped that highlighting this myth in that review would reduce references to the Pyramid in future medical education literature. This study aimed at determining what change in Pyramid citation has occurred in the past 5 years.

METHODS:

A documented literature review, following the same process as the original review, was conducted. The search dates were September 2012 to April 2018, and the databases were Academic Search Complete, CINAHL, Medline and Google Scholar. Sources were from peer-reviewed journals or conferences, in English.

RESULTS:

From an initial search result of 992 documents, 41 were found to match the criteria. Trends discovered are: the number of Pyramid citations in medical education literature is increasing dramatically, new sources of the Pyramid are now being used, refutations of the Pyramid are being used to support it, and even researchers who acknowledge the weakness of the Pyramid still cite it.

DISCUSSION AND CONCLUSION:

In spite of the 2013 review, the situation has become worse. One possible reason is that refutations use too polite academic wording, and other researchers then consider the Pyramid to be merely "disputed" or "debated." To kill the myth of the Pyramid, it is necessary for this article's Abstract to state unequivocally: The Pyramid is rubbish, the statistics are rubbish, and they do not come from Edgar Dale. Until the NTL can provide details about the original research, their version must also be treated as rubbish.

PMID:
 
31576610
 
DOI:
 
10.1111/medu.13813


의학교육에서 에드가 데일의 배움의 피라미드: 문헌 고찰(Med Teach, 2013)

Edgar Dale’s Pyramid of Learning in medical education: A literature review

KEN MASTERS






도입

Introduction


교육훈련서적, 컨퍼런스 리포트, 동료 리뷰 저널 기사에서는 문제 기반 학습(북)과 같은 활동에 참여하는 학생들의 체류율을 설명할 때까지 학생들이 듣는 것의 10%, 읽은 것의 20%, 그리고 이러한 보유 비율의 10배 증가를 기억한다는 것이 널리 인용되고 있다.목재 외, 2003; 우드 2004; 우즈 2006; 예 외, 2011), 컴퓨터 기반 훈련 및 시뮬레이션(Barnes 2001; Buler 외, 2001; Chen et al. 2007; Krain & Lantis 2006) 사례 기반 학습(Golich et al. 2000년) 및 기타 구성주의 활동(Harker 2008; Khan et al. 2012; Pinto et al. 2012).

In education and training books, conference papers and peer-reviewed journal articles, it is widely cited that students remember 10% of what they hear, 20% of what they read, and these percentages of retention increase in multiples of 10 until they describe the retention rates of students involved in activities such as problem-based learning (Northwood et al. 2003; Wood 2004; Woods 2006; Yeh et al. 2011), computer-based training and simulation (Barnes 2001; Buehler et al. 2001; Chen et al. 2007; Krain & Lantis 2006) case-based learning (Golich et al. 2000) and other constructivist activities (Harker 2008; Khan et al. 2012; Pinto et al. 2012).


이러한 컨퍼런스 및 저널 기사의 비율은 WHO(PAHO 1997), 유네스코(Obana 2010), 세계은행(n.d.), 유럽생명공학 가상캠퍼스(Kybartaite et al. 2007), 뉴캐슬 온 타이인 대학(2004) 및 주립 대학, 주립 지원 뉴스레터들 (Iowa Department on Aging 2009) 등과 같은 유명 비학문 자료에서도 나타난다.

The percentages from these conference and journal articles are also supported in documents from well-respected, non-academic sources such as the WHO (PAHO 1997), UNESCO (Obanya 2010), the World Bank (n.d.), the European Virtual Campus for Biomedical Engineering (Kybartaite et al. 2007), the University of Newcastle Upon Tyne (2004) and even State sponsored newsletters (Iowa Department on Aging 2009).


이 퍼센트의 근거가 되는 연구를 인용할 때, 저자들은 때때로 이차적 출처를 인용한다(Lagowski 1990; Golich 등). 2000; Buler 등; 2001; Joss 2001; Obana 2010; Pinto 등. 2012) 또는 전혀 출처가 없는 경우도 있다.(PAHO 1997; Barnes 2001; Iowa Department on Aging 2009; Martinez & Jaganathan 2010).

When citing the research on which these percentages are based, authors sometimes cite secondary sources (Lagowski 1990; Golich et al. 2000; Buehler et al. 2001; Joss 2001; Obanya 2010; Pinto et al. 2012) or no sources at all (PAHO 1997; Barnes 2001; Iowa Department on Aging 2009; Martinez & Jagannathan 2010).


이 연구의 가장 흔한 두 가지 주요 원천은 응용 행동과학의 학습 피라미드(Kybartite et al. 2007; World Bank n.d)를 위한 국립 훈련 연구소(NTL)에드거 데일의 학습 원뿔 또는 학습 피라미드 (Campbell 1993; Bernardo 2003; Learning Northwood et al.; Krain & 2006; Wood et; 2006; 2006; 2006; Wood 첸 등이다.007; Harker 2008; Elouarat 외, 2011; Yeh 외, 2011; Khan 외, 2012; Pinto 외, 2012). 때때로 소코니-진공 석유 회사의 연구도 인용된다(Golich et al. 2000).

The two most common primary sources of the research are the National Training Laboratories (NTL) for Applied Behavioral Science’s Pyramid of Learning (Kybartaite et al. 2007; World Bank n.d.), and Edgar Dale’s Cone of Learning or (Campbell 1993; Bernardo 2003; Pyramid of Learning Northwood et al. 2003; Krain & Lantis 2006; Woods 2006; Chen et al. 2007; Harker 2008; Elouarat et al. 2011; Yeh et al. 2011; Khan et al. 2012; Pinto et al. 2012). Occasionally, the Socony-Vacuum Oil Company’s research is also cited (Golich et al. 2000).


이 퍼센트의 분명한 신뢰성이 확고히 확립되어 있기 때문에, 의심받을 만한 것은 아무것도 없어 보인다. 그러나 이러한 백분율을 대충 훑어보는 것은 분명히 경보를 유발해야 한다. 인간의 행동은 거의 모든 경우에 5 또는 10의 배수로 깔끔한 비율로 분류될 수 없다.

With the apparent credibility of these percentages firmly established, there appears nothing to be questioned. A cursory glance at these percentages, however, should surely trigger an alarm: human behaviour can seldom, if ever, be classified into neat percentages in multiples of 5 or 10.


피라미드를 지지하는 증거를 검토하는 것은 매우 중요하며, 분명한 출발점은 피라미드의 주요 출처에 상세히 기술된 연구다.

it is crucial to examine the evidence supporting the Pyramid, and the obvious starting point is the research detailed in the primary sources of the Pyramid.


피라미드의 출처와 백분율

The sources of the Pyramid and the percentages


이러한 백분율의 첫 번째 가능한 주요 출처인 NTL은 이 피라미드를 지원하는 출판되거나 출판되지 않은 연구 데이터를 가지고 있지 않다. NTL(Raymond 2012)의 이메일 서신에 따르면, 학습의 피라미드는 자체 연구에 기초하고 있다고 한다. 그러나 이 연구가 어디있는지 찾을 수 없었고 이 연구의 세부사항을 제공할 수 없다고 답했다. 그럼에도 불구하고, NTL은 "'학습 피라미드'가 변형된 것으로 보이며, 오랫동안 NTL의 결과로 귀속되어 왔다"고 설명한다. NTL 학습 피라미드(때로는 약간 다른 백분율로 표시됨)는 [그림 1]과 같다.(레이몬드 2012)

The first possible primary source of these percentages, the NTL, does not have any research data, published or unpublished, supporting its Pyramid. According to email correspondence from the NTL (Raymond 2012), its Pyramid of Learning is based upon its own research, although it has never been able to locate this research and has not been able to provide details of this research. Nevertheless, the NTL explains that ‘‘the Learning Pyramid as such seems to have been modified and has long been attributed to NTL. The NTL Learning Pyramid, sometimes with slightly different percentages, appears as [Figure 1].’’ (Raymond 2012)



NTL은 또한 에드거 데일이 1954년(레이몬드 2012) 본문에서 '조금 다른 숫자들을 가진 비슷한 피라미드'를 제작했다는 것을 인정한다. NTL에 따르면, "다음 [그림 2]는 Edgar Dale의 강의에서 시청각적 방법에 기인한 피라미드다." (레이몬드 2012).

The NTL further acknowledges that Edgar Dale produced ‘‘a similar pyramid with slightly different numbers’’ in his 1954 text (Raymond 2012). Audio-Visual Methods in Teaching According to the NTL, ‘‘The following [Figure 2] is the pyramid attributed to Edgar Dale’s Audio-Visual Methods in Teaching.’’ (Raymond 2012).



그러나 이 두 번째 (잠정적) 1차 출처를 보면 뭔가 다른 것을 볼 수 있다. Edgar Dale은 교수법에 관한 그의 책, 시청각 방법에서(Dale 1946, 1954, 1969), 에드가 데일은 '경험의 원뿔'(그림 3)을 제시하였지, 피라미드를 제기한 것이 아니다. (그의 텍스트의 다른 판들을 통해 텔레비전의 포함과 같은 몇 가지 업데이트가 있었을 뿐이다). 가장 중요한 것은 NTL에 의해 데일에 귀속된 학습의 피라미드와는 달리 데일의 경험론은 숫자나 백분율이 없으며, 특정 유형의 입력 소스나 활동에서 또는 어떤 기간 동안 또는 어떤 입력 소스나 활동으로부터도 정보를 보존할 수 있는 제안도 없다.

When we look at this second possible primary source, however, we see something different. In his text Audiovisiual methods in teaching (Dale 1946, 1954, 1969), Edgar Dale presents a ‘‘Cone of Experience’’ (Figure 3), and not a Pyramid of Learning. (Through the different editions of his text, there were some updates, such as the inclusion of television.) Most importantly, unlike the Pyramid of Learning attributed to Dale by the NTL, Dale’s Cone of Experience has no numbers or percentages, and no suggestion of retention of information from any input source or activity of any type, or for any length of time.



데일의 경험론은 단지 분류 도표일 뿐이다. 이 원뿔은 '각각의 경험이 제공할 수 있는 상대적 수준의 concreteness에 따라 다양한 유형의 교육 자료를 분류'한 것이다(1969년 데일). 데일은 자신의 경험담을 '모형일 뿐'이라고 말하며, "시각적 아날로지"로서 두뇌의 기능을 이해하기 위한 컴퓨터의 비유와 비교한다.

Dale’s Cone of Experience is merely a classification diagram. It ‘‘classifies various types of instructional materials according to the relative degree of concreteness that each can provide.’’ (Dale 1969). Dale presents his Cone of Experience as ‘‘only a model,’’ a ‘‘visual analogy,’’ comparing it to the analogy of the computer for understanding the functioning of the brain.


또한 일반적으로 인용되는 학습의 피라미드와 달리, 원뿔의 하단부에 있는 경험이 상단부에 있는 경험보다 우수하다는 암시도 없다. 반대로 데일의 토론에서는, 비록 모든 학습이 그렇게 이루어지는 것은 아니지만, 암시적으로 원하는 지향하는 방향을 굳이 말하자면, 상단부apex의 경험이 추상화abtraction를 지향하는 경향이 있다.

In addition, unlike the Pyramid of Learning commonly cited, there is no suggestion that the experience at the base is superior to the experience at the apex. On the contrary, in Dale’s discussion, if there is an implied desired direction of movement, it tends towards the abstraction at the apex, although not all learning happens like that.


데일은 '직접적이고 직접적인 경험'이 우리 학문의 기초를 이루는 가치를 기술하지만, 그는 또한 '물론 인간의 삶은 직접적이고, 구체적이며, 감각적인 수준에서 독점적으로 살 수 없다'고 분명히 언급하고 있으며, 많은 경우 학습은 더 높은 수준의 추상화를 지향하는 경향이 있다. 원뿔은 '교육 메시지instructional message를 얼마나 더 혹은 덜 추상적인가의 관점에서만 분류'하고 있을 뿐이며, 이것은 '학습 과정의 실제적인 서열'이 아니다. 교수자와 학습자는 모든 단계를 통과할 수 있어야 한다.

While Dale describes the value of the ‘‘direct, firsthand experiences that make up the foundation of our learning,’’ he also makes it clear that ‘‘human life cannot, of course, be lived exclusively on the direct, concrete, sensory level,’’ and frequently learning tends towards higher levels of abstraction. The Cone ‘‘classifies instructional messages only in terms of greater or lesser concreteness or abstractness,’’ and it is not an ‘‘exact rank order of learning processes.’’ The teacher and learner must be able to move through all levels.


요컨대(상식과 모순되는 것은 둘째치고라도) 이러한 비율의 숫자도 의심스럽다.

In short (apart from contradicting common-sense), these percentages are questionable


그렇다면 피라미드 구조와 퍼센티지는 그 어떤 실질적인 것에도 기초하고 있지 않은 것으로 보인다.

It appears, then, that the pyramid structure and the percentages are based on nothing substantial.


의학교육의 문제

The problem for medical education


의학교육을 뒷받침하기 위한 강력한 교육 연구와 이론의 필요성은 잘 알려져 있다(Pauli et al. 2000, 콜린스 2006, 깁스 외 2011). 게다가 의료 교육 관행은 가정과 잘못된 가정이 아니라 진정한true 연구에 기초해야 한다는 것이다.

The need for strong education research and theory to underpin medical education is well-recognised (Pauli et al. 2000; Collins 2006; Gibbs et al. 2011). It follows, moreover, that medical education practice must be based on true research, and not on suppositions and invalid assumptions.


방법

Methods


학습의 피라미드는 (실제 피라미드 없이 포함) 다양한 방법으로 표시될 수 있고, 다양한 출처로부터 참조될 수 있기 때문에 검색 용어는 넓었다.

Because the Pyramid of Learning might be displayed in a variety of ways (including without an actual pyramid), and might be referenced from a range of sources, the search terms were broad.



결과

Results


전반

Overall


피라미드의 기원과 적용가능성에 대해 의문을 제기하는 논문은 단 하나만 찾을 수 있었다. 갤러거 외 연구진(2012년)은 '학습 보존 피라미드'의 권한과 기원에 대해 일부에서 논란이 되고 있다고 지적하고 랄리와 밀러(2007년)를 인용한다. 그럼에도 불구하고 갤러거 외 연구진은 여전히 피라미드의 백분율을 워크숍에서 토론을 자극하기 위해 사용했다.

This search could find only one article that questioned the origin and applicability of the Pyramid. Gallagher et al. (2012) noted that the ‘‘authority and origins of the [Learning Retention Pyramid] are disputed in some quarters,’’ and cite Lalley and Miller (2007). Nevertheless, Gallagher et al. still used the percentages in the Pyramid to stimulate discussion in their workshop.


기사 및 그 출처

Articles and their sources


표 1은 발견된 기사의 요약을 제시한다.

Table 1 gives a summary of the articles found.



데일의 1차 텍스트(Dale 1946, 1954, 1969년) 외에도 편집된 텍스트에서 그의 피라미드 복제에 대한 언급은 세 가지가 있다.

In addition to Dale’s primary text (Dale 1946, 1954, 1969), there are three references to a replication of his Pyramid in an edited text.


본문을 점검한 결과, Donald Stewart(1969)의 한 챕터를 발견하게 되는데, 이 장에서는 Dale의 경험론에 대해 상세히 설명하고 자신의 해석 도표를 제공한다(그림 5).

Upon inspecting the text, one finds a chapter by Donald Stewart (1969) in which he elaborates on Dale’s Cone of Experience and supplies a diagram of his own interpretation (Figure 5).





백분율

The percentages


Hazlett(2009년)를 제외한 모든 저자들은 백분율을 인용한다.

All of the authors, apart from Hazlett (2009), quote percentages.


그러나 연구자들이 제시한 퍼센트는 표 2에 제시되어 있지만, 다양한 활동을 통해 몇 퍼센트의 정보를 보유하는지에 대해서는 합의가 이루어지지 않은 것으로 보인다. 

The percentages, as given by the researchers, are given in Table 2.  however,  it appears that there is no agreement on what percentage of information is retained through the different activities. 



고찰

Discussion


2012년에 논문의 상당 부분이 출판되었다는 사실은 피라미드와 그 비율이 현재 의학 교육 문헌에서 여전히 인용되고 있다는 것을 보여준다. 또한, 의학교육을 다루는 새로운 기사와 책을 통해 오류는 더 강화되고 있다(Frith 2013; Risavi et al. 2013; Sewell 2013).

The fact that a sizable proportion of the articles was published in 2012 indicates that the Pyramid and its percentages are still currently being cited in medical education literature. Further, the error is being reinforced in new articles and books dealing with medical education (Frith 2013; Risavi et al. 2013; Sewell 2013).


형편없는 참고문헌

Poor referencing


일부 작가들은 선의로 존경할 만한 2차 자료를 인용하고 있는 것이 분명하다. 2차 출처를 언급하는 것은 거의 권장되지 않지만, 반드시 받아들일 수 없는 학문적 관행을 나타내는 것은 아니다. 마찬가지로, 몇몇 저자들은 NTL 도표를 주요 소스로 인용했고, 따라서, 그 피라미드에 존재할 수 있는 오류에 대해서까지 비난할 수는 없다.

It is apparent that some authors are citing respectable secondary sources in good faith. While citing secondary sources is seldom advisable, it does not necessarily indicate an unacceptable academic practice. Similarly, several authors have cited the NTL diagram as a primary source, and, therefore, cannot be blamed for errors that may exist in that Pyramid.


그러나 다른 많은 경우에서는 부실한 참고문헌 인용 패턴이 발견되었다. 이는 연구를 저해하고 모순된 비율에 기여하기도 한다. 이것들은 사소한 인쇄상의 오류나 잘못 배치된 구두점 오류가 아니라 더 깊은 무언가에 대한 증거다. '사기'라는 단어는 아마도 너무 지나칠지 몰라도, 연구 결과는 명백히 학문적으로 만족스럽지 못한 모습을 보여주고 있다. 

In many other cases, however, there is a pattern of poor referencing, and this serves to undermine the research and also contributes to the contradictory percentages. These are not minor typographical errors or misplaced punctuation errors (for which students are routinely berated), but evidence of something deeper. The word ‘‘fraud’’ is probably too strong, but the evidence does point to something academically unsatisfactory. 


또한 일부 저자들이 2차 출처(예: Pei 2003)를 인용한 경우에는, 그 2차 출처문헌 자체가 존재하지 않기 때문에 저자들이 그 text를 consult했을 가능성은 낮다. 

In addition, where some authors have cited secondary sources (e.g. Pei 2003 citing Lagowski 1990), it is unlikely that the authors had consulted the text, as it does not exist. 


2차 출처가 존재하는 경우에서도, 여러 문헌들은 학술 논문의 주장을 뒷받침하는 텍스트로서 의심스럽다. 일부 경우(예: Gordon 1996; Murphy 1998; Kumar et al. 2009), 백분율은 아무런 언급이나 인용 없이 제시되기도 하는데, 이 말은 그 백분율이 자명하게 정확하다는 것을 함축한다. 이러한 텍스트는 그 후에 또다시 다른 텍스트(예: Keulers & Spauwen 2003)의 데이터 출처가 된다.

In cases where the secondary sources do exist, many are questionable as texts supporting arguments in an academic paper. In some cases (e.g. Gordon 1996; Murphy 1998; Kumar et al. 2009), the percentages are given without any reference or citation, and the implication is that they are self-evidently correct. These texts then become a source of data for other texts (e.g. Keulers & Spauwen 2003).


의학교육의 시사점

Implication for medical education


증거의 질에 대한 하든 외 연구진(1999)의 논의에 따르면, 에드거 데일의 경험의 원뿔(Cone of Experience)은 그의 전문적인 경험과 관찰에 기초하고 있기 때문에 여전히 분류 체계로서의 가치를 지닐 수 있다. 그러나 단순한 분류 체계를 넘어, 학습에서 retention 비율까지 적용하는 지점으로 이동하면, 이는 어떤 종류의 '측정'을 가정하고 있는 것이다. 즉, 우리는 이 비율에 대해 질문을 해야 한다. '이 측정은 어떻게 수행되었는가?’’

Harden et al.’s (1999) discussion of the quality of evidence supports the idea that Edgar Dale’s Cone of Experience still has value as a classification system, as it is based on his professional experience and observation. A move to a point at which we apply percentages of learning retention, however, assumes measurement, and we should ask the pertinent question that we would ask of any medical research: ‘‘howwas this measurement performed?’’


학습의 피라미드나 그 비율이 연구에 근거하는 것으로 검증될 수 있을 때까지, 의료 교육 연구자들은 이와 관련된 정보를 사용하는 것을 경계할 필요가 있다. 또한 아제르 외 연구진(2012년)이 지적한 바와 같이, 의료 교육 저널의 동료 검토자들이 참고문헌이 정확하게 보고되도록 할 필요가 있다.

Until the Pyramid of Learning or its percentages can be verified as grounded in research, there is a need for medical education researchers to be wary of using the information associated with them. There is also a need, as noted by Azer et al. (2012), for peer-reviewers of medical education journals to ensure that references are accurately reported.


이 연구를 통해, NTL을 인용하든, Edgar Dale을 인용하든 간에 학습의 피라미드는 실체가 없다는 것은 명백하다.

From this study, it is obvious from the background that, whether citing the NTL or Edgar Dale, the Pyramid of Learning has no substance.


함의

Implication for the NTL


적어도 연구자들과의 교신에서, 존재하지 않는 에드거 데일의 학습 피라미드에 대한 언급은 중단해야 한다.

At the very least, it should stop referring to Edgar Dale’s non-existent Pyramid of Learning in its correspondence with researchers.


Limitations


Conclusion















 2013 Nov;35(11):e1584-93. doi: 10.3109/0142159X.2013.800636. Epub 2013 Jun 11.

Edgar Dale's Pyramid of Learning in medical education: a literature review.

Author information

1
Sultan Qaboos University , Sultanate of Oman.

Abstract

BACKGROUND:

Edgar Dale's Pyramid of Learning and percentages of retained learning are cited in educational literature in a range of disciplines. The sources of the Pyramid, however, are misleading.

AIMS:

To examine the evidence supporting the Pyramid and the extent to which it is cited in medical education literature.

METHODS:

review of literature (1946-2012) based on a search utilising Academic Search Complete, CINAHL, Medline and Google Scholar conducted from September to November 2012.

RESULTS:

A total of 43 peer-reviewed medical education journal articles and conference papers were found. While some researchers had been misled by their sources, other authors' interpretations of the citations did not align with the content of those citations, had no such citations, had circular references, or consulted questionable sources. There was no agreement on the percentages of learning retention, in spite of many researchers' citing primary texts.

DISCUSSION AND CONCLUSION:

The inappropriate citing of the Pyramid and its associated percentages in medical education literature is widespread and continuous. This citing undermines much of the published work, and impacts on research-based medical education literature. While the area of learning/teaching strategies and amount of retention from each is an area for future research, any reference to the Pyramid should be avoided.

PMID:
 
23758180
 
DOI:
 
10.3109/0142159X.2013.800636


멀티미디어 설계 원칙을 이용하여 수업 진행한 효과(Med Educ, 2013)

Teaching for understanding in medical classrooms using multimedia design principles

Nabil Issa,1 Richard E. Mayer,2 Mary Schuller,1 Edward Wang,1 Michael B. Shapiro1 & Debra A. DaRosa1




도입

INTRODUCTION


메이어의 멀티미디어 원칙의 사용4는 대학생들의 retention(표시된 것을 기억)와 transfer(새로운 문제를 해결하기 위해 배운 것을 적용)을 향상시키는 것으로 나타났다.5–7

use of Mayer’s multimedia principles4 has been shown to improve retention (remembering what was presented) and transfer (applying what was learned to solve novel problems) in undergraduate college students.5–7


'멀티미디어'라는 용어는 시각 자료(예: 삽화와 같은 정적 이미지 또는 애니메이션과 같은 동적 이미지)와 언어 자료(예: 인쇄된 단어 또는 구어)의 존재를 말하며, 이러한 맥락에서 학습자에게 새로운 지식을 알리고 설명하는 데 사용된다.

The term ‘multimedia’ refers to the presence of both visual material (i.e. static images suchas illustrations or dynamic images such as animation) and verbal material (i.e. printed words or spoken words), which, in this context, are used to inform and explain new knowledge to learners.


교육자들은 빠르게 진화하는 기술을 교육의 새로운 잠재적 환경으로 사용하는 것의 중요성을 인식했다.8–10

Educators have recognised the importance of using rapidly evolving technologies as new potential environments for education.8–10


효과적인 멀티미디어 교육 자료의 설계는 인간이 어떻게 정보를 배우고 보유하는지에 관련된 선천적인 인지 프로세스를 강화해야 한다.8-10 잘못 설계된 멀티미디어 자료는 학습자들을 압도하고 혼란스럽게 할 수 있다.1 멀티미디어 학습의 인지 이론은 효과적인 증거 기반 멀티미디어 메시지를 개발하기 위한 원칙을 제공한다. 이 이론은 Mayer4에 의해 인지 부하 이론 12와 이중 채널 코딩 이론 13,14에 의해 개발되었으며, 경험적 연구15와 실제 적용에 의해 뒷받침된다.8,16,17

The design of effective multimedia educational materials should enhance the innate cognitive processes involved in how humans learn and retain information.8–10 Poorly designed multimedia materials may overwhelm and confuse learners.11 The cognitive theory of multimedia learning provides principles for developing effective, evidence-based multimedia messages. The theory was developed by Mayer4 based on cognitive load theory12 and dualchannel coding theory,13,14 and is backed by empirical research15 and real-world applications.8,16,17


초기 연구18,19는 rote learning이 즉각적인 보유 후 테스트에서 좋은 성과를 얻는 반면, 의미 있는 학습은 지연된 보유 후 테스트와 즉각적이고 지연된 이전 테스트에서 좋은 성과를 이끌어 낸다는 것을 보여주었다.20,21

Early research18,19 showed that rote learning leads to good performance on immediate retention post-tests, whereas meaningful learning leads to good performance on delayed retention post-tests and on immediate and delayed transfer post-tests.20,21


이전의 연구에서, 5 우리는 메이어의 멀티미디어 설계 원리에 따라 강의 슬라이드를 수정하는 것이 강의 완료 후 1시간 이내에 주어진 체재 후 시험에서 의대생들의 성과를 현저하게 향상시킨다는 것을 발견했다.4

In a previous study,5 we found that revising lecture slides according to Mayer’s multimedia design principles significantly improved medical students’ performance on a post-test of retention given within an hour of lecture completion.4


방법들

METHODS


참가자 및 설계

Participants and design


참가자는 외과 임상실습을 통해 진행 중인 사립 중서부의 한 의대 3학년 학생 80명이었다. 총 학급 크기는 156명이었고, 모든 학생들은 12주간의 외과 임상실습에서 교대로 수업을 받아야 했다.

The participants were 80 third year medical school students at a private, midwestern medical school progressing through their surgery clerkship. Total class size was 156 students, with all students having to rotate through the 12 week surgery clerkship.


재료 및 절차

Materials and procedure


매주 열리는 필수 핵심 커리큘럼 강의 시리즈가 이 연구의 장으로 작용했다. 초점은 쇼크라는 주제에 대한 1시간 강의였다. 이 강의의 전통적인 슬라이드는 Mayer의 멀티미디어 설계 원리를 배우기 전에 강사lecturer가 개발했으며, 대부분 글자가 글머리 기호 형식으로 포함되었다. 수정된 슬라이드는 강사instructor가 개발했으며, 그 후 다른 콘텐츠 전문가(MSh)와 함께 멀티미디어 원칙(REM)의 권위 및 교육자(DD)와 함께 변경 사항을 검토했다.

The required weekly core curriculum didactic lecture series served as the venue for this research. The focus was a 1-hour lecture on the topic of shock. The traditional slides for this lecture had been developed by the lecturer before learning about Mayer’s multimedia design principles and included mostly text in a bullet point format.4 The modified slides were developed by the instructor and changes were then reviewed with another content expert (MSh), as well as with an authority in multimedia principles (REM) and an educator (DD).


강사instructor는 전통적인 강의에서 각 슬라이드의 핵심 사항이나 메시지를 분석한 다음 관련 멀티미디어 원칙을 적용하여 이해를 증진하기 위해 어떻게 변경할 수 있는지 결정했다. 두 강의가 동일한 학습 목표에 대해 미리 제시되었기 때문에 각 슬라이드의 교육 포인트를 변경하지 않고 메이어의 설계 원칙을 적용하는 데 초점을 맞췄다.4

The instructor analysed the key point or message of each slide in the traditional lecture and then determined how it could be changed to enhance understanding through the application of relevant multimedia principles. The modification process focused on adapting Mayer’s design principles without changing each slide’s teaching points as both lectures were premised on the same learning objectives.4


50분 강의의 학습 목표에는 쇼크의 정의, 쇼크의 세포 기능 장애 메커니즘, 쇼크의 aetiology와 분류, 쇼크의 진단 및 관리 원칙이 포함되었다.

Learning objectives for the 50-minute lecture included: definition of shock; mechanisms of cellular dysfunction in shock; aetiology and classification of shock, and principles of diagnosis and management of shock.


그림 1은 Mayer의 설계 원칙을 사용한 수정 프로세스의 대표적인 슬라이드를 보여주고 있다. 

글머리 기호 텍스트가 시각적 표현으로 대체되었다.4 이것은 멀티미디어 원리로 알려져 있다. 

    • 메이어의 일관성 원칙에 따라 내용과는 직접 관련이 없는 모든 사진과 텍스트를 삭제했다. 

    • 우리는 메이어의 신호 전달 원리에 기초해 더 큰 글꼴과 다른 색상을 사용하여 중요한 교육 포인트를 강조하였다. 

    • 그래프와 관련 텍스트는 공간적 인접성 원리에 따라 화면에 인접하게 나타났다. 

    • 폐동맥 카테터를 띄울 때 발생하는 전파나 산소전달, 소비, 조직추출의 관계 등 복잡한 내용을 보여주는 슬라이드를 그림이나 그래프의 형태로 시각적 표현으로 제시하였고, 모달리티 원리에 따라 음성 내레이션을 사용하여 이러한 복잡한 현상을 설명하였다.

Figure 1 shows a representative slide for the modification process using Mayer’s design principles. 

    • Bulleted text was replaced by visual representation.4 This is known as the multimedia principle

    • All pictures and text not directly related to the content were deleted based on Mayer’s coherence principle. We highlighted important teaching points by using a larger font and a different colour based on Mayer’s signalling principle

    • Graphs and related text appeared contiguously on the screen in accordance with the spatial contiguity principle

    • Slides reflecting complex materials, such as the electric waves generated when floating a pulmonary artery catheter, or the relationships between oxygen delivery, consumption and tissue extraction, were presented as a visual representation in the form of a picture or graph, and used vocal narration to explain these complex phenomena based on the modality principle





슬라이드 수정 프로세스를 거치며 기존 슬라이드 세트의 총 35개의 슬라이드에서 수정된 슬라이드 세트의 28개의 슬라이드로 전체 슬라이드 수를 줄이는 데 도움이 되었다(그림 2).

The process of slide modification helped to decrease the total slide count from a total of 35 slides in the traditional slide set to 28 slides in the modified slide set (Fig. 2).



강사는 두 세션 동안 필수적인 자료 처리와 이해를 관리하기 위해 대화 방식의 전달 방식을 사용했다. 두 가지 조건 모두에서 강사는 강의가 끝날 때 데이터가 점진적으로 공개되는 임상 vignette를 제시했다. 각 데이터가 제시된 후, 학생들은 진단에 도달하거나 개입을 하고 그들의 대답에 대한 적절한 정당성을 제시하도록 요청받았다. 강사는 학생들이 주제에 대한 세대적 이해를 증진할 수 있도록 돕기 위해 in-time 피드백을 제공했다. 이전 연구에서 사용된 것과 유사한 방식으로, 두 개의 강의 형식은 강사의 스타일 차이를 통제하기 위해 동일한 강사에 의해 진행되었다.

The lecturer used a conversational style of delivery during both sessions to help manage essential processing and understanding of the material. In both conditions, at the end of the lecture, the lecturer presented a clinical vignette in which data were progressively disclosed. After each set of data had been presented, students were asked to reach a diagnosis or provide an intervention and give appropriate justification for their answer. In-time feedback was provided by the lecturer to help students foster generative understanding of the topic. In a manner similar to that used in our previous research,5 both lecture formats were given by the same lecturer to control for lecturer style differences.


종이로 된 시험 자료에는 종전에 개발된 5항목 보유시험과 5항목 이전시험이 포함되었다.5 

    • Retention 시험은 '충격의 정의' '충격의 종류별 열거' '폐동맥 카테터가 제공하는 수치변수는 무엇인가?'와 같은 개방형 질문을 사용하여 평가되었다. 

    • Transfer 시험질문도 개방형이었고 응답자는 제시된 정보를 담낭염에 대한 환자성 패혈성 쇼크 2차 임상 증상에 적용해야 했다.

The paper-based test materials included the previously developed five-item retention test and five-item transfer test.5 

    • Retention was assessed using open-ended questions, such as: ‘Define shock’; ‘Enumerate the different types of shock’, and ‘What are the numerical variables provided by the pulmonary artery catheter?’ 

    • Transfer test questions were also open-ended and required respondents to apply the presented information to a clinical vignette of a patient inseptic shock secondary to cholangitis.


보유 및 이전 시험 문제는 강의 직전(사전 시험)과 강의 후 1시간(즉시 시험) 후 1시간 후에 1회 시험으로 관리되었다. 테스트는 1주, 4주 후에 다시 등록되었다(후 테스트 지연). 같은 시험이 도처에서 사용되었다. 학생들은 10분 동안 시험을 끝마쳤다. 테스트 형식과 질문 유형은 Mayer의 연구에서 멀티미디어 학습 효과에 대한 질문 형식을 사용하여 개발되었다.4 각각의 질문에 대해 1점, 반점 또는 무점이 주어졌다.

The retention and transfer test questions were administered as one examination just before (pre-test) and 1 hour after the lecture (immediate post-test). The tests were re-administered 1 week and 4 weeks later (delayed post-tests). The same examination was used throughout. Students were given 10 minutes to complete the examination. The test format and question types were developed using the question format published in Mayer’s studies on the effects of multimedia learning.4 One point, half a point or no point was given for each of the questions.


이전 연구에서 테스트와 해당 응답 키가 모두 개발 및 현장 테스트되었음.5 원래 연구5에서 시험을 채점한 두 명의 컨텐츠 전문가(NI와 MSc)도 첫 번째 연구를 위해 개발한 채점 루빅을 사용하여 이번 연구에서 점수를 매겼다.

Both the tests and corresponding answer keys were developed and field-tested in our previous study.5 The two content experts (NI and MSc) who scored the tests in the original study5 also scored the tests in the current study using the scoring rubric they had developed for the first study.


본 연구에서는 모든 시험 문제를 한 연구자가 채점했으며 무작위로 선택한 20%의 표본은 다른 임상 연구자에 의해 독립적으로 재점화하여 층간 합의를 검증하였다.

In the current study, all test questions were scored by one researcher and a randomly chosen sample of 20% of the scored tests were re-scored independently by another clinician-researcher to verify inter-rater agreement.


통계학적 분석

Statistical analysis


또한 시험 전 점수를 공변량으로서 사용하여 시험 후 시험 점수의 그룹 차이를 비교하기 위해 공분산(ANCOVA) 분석을 수행했다. 마지막으로, 그룹, 시간 및 그룹*시간 상호작용을 고정 효과로 사용한 일반화된 혼합 효과 모델을 채택하고, 수정 조건 그룹과 기존 조건 그룹 간의 시간에 따른 시험 점수의 변화 궤적을 비교하기 위해 요격을 무작위 효과로 채택했다.

We further conducted analysis of covariance (ANCOVA) to compare the group differences in transfer and retention post-test scores using the pre-test score as a covariate. Finally, we employed a generalised mixed-effects model using group, time and the group*time interaction as fixed effects, and intercept as a random effect to compare the trajectory of change in test scores over time between the modified and the traditional condition groups.


결과

RESULTS


표 1은 사전 테스트, 즉석 테스트, 1주 후 테스트 및 4주 후 테스트의 두 그룹에 대한 전송 테스트의 평균 ± SD 점수를 보여준다.

Table 1 shows the mean ± SD scores on the transfer test for the two groups on the pre-test, immediate post-test, 1-week post-test and 4-week post-test.



그림 3에서 알 수 있듯이, 강의 직후 기존 조건군은 이전(d = 0.61, p < 0.01)과 보유(d = 1.89, p < 0.01) 점수의 유의한 증가를 보였으며, 이와 유사하게 수정된 조건군은 이전 테스트(d = 1.25, p < 0.01)와 보유(d = 유지)에 대한 즉각적인 사후 테스트(d = 1.25, p < 0.01))로 유의한 증가를 보였다. 2.67, p < 0.01>.

As Fig. 3 shows, immediately after lectures, the traditional condition group showed a significant increase in transfer (d = 0.61, p < 0.01) and retention (d = 1.89, p < 0.01) scores; similarly, the modified condition group showed a significant increase from the pre-test to the immediate post-test on transfer (d = 1.25, p < 0.01) and retention (d = 2.67, p < 0.01).



고찰

DISCUSSION


현재의 연구는 멀티미디어 학습의 인지 이론에 기초한 설계 원리를 적용하면 의대생들의 장기 보유와 전이가 개선된다는 것을 보여주었다. 단기 retention 및 transfer과 관련된 우리의 연구 결과는 메이어의 이전 연구와 일치한다.

The current study showed that applying design principles based on the cognitive theory of multimedia learning improves medical students’ long-term retention and transfer. Our findings regarding shortterm retention and transfer are consistent with those noted in Mayer’s earlier work


현재의 연구는 의대생들에게 주어진 멀티미디어 강의에 메이어의 디자인 원칙4를 적용하면 단기 retention뿐만 아니라, 학생들 사이에서 의미 있는 학습의 중요한 지표가 되는 연구 자료의 장기 transfer을 개선할 수 있다는 것을 보여줌으로써 그러한 초기 발견들에 대해 확대한다.

The current study expands on those early findings by showing that applying Mayer’s design principles4 to a multimedia lecture given to medical students improves not just short-term retention, but also longterm transfer of the study material, which researchers have identified as an important indicator of meaningful learning among students.


현재의 연구에는 몇 가지 한계가 있었다. 이번 연구는 단일 학술기관에서 강의 주제와 강사 1명을 이용해 진행됐으며, 강의 발표에 무의식적인 편견을 도입했을 가능성이 있다. 그러나 학생들의 강의 등급은 다르지 않았다. 5.00점 만점에 4.73점, 4.67점이라는 등급은 각각 변형된 조건군과 전통적인 조건군에서 얻었다.

There were several limitations to the current study. The study was carried out at a single academic institution using a single lecture topic and a single lecturer, which may have introduced unconscious bias into the lecture presentations. Ratings of the lectures by the students, however, did not differ; ratings of 4.73 and 4.67 out of 5.00 were obtained from the modified condition group and the traditional condition group, respectively


이 연구를 구축하거나 복제하는 데 관심이 있는 연구원들은 두 그룹의 테스트에 대해서 평가자가 완전히 blind 하기 위해 연구자에 의해 그룹 유형을 지정하도록 코드화되었는지 확인해야 한다.

Researchers interested in building or replicating this research should ensure that the tests from both groups are coded to designate group type by a researcher in order to ensure graders are completely blinded.


멀티미디어 원칙의 채택 정도에 관계없이, 교육 개입에 대한 어떤 종류의 검토와 피드백이든 학습 성과에 차이를 일으킬 가능성이 있다.

It is plausible that any kind of review and feedback on any teaching intervention will cause differences in learning outcomes, regardless of the extent to which multimedia principles were employed.


현재 연구 설계는 독립 변수로서의 시간 인자를 강화하기 위해 이전 연구5에서 사용된 방법론과 자료에 충실했다. 따라서, 시험 문제는 데이터 수집 지점들 사이에 차이가 없었으며, 이는 일부 학생들이 자신의 답을 기억하고 수정하지 않는 것을 선택할 수 있게 했을 수도 있다. 동일한 코호트 학습자에게 동일한 테스트 자료를 반복적으로 사용하는 것은 테스트 효과를 도입했을 수 있다. 기존 연구에서, 학습자는 자료를 단순히 두 번 학습하는 것보다, 자료를 공부하고 연습 시험을 보는 상황에서 지연 시험delayed test(유지 및 전달 기능 모두)를 더 잘 수행하는 것으로 나타났다. 이 연구들의 연구결과는 [시험이 학습에 더 큰 영향을 미친다는 것]을 시사하고 있으며, [시험을 일종의 교육 도구로 채택하는 것]을 옹호하고 있다.

The current study design remained faithful to the methodology and materials used in the earlier study5 in order to enhance the factor of time as the independent variable. Thus, the test questions did not vary between the data collection points, which may have allowed some students to remember their own answers and choose not to revise them. Using the same test material in a repeated fashion for the same cohort of learners may have introduced a testing effect. Learners have been shown to perform better on delayed tests (for both retention and transfer functions) after studying the material and taking a practice test than after studying the material twice.24,25 The findings of those studies24,25 suggest that tests have an enhancing effect on learning and advocate for the adoption of tests as a teaching tool.


본 연구에서, 두 그룹의 학생들 모두 그들이 세션에서 배운 것을 기억했지만, 수정된 강의 조건의 학생들은 전통적인 강의 조건의 학생들보다 훨씬 더 높은 수준의 학습 증가와 지속적 학습을 보여주었다. 이러한 데이터는 교수진이 멀티미디어 원칙을 배우고 이를 강의실 강의나 원격 학습 모듈에 포함하도록 동기를 부여할 수 있는 설득력 있는 증거를 제공해야 한다.

In the present study, students in both groups remembered what they had learned in the session, but students in the modified lecture condition demonstrated increased and sustained learning to a greater extent than students in the traditional lecture condition. These data should provide convincing evidence to motivate faculty staff to learn about multimedia principles and to include them in their classroom lectures or distance learning modules.


3 Association of American Medical Colleges. Effective Use of Educational Technology in Medical Education. Candler C, ed. Institute for Improving Medical Education. Washington, DC: AAMC, 2007.








 2013 Apr;47(4):388-96. doi: 10.1111/medu.12127.

Teaching for understanding in medical classrooms using multimedia design principles.

Author information

1
Department of Surgery, Northwestern University Feinberg School of Medicine, Chicago, Illinois 60611, USA. nissa@northwestern.edu

Abstract

OBJECTIVES:

In line with a recent report entitled Effective Use of Educational Technology in Medical Education from the Association of American Medical Colleges Institute for Improving Medical Education (AAMC-IME), this study examined whether revising a medical lecture based on evidence-based principles of multimedia design would lead to improved long-term transfer and retention in Year 3 medical students. A previous study yielded positive effects on an immediate retention test, but did not investigate long-term effects.

METHODS:

In a pre-test/post-test control design, a cohort of 37 Year 3 medical students at a private, midwestern medical school received a bullet point-based PowerPoint™ lecture on shock developed by the instructor as part of their core curriculum (the traditional condition group). Another cohort of 43 similar medical students received a lecture covering identical content using slides redesigned according to Mayer's evidence-based principles of multimedia design (the modified condition group).

RESULTS:

Findings showed that the modified condition group significantly outscored the traditional condition group on delayed tests of transfer given 1 week (d = 0.83) and 4 weeks (d = 1.17) after instruction, and on delayed tests of retention given 1 week (d = 0.83) and 4 weeks (d = 0.79) after instruction. The modified condition group also significantly outperformed the traditional condition group on immediate tests of retention (d = 1.49) and transfer (d = 0.76).

CONCLUSIONS:

This study provides the first evidence that applying multimedia design principles to an actual medical lecture has significant effects on measures of learner understanding (i.e. long-term transfer and long-term retention). This work reinforces the need to apply the science of learning and instruction in medical education.

PMID:
 
23488758
 
DOI:
 
10.1111/medu.12127


가르침을 배움 프로세스에 맞추기 (Acad Med, 2004)

Orienting Teaching Toward the Learning Process

Olle ten Cate, PhD, Linda Snell, MD, Karen Mann, PhD, and Jan Vermunt, PhD





의료 커리큘럼의 급속한 진화는 교사들의 혼란을 초래할 수 있다. 커리큘럼 내용과 과정이 교육 이론에 의해 보다 중앙에서 통제되고 지도됨에 따라, 교사들은 커리큘럼 변화의 기초가 되는 철학을 파악하여 그것들을 매일 실천하는 데 어려움을 겪을 수 있다. 예를 들어, 학생 중심의 교육은 많은 경험 있는 교사들에게는 좋은 접근법으로 들릴 수도 있지만, 일상적 연습에 쉽게 도입되지는 않는다.

The rapid evolution in medical curricula can lead to confusion in teachers. As curricular content and process become more centrally controlled and guided by educational theory, teachers may have difficulty grasping the philosophies underlying curricular change and putting them into daily practice. Student-centered teaching, for example, may sound to many experienced teachers like a laudable approach, but not easily put into day-to-day practice.


이 글에서, 우리는 교사들에게 [무엇이 학생들에게 동기를 부여하고 왜 학습자가 교육 활동의 중심이 되어야 하는지]를 이해하는 데 도움을 줄 수 있는 가르치는 모델을 제안한다.

In this article, we propose a model for teaching that can help teachers understand what motivates students and why learners should be the central focus of teaching activities.


교육이 학습을 촉진하기 위한 것이라면, 분명히, 교육 활동은 학습 과정을 지향해야 한다. 우리 모델의 중심에는 두 가지 차원이 있다. 

  • (1) 학습 과정의 핵심 특징 분석과 이러한 특징과 교사 기능의 연결, 그리고 

  • (2) 학습의 외부 규제와 자율 규제 사이의 상호 작용이다.

If teaching is to facilitate learning, clearly, teacher activities should be oriented toward the learning process.1 Two dimensions are central to our model: 

  • (1) the analysis of critical features of the learning process and the linking of teacher functions to these features, and 

  • (2) the interplay between external regulation and self-regulation of learning.


DIMENSION 1: 관련 절차의 중요 구성 요소

DIMENSION 1: CRITICAL COMPONENTS OF THE LEARNING PROCESS


몇몇 저자들은 Bloom의 교육목표 영역을 연상시키는 학습의 인지적 요소, 정서 요소, 그리고 메타인지적 요소들을 구별한다.7–9 최근 메이어는 효과적인 문제해결에 필요한 기술, 의지 및 메타스킬10에 대해 자세히 설명하였다. 이는 우리의 초점의 요소들과 명확하게 상응하며, 우리의 학습중심교육(LOT) 모델에서 핵심 요소들이다.


Several authors distinguish among cognitive, affective, and metacognitive components of learning reminiscent of Bloom’s domains of educational objectives.7–9 More recently, Mayer elaborated on the skill, will, and metaskill10 needed for effective problem solving, which clearly parallel the components of our focus and are core elements in our learning-oriented teaching (LOT) model.


학습 프로세스의 인지 구성요소

Cognitive Component of the Learning Process


본질적으로, 학습자는 책, 생활 사례, 인터넷, 다른 매체 등 방대한 외부 지식체로부터 (무엇을 배울지) 선택을 해야 한다. 이러한 학습의 측면은 "무엇을 배워야 하는가?"라는 질문으로 요약할 수 있다. 즉, 학습의 내용이나 목표는 무엇이며, 이 내용을 어디에서 찾아야 하며, 정보를 적절하게 처리하기 위해 어떻게 구성되어야 하는가?

Essentially, the learner must make a selection from the vast external body of knowledge—from books, living examples, the Internet, other media, etc. This aspect of learning can be summarized by the question “What should be learned?” That is, what is the content or objective of the learning, where should this content be found, and how should it be structured to adequately process the information?


학습 프로세스의 영향력 있는 구성 요소

Affective Component of the Learning Process


학습의 영향을 주는 요소는 집중적인 학습을 시작하고 지속하려는 학습자의 동기를 다룬다. 이 구성 요소는 외재적이거나 내재적인 동기, 학습내용과의 감정적 관계, 그리고 공부할 준비와 관련이 있다. 영향을 미치는 구성 요소는 "왜 배우는가?"라는 질문으로 요약할 수 있다.

The affective component of learning deals with the learner’s motivation to start and persist in concentrated learning. This component pertains to extrinsic and intrinsic motivation, to emotional relationship to the content materials, and to readiness to study. The affective component may be summarized with the question “Why learn?”


학습 과정의 인지 구성요소

Metacognitive Component of the Learning Process


학생이 배울 수 있는 충분한 동기 부여와 함께 인지 능력과 정보의 존재는 학생이 배우는 방법을 모른다면 적절한 학습 과정을 초래하지 않을 수 있다. 정보를 처리하기 위해 학습자는 메타인지 능력이 필요하다: 학습 활동을 계획하고, 진행 상황을 모니터링하고 평가하며, 개인의 지식 부족을 진단하고 해결할 수 있어야 한다. 이것들은 또한 메타인지적 조절활동이라고 불려왔다.14

The presence of cognitive skills and information combined with sufficient motivation to learn may not result in an adequate learning process if the student does not know how to learn. A learner needs metacognitive skills to process information: he or she must be able to plan study activities, to monitor and evaluate progress, to diagnose and address personal lack of knowledge. These have also been called metacognitive regulation activities.14


표 1은 학습자의 매크로 및 마이크로 관점에서 세 가지 구성 요소 모두에 대한 이러한 질문의 예를 제시한다. 우리는 6개의 라벨을 선택했는데, 그 중 대부분은 배움의 내용 개념, 배움의 목적 개념, 그리고 배움의 방법 개념이다.

Table 1 gives examples of these questions on all three components, from the learners’ macro and micro perspectives. We have chosen six labels in the cells, most of which were adapted from the work of Vermunt15: the content conception of learning, the purpose conception of learning , and the method conception of learning


  • 거시적 관점은 학습자의 성격 특성과 의견을 반영하고 지배적인 학습 스타일을 형성하는 반면, 

  • 미시적 관점은 반드시 달성해야 하는 특정 학습 활동과 관련이 있다.

  • The macro perspective reflects personality characteristics and opinions of the learner and shapes a dominant learning style, whereas 

  • the micro perspective pertains to specific learning activities that must be accomplished.



설명 2: 학습의 외부규제 및 자기규제의 상호작용

DIMENSION 2: THE INTERPLAY BETWEEN EXTERNAL REGULATION AND SELF-REGULATION OF LEARNING


교육은 두 가지 주요 목적에 기여한다. 

  • (1) 학생들에게 도메인 의존적 지식과 기술을 창출하는 것과 

  • (2) 학생들이 독자적으로 더 발전할 수 있는 지역사회의 성인 책임 있는 구성원으로 발전하는 것을 돕는 것(즉, 교사나 학교의 지도 없이)

후자의 목적은 대개 많은 교사들에게 받아들여지는 암묵적인 목표인 경향이 있지만, 커리큘럼에서 구체적인 관심을 덜 받게 된다.

Education serves at least two major two purposes: 

  • (1) generating domain-dependent knowledge and skills in students, and 

  • (2) helping them develop into adult, responsible members of the community, who can further develop independently (i.e., without guidance by a teacher or a school). 

The latter purpose usually tends to be an implicit goal, accepted by many teachers, but given less specific attention in the curriculum.


학생들을 사회에 대비시키려면 교사와 [학교에 의한 학생 학습 과정의 규제]에서 [배움의 자율화]로 전환해야 한다. 고등교육에 들어가기 전에, 대부분의 학생들은 무엇을, 왜, 어떻게 배워야 하는지에 대한 제한된 생각을 가지고 있다. 대학 수준의 교육은 졸업생들의 훈련 영역뿐만 아니라 더 넓은 의미에서의 독립적 기능independent functioning을 목표로 한다.

If we want to prepare our students for society, a shift from a regulation of the student learning process by teachers and school to self-regulation of learning should therefore take place during the period of formal education. Before entering higher education, most students have limited ideas of what, why, and how to learn.16 Education at the university level aims at independent functioning of graduates, not only in the domain of their training, but also in a broader sense.


우리는 의학 교육에서 의대에서 인턴쉽으로의 전환될 때 학생들은 완전히 훈련받은 않은 책임에 갑작스럽게 당면하게 된다는 것을 알고 있다. 학생 스스로 학습목표를 정하도록 하는 PBL에서도 주제와 자료는 여전히 치밀하게 계획되어 있으며, 학습목표는 커리큘럼의 일반적인 목표에 맞게 설계되어 있다. 따라서 외부-지도에서 자기-지도로의 더 부드러운 전환은 공식적인 교육의 목표가 되어야 한다.

We know that in medical education, the transition from medical school to internship is a sudden confrontation with responsibilities for which students have only partially been trained. Even in PBL, where students are asked to set their own learning goals, the themes and materials are still carefully planned, and learning goals are designed to fit within the general objectives of the curriculum. A better shift from external guidance to self-guidance should therefore be a goal of formal education.


교육심리에 따르면, 학생들은 자기조절학습 능력이나 독립을 성장에 대한 능력이 서로 다르다. 그 결과로, 필요 이상으로 많은 지침guidance을 받는 것은 에너지 낭비로 이어질 수 있으며, 부작용도 발생할 수 있다.

According to educational psychology, students vary in their capability for self-regulated learning and growth towards independence.17 Consequently, their need for external guidance may differ. Receiving more guidance than needed will lead to wasted energy, and adverse effects may result.


Guidance과 self-regulation 사이에서 균형을 찾아야 한다. Vermunt와 Verloop은 이것을 학문과 교수 사이의 건설적인 마찰에 대한 탐색이라고 불렀다. 이는 Vygotsky의 근위부 발달 구역에 필적할 만한 것이다: [독립적인 문제 해결 능력에 의해 결정되는 실제 개발 수준]과 [다른 사람의 도움을 받는 잠재적 개발 수준] 사이의 거리.18. 건설적 마찰constructive friction은 학생들에게 일상적인 활동보다 더 많은 지적 노력을 요구함으로써 새로운 지식과 기술을 익히려는 노력으로 이어지게끔 한다.

A balance should be found between guidance and self-regulation. Vermunt and Verloop have called this the search for constructive friction between learning and teaching, comparable to Vygotsky’s zone of proximal development: the distance between the actual developmental level as determined by independent problem solving capability, and the level of potential development with the assistance of others.18 Constructive friction leads to an effort by the student to master new knowledge and skills, by demanding more intellectual effort than routine activities take.


학생의 요구에 비해 너무 적거나 너무 많은 지도는 파괴적인 마찰destructive friction로 이어질 것이며, 따라서 불충분한 학습은 분명히 우리의 목표는 학생들의 자기주도 능력에 최적의 민감성을 유지하면서 완전한 외부 규제에서 학습의 자율규제로 전환하는 것이다..이 변화를 어떻게 실현할 수 있을까?

Too little or too much guidance, relative to the needs of the student, will result in what has been called destructive frictions and, therefore, inadequate learning.11 Clearly, our goal is a transition from full external regulation to self-regulation of learning, while maintaining an optimal sensitivity for the student’s capacity for self-direction. How can we realize this transition?


Shared Guidance의 개념

THE CONCEPT OF SHARED GUIDANCE


이제 선생님이 사진 속으로 들어온다. 현대의학 커리큘럼은 그들 자신의 학습에 대한 학생들의 개인적인 책임을 강조한다. 더 작은 그룹 학습과 더 많은 자기 주도적인 학습으로 바뀌면서, 학생들에게 성숙한 학습 능력이 요구된다. 구성주의 철학(즉, 학습자가 이미 알고 있는 것에 근거하여 자신의 지식을 "구축"한다)19은 커리큘럼의 발견학습 접근방식을 정당화하기 위해 사용될 수 있다. 즉, 학생들은 배우는 법을 배워야 한다. 교사들이 할 수 있는 일은 과제 제공, 독립적인 학습 시간 스케줄링, 또는 임상적 순환에서 학생들을 스케줄링하는 것 외에 무엇인가?

Now the teacher comes into the picture. Modern medical curricula emphasize the personal responsibility of students for their own learning. With a shift towards more small-group learning and more self-directed learning, mature studying skills are required of students. A constructivist philosophy (i.e., learners “construct” their own knowledge on the basis of what they already know)19 may be used to justify a discovery-learning approach in the curriculum: students should learn to learn. What can teachers do besides providing assignments, scheduling independent learning time, or scheduling students in clinical rotations?


전통적인 커리큘럼은 많은 학생들에게 비효율적인 것으로 보일 수 있다. 왜냐하면 정보의 제시와 계획 요건이 학생들의 요구를 충족시키기 부족하기 때문이다. 반대로 커리큘럼은 또한 너무 많은 학생들에게 요구함으로써 해를 끼칠 수도 있다. 두 가지 모두 역량과 요구 사이의 "파괴적 마찰"을 초래할 수 있다.

A traditional curriculum may be viewed as inefficient for many students because the presentation of information and the planning requirements are insufficiently geared to the students’ needs; however, a curriculum may also do harm by asking too much of students. Both may result in “destructive friction” between competence and demands.


이상적으로는 교사와 학생이 함께 일하는 학습과정의 공유지침 단계가 있다. Collins 등.3은 이 단계를 코칭 또는 비계라고 부른다; Vermunt는 배움의 규제에 대한 공유된 통제에 대해 말한다.20

Ideally, there is a stage of shared guidance of the learning process in which the teacher and the student work together. Collins et al.3 call this stage coaching or scaffolding; Vermunt speaks of shared control over the regulation of learning.20


Shared guidance은 학생들이 무엇을 알고 있는지, 무엇을 추진하는지, 그리고 그들이 어떤 인지능력을 가지고 있는지에 대한 인식을 필요로 하며, 건설적인 마찰로 이어지는 교육 환경을 조성해야 한다. 학습의 초기 단계에서 건설적인 마찰은 훈련의 후반 단계에서 발견되는 요구와는 상당히 다른 요구에서 비롯되기 때문에, shared guidance은 고정된 일련의 교육 활동으로 이어지지 않는다. 대신, shared guidance은 학생들과의 대화, 진행 상황의 모니터링, 그리고 그들의 인식된 필요에 따라 가르치는 것이 필요하다.

Shared guidance requires awareness of what students know, what drives them and what metacognitive skills they possess, to generate an educational environment that leads to constructive friction. Shared guidance does not lead to a fixed set of teaching activities, since constructive friction at early stages of learning stems from demands that are quite different from those found in late stages of training. Instead, shared guidance requires a dialogue with students, a monitoring of their progress, and an adapting of the teaching to their perceived needs.


교수자의 기능을 학습에 지향시키기

ORIENTING THE TEACHER’S FUNCTIONS TOWARD LEARNING



우리는 어떻게 선생님들이 배움을 이해하고 쉽게 할 수 있도록 도울 수 있을까?

How can we help teachers to understand and facilitate learning?


"무엇을 공부할 것인가?"는 주제, 책, 주제, 기타 등등, 또한 습득해야 할 지식과 기술의 본질에 관한 것이다. 간단히 말해서 학교는 물론 학생에 의해 정해진 모든 교육적 목표. 이에 따른 교사들의 활동은 발표presenting로 요약된다. 여기에는 구두 설명, 교육 목적 정의, 독서 자료 선택, 관련 시험 항목 생성, 환자와의 모델 상호작용, 보고서나 프레젠테이션 내용에 대한 피드백 등이 포함될 수 있다. 이러한 활동은 또한 적절한 학습 환경을 선택하는 것을 포함할 수 있다. 특히 이는 임상실습과 같이 맥락이 학습할 내용의 일부인 경우 그러하다. 요컨대, 콘텐츠 문제의 적절한 제공과 관련된 모든 활동을 포함한다.

“Study what?” pertains to topics, books, subject matter and the like, but also to the nature of the knowledge and skills to be acquired; in short all educational objectives that are set by the school as well as the student. The parallel teachers’ activities are summarized as presenting. This may include 

  • verbal explanations, 

  • defining of educational objectives, 

  • selection of reading materials, 

  • generation of relevant test items, 

  • modeling interactions with a patient, 

  • giving feedback on the content of papers, presentations, etc. 

These activities may also include choosing an appropriate learning environment, if that context is deliberately part of what is to be learned, such as is the case in clinical clerkships. In short, all activities that pertain to the adequate provision of content matter are included.


"왜 공부하는가"는 공부 활동의 시작과 유지와 관련된 모든 감정을 말한다. 그러므로 동기부여는 

  • 시험 기획에서부터 

  • 강의실에서 학생들에게 열정을 전달하는 것, 

  • 설득력 있는 경험을 조직하는 것, 

  • 학생들이 자기효능감을 습득하는 것을 돕는 것, 

  • 현재의 행동에 따른 미래의 결과를 생각하도록 돕는 것, 그리고 

  • 공부 활동의 중요성을 강조하는 것까지 다양할 수 있다.

“Why study?” refers to all feelings that are related to starting and maintaining study activities. Motivating, therefore, may range from 

  • planning tests, to 

  • conveying enthusiasm to students in a lecture hall, 

  • organizing compelling experiences, 

  • helping students to acquire self-efficacy, 

  • helping them think of future consequences of current behavior, and 

  • stressing the importance of study activities.


마지막으로, "어떻게 배우는가?"학습을 조절하는 것과 관련된 모든 메타인지적 활동을 포함한다. 우리는 "instructing"이라는 용어를 [학생의 메타인지적 활동과 기술을 돕는 모든 교사 기능을 포함하는 의미]로서 선택했다. 우리는 이 용어를 교육 문헌에서 일반적으로 행해지는 것보다 더 엄격한 의미로 사용하며, 현재적 요소와 동기적 요소를 제외하고 그것의 교육의 "how to" 특성만을 강조하기 위해 사용한다.21

Finally, “How to learn?” includes all relevant metacognitive activities that regulate learning. We have chosen the term instructing to include all teacher functions that help students with such metacognitive activities and techniques as 

  • devising learning tasks, 

  • explaining how to go about acquiring knowledge and skills, 

  • organizing or giving personal feedback, etc. 

We use this term in a more strict sense than is usually done in educational literature, to stress the instrumental “how to” nature of it, excluding presentational and motivational elements.21


학습-지향 교육 모델

THE LEARNING-ORIENTED TEACHING MODEL


Table 4.


인지 수준에서의 전환

Transitions at the Cognitive Level


입학하는 학생들은 교육목표가 정해지고 교육내용이 결정되기를 기대할 것이다. 그들은 교사들이 과목에 대해 설명하는 것을 듣고, 학습에 가장 관련성이 있는 문제들로 신중하게 구성된 학습 자료를 기대하고, 학습 과제를 반영하는 시험을 예상할 준비가 되어 있다. 교사들은 중요한 역할 모델로서 행동할 수도 있고 예시 역할을 할 수도 있지만, 무엇보다도 그들은 선택된 주제 내에서 무엇을 공부하고 우선순위를 정하는 데 앞장선다. 좀 더 시간이 지나게 되면, 학생들은 개인적인 학습 목표를 세우고 피드백을 받도록 요청받을 수 있다. 독서 과제는 [제안된 참고문헌]이 아니라 [학생들이 스스로 찾은 정보 출처에서 탐구해야 할 주제]로 바뀔 수 있다. 

Entering students will expect educational objectives to be set and the content of the education to be determined for them. They are prepared to listen to teachers explaining subject matter, expect study materials that are carefully composed of the most relevant issues for learning, and anticipate examinations that reflect learning assignments. Teachers may act as important role models and serve as examples, but above all they take the lead in determining what to study and prioritizing within the chosen subject matter. Later, students can be asked to generate personal learning objectives and receive feedback. Reading assignments can shift via suggested literature references to topics to be explored from sources of information the students themselves find. 


따라서 교사의 Guidance는 학생들이 검색하고 발견한 주제를 검증하는 데 초점을 맞출 수 있으며, 이는 다음과 같은 질문에 대한 피드백을 내포한다:

    • 이 문제들은 내 교육에 적합한가? 

    • 획득한 정보가 정확하고, 품질이 우수하며, 최신 상태인가? 

    • 어떤 대체 정보원이 존재하는가? 

    • 이 정보가 어떻게 나의 학습 문제를 해결하거나 나의 학습 요구를 해결하는데 적용될 수 있는가?

Guidance may then focus on validating the subject matter searched for and found by students, which implies feedback on questions such as: 

    • Are these relevant issues for my education? 

    • Is the obtained information correct, of high quality, and up to date? 

    • What alternative information sources exist? 

    • How can this information be applied to solving my learning problem or addressing my learning need?


정서 수준에서의 전환

Transitions at the Affective Level


학습을 위한 외부 동기에는 필수 출석, 필기 시험, 논문, 프레젠테이션 및 외부적으로 규정된 다른 모든 과정 요건과 같은 과제가 포함된다. 선생님들과 교육 기관들은 이 모든 것들을 제공할 수 있다. 따라서, 그러한 학교가 제시하는 요건을 넘어서지만 아직 학생들이 자신의 학습에 전적으로 책임이 있다고 생각하지 않는 학습 동기로는 다음의 것들을 사용할 수 있다.

    • 사회적 인센티브(예: PBL의 공유 과제, 동료 교육 준비), 

    • 명백한 비학문적 보상(교사와의 개인적 접촉), 

    • 미래의 practice 맥락을 통해 동기 부여적 경험에 노출

    • 열정적인 롤모델에 노출 

지속적인 학습에 대한 현실적인 필요성을 느끼도록 하기 위해, 점차적으로 고급 학생들에게 제한된 직업적 책임을 부여할 수 있다.

External motivations for learning include required attendance, written examinations, and assignments such as papers, presentations, and all other course requirements that are externally prescribed. Teachers and the educational institution can provide all these. Subsequently, motives for learning that extend beyond such school requirements but do not yet consider students fully responsible for their own learning may include the use of 

    • social incentives (e.g., shared tasks as in PBL, peer teaching arrangements), 

    • overt nonacademic rewards (personal contact with teachers), 

    • exposure to motivating experiences in contexts of future practice, and 

    • exposure to enthusiastic role models. 

Limited professional responsibilities can gradually be given to advanced students to create the feeling of the realistic need for continued learning.


교사들은 내적 동기의 출현이 장려될 수 있도록 의대 졸업 전에 학생들에게 입원환자 치료, 과학 연구 또는 기타 미래의 역할 등 진정한genuine 책임을 부여해야 한다. 이것은 자기 결정 이론과 일치한다.

Teachers should give students genuine responsibilities—inpatient care, scientific research, or any other future role—before the end of medical school so that the emergence of internal motives can be encouraged. This accords with self-determination theory,


메타인지 수준에서의 전환

Transitions at the Metacognitive Level


학생들은 초기 단계에서 다소 다양한 학습 능력과 전략을 가질 수 있다. 이는 명확한 지침을 통해 보상될 수 있다(예: 사전 준비자, 요약, 키워드 목록, 세부 학습 목표, 학습 텍스트의 레이아웃 기능, 형태별 테스트 및 기타 피드백).

Students may have rather variable learning skills and strategies in the early stages. These can be compensated by explicit guidelines (e.g., advance organizers, summaries, keyword lists, detailed learning objectives, lay-out features in study texts, formative tests and other feedback).


나중에, 공부 기술에 대한 도움은 학생들의 학습 과정의 발전과 정교함을 촉진할 수 있다. 이는 학습자가 자신의 사고 과정을 (예를 들어 개념 맵을 사용하여) 명확히 할 때 학습자에게 도움과 피드백을 제공함으로써 그들의 지식 기반 개발과 조직을 평가하고 촉진할 수 있도록 함을 의미한다. 학생들은 특히 새로운 지식과 경험이 기존 지식과 어떻게 관련되는지, 그리고 새로운 지식이 어떻게 적용될 수 있는지에 대해 성찰하는 법을 배워야 한다.

Later on, help with study skills may foster development and refinement of students’ learning process. This implies providing help and feedback to learners when making their thought processes explicit (e.g., with concept maps), so the development and organization of their knowledge base can be assessed and facilitated. Students should learn to reflect, particularly upon how new knowledge and experience relate to existing knowledge and upon how new knowledge can be applied.


졸업생은 지식의 결함을 확인할 수 있어야 하며, 이러한 결함을 시정할 수 있는 방법을 찾는 데 능숙해야 한다. 개인 지식의 결핍을 인식하기 위한 메타인지적 기술은 습득하기 어려우며, 이를 수정하기 위한 동기 부여는 오랜 기간 동안의 shared guidance을 필요로 할 수 있다. 추가 지식을 습득하기 위해 학습과 계획을 평가하는 학생의 능력은 성숙하고, 내면화되고, 메타인지적인 기술의 집합으로 간주될 수 있다. 

The graduate should be capable of identifying knowledge deficits and be proficient in finding ways to correct these. The metacognitive skill to recognize personal knowledge deficiencies is difficult to acquire, and the motivation to correct these maybe hard to generate; this may require a long period of shared guidance. The student’s ability to assess his or her learning and planning for acquiring further knowledge may be considered a collection of mature, internalized, metacognitive skills.24 



LOT모델을 실천으로 옮기기

PUTTING THE LOT MODEL INTO PRACTICE


모델의 중요한 특징은 타임라인이 정의되어 있지 않다는 것이다. 외부규제에서 내부규제으로의 전환은 수년에 걸쳐 커리큘럼 수준에서 이루어질 수 있지만, 며칠 이내에 과정 수준에서 이루어질 수 있으며, [크고 넓거나] [작거나 구체적]일 수 있는 교육의 대상에 따라 동시에 일어날 수도 있다. 동시에 학습자는 한 분야의 전문가가 될 수 있지만 다른 분야의 초보자가 될 수 있다.

An important feature of the model is that a timeline is not defined. Transition from external to internal regulation may take place at the curricular level over several years, but at the course level within days, and it may even happen simultaneously, depending on the object of education, which may be large and broad or small and specific. Learners can, at the same time, be experts in one field but novices in another field.


예시

EXAMPLES


예 1: 커리큘럼 레벨에서 학습 지향적 학습 접근법

Example 1: A Learning-Oriented Teaching Approach at the Curricular Level


예 2: 과정 레벨에서 학습 지향적 학습 접근 방식

Example 2: A Learning-Oriented Teaching Approach at the Course Level


예 3: 세션 레벨에서 학습 지향적 접근 방식

Example 3: A Learning-Oriented Approach at the Session Level


LOT 모델의 관점에서 베드사이드 교육을 고려할 수 있다. 1시간에서 2시간 동안 임상 지도 교사 또는 주치의가 학습자 및 환자와 상호 작용한다. 

    • 임상 문제를 논의한다. 

    • 의료 이력 및 신체 검사를 관찰하거나 시연한다. 

    • 임상 데이터를 종합하고 

    • 임상 환경에 기초과학 원리를 적용하고, 

    • 문제 목록, 차등 진단 및 

    • 조사나 경영을 계획하다33

Bedside teaching may be considered from the perspective of the LOT model. During these one- to two-hour sessions, a clinical tutor or attending physician–teacher interacts with learners and a patient to 

    • discuss the clinical problem, 

    • observe or demonstrate medical history-taking and physical examination, 

    • synthesize the clinical data, 

    • apply basic science principles to the clinical setting, and 

    • develop a problem list, differential diagnosis, and 

    • plan for investigation or management.33 


학습자는 서로다른 레벨(예: 하위 또는 상급 학생, 인턴, 주니어 또는 선임 레지던트 또는 동료로 구성된 병동 팀)이거나 명목상 모두 동일한 레벨(예: 3-6명의 상급 임상 학생 또는 사무원)일 수 있다. 그러나 실제로는 모든 학습자가 다양한 배경과 임상 경험을 가지고 있으며, 외부 지침(즉, 내부 지침의 가변 능력)에 대한 가변적 필요성뿐만 아니라 서로 다른 인지적 필요성, 학습 동기 및 학습 기술과 전략을 가진 동일한 환자 및 임상 문제에 접근한다.

The learners may be at different levels (e.g., a ward team consisting of junior or senior students, interns, junior or senior residents, or fellows) or may all be nominally at the same level (e.g., three to six senior clinical students or clerks). However, in reality, all learners come with varied backgrounds and clinical experiences, and they will approach the same patient and clinical problem with different cognitive needs, motives to learn, and learning skills and strategies, as well as variable needs for external guidance (i.e., variable competence at internal guidance).


외부 지침이 필요한 주니어 학생은 튜터가 환자와 주제(예: 일반적인 신체 검사를 효율적으로 수행하는 방법, 일반적인 임상 문제의 진단 및 관리에 대한 접근 방법)를 선택하고 토론을 지시하고 임상 역할 모델 역할을 하며 질문을 통해 학생의 사고 과정을 촉진하는 경우에 가장 잘 배울 수 있다. 

Junior students, in need of external guidance, may learn best if the tutor chooses the patient and topics (e.g., how to perform a general physical examination efficiently, the approach to the diagnosis and management of a common clinical problem), directs the discussion, acts as a clinical role model and prompts the student’s thinking process with questions.


반면에 시니어 레지던트는 자신의 특정한 학습 필요성(예: 이 치료가 환자의 즉각적인 치료나 향후 관행에 적합한지 여부를 알기 위해 신체 검사에서 어려운 기술을 정비하는 방법)을 가지고 세션에 올 수 있다. 그들은 의료 문헌을 검색하고 컨설턴트를 사용하여 대부분의 환자 관련 질문을 하고 대답할 수 있으며, "주니어 동료"로서 스스로 정의내린 시점에 대해서만, 아주 작은, 구체적인 지침만 필요할 수 있다. 그들은 주로 인지적, 정서적, 그리고 메타인지적 수준에서 내면화된 지도internalized guidance를 통해 배운다. BST에서 가르치는 동안 그들은 학습을 할 수 있고, 동시에 후배들을 위한 역할 모델 역할을 할 수도 있다.

Senior residents, on the other hand, may come to the session with their own specific learning needs (e.g., how to refine a difficult maneuver in the physical examination to find out whether this treatment will be appropriate on their patient) relevant to the immediate care of their patient or to their future practice. They may ask and answer most of their own patient-related questions by searching the medical literature and using consultants and may need—as “junior colleagues”—only scarce and specific guidance at self-defined moments. They learn predominantly through internalized guidance, at the cognitive, affective and metacognitive levels. During the bedside teaching session they may learn, and act as role models for junior students, at the same time.


"중간" 학습자(예: 상급생, 주니어 레지던트)는 이 두 극 사이의 어딘가에 있을 수 있으며, 학습의 세 가지 구성 요소에 관여할 때 shared guidacne가 필요할 수 있다. 교사는 인턴에게 사례 기반 질문을 작성하고 집중하도록 한 다음, 인턴이 답을 평가하고 적용하도록 도울 수 있다. 인턴은 선생님의 열성 때문에 혹은 그것이 "팀" 활동으로 인식되기 때문에 침대 머리맡에 있는 세션에 참석하기를 원할 수도 있다. 교사는 지식과 추론 능력이 명시적으로 설명될 수 있도록 인턴에게 "think out loud"를 요청할 수 있다. 인턴에게 자신의 학습 과정을 되돌아보라고 한다면, 메타인지적 요소가 도입되는 것이다.

“Intermediate” learners (e.g., senior students, junior residents) may be somewhere between these two poles, needing shared guidance when involved in the three components of learning. The teacher may ask an intern to generate and focus case-based questions, and then help the intern to appraise and apply the answers. The intern may want to attend a bedside session because of the teacher’s enthusiasm or because it is perceived as a “team” activity. The teacher may ask the intern to “think out loud” so that knowledge and reasoning skills can be addressed explicitly. If the intern is also asked to reflect on his or her learning process, a metacognitive element has been introduced.


문제를 더 혼란스럽게 만드는 것은, 모든 학습자는 학습 과정의 각 구성 요소에 대해 외부-내부 연속체의 다른 지점에 있을 수 있다는 것이다. 심지어 이는 이전의 경험이나 향후 계획에 따라서, 또는 당면한 사례 자료에 따라서도 달라질 수 있다. 시니어 학생이라면 제2형 당뇨병에 걸린 환자를 다루는 데 전문지식을 가지고 있을 수 있고, 여기에 대한 지도는 거의 필요하지 않지만, 동시에 자살성 우울증을 앓고 있는 환자들을 다루는 데는 초보자일 수도 있다. 이러한 상황이 임상튜터에게 어려울 수 있으며, 동시에 다른 학습자에게 서로 다른 지원을 제공해야 하며, 동일한 학습자의 역할이 일련의 세션 과정에서 변할 수 있다. 그러나 다른 수준의 학습 과정과 지도에 대한 다양한 필요성에 대한 인식은 이러한 복잡성을 다루는 지도자의 민감성과 능력을 향상시킬 것이다.

To confound the issue further, any single learner may be at a different point of the external–internal continuum for each of the components of the learning process (e.g., be at the shared-guidance point for the cognitive component, the external-guidance point for the metacognitive component, and the internal-guidance point for the affective component), and these may vary according to the case material at hand, depending on prior experiences or future plans. A senior student may have expertise in handling a patient with type 2 diabetes, needing little guidance here, while being a novice in dealing with patients with a suicidal depression. This can prove challenging for the clinical tutor, who must provide different support for different learners at the same time, and whose role with the same learner may change over the course of a series of sessions. Awareness of different levels of the learning process and variable needs for guidance, however, will enhance the tutor’s sensitivity and ability to deal with these complexities.



고찰

DISCUSSION


이러한 개념은 전통적인 교사-강의(표 4의 왼쪽 상단 셀에 표시됨)에서 [학생들의 지도 필요성에 민감하게 반응하는 학습 과정의 세 가지 요소 모두에 대한 기술을 가진 교사]로 이동하면서 [교사의 특성과 행동을 재정의]하는 것으로 이어질 수 있다.

These concepts may, lead to a redefinition of the teacher’s characteristics and actions, moving from the traditional teacher–lecturer (indicated in the upper-left cell of Table 4) to a teacher with skills in all three components of the learning process, who has sensitivity to students’ need for guidance.


이 모델은 학습과 교수 사이의 상호 작용에 초점을 맞추고 있지만, 어느 한 과정에 대해서도 완전한 설명을 하지는 않는다. 예를 들어 인지 처리는 정보의 선택뿐만 아니라 사실과 통찰력의 인코딩, 처리, 기억, 회수까지 포함한다.

The model focuses on the interplay between learning and teaching, but does not give a complete description of either process. For example cognitive processing includes not only selection of information, but also encoding, processing, remembering, and recall of facts and insights.


한 가지 실제적인 적용은 이 모델을 진단 기구로 사용할 수 있다는 것이다. 불만족스러운 과정이나 단일 교육 세션에서는 모델의 모든 면에서 문제를 분석하려고 할 수 있다. 학습 과정의 각 구성 요소에 적절한 과제가 있는가? 지침이 너무 적거나 너무 많이 제공되었는가?

One practical application is that the model could be used as a diagnostic instrument. In unsatisfactory courses or a single teaching session, one may try to analyze problems in all facets of the model: Are adequate challenges present in each of the components of the learning process? Is too little or too much guidance provided?


여기에 기술된 모델은 제시된 예에서 보듯이 커리큘럼과 과정 개발 및 실제 교실 교육의 프레임워크 역할을 할 수 있다. 또한, 우리가 배우고 관찰할 수 있는 발표, 동기 부여 및 교육 수준에서 교사 행동을 기술하는데 성공한다면, 이 모델은 교사 훈련과 교사 평가에 사용하는 것이 도움이 될 수 있다. 학생들이 필요로 하는 지도의 양(즉, 원하는 건설적 마찰량)을 측정할 수 있다면, 외부-내부 지도의 차원을 현실화할 수 있을 것이다.

The model described here could serve as a framework for curriculum and course development and actual classroom teaching as shown in the examples presented. In addition, teacher training and teacher evaluation may profit from the use of this model, if we succeed in describing teacher behaviors at presentational, motivational, and instructional levels that can be learned and observed. If we can measure the amount of guidance students need (i.e., the desired amount of constructive friction), it might be possible to actualize the dimension of external–internal guidance.





 2004 Mar;79(3):219-28.

Orienting teaching toward the learning process.

Author information

1
School of Medical Sciences, University Medical Center Utrecht, Utrecht, The Netherlands. t.j.tencate@med.uu.nl

Abstract

Based on developments in educational psychology from the late 1980s, the authors present a model of an approach to teaching. Students' learning processes were analyzed to determine teacher functions. The learning-oriented teaching (LOT) model aims at following and guiding the learning process. The main characteristics of the model are (1) the components of learning: cognition (what to learn), affect (why learn), and metacognition (how to learn); and (2) the amount of guidance students need. If education aims at fostering one's ability to function independently in society, an important general objective should be that one learns how to fully and independently regulate his or her own learning; i.e., the ability to pursue one's professional life independently. This implies a transition from external guidance (from the teacher) through shared guidance (by the student together with the teacher) to internal guidance (by the student alone). This transition pertains not only to the cognitive component of learning (content) but also to the affective component (motives) and the metacognitive component (learning strategies). This model reflects a philosophy of internalization of the teacher's functions in a way that allows optimal independent learning after graduation. The model can be shown as a two-dimensional chart of learning components versus levels of guidance. It is further elaborated from learners' and teachers' perspectives. Examples of curriculum structure and teachers' activities are given to illustrate the model. Implications for curriculum development, course development, individual teaching moments, and educational research are discussed.

PMID:
 
14985194
 
DOI:
 
10.1097/00001888-200403000-00005


PBL의 문제: 의학교육에서 변혁적 비판적 교육을 향하여(Perspect Med Educ, 2019)

Problems of problem-based learning: Towards transformative critical pedagogy in medical education

Alice Cavanagh1,2 · Meredith Vanstone3,4 · Stacey Ritz5,6




의료 행위를 통해 건강의 불평등을 해결하라는 요구는 19세기에 비르초우가 설명한 인도적이고, 정치적, 사회적 의술에 더 가까이 의료 행위를 옮길 수 있는 가능성을 가진 전문 교육에 대한 새로운 접근법을 찾는 데 박차를 가하고 있다[1]. 이러한 노력은 제한된 성공을 달성했다. 비록 '건강 옹호'가 의료 학습자의 핵심 역량으로 인정되었지만[2–4], 교육자와 훈련자에게는 이 역할과 관련된 책임의 범위가 불명확하다[5, 6]. 비록 사회적 책임에 대한 의무는 의과대학의 공식적인 의무[7]에서 공통적이지만, 의대생들은 그들을 발생시키는 정치적 현실을 이해하지 않고 건강상의 불균형을 '인식'하도록 교육 받는다[8].

Calls to address health inequity through medical practice have spurred a search for new approaches to professional training with the potential to shift medical practice closer to the humane, political, social medicine described by Virchow in the nineteenth century [1]. These efforts have achieved limited success: although ‘health advocacy’ has been recognized as a core competency for medical learners [2–4], the scope of responsibilities attached to this role remain unclear to educators and trainees [5, 6]; although commitments to social accountability are common in medical schools’ formal mandates [7], medical students are taught to be ‘aware’ of health disparities without understanding political realities that give rise to them [8].


비판적 교육학의 관점이 의학 교육에 큰 영향을 미치지는 않았지만, 우리는 Paolo Freire의 문제 제기 교육 모델이 현대 의학 교육자들에게 많은 것을 제공할 것이라고 주장한다. 사회문화적, 정치적, 환경적 결정요인이 인간의 건강에 미치는 영향을 인정하면서, Freire의 중요한 교육학은 훈련생들을 사회정의에 봉사하는 데 그들의 비상한 전문적 힘을 사용하는데 참여하도록 하는 노력에 중추적인 개입을 나타낼 수 있다.

While perspectives from critical pedagogy have not made significant inroads in medical education, we argue that Paolo Freire’s model of problem-posing education has much to offer contemporary medical educators. Acknowledging the influence of sociocultural, political, and environmental determinants on human health, Freire’s critical pedagogy could represent a pivotal intervention in efforts to engage trainees in using their emergent professional power in service of social justice.


'억압당한 자의 페다고지' 소개

Introducing the ‘Pedagogy of the Oppressed’


처음 출판된 지 50년이 지난 지금, 프리레의 '억압당한 자의 페다기'는 현대 교육에 있어 중요한 요소로 남아 있다. Freire의 연구의 핵심에는 급진적인 명제가 있다. 교육은 그 학습자들 사이에서 '비판적 인식'을 발전시키는 '자유 실천'이 되어야 한다[9]. Freire에게 이것은 교사들이 실제로 그들의 학생들의 마음에 지식을 '예탁'하는 교육에 대한 전통적인 '은행' 접근방식을 특징짓는 규율과 암기를 거부하는 것을 의미한다. 대신, 프리어는 사회적 불평등을 유발하는 조건을 탐구하면서 학생들에게 자기 주도적인 조사에 참여하도록 요구하는 '문제제기 교육'(PPE)을 상상한다[9]. 

Fifty years after its initial publication, Freire’s Pedagogy of the Oppressed remains a vital force in contemporary education. At the heart of Freire’s work lies a radical proposition: education must become a ‘practice of freedom’ that develops ‘critical consciousness’ amongst its learners [9]. For Freire, this means rejecting the discipline and rote memorization that characterize conventional ‘banking’ approaches to education, in which teachers didactically ‘deposit’ knowledge into the minds of their students. Instead, Freire imagines a ‘problem-posing education’ (PPE) that asks students to engage in self-directed inquiry, exploring conditions responsible for social inequities [9]. 


이를 위해 Freire는 학생들과 교사들이 '세상에 의해 중재되어, 서로 가르치고' 있는 전통적인 계층 구조를 뒤집은 교실을 상상한다[9]. Freire에 따르면, 이러한 평등주의적인 재구성은 학습자들이 자신들이 지역사회로부터 고립되어 있거나, 변화를 가져올 수 없다는 인식을 없애는 데 도움이 된다. 오히려 학생들은 그들 공동체의 운명에 대한 책임을 분담하면서 스스로를 학습과 행동의 유능한 주체agent으로 보게 된다[9].

To this end, Freire imagines classrooms that upend traditional hierarchies, where students and teachers ‘teach each other, mediated by the world’ [9]. According to Freire, this egalitarian reconfiguration helps learners dispel the perception that they are isolated from their communities, or are unable to effect change; rather, students come to view themselves as capable agents of learning and action, holding a share of responsibility for the fate of their communities [9].


비록 억압받는 자의 페다기학이 수십 년 이상 학자들에게 반향을 일으켰지만, 비평가들은 오랫동안 프리어의 빈곤에 대한 집중이 나쁜 건강 결과를 초래할 수 있는 다른 유형의 억압을 모호하게 만드는 효과가 있다고 주장해왔다[10–12].

Although Pedagogy of the Oppressed has resonated with scholars over decades, critics have long argued that Freire’s focus on poverty works to obscure other types of oppression that can result in poor health outcomes [10–12].


우리는 Freire의 작품을 재해석한 것이 의학 교육자와 학습자에게 많은 것을 제공할 것이라고 주장한다: Freire가 설명한 은행 모델과 유사하게, 의학교육은 오래 전부터 '라이프스타일 선택'을 형성하는 사회적 맥락을 인정하지 않고, 환자의 건강악화에 대한 개인적 책임을 강조하는 방향으로 학생들을 암묵적으로 사회화시킨 혐의를 받아왔다[17, 18].

We contend that this reinterpretation of Freire’s work has much to offer medical educators and learners: akin to the banking model described by Freire, medical education has long been charged with tacitly socializing students to stress patients’ personal responsibility for poor health, without acknowledging the social contexts that shape their ‘lifestyle choices’ [17, 18]. 


우리는 Freire의 '압박받는 사람들에 대한 페다고지'을 연습생-의사들만큼 특권을 가진 사회 집단을 위해 구현하는 것은 그 나름의 과제를 가지고 있을 수 있다는 것을 알고 있다[19–21]. 그러나 의대생들에게 전문적인 배제의 사회적 패턴과 나쁜 건강의 구조적 원인에 대해 비판적으로 생각해 보라고 요구함으로써, 우리는 전문적 지지에 대한 학습자들의 비판적 성찰적 헌신을 촉진할 수 있는 비옥한 기회를 보게 된다.

We are aware that implementing Freire’s ‘pedagogy of the oppressed’ for a social group as privileged as trainee-physicians may have its own challenges [19–21]. However, by asking medical students to think critically about social patterns of professional exclusion and the structural causes of ill health, we see fertile opportunities for fostering learners’ critically reflexive commitments to professional advocacy.


의료 교육을 위한 문제기반 및 문제제기 교육 과정

Problem-based and problem-posing pedagogies for medical education


Freire의 문제 제기적 교육 접근은 1960년대 후반에 등장한 교육에 대한 급진적인 개입만이 아니었다. 문제 기반 학습(PBL) 모델은 1969년 맥매스터 대학에서 처음 시행되었고, 당시 의학교육을 위한 의료 혁신의 가속화에 대한 해답으로 등장했다. PBL의 지지자들은 독특하고 진행 중인 사례 연구를 중심으로 의학 학습을 구조화하는 것이 새로운 의사들이 '변화하는 개념과 새로운 지식을 따라갈 수 있도록' 준비했고, 자기 주도적인 연구와 평생 학습을 위한 기술을 개발했다고 주장했다[23]. 50년 후, 문제 기반 커리큘럼은 전 세계 의료 제공자를 위한 교육 프로그램에서 구현되었다[24–26]. 메타 분석과 체계적인 리뷰는 의료 교육의 모델로서 PBL을 많이 지원한다[22, 27, 28]. 문제 기반 교육 기관은 더 나은 비판적 사고자[30], 더 효과적인 팀 작업 [31] 및 임상 사례에서 발생하는 공중 보건 우려에 더 부합하는 훈련 내과의로 인정받았다[32].

Freire’s problem-posing approach to education was not the only radical intervention in education to emerge during the late 1960s. First implemented at McMaster University in 1969 [22], the problem-based learning (PBL) model for medical education emerged as an answer to the accelerating pace of innovation in healthcare. Advocates for PBL argued that structuring medical learning around unique, progressing case studies prepared new physicians ‘to keep up with changing concepts and new knowledge’ [23], developing skills for self-directed research and life-long learning. Fifty years later, problem-based curricula have been implemented in training programs for healthcare providers worldwide [24–26]. Meta-analyses and systematic reviews plentifully support PBL as a model for medical education [22, 27, 28], and the competence of graduates of problem-based medical programs is comparable to their conventionally trained colleagues [29]. Problem-based pedagogies have been credited with training physicians who are better critical thinkers [30], more effective working in teams [31], and more attuned to public health concerns that arise in their clinical practice [32].


PBL과 Freire의 PPE는 일시적 유사점 이상의 것을 공유한다. 두 페다고지는 [학생들의 수동성과 직위에 대한 반응성을 단념하며], [자기 주도적인 탐색이 커리큘럼의 중심]에 있다, 두 페다고지는 강의에 기초한 콘텐츠 전달을 거부하고 학습자의 탐구를 지원하는 교사로서의 역할을 다시 생각해낸다. 

PBL and Freire’s PPE share more than a passing resemblance: both pedagogies renounce student passivity and position responsive, self-directed inquiry at the heart of their curricula; both pedagogies reject lecture-based content delivery, and re-imagine the role of teachers as supportive facilitators of learners’ exploration. 


그러나 도구적 방법으로 두 가지 접근방식은 서로 달라져 의학교육에 매우 다른 의미를 갖게 된다. 아래에서는 Freire의 교육학을 의학 교육에 통합하는 것의 가치를 예시하는 문제점과 문제 기반 접근법을 비교하는 세 가지 예를 제공한다.

In instrumental ways, however, the two approaches diverge, resulting in profoundly different implications for medical education. Below, we offer three examples comparing problem-posing and problem-based approaches to medical learning that illustrate the value of incorporating Freire’s pedagogy into medical education.



예제 1: 문제 재구성

Example one: reconceptualizing problems


PBL과 PPE 사이의 차이점 중 하나는 각 모델이 각 교육학의 중심에 있는 '문제'를 어떻게 개념화하느냐에 있다[33]. PBL에서 매주, 문제는 짧은 사례의 형태로 나타나며, 제시된 환자의 이야기와 그들의 의료 기록을 간략하게 스케치한다. 이러한 사례연구는 이중의 목적에 기여하며, 학생들이 각 사례를 이해하기 위해 의학지식 학습 목표를 설정하도록 하는 동시에, 의학의 사고 관행에 맞춰 '인식적 견습'에 참여하게 한다[34]. 

One central point of divergence between PBL and PPE lies in how each model conceptualizes the ‘problems’ at the heart of each pedagogy [33]. Week-to-week in PBL, problems come in the form of short case vignettes, briefly sketching the story of a presenting patient and their medical history. These case-studies serve a dual purpose, prompting students to set medical knowledge learning objectives in order to understand each case while enlisting them into a ‘cognitive apprenticeship’ that acculturates them‘into the thinking practices of medicine’ [34]. 


이러한 틀은, 가족이나 생활 환경으로부터 대부분 추상화된, 개별 환자의 생명 의학적인 문제로서, 문제 해결사의 역할에 연습생-의사들을 캐스팅한다. PBL은 학습자와 교사 사이의 계층을 불안정하게 하지만, 전문 기관의 힘이 어떻게 임상적 접점을 형성하는지 또는 질병의 구조적 원인을 해결하기 위한 모든 요법의 의료 개입의 좁은 능력을 형성하는지에 대해서는 거의 문제 삼지 않는다[35].

This framing—of individual patients as biomedical problems, largely abstracted fromtheir family or life circumstances—by definition casts trainee-physicians in the role of problem-solvers. Although PBL destabilizes the hierarchy between learners and teachers, it does little to problematize how the power of professional authority shapes the clinical encounter or the narrow capacity of allopathic medical interventions to address structural causes of ill-health [35].


이와는 대조적으로, 의학 학습에 대한 PPE 접근방식에서 학생들은 인구 건강 불균형을 야기하는 정치적 맥락의 문제화 및 환자 치료에 대한 협력적 접근방식에 대한 학습을 담당할 것이다[33]. PPE를 통해, 치료 제공자의 역할은 '가장 책임 있는 문제 해결자'에서 통찰력과 대리력을 갖춘 다수의 situated actor 중 한 명으로 재구성될 수 있으며, 환자와의 협력 관계를 촉진할 수 있다[35].

By contrast, in a PPE approach to medical learning, students would be tasked with problematizing political contexts that give rise to population health disparities, and learning about collaborative approaches to patient care [33]. Through PPE, the role of care provider could be reconfigured from that of ‘most responsible problem-solver’ to one of multiple situated actors with insight and agency, promoting collaborative engagement with patients [35].


예를 들어, 치료에 '부적합'한 당뇨병 환자를 대상으로 한 사례 연구에서 PPE 강사는 학생들이 좋지 않은 글리카메믹 제어의 생물학적 영향 외에 규정된 치료 요법으로 환자의 사회적 맥락을 형성하는 후속 조치를 취하도록 도울 것이다[36, 37]. 자신의 직업적 정체성과 개인적 정체성이 임상적 만남을 어떻게 형성하는지 탐구하도록 요청받은 학생들은 또한 그들의 경력 동안 지속되는 성찰적 실천의 습관을 기르도록 격려받을 것이다[38]. 훈련 초기부터 의학 인식론의 기원, 누락, 함축적 의미를 비평하는 연습을 함으로써, PPE 교육을 받은 의사들은 그들 지역사회의 질병의 기원을 파악하고 정책 변화를 옹호하기 위해 잘 갖춰진 의대를 졸업할 것이다.

In a case study featuring a diabetic patient who is ‘nonadherent’ to treatment, for instance, PPE tutors would help students interrogate social contexts shaping patients’ follow-through with prescribed treatment regimens, in addition to biomedical implications of poor glycaemic control [36, 37]. Asked to explore how their professional and personal identities shape clinical encounters, students would also be encouraged to forge habits of reflexive practice that last throughout their careers [38]. Practised in critiquing the origins, omissions and implications of medical epistemology from their earliest days of training, PPE-trained physicians would graduate medical school well-equipped to identify the origins of ill-health in their communities and to advocate for policy change.


예제 2: 지식 재구성

Example two: reconceptualizing knowledge


PPE와 PBL 사이의 또 다른 긴장점은 knowledge과 kknower 사이의 관계에 있다. PBL에서 학생들은 출처를 평가할 때 증거 기반 의학(EBM)의 원리를 이용하여 자기 주도 연구[23]를 통해 기초 의학 지식을 배운다. PBL은 비판적 연구 평가와 응답성 조사를 가르치는 유용성에 대해 찬사를 받아왔으나[39], 학생들이 과학적 지식의 우월성을 뒷받침하는 객관성과 일반성의 가정에 대해 비판적으로 생각하도록 장려하지는 않는다[40].

Another point of tension between PPE and PBL lies in the relationship each understands between knowledge and the knower. In PBL, students learn foundational medical knowledge through self-directed research [23], using principles of evidence-based medicine (EBM) in their evaluation of sources [39]. Although PBL has been lauded for its utility teaching critical research appraisal and responsive inquiry [39], it does not encourage students to think critically about assumptions of objectivity and generalizability that support the supremacy of scientific knowledge over other ways of approaching health problems [40].


억압된 자의 페다지학에서 프리어는 과학이 (그 권위의 기초인) 내부 논리가 '의문의 여지없이 강력한' 것으로 가려질 때 억압의 도구가 된다고 이 비판을 presage한다[9]. EBM에서는 집계된 연구가 가치 계층에 따라 순위를 매긴 후, 연구 결과를 임상적 맥락에서 광범위한 적용 지침으로 재구성하는데, 이 과정에서 생물 의학적 '사실fact'은 그 fact가 생성된 상황에서 소외alienated된다. 과학적 지식은 sedimented 되고, 그것의 과학적 권위의 복합적인 힘을 바탕으로 점점 더 도전에 대한 저항력을 갖는다[40].

In Pedagogy of the Oppressed, Freire presages this critique, writing that science becomes an instrument of oppression when its internal logics—the basis for its authority—are obscured as ‘unquestionably powerful’ [9]. In EBM, after aggregated studies are ranked according to hierarchies of value, research findings are reformulated as guidelines for broad application in the clinical context; through this process, biomedical ‘fact’ is alienated from the circumstances in which it was generated. Scientific knowledge becomes sedimented, and increasingly resistant to challenge through the compounded force of its scientific authority [40].


성별 기반 분석을 배제한 우울증 치료에 대한 무작위 통제 실험은 이러한 현상의 한 가지 예를 제공한다. 즉, 연구는 EBM의 원칙에 따라 '고품질'으로 간주되는 방법론적 기준을 충족할 수 있는 반면, 여전히 우울증 증상학 및 치료 효과에 있어 두드러진 차이를 모호하게 한다.임상 실습에 대한 원인의 함축적 의미 [41]. 따라서 PPE에서 학생들은 (의학적 성차별, 인종 차별, 그리고 역사적으로나 현대의학 연구 관행에 고착된 능력주의 유산을 포함하여) 생물 의학 지식이 어떻게 구성되는지에 대한 사회적, 정치적 의미를 고려하는 임무를 맡게 될 것이다[42, 43].

Randomized controlled trials of treatment for depression that exclude gender-based analyses offer one example of this phenomenon at work: research can meet methodological criteria to be deemed ‘high quality’ according to the principles of EBM, while still obscuring gendered differences in depression symptomatology and treatment effects that have profound implications for clinical practice [41]. In PPE, therefore, students would be tasked with considering the social and political implications of how biomedical knowledge is constructed, including the legacies of medical sexism, racism and ableism entrenched in historical and contemporary medical research practices [42, 43]. 


생물 의학 연구를 [사회적으로 구성된 것으로서, 연구가 실제로 사용되는 동안에도 지속적인 revision의 대상이 되는 것]으로 간주하도록 훈련된다면, 문제제기적 의학 훈련PPE의 졸업생들은, 지식을 권력의 한 형태로 여기며, 그 권력을 사회 정의에 봉사하는 데 활용하는 연구 의제를 개발하기에 적합할 것이다.

Trained to consider biomedical research as socially constructed and subject to continual revision even as they use it in their practice, graduates of problem-posing medical training would be well-equipped to develop research agendas that account for knowledge as a form of power and that harness that power in service of social justice.


예제 3: 환자 재구성

Example three: reconceptualizing patients


PBL과 PPE의 마지막 차이점은, 의료의 핵심에서 환자의 삶과 건강을 이해하기 위해 건강 프레임워크의 사회적 결정요인을 사용하는 것이다. PBL에서, 건강의 사회적 결정 요인은 종종 학생들이 환자의 개인 특성이 그들의 병을 어떻게 형성하고 있는지를 고려하도록 장려한다[16, 44]. 이 분석은 학생들에게 질병의 역학 및 사회적 불평등의 위치와 어떻게 일치하는지를 고려할 수 있는 기회를 제공하지만, 이러한 대화를 고무시키기 위한 사례 설명은 실제 환자의 사회적 정체성에 대한 주목할 만한 질감과 매우 일치하지 않는다.

A final point of difference between PBL and PPE lies in the use of social determinants of health frameworks to understand the lives and ill-health of patients at the heart of medicine. In PBL, the social determinants of health are often invoked to encourage students to consider how a patient’s individual characteristics might be shaping their illness [16, 44]. Although this analysis affords students an opportunity to consider the epidemiology of illness and how it aligns with positions of social inequity, case descriptions meant to inspire these conversations fall well short of matching the appreciable texture of real patients’ social identities.


이러한 (PBL의) 접근법은 학생들이 인종 차별, 소외, 문화적으로 관련된 의료 서비스에 대한 접근성의 결여로 인해 초래되는 건강에 대한 해악을 탐구할 기회가 거의 없게 한다. 이러한 [정체성에 대한 사려깊음의 부재]는 다음과 같은 가정을 부채질한다. 즉, 건강상의 불평등은 [강압적인 정치적 강요에 의한 유동, 진화, 구축되는 것]이 아니라, 역사적 또는 생물학적으로 불가피한 것이라고 보는것이다 [44]

This approach leaves students with little opportunity to explore harms to health borne of racism, marginalization, and lack of access to culturally relevant healthcare services. The absence of thoughtfulness about these identity features fuels assumptions that embodied health inequities are trans-historical or biologically inevitable rather than fluid, evolving, and forged by oppressive politically imposed structures [44]. 


이 모든 것을 강조하는 것은 PBL 학생들이 사건을 해독해야 하는 현실이며, 이 때 질병의 사회적 맥락을 이해하는 것은 종종 의학적 학습 목표를 숙달하는 것에 밀려난다. 사회적 맥락은 '인지recognize'하거나 '인정acknowledge'할 것이 되며, 전문적 개입이나 advocacy해야 할 것이 아니게 된다.

Underscoring all this lies the reality that when PBL students are tasked with deciphering a case, understanding the social context of illness often falls second to mastering biomedical learning objectives. The social context becomes something to ‘recognize’ or ‘acknowledge’ as opposed to a site of possible professional intervention or advocacy. 


이와는 대조적으로, PPE 교실의 학생들은 그들의 직업적, 개인적 역할뿐만 아니라 사회정치적 구조도 이러한 건강 격차를 유지하는 데 어떻게 관련되어 있는지에 대해 의문을 제기하면서, 그들이 진료소에 구현되어 있는 건강의 구조적 원인을 조사하도록 장려될 것이다. 학생들은 인종, 성별, 이민 지위, 그리고 사회적 정체성의 다른 측면들이 어떻게 그들의 환자에게 영향을 미치는지에 대한 더 풍부한 이해를 개발해야 할 것이다. 궁극적으로, PPE 교실에 있는 학생들은 그들의 모든 사회 현실을 우연히 보게 되면서, 사회의 가장 아픈 사람들과의 연대 속에서, 그들의 전체 사회 현실을 우연히 보게 될 것이다.

By contrast, students in PPE classrooms would be encouraged to probe the structural causes of ill health they see embodied in the clinic, questioning how their professional and personal roles, as well socio-political structures, are implicated in upholding these disparities in health. Students would be tasked with developing a richer understanding of how lived experiences of race, gender, migration status and other facets of social identity combine to affect their patients. Ultimately, students in a PPE classroom would challenge deterministic conceptions of health, coming to see their whole social reality as contingent—a site of activist intervention, in solidarity with the sickest in society.



문제 제기 의료 교육으로 나아가다

Moving towards a problem-posing medical education


이러한 목적을 달성하기 위해 PPE의 학생들은 그들의 생물학적 지식과의 관계를 복잡하게 하고, 클리닉의 계층적 역학 관계를 재고하고, 그들의 미래의 의료 및 의학 연구의 핵심에 질병의 사회적 원인에 대한 투쟁을 다시 집중하도록 요청받을 것이다.

To achieve this end, students of PPE would be asked to complicate their relationships to biomedical knowledge, reconsider hierarchical dynamics in the clinic, and re-centre the struggle against social causes of illness at the heart of their future practices of medicine and medical research.






 2019 Feb;8(1):38-42. doi: 10.1007/s40037-018-0489-7.

Problems of problem-based learningTowards transformative critical pedagogy in medical education.

Author information

1
Health Policy PhD Program, McMaster University, Hamilton, ON, Canada. cavanaam@mcmaster.ca.
2
Michael G. DeGroote School of Medicine, McMaster University, Hamilton, ON, Canada. cavanaam@mcmaster.ca.
3
Department of Family Medicine, McMaster University, Hamilton, ON, Canada.
4
McMaster Program for Education Research, Innovation & Theory (MERIT), Hamilton, ON, Canada.
5
Department of Pathology and Molecular Medicine, McMaster University, Hamilton, ON, Canada.
6
Medical Sciences Division, Northern Ontario School of Medicine, Sudbury, ON, Canada.

Abstract

Problem-based medical education is based in a biomedical worldview that works to entrench deterministic ways of thinking about socioculturally-influenced health disparities in the minds of medical trainees. This perspective paper considers the utility of Paolo Freire's critical pedagogy as a means of redressing this issue, as it may enable medical learners to perceive and address the social sources of illness that shape their patients' lives. With an eye to advancing health equity, and educating health professionals who are responsive to marginalized and vulnerable communities, this paper considers how a problem-posing medical education could redefine physicians' relationships to knowledge, identity, and to their patients.

KEYWORDS:

Critical pedagogyProblem-based learning; Undergraduate medical education

PMID:
 
30632061
 
PMCID:
 
PMC6382617
 
DOI:
 
10.1007/s40037-018-0489-7


공손함, 체면, 피드백: 기관의 피드백 문화가 피드백 실천에 미치는 영향 (Acad Med, 2018)

About Politeness, Face, and Feedback: Exploring Resident and Faculty Perceptions of How Institutional Feedback Culture Influences Feedback Practices

Subha Ramani, MBBS, MMEd, MPH, Karen D. Könings, PhD, Karen V. Mann, PhD, Emily E. Pisarski, MSc, and Cees P.M. van der Vleuten, PhD






피드백은 학습자의 자기평가에 inform하고, 수행능력에 영향을 줄 때 가장 효과적이다.1–5 점점 더 피드백의 초점이 제공자의 피드백 제공 기술에서 벗어나 [수신자의 수용, 통합 및 행동 변화]로 바뀌고 있다.2,6,7 그러나 피드백 대화는 (제공자와 수신자 사이에 공유된 정신적 모델이 부족하면) 방어, 분노 또는 심지어 정보에 대한 거부로 이어질 수 있는 복잡한 대인 교환interpersonal exchange이다.

Feedback is most effective when it informs learners’ self-assessment and influences performance.1–5 Increasingly, the focus of feedback is shifting away from providers’ feedback-giving skills toward receivers’ acceptance, incorporation, and behavior change.2,6,7 However, feedback conversations are complex interpersonal exchanges, where the lack of a shared mental model between the provider and receiver may result in defensiveness, anger, or even rejection of the information.8–11


사회문화적 요인도 피드백을 주고 받는 방법에 영향을 미칠 수 있다. 다음이 포함된다.

  • 자존감과 자율성에 대한 위협 인식

  • 관계를 해칠까 봐 두려움

  • 출처의 신빙성에 대한 회의적 견해

  • 자체 평가와 피드백 데이터의 불일치

또한 기관의 학습 문화도 영향을 준다.

Sociocultural factors—including perceived threats to self-esteem and autonomy, fear of damaging relationships, skepticism about the credibility of the source, and incongruence of feedback data with self-assessments,11–18 as well as the institutional learning culture9,19,20—can also impact how feedback is given and received.


조직문화, 이미지 및 아이덴티티

Organizational Culture, Image, and Identity


조직문화는 그들의 인식과 행동을 지도guide하는 공동체의 신념을 말한다.22,23 셰린22는 조직의 행동에 영향을 미치는 세 가지 수준을 묘사했다.

  • 정과 믿음, 문화의 기초를 이룬다. (레벨 1)

  • 옹호되는 가치, 이는 조직의 목적과 바람직한 이미지를 강조한다 (레벨 2)

  • 으로 보이는 일상적 행동, 흔히 명시된 가치와 실제적인 필요성 사이에서 타협한다 (레벨 3) 

Organizational culture refers to the beliefs of a community, which guide their perceptions and behavior.22,23 Schein22 described three levels of culture that influence organizational behavior: 

assumptions and beliefs that form the foundation of the culture (level 1), 

espoused values that underscore the organization’s purpose and desired image (level 2), and visible day-to-day behavior, which is often a compromise between stated values and practical needs (level 3).


교육기관은 전반적인 조직문화 외에도, 직업에서 [학습자의 교육을 설계하는 방법을 뒷받침하는 공유된 믿음, 실천, 가치]로 정의되는 학습문화를 가지고 있다.

In addition to an overall organizational culture, educational institutions have a learning culture, defined by Watling and colleagues19 and Watling20 as the shared beliefs, practices, and values that underpin how the profession designs the education of its learners.


피드백에 대한 얼굴 및 연관성의 개념

The Concept of Face and Relevance to Feedback


고프만26은 [승인된 사회규범에 따라 자신의 이미지가 어떻게 구성되는지]를 설명하기 위해 "체면face"이라는 용어를 만들어냈으며, 따라서 전문직업적 행동은 다른 사람들에게 긍정적인 이미지를 투영하려는 욕구에 의해 유도될 수도 있다. 브라운과 레빈슨27은 face 개념을 확장하여 

  • 정적인 face을 자기 확인이나 자기 효능감에 대한 욕망으로, 

  • 정적인 face을 행동의 자유나 자율에 대한 욕망으로 규정하였다. 

Goffman26 coined the term “face” to describe how an image of oneself is constructed per approved social norms, hence professional behavior may be guided by the desire to project a positive image to others. Brown and Levinson27 expanded on the concept of face, defining 

  • positive face as a desire for self-affirmation or self-efficacy and 

  • negative face as a desire for freedom of action or autonomy. 

브라운과 레빈슨에 의해 묘사된 공손함 이론[대부분의 대화가 듣는 사람이나 말하는 사람에게 위협적일 수 있다고 가정]하며, 공손함을 [체면을 위협하는 행동을 완화하기 위한 시도]로 본다.

Politeness theory, as described by Brown and Levinson,27 assumes that most conversations can be face threatening to either the hearer or speaker and views politeness as an attempt to mitigate face-threatening acts.


체면 또한 개인보다는 관계적인 것으로 묘사되어 왔으며, 다른 사람들과의 관계를 통해 자신의 자아 감각이 발달한다.28 긍정적인 얼굴, 부정적인 얼굴, 그리고 체면의 관계적 개념화는 [팀과 팀 구성원이 서로 의지하여 전문적으로 성장하는] 임상 환경과 관련이 있다.    

Face has also been described as relational rather than individual, where one’s sense of self develops through relationships with others.28 Positive face, negative face, and the relational conceptualization of face are relevant to clinical settings where patient care and learning occur on teams and members of the team rely on each other to grow professionally.


긴스버그와 동료29,30은 훈련생 성과에 대한 서면 논평에서, 교수들이 모호한 언어를 사용하는 것에 대한 한 가지 설명으로 기존의 간접성의 공손성 개념을 도입한다. 그들은 ITER에 나오는 교수진 서술이 종종 모호하고, nonliteral한 언어를 포함하고 있으며, 다른 교수진들은 종종 "행간을 읽음"으로서 이 말을 해독한다고 보고한다.

Ginsburg and colleagues29,30 invoke the politeness concept of conventional indirectness as one possible explanation for vague language used by faculty in written comments about trainee performance. They report that faculty narratives on in-training evaluation reports frequently contain vague and nonliteral language, and that other faculty often decode this language by “reading between the lines.”


예를 들어, 다른 연구에서는 "좋은good"이라는 표현이 성적이 낮은 연습생을 지칭할 수 있고 "뛰어난excellent"라는 표현도 가장 우수한 수행자를 지칭하지 않을 수 있다는 것을 발견했다.31 그러나 평가에 비특정nonspecific 언어를 사용하는 것은 연습생이 평가 언어를 잘못 해석하는 결과를 초래할 수 있다.29 학습자가 단어 그대로 평가를 해석한다면, 셀프이미지는 보존하지만 진짜 메시지를 놓칠 수 있다. 반대로 행간을 읽는다면 체면이 깎일 수 있다.30

For example, another study found that “good” may refer to underperforming trainees and “excellent” may not refer to the highest performers.31 However, the use of nonspecific language in assessments could result in trainees misinterpreting the assessment language.29 If learners take the comments literally, they may miss the message and preserve their self-image; if they read between the lines, they could lose face.30


피드백을 긍정적으로 프레이밍 하는 것은 학습자의 만족도와 자기 효율을 향상시키기 때문에,  피드백 제공자는 학습자의 자존심을 위협할 수 있는 언어를 피하려고 하는 반면, 학습자는 자신의 자기 효율성(긍정적 체면) 또는 자율성(부정적 체면)을 위협하는 정보를 거부할 수 있다.26 긍정적이고 부정적인 체면의 개념은 위계적 학습 환경에서 양방향 피드백에 직접적인 위협이다. 숨겨진 코드, 위험회피, 간접성과 같은 것이 면대면 대화 중에 증폭될 수 있다.30 또한 피드백 언어는 피드백 대화에 대한 (암묵적이거나 명시적인) 기대와 상충되는 기관적 또는 전문적 문화에 의해서도 영향을 받을 수 있다.22,33

Because positive framing of feedback enhances learner satisfaction and self-efficacy,32 feedback providers may avoid language that could threaten learners’ self-esteem, while learners could reject information that threatens their self-efficacy (positive face) or autonomy (negative face).26 The concepts of positive and negative face are also direct threats to bidirectional feedback in a hierarchical learning environment. Hidden codes, hedging, and indirectness may well be amplified during face-to- face conversations.30 Further, feedback language may also be influenced by institutional or professional culture with conflicting implicit and explicit expectations for such conversations.22,33


방법

Method


구성주의 기반 이론 접근법,37–39를 사용하여, 우리는 기관 피드백 문화에 대한 그들의 견해를 탐구하기 위해 주민들과의 집중 그룹 토론, 집중 그룹 토론 및 교수들과의 개별 인터뷰를 실시했다.

Using a constructivist grounded theory approach,37–39 we conducted focus group discussions with residents, and focus group discussions and individual interviews with faculty, to explore their views on institutional feedback culture.


세팅

Setting


하버드 의과대학의 교수 계열인 브리검 및 여성병원의 내과 레지던트 프로그램은 많은 훈련 트랙에 걸쳐 약 160명의 레지던트로 구성된 대규모 도시 훈련 프로그램이다. (예: 연구, 글로벌 보건, 1차 진료, 관리 및 리더십, 의료 교육, 의료-피디아틱스).

The internal medicine residency program in the Department of Medicine at Brigham and Women’s Hospital, a teaching affiliate of Harvard Medical School, is a large urban training program consisting of approximately 160 residents across many training tracks (e.g., research, global health, primary care, management and leadership, medical education, medicine–pediatrics).


프레임워크, 참가자 및 샘플링

Framework, participants, and sampling


우리는 참여자의 서술로부터 의미를 반복적으로 분해하고 재구성하는 구성주의 기반 이론 접근법이 피드백 관행에 대한 기관 문화의 영향에 관한 이론을 개발하는 데 가장 적합하다고 믿었다.

We believed that a constructivist grounded theory approach, which iteratively deconstructs and reconstructs meaning from participant narratives, was most appropriate to develop a theory regarding the influence of institutional culture on feedback practices.40–42


세부전공 교수로 포커스 그룹을 소집하는 것은 불가능했기 때문에, 교수진이 만날 수 있는 시간에 개별 인터뷰를 실시했다(샘플 질문은 디지털 부록 1 참조. http://links.lww.com/ACADMED/A537).44,45

It was not feasible to convene focus groups with subspecialist faculty, so we conducted individual interviews at times when faculty were available to meet (for sample questions, see Supplemental Digital Appendix 1 at ).44,45


데이터 수집

Data collection


개방형 질문을 사용하여, 우리는 무엇이 피드백 문화를 구성하는지, 기관 피드백 문화(예절 개념 포함)가 피드백 품질과 영향, 피드백 탐색, 피드백에 대한 수용성 및 양방향 피드백에 어떻게 영향을 미칠 수 있는지에 대한 그들의 인식을 탐구했다.

Using open-ended questions, we explored participant perspectives on what constitutes feedback culture, their perceptions of how institutional feedback culture (including politeness concepts) might influence the quality and impact of feedback, feedback seeking, receptivity to feedback, and bidirectional feedback.


포커스 그룹 토론은 약 60분이었고 인터뷰는 약 30분이었다. 1차 조사관(S.R)이 논의를 주도했고, 연구 보조자가 그룹 상호작용을 관찰하고 현장 메모를 했다. 연구팀이 사전에 논의한 트리거 질문을 사용하여 대화를 개시했다(목록 1). 응답은 조사되었고, 토론의 내용이 연구 질문을 다루는지 확인하기 위해 더 많은 개방형 질문들이 제기되었다. 참가자들이 자발적으로 피드백 문화, 피드백 추구 및 수용성, 양방향 피드백 등 세 가지 주요 분야를 다루었다면 추가 질문은 제기되지 않았다.

Focus group discussions were approximately 60 minutes, and interviews were approximately 30 minutes. The primary investigator (S.R.) led the discussions while a research assistant observed group interactions and made field notes. Trigger questions, discussed in advance by the research team, were used to initiate conversations (List 1). Responses were probed, and further open-ended questions were posed to ensure that the content of the discussions covered the study questions. Further questions were not posed if participants spontaneously covered three main areas: feedback culture, feedback seeking and receptivity, and bidirectional feedback.


Reflexivity은 구성주의적 접근방식에 필수적이기 때문에 샘플링, 데이터 수집 및 분석에 대한 접근방식에서 연구자 역할과 가정들의 영향을 반영했다성찰2,46

Because reflexivity is essential for a constructivist approach, we reflected on the influence of researcher roles and assumptions in our approach to sampling, data collection, and analysis.42,46


그 학과 교수인 주저자는 프로그램 관리자로서 레지던트의 승진이나 졸업 결정을 하지 않으며, 동료 교수에 대한 지배적 위치에 있지도 않다. 그녀의 협력자는 비의학자로 외부 기관(C.P.M.V, K.V.M., K.D.K)의 전문가 건강 전문가와 연구원들을 포함한다. 연구팀에는 두 명의 연구 보조자(한 명은 포커스 그룹을 관찰하고 현장 메모를 했으며, 다른 한 명은 독립 데이터 분석[E.E.P]에 참여하였다.) 우리는 관찰자와 조사자 사이에 토론 후 토론을 벌였고, 질문이 공개되었고 다양한 의견의 표현이 허용되었는지 확인하기 위해 각 녹취록을 검토했다.

The lead author (S.R.), a faculty physician in the department, is neither a program director responsible for promotion or graduation decisions for residents nor in a position of power over fellow faculty. Her collaborators are nonphysicians and include expert health professions educators and researchers from outside institutions (C.P.M.V., K.V.M., K.D.K.). The team also included two research assistants (one observed the focus groups and took field notes, and the other participated in independent data analysis [E.E.P.]). We held postdiscussion debriefings between the observer and investigator and examined each transcript to ensure that questions were open-ended and allowed for the expression of a wide range of opinions.




자료 분석

Data analysis


우리는 어떤 이름도 포함하지 않고 포커스 그룹 토론과 인터뷰를 감사하고 기록했으며, 근거이론 연구에 필요한 지속적인 비교 접근법을 사용하여 동시 데이터 수집과 분석을 수행했다.37,38,40–42 트리거 질문 및 탐침을 관련 주제로서 향후 토론에 필요에 따라 수정했다. 우리의 연구 문제와 관련된 중요한 추가적인 주제를 얻지 못하고 이론적 이해를 구축하기에 적절한 정보가 있을 때 데이터 수집이 중단되었다.47 두 명의 조사관(S.R, E.E.P)이 처음 세 개의 기록을 독립적으로 검토하여 코드화했다. 

We audiotaped and transcribed focus group discussions and interviews verbatim without inclusion of any names, and performed concurrent data collection and analysis using the constant comparative approach required for grounded theory research.37,38,40–42 Trigger questions and probes were modified as needed for future discussions as relevant themes were identified. Data collection was stopped when we did not obtain significant additional themes related to our study questions and there was adequate information to construct a theoretical understanding.47 Two investigators (S.R., E.E.P.) independently reviewed and coded the first three transcripts. 


이 세 개의 녹취록은 다른 수사관들 중 한 명(C.P.M.V., K.V.M., K.D.K)에 의해서도 코드화되었다. 그들의 코드를 비교하고 논의함으로써, 팀은 나머지 데이터 세트의 코딩 시스템에 대한 합의를 이루었는데, 이 시스템은 NVivo 10 Pro for Windows(QSR International Pty)를 사용하여 분석 및 구성되었다. S.R.과 E.E.P. 현장 노트와 조사관 관찰은 유사한 전략을 사용하여 코드화되었고 주제 생성에 기여하였다. 이러한 해석은 member checking으로 confirm하고 참여자의 quote로 backed되었다. 

These three transcripts were also coded by one of the other investigators (C.P.M.V., K.V.M., K.D.K.). By comparing and discussing their codes, the team reached consensus on a coding system for the rest of the dataset, which was analyzed and organized using NVivo 10 Pro software for Windows (QSR International Pty. Ltd., Melbourne, Australia) by S.R. and E.E.P. Field notes and investigator observations were coded using a similar strategy and contributed to the generation of themes; these interpretations were confirmed by member checking and backed by participant quotes. 


1단계 코딩 중에 특정 이슈를 참조하는 각 데이터 유닛에 체내(참가자의 단어 사용) 코드가 할당되었다. 2단계 코딩 동안 주요 주제 영역별로 코드를 그룹화한 코딩 카테고리를 설정하여 코드 수를 줄였다. 그런 다음 우리는 사전 가설보다는 참가자 서술어에 기반을 둔 주요 주제를 식별하기 위해 주제 분석을 수행했다.48 식별된 테마를 전체 연구팀과 논의했고 추가적인 포커스 그룹 토론 및 인터뷰 질문을 통해 후속 데이터 수집에 영향을 미쳤다. 

During first-level coding, each data unit referring to a specific issue was assigned an in vivo (using participants’ words) code. During second-level coding, the number of codes was reduced by establishing coding categories which grouped codes by major subject areas. We then performed thematic analysis to identify major themes grounded in participant narratives rather than through a priori hypotheses.48 Identified themes were discussed with the entire research team and influenced subsequent data collection through additional focus group discussion and interview questions. 


일단 완전한 데이터 세트를 코드화한 후, 우리는 범주적 수준(의견과 신념 탐색)에서 개념적 수준(범주 간 관계 탐색 및 의미 이해)으로 이동했다.41,42 코딩과 주제 생성에 있어서의 의견 불일치 또는 불일치는 연구팀 회의에서 합의로 해결되었다. 마지막으로, 우리는 예의 이론26과 조직 문화의 렌즈를 통해 우리의 분석을 고려했다,22,23 우리의 발견이 어떻게 이러한 구성들과 일치하는지 또는 어떻게 도전하는지 탐구한다.

Once we coded the complete dataset, we moved from the categorical level (exploring opinions and beliefs) to the conceptual level (exploring relationships between categories and understanding meaning).41,42 Ambiguities or disagreements in coding and generation of themes were resolved by consensus at research team meetings. Finally, we considered our analysis through the lens of politeness theory26 and organizational culture,22,23 exploring how our findings aligned with or challenged these constructs.


결과

Results


초청 대상자 중 레지던트 29명과 일반내과 교수 22명이 포커스 그룹 토론에 참여했고, 8명의 세부전공 교수들이 인터뷰에 참여했다(표 1).

Of those we invited, 29 residents and 22 general medicine faculty participated in focus group discussions, and 8 subspecialty faculty participated in interviews (Table 1).


우리는 보다 광범위한 문화적 맥락 안에 위치한 세 가지 핵심 테마를 발견했다. 즉, 

  • 성장의 문화를 촉진하기 위한 건설적인 피드백을 정상화하는 것, 

  • 피드백을 추구하기 위해 정신적 블록을 극복하는 것

  • 양방향 피드백을 방해하는 계층적 문화.

We discovered three key themes situated within the broader cultural context: 

  • normalizing constructive feedback to promote a culture of growth, 

  • overcoming the mental block to feedback seeking, and 

  • hierarchical culture impeding bidirectional feedback.


이러한 주제와 각 주제에 대해 보고된 장벽 및 촉진자는 그림 1에 나타나 있다.

These themes as well as reported barriers and facilitators for each theme are depicted in Figure 1.





문화적 배경: 제도적 피드백 문화

Cultural context: The institutional feedback culture


많은 참여자들은 피드백 문화를 [정기적인 양방향 피드백 대화를 촉진하는 명확한 기관의 기대를 전달하는 문화]로 정의했다.

Many participants defined a feedback culture as one that communicated clear institutional expectations promoting regular two-way feedback conversations.


교직원과 레지던트 모두 피드백 문화의 정의에서 양방향적인 측면을 강조했다.

both faculty and residents emphasized the bidirectional aspect in their definition of feedback culture.


예의 문화.

Culture of politeness.


대부분의 레지던트와 교직원들 사이에서는 그 기관이 레지던트들의 자존심에 잠재적으로 위협적일 수 있는 피드백 언어를 암묵적으로 단념시키는 강한 예의 문화를 가지고 있다는 것에 대한 일반적인 합의가 있었다. 교직원은 레지던트를 위한 [따뜻하고 안락한 고치]라고 했고, 레지던트는 '가족 같은 분위기'라고 표현했다.

There was general agreement among most residents and faculty that the institution had a strong culture of politeness that implicitly discouraged feedback language that might be potentially threatening to residents’ self-esteem. It was described by faculty as a “warm and fuzzy cocoon” for residents and by residents as a “family- like atmosphere.”


임상 교육 환경과 대조적으로, 몇몇 교수들은 연구 훈련생들에게 건설적인 피드백을 제공하는 것이 덜 어렵다고 말했다. 임상 연수생에 대한 피드백은 의사소통 기술이나 전문성과 같은 복잡한 역량과 관련되기 때문에, 이러한 역량에 대한 건설적인 피드백을 제공하는 것은 이러한 수련생들에게 더 위협적일 수 있고, 연구 연수생들에 대한 피드백보다 받아들이기가 더 어려울 수 있다고 느꼈다.

In contrast to the clinical teaching environment, a few faculty stated that it was less challenging to provide constructive feedback to research trainees. It was felt that because feedback to clinical trainees involves complex competencies, such as communication skills and professionalism, giving constructive feedback on these skills could be more threatening for these trainees and harder for them to accept than is feedback for research trainees.


[의미 있고 건설적인 피드백을 제공하고자 하는 욕구]와 [대인관계를 해칠 염려] 사이의 긴장이 자주 언급되었다.

The tension between the desire to provide meaningful, constructive feedback and the fear of damaging interpersonal relationships was alluded to frequently.


탁월함의 문화

Culture of excellence.


그 문화는 또한 그 기관의 뛰어난 학문적 명성과 그 훈련생들의 혈통 때문에 건설적인 피드백이 억제되는 문화로 묘사되었다.

The culture was also described as one where the institution’s outstanding academic reputation and the pedigree of its trainees inhibited constructive feedback.


교수진은 레지던트들의 자기효능을 위협할 수 있는 건설적인 발언을 할 경우 비난받을take to task 수 있다는 우려를 나타냈다.

Faculty expressed concern about being taken to task if they provided constructive comments that could be threatening to residents’ self-efficacy.


교수진은 또 유명 기관 레지던트들이 임포스터 콤플렉스에 시달릴 수 있고 부정적 발언이 가뜩이나 취약한 자존심을 위협할 수 있다고 지적했다.

Faculty also alluded to the possibility that residents at prestigious institutions may suffer from an imposter complex and that “negative” comments could threaten their already fragile self-esteem.


교직원들은 레지던트의 "pedigree"와 [탁월성에 대한 의심의 여지가 없다는 가정]이 건설적인 피드백을 제공하는 데 상당한 장벽으로 간주했다.

The “pedigree” of residents and unquestioned assumptions of their excellence were viewed by faculty as significant barriers to providing constructive feedback.


주제 1: 성장문화를 촉진하기 위한 건설적 피드백의 정상화

Theme 1: Normalizing constructive feedback to promote a culture of growth


기관 피드백 문화는 건설적인 피드백의 장벽으로 주목되었지만, 레지던트들과 교수진 참여자들은 부정적인 암시가 없는 대화를 하는 것이 중요하다고 생각했다.

Though the institutional feedback culture was noted to be a barrier to constructive feedback, both residents and faculty participants felt that it was important to have those conversations without negative connotations.


레지던트 훈련 중에 흔히 볼 수 있는 짧은 업무 관계는 건설적이고 목표 지향적인 피드백 대화의 주요 장애물로 간주되었다. 교수진은 또 [종단적 관계가 없는 레지던트의 성장]에 대해서는 오너십이 떨어진다고 지적했다.

Short working relationships, common during residency training, were viewed as a major obstacle to constructive, goal- directed feedback conversations. Faculty also indicated that they felt a decreased sense of ownership for a resident’s growth without a longitudinal relationship.


직접적인 성과관찰이 의미 있는 피드백에 기여할 수 있는 정도는 많은 레지던트와 교직원을 놀라게 하는 것 같았다.

The degree to which direct performance observation could contribute to meaningful feedback seemed to surprise many residents and faculty.


그러나 일부 레지던트들은 교수진 관찰자의 존재를 침해적이고 자율성에 대한 잠재적 위협으로 보았다.

However, a few residents saw the presence of a faculty observer as intrusive and a potential threat to autonomy.


건설적인 피드백 교환과 전반적인 성장 문화를 촉진하기 위한 제안에는

    • 언어에 대한 주의와 피드백 톤, 

    • 종적 관계, 

    • 자율성을 위협하지 않고 직접적인 성과 관찰, 

    • 목표 지향 피드백, 

    • 개선을 위한 조치 계획 및 

    • 건설적인 피드백 교환을 정상화

Suggestions for promoting constructive feedback exchanges and an overall culture of growth included 

    • attention to language and tone of feedback, 

    • longitudinal relationships, 

    • direct performance observation without threatening autonomy, 

    • goal-directed feedback, 

    • action plans for improvement, and 

    • normalizing constructive feedback exchanges.


주제 2: 피드백을 구하는 데 있어 정신적 블록 극복

Theme 2: Overcoming the mental block to feedback seeking


적극적으로 피드백을 구하는 것은 전반적으로 비교적 드문 일이었다. PGY-1 레지던트들은 정신적으로 피드백을 구할 준비가 되어 있지 않았는데, 그들은 일반적으로 피드백을 비판과 동일시했다. 이는 그들이 정신없는 임상 약속에 압도되었기 때문이다.

Active feedback seeking was a relatively rare occurrence overall. PGY-1 residents were mentally unprepared to seek feedback, which they generally equated to criticism, because they were overwhelmed by hectic clinical commitments.


시니어 레지던트일수록 이런 정서에 공감했다. 그들 역시 약간의 두려움을 가지고 피드백 대화에 접근했기 때문이다.

The more senior residents empathized with these sentiments as they too approached feedback conversations with some trepidation.


그러나 PGY-2와 PGY-3 레지던트들도 인턴십 기간 동안 피드백을 구하는 데 좀 더 적극적으로 임했어야 했다고 생각했다. 아이러니하게도, 그들의 기술과 역량에 관한 불확실성은 특히 PGY-3에 접근하여 독립적 실천practice을 향해 나아가면서 증가하는 것처럼 보였고, 따라서 그들은 그들의 강점과 개선될 수 있는 분야를 평가하기 위해 더 자주 피드백을 구했다.

However, PGY-2 and PGY-3 residents also wished that they had been more proactive at seeking feedback during internship. Ironically, uncertainty regarding their skills and competence seemed to rise especially as they approached PGY-3 and were heading toward independent practice, and thus, they sought feedback more frequently to assess their strengths and areas in which they could improve.


몇몇 교직원들은 학습 목표와 관련된 구체적인 피드백을 제공할 수 있다고 생각했기 때문에 레지던트가 피드백을 seeking하면 환영했다.

Several faculty welcomed feedback seeking from residents as they felt it allowed them to provide specific feedback related to learning goals.


레지던트들은 직접적인 수행 관찰목표 지향 피드백이 피드백의 신뢰성과 수용성을 높일 것이라고 지적했다.

Residents indicated that direct performance observation and goal- directed feedback would enhance the credibility and acceptability of feedback.


피드백 훈련에는 피드백을 구하고 받기 위한 전략과 피드백을 제공하는 기술에만 집중하기보다는 수용성을 높이기 위한 프레임 피드백이 포함될 것이 권고되었다.

It was recommended that feedback training include strategies for seeking and receiving feedback, and framing feedback to enhance receptivity rather than focusing solely on the techniques of providing feedback.


PGY-1 주민들은 건설적인 피드백을 받을 준비가 되어 있지 않은 것 같았다. 참가자들에 따르면, 성과 관련 피드백을 수행할 인지 및 감정적 공간을 개선하는데는 다음이 도움될 것이다.

    • 명백한 제도적 기대, 

    • 피드백 탐색 및 수신에 대한 교육, 

    • 성찰 전용 시간, 

    • 피드백을 구하고 받는 교수진 모델링, 

    • 성과 개선 계획을 이행할 기회

PGY-1 residents seemed mentally unprepared to seek and receive constructive feedback. According to participants, this lack of cognitive and emotional space to engage in performance-related feedback could benefit from 

    • explicit institutional expectations, 

    • training in feedback seeking and receiving, 

    • dedicated time for reflection, 

    • faculty modeling seeking and receiving feedback, and 

    • opportunities to implement performance improvement plans.


주제 3: 양방향 피드백을 방해하는 계층적 문화

Theme 3: Hierarchical culture impeding bidirectional feedback


대부분의 레지던트들은 가장 nurturing한 분위기의 임상 환경조차도 근본적으로 위계적이라고 말했으며, 이는 윗사람에 대한 건설적인 피드백의 제공을 방해한다고 말했다.

Most residents stated that even the most nurturing clinical environment is fundamentally hierarchical, which impedes the provision of constructive feedback to upper levels.


레지던트들 사이에는 주치의들이 건설적인 피드백을 수용하지 않거나 그들의 행동을 기꺼이 바꾸지 않을 것이라는 인식이 공통적으로 있었다.

There was a common perception among residents that attendings would not be receptive to constructive feedback or willing to change their behaviors.


교직원들이 피드백을 구했을 때 레지던트들은 여전히 대화에 어떻게 접근해야 할지 몰랐다.

On those occasions when faculty sought feedback, residents still did not know how to approach the conversation.


교직원들이 대화를 시작했다는 전제 하에, 양방향 피드백 문화를 촉진하는 데 찬성하는 많은 레지던트들이 있었다.

There were many residents in favor of promoting a culture of bidirectional feedback provided that faculty initiated the dialogue.


주치의 의사의 성격과 그들의 팀 상호작용 방식은 양방향 피드백에 영향을 미치는 것으로 여겨졌다. 레지던트들은 또한 기관의 teaching expectation를 아는 것이 교수진에게 보다 구체적인 피드백을 제공하는 데 도움이 될 수 있다고 말했다.

The personality of attending physicians and the manner of their team interactions were believed to influence bidirectional feedback. Residents also stated that knowing institutional teaching expectations could help them to provide more specific feedback to their faculty.


반면 교수진은 피드백을 구할 때도 솔직하거나 구체적인 언급을 받지 못했다고 밝혔다.

Faculty, on the other hand, stated that even when they sought feedback, they did not get honest or specific comments.


교수진들은 레지던트들의 피드백을 얻는데 더 많은 노력을 기울이는 것 같았다. 한 교수 참여자는 자신의 한계를 인정하는 것이 계층적 장벽을 허물고 레지던트가 피드백을 제공할 수 있도록 하는 경향이 있다고 말했다.

Faculty seemed willing to engage more in seeking feedback from residents. One faculty participant stated that admitting his own limitations tended to break down hierarchical barriers and allow residents to provide feedback:


양방향 피드백이 레지던트과 교수진의 피드백 문화의 정의에 포함되었음에도 불구하고, 양방향 피드백에 대한 개방성의 정도는 다양했다. 그러나 교수진이 적극적으로 피드백을 구하고, 피드백을 계층 위로 장려하고 촉진하는 제도적 피드백 문화가 존재한다면 전반적으로 레지던트들은 기꺼이 참여하였다.

There were varying degrees of openness to bidirectional feedback despite its inclusion in the definition of a feedback culture by residents and faculty. However, overall residents were willing to engage provided that faculty actively sought feedback and there was an institutional feedback culture that encouraged and promoted feedback up the hierarchy.


고찰

Discussion


몇몇 교직원과 레지던트 참여자들은 "우수함의 문화"와 "예의 문화"라는 용어를 사용했다. 

  • 우수함의 문화란 그 기관의 뛰어난 학문적 명성과 수련자의 혈통을 말하며, 이는 건설적인 피드백을 억제할 수 있다. 

  • 예의 문화는 자존심에 부정적인 영향을 미칠 가능성이 있는 언어를 사용하는 것을 억제하는 collegial하고 supportive한 업무 환경을 말한다.

Several faculty and resident participants used the terms “culture of excellence” and “culture of politeness.” 

  • The culture of excellence refers to the outstanding academic reputation of the institution and pedigree of its trainees, which can inhibit constructive feedback. 

  • The culture of politeness refers to the collegial and supportive work environment that could prevent the use of language that has the potential to adversely affect self-esteem.


이러한 설명과 테마를 사용하여 건설적인 피드백 교환, 피드백 탐색 및 양방향 피드백에 대한 상주 및 교직원의 개방성 정도를 기반으로 그림 2에 표시된 프레임워크를 개발했다(축과 사분면에 대한 설명은 그림 범례 참조).

Using these descriptions and themes, we developed the framework shown in Figure 2, based on varying degrees of resident and faculty openness to constructive feedback exchanges, feedback seeking, and bidirectional feedback (see the figure legend for an explanation of the axes and quadrants).



우리의 틀에서, 교직원과 레지던트가 피드백을 구하는 것, 건설적인 피드백 교환, 양방향 피드백에 관여하는 것을 주저하는 하단 왼쪽 사분면의 제목은 "가정된 우수성의 문화"이다. 이러한 유형의 문화에서는 학문적 우수성과 임상적 우수성에 대한 제도적 명성으로 인해서, [그 기관의 모든 사람이 뛰어나며, 피드백 대화에서 "부정적" 진술은 허용되지 않는다는 가정]으로 이어지는 것으로 보인다. 종단적 관계의 결여와 위계적 기관문화 또한 그러한 문화에서 정직하고 의미 있는 대화의 장벽이 될 것이다. 또한, 보복에 대한 두려움은 교직원들에 대한 레지던트들의 피드백을 억제할 수 있고, 자신감의 결여사기꾼 콤플렉스는 피드백을 구하고 수용하는 것의 부족을 뒷받침할 수 있다. 이러한 요소들은 개방적인 피드백 문화를 촉진하기 위해 탐구하고 다루는데 중요하다.

In our framework, the bottom left quadrant, where faculty and residents hesitate to engage in feedback seeking, constructive feedback exchanges, and bidirectional feedback, is titled the “culture of assumed excellence.” In this type of culture, the institutional reputation for academic and clinical excellence appears to lead to assumptions that everyone is outstanding and that “negative” statements are not permitted in feedback conversations. A lack of longitudinal relationships and a hierarchical institutional culture would also be barriers to honest and meaningful dialogue in such a culture.4,9,49 Further, a fear of retaliation would inhibit feedback from residents to faculty, and an underlying lack of self-confidence or the imposter complex could underlie the lack of feedback seeking and receptivity. These factors are important to explore and address to promote an open feedback culture.


우측 하단의 사분면은 레지던트가 피드백을 구하고 양방향 피드백을 제공하지만 교직원들이 건설적이고 양방향 피드백을 참여하기를 꺼리는 것이 특징이다. 이 조합은 긍정적인 체면이나 자존심을 유지하는 데 초점을 맞추어 비특정적인 대화를 촉진할 수 있다. 긍정적인 얼굴을 유지하는 것, 다른 사람의 승인을 받고자 하는 욕망, 그리고 피드백 상호작용에서 언어 사용을 형성하는 방법은 이 연구의 결과와 매우 관련이 있다. 다른 조사자들은 또한 이런 종류의 문화에 적용될 건설적인 피드백을 교환하는 데 있어 주요한 장벽으로 피드백 수신자의 감정을 상하게 하고 업무 관계를 손상시킬 수 있다는 두려움을 보고했다. 흥미롭게도, 우리의 연구에서, 일부 교수진들은 레지던트들의 자존심에 위협적인 것으로 인식되는 피드백을 제공했다는 이유로 부서 지도부의 질책을 들었고 말했다. 이러한 보복에 대한 두려움은 정직하고 건설적인 피드백의 제공을 방해할 것이다.

The bottom right quadrant can be characterized by residents’ openness to feedback seeking and bidirectional feedback but faculty reluctance to engage in constructive and bidirectional feedback. This combination could promote nonspecific conversations with a focus on preserving positive face or self-esteem. Preserving positive face, the desire to be approved by others, and how it shapes language use in feedback interactions are very relevant to the findings of this study.26,27 Other investigators have also reported the fear of hurting feedback receivers’ feelings and damaging working relationships as major barriers to the exchange of constructive feedback, which would apply in this type of culture.4,10,50 Interestingly, in our study, some faculty stated that providing feedback perceived as threatening to residents’ self-esteem had led to admonishment from departmental leadership; this fear of retaliation would impede the provision of honest, constructive feedback.


왼쪽 상단 사분면교수진은 피드백을 구하고 양방향 피드백을 받는 것에 개방적이지만, 레지던트들은 건설적인 피드백을 구하고 받기를 꺼린다. (실제하는 것이든 인식만 그렇든) 위계적 학습 문화, 그리고 건설적인 피드백을 받기 위한 인지 및 감정적 공간의 부족은 또한 이러한 유형의 문화에서 피드백을 구하고 수용하는 것을 방해할 것이다. 레지던트가 수용적이지 않은 것으로 인식될 경우, 교수진은 레지던트의 자율성에 대한 위협(즉 부정적인 얼굴)을 피하여 대화를 시작할 것이다.26 와틀링51과 와틀링과 동료4는 의학교육에서 자율성과 자주적 실천에 대한 강조는 교수진의 [직접적인 수행 관찰]을 방해하고, [개방적 피드백 문화의 장벽] 역할을 하는 경우가 많다고 보고했다. 피드백 제공자가 자존감이나 자율성을 보존하기 위한 예의 바른 언어를 사용하여 메시지를 암호화하는 경우, 피드백 수신자는 원래의 메시지와 상반되는 의미를 해독하고 구성할 수 있다.28 긴즈버그와 동료29,30에 의해 기술된 hedging and hidden code는 그러한 문화에서 직접 피드백 대화에 매우 적용된다.

The top left quadrant is characterized by faculty openness to engage in feedback seeking and bidirectional feedback but residents’ reluctance to seek and receive constructive feedback. A hierarchical learning culture, real or perceived, and a lack of cognitive and emotional space to receive constructive feedback would also impede feedback seeking and receptivity in this type of culture. If residents are perceived as not receptive, faculty might initiate conversations that avoid any threat to resident autonomy—that is, negative face.26 Watling51 and Watling and colleagues4 reported that the emphasis on autonomy and independent practice in medical education often prevents direct observation of performance by faculty and acts as a barrier to an open feedback culture. If feedback providers encode their messages using polite language aimed at preserving self-esteem or autonomy, the feedback receiver may decode and construct a meaning that contradicts the original message.28 The hedging and hidden code described by Ginsburg and colleagues29,30 are very applicable to in-person feedback conversations in such a culture.


우리는 상단 우측 사분면에 피드백을 구하고 양방향 피드백을 얻기 위해 한 레지던트가 사용한 용어인 "성장의 문화"라는 제목을 붙였다. 이러한 행동들은 [전문적 목표, 목표 지향적 피드백, 개선을 위한 행동 계획의 토론과 교환]을 자극하는 교육적 동맹 구축의 기조를 설정한다.52 이러한 문화가 먹히려면 교직원들과 레지던트들 모두가 스스로를 취약하게 만들고, 그들의 약점들에 대한 토론에 개방적이어야 하며, 이를 통해 그들은 전문직업적 성장을 포용할 수 있다.

We titled the top right quadrant the “culture of growth,” a term used by one resident participant, to indicate openness to feedback seeking and bidirectional feedback on both sides. These behaviors set the tone for educational alliance building, which stimulates discussion and exchange of professional goals, goal-directed feedback, and action plans for improvement.52 For this culture to work, both faculty and residents need to make themselves vulnerable and open to discussion of their weaknesses so that they can embrace professional growth.


기존 피드백 문화에 대한 거주자 및 교직원 인식에 대한 Schein의 수준 적용 

  • 근원적 가정은 평판에 존재하는 예의와 기관적 자긍심을 포함할 수 있다(레벨 1)

  • 제시된 가치는 정기적인 피드백과 전문성 성장에 대한 서면 기대일 수 있다(레벨 2) 그러나, 

  • 보다 눈에 띄는 일상 행동은 체면을 지키고자 하는 욕구가 지배할 수 있다(레벨 3). 

Applying Schein’s22 levels of culture to resident and faculty perceptions of the existing feedback culture, 

  • underlying assumptions may involve politeness and institutional pride in its reputation (level 1), and 

  • espoused values may be the written expectations of regular feedback and professional growth (level 2); however, 

  • the more visible day-to- day behaviors may be dominated by the desire to preserve face (level 3). 


이 개념은 "피드백에 대한 가치와 필요성은 전달되지만 의도를 행동으로 옮기는 데는 차이가 있다"는 교수 참여자의 논평에 반영된다. 세 가지 수준의 문화 사이의 불일치를 다루지 않으면, 변화 전략은 성공적이지 못할 수 있다.

This concept is reflected in a faculty participant comment, “the value and need for feedback is communicated but there is a gap in translating the intent to action.” Without addressing the mismatch between the three levels of culture,22 change strategies may not be successful.


한계와 강점

Limitations and strengths


수술과 마취과 같은 학과에서 배우는 문화는 내과와 현저하게 다를 것이다. 예를 들어, 수술에서, 핵심 학습 설정은 문화가 더 계층적이고, 감독자와 훈련자 사이의 접촉은 절차의 기간으로 제한되며, 주로 건설적일 수 있는 특정한 기술 기반 피드백에 초점을 맞춘다. 

learning cultures in departments such as surgery and anesthesiology would be markedly different from internal medicine. For example, in surgery, a key learning setting is the operating room where the culture is more hierarchical, contact between supervisors and trainees is limited to the duration of the procedure, and the focus would be on specific skills-based feedback that would likely mostly be constructive.53,54


우리는 발견된 주제가 한 명의 조사자의 가정이나 편견에 의해 유도되지 않도록 하기 위해 복수의 데이터 소스를 사용하여 조사 결과를 삼각측량하고 두 명 이상의 조사자에 의한 독립적인 데이터 분석을 통해 조사 결과의 신뢰성을 높이려고 노력해왔다. 그 결과는 피드백에 영향을 미치는 사회문화적 요인을 기술한 기존의 피드백 문헌과 일치하는 것으로 보인다.4,10,11,15,16,18,51

We have tried to enhance the credibility of the findings by using multiple data sources to triangulate the findings and independent data analysis by two or more investigators to ensure that the discovered themes were not guided by a single investigator’s assumptions or biases. The findings appear to resonate with existing feedback literature describing sociocultural factors that influence feedback.4,10,11,15,16,18,51



추가 연구를 위한 제안

Suggestions for further research


피드백 이니셔티브가 적극적으로 피드백을 추구하는 행동을 촉진하고, 건설적인 피드백 교환을 정상화하며, 후배 학습자가 상급 레벨에 피드백을 제공할 수 있도록 하고, 모든 레벨에서 제한사항에 대한 인식을 장려하더라도, 후배 연습생들은 여전히 [평가 중 총괄적 피드백]과 [전문직업적 성장을 목표로 하는 형성적 피드백]을 구별하지 못할 수 있다. 따라서, 그들은 경쟁적인 업무 환경에서 미래의 동료 관계나 직업 전망을 망칠 것을 우려하여 교수진에게 형식적인 피드백을 주는 것을 피할 수 있다.

Even if feedback initiatives actively promote feedback-seeking behaviors, normalize constructive feedback exchanges, empower junior learners to give feedback to senior levels, and encourage acknowledgment of limitations at all levels, junior trainees still may not distinguish between summative feedback during assessment and formative feedback aimed at professional development. Thus, they may avoid giving formative feedback to faculty for fear of ruining future fellowship or career prospects in competitive work settings.


Conclusions






 2018 Sep;93(9):1348-1358. doi: 10.1097/ACM.0000000000002193.

About PolitenessFace, and FeedbackExploring Resident and Faculty Perceptions of How Institutional Feedback Culture Influences Feedback Practices.

Author information

1
S. Ramani is director, Scholars in Medical Education Pathway, Internal Medicine Residency Program, Brigham and Women's Hospital, and assistant professor of medicine, Harvard Medical School, Boston, Massachusetts; ORCID: http://orcid.org/0000-0002-8360-4031. K.D. Könings is associate professor, School of Health Professions Education, Maastricht University, Maastricht, the Netherlands; ORCID: http://orcid.org/0000-0003-0063-8218. K.V. Mann was professor emeritus, Division of Medical Education, Faculty of Medicine, Dalhousie University, Halifax, Nova Scotia, Canada. E.E. Pisarski is research associate, Global Health and Social Medicine, Harvard Medical School, Boston, Massachusetts. C.P.M. van der Vleuten is director, School of Health Professions Education, and professor of education, Department of Educational Development and Research, Faculty of Health, Medicine and Life Sciences, Maastricht University, Maastricht, the Netherlands; ORCID: http://orcid.org/0000-0001-6802-3119.

Abstract

PURPOSE:

To explore resident and faculty perspectives on what constitutes feedback culture, their perceptions of how institutional feedback culture (including politeness concepts) might influence the quality and impact of feedbackfeedback seeking, receptivity, and readiness to engage in bidirectional feedback.

METHOD:

Using a constructivist grounded theory approach, five focus group discussions with internal medicine residents, three focus group discussions with general medicine faculty, and eight individual interviews with subspecialist faculty were conducted at Brigham and Women's Hospital between April and December 2016. Discussions and interviews were audiotaped and transcribed verbatim; concurrent data collection and analysis were performed using the constant comparative approach. Analysis was considered through the lens of politeness theory and organizational culture.

RESULTS:

Twenty-nine residents and twenty-two general medicine faculty participated in focus group discussions, and eight subspecialty faculty participated in interviews. The institutional feedback culture was described by participants as (1) a culture of politeness, in which language potentially damaging to residents' self-esteem was discouraged; and (2) a culture of excellence, in which the institution's outstanding reputation and pedigree of trainees inhibited constructive feedback. Three key themes situated within this broader cultural context were discovered: normalizing constructive feedback to promote a culture of growth, overcoming the mental block to feedback seeking, and hierarchical culture impeding bidirectional feedback.

CONCLUSIONS:

An institutional feedback culture of excellence and politeness may impede honest, meaningful feedback and may impact feedback seeking, receptivity, and bidirectional feedback exchanges. It is essential to understand the institutional feedback culture before it can be successfully changed.

PMID:
 
29517523
 
DOI:
 
10.1097/ACM.0000000000002193


학습목표 개발에서 Informed self-assessment의 영향(Med Teach, 2017)

The impact of informed self-assessment on the development of medical students’ learning goals

Margaret Wolffa,b, Jennifer Stojanb,c, James Cranfordd, Laurie Whitmane, Stacie Bucklere, Gruppen Larryf and

Sally Santena,f




도입

Introduction


불행하게도 많은 practicing physician들이 학습에 정체되고 새로운 임상적 도전에 적응하지 못하여 시간이 지남에 따라 점점 더 큰 practice gap를 야기한다(Reger, Mylopoulos et al. 2016). 이러한 정체 경향은 다요소적이지만, 부분적으로는 [결함 있는 자기 평가(SA)][미래 학습에 대한 준비 부족]과 결합하기 때문이다(Cordes 1984; Friedman et al. 2005; Davis et al. 2006; Eva and Regehr 2008; Regehr and Mylopoulos 2008; Mylopoulos et al. 2016; Gude et al. 2017). 이는 [가장 숙련되지 않고 가장 자신감 있는 의사]가 낮은 자기 평가 능력을 가지고 있다는 발견에 의해 더욱 복잡해진다(Davis et al. 2006).

Unfortunately, many practicing physicians stagnate in their learning and fail to adapt to new clinical challenges, creating increasingly large practice gaps over time (Regehr and Mylopoulos 2008; Mylopoulos et al. 2016). This tendency to stagnate is multi-factorial, but is due in part to poor preparation for future learning paired with flawed self-assessment (SA) (Cordes 1984; Friedman et al. 2005; Davis et al. 2006; Eva and Regehr 2008; Regehr and Mylopoulos 2008; Mylopoulos et al. 2016; Gude et al. 2017). This is further compounded by the finding that physicians who are the least skilled and the most confident have a limited ability to self-assess (Davis et al. 2006).


미래 학습을 위한 준비는 "새로운 정보를 배우고, 자원을 효과적이고 혁신적으로 사용하며, 실제로 학습과 문제 해결을 위한 새로운 전략을 개발하는 능력"으로 정의된다(Mylopoulos et al. 2016).

Preparation for future learning is defined as “the capacity 

  • to learn new information, 

  • to use resources effectively and innovatively, and 

  • to invent new strategies for learning and problem solving in practice” (Mylopoulos et al. 2016).


정기적인 피드백이 없으면 의사는 [실제 약점이 있는 영역을 강점 영역으로 인지]하거나, 반대로 [이미 지식이 있는 영역에서 학습을 추구]하는 식으로 제한된 시간을 허비할 수 있다(Eva et al. 2004; Davis 등, 2006).

Without regular feedback, physicians may perceive areas of strength in areas of actual weakness or conversely spend their limited time pursuing learning opportunities in areas in which they are already knowledgeable (Eva et al. 2004; Davis et al. 2006).


실천 기반 학습 및 개선(PBLI) 역량 영역은 자기 주도 학습의 개념을 포함한다(Knowles 1975; ACGME 2011; Burke et al. 2014).

the practice-based learning and improvement (PBLI) competency domain encompasses the tenets of self-directed learning: 

  • identification of strengths and deficiencies, 

  • goal-setting, 

  • systematically analyzing performance, and 

  • incorporating formative feedback into daily practice (Knowles 1975; ACGME 2011; Burke et al. 2014).


Informed SA는 "명확하고 시기적절하며 구체적이며 건설적인" 외부 피드백을 통합하는 것이며, 이상적으로는 "신뢰되고 신뢰할 수 있는" 출처를 통해 trainee의 SA에 inform한다 (Sargeant et al. 2010).

Informed SA incorporates external feedback that is “clear, timely, specific, and constructive” and ideally is given by “trusted, credible” sources to inform a trainee’s SA (Sargeant et al. 2010).


SA가 학습 목표(LG) 생성(Eva et al. 2010; Chang et al. 2011; Bounds et al. 2013)에서 주요 역할을 한다는 것을 뒷받침할 수 있는 몇 가지 증거가 있지만, 연습생들의 외부 피드백을 어떻게 인지하고 피드백을 LG에 통합할 것인지를 결정하는 방법에 대해서는 덜 알려져 있다(Chang et al. 2011; Bounds et al. 2013).

There is some evidence to support that SA plays a predominant role in learning goal (LG) generation (Eva et al. 2010; Chang et al. 2011; Bounds et al. 2013), but less is known about how trainees’ perceive external feedback and determine when to incorporate the feedback into their LGs (Chang et al. 2011; Bounds et al. 2013).


방법

Methods


스터디 설정 및 모집단

Study setting and population


SA, 피드백, LG 개발의 근거로 표준화된 환자(SP) 시나리오를 이용한 다단계 교육 개입 연구를 실시했다.

We performed a multi-step educational intervention study using a standardized patient (SP) scenario as the basis for SA, feedback, and development of LGs.


스터디 프로토콜

Study protocol


기초적인 임상 기술 과정에 등록한 졸업생들은 금연 상담에 초점을 맞춘 formative SP 만남에 참여했다. 그림 1은 학생들의 경험을 개략적으로 보여준다. 학생들은 표준화된 체크리스트를 사용하여 SP에 의해 의사소통 기술, 동기식 인터뷰 기법 사용, 금연 요법에 대한 지식을 평가받았다.

Preclerkship students enrolled in a foundational clinical skills course participated in a formative SP encounter focused on smoking cessation counseling. Figure 1 outlines the student experience. Students were assessed on communication skills, use of motivational interviewing techniques, and knowledge of smoking cessation therapies by SPs using standardized checklists.



이 만남 이후, 학생들은 수행된 행동의 체크리스트, 전지구적 등급 척도 및 선택적 서술 부분을 포함하는 SA를 완료했다. 글로벌 등급 척도 점수는 학생들의 성적 인식을 위한 자료로 사용되었다. 그 직후 각 학생은 자신이 인터뷰한 SP를 만나 개별화된 언어 성과 피드백을 받았다. 그리고 나서 각 SP는 그들이 학생들에게 준 피드백을 기록했다.

Following this encounter, students completed a SA that included a checklist of performed behaviors, a global rating scale, and an optional narrative portion. The global rating scale score was used as the data for the student perception of performance. Immediately following this, each student met with the SP they had interviewed and received individualized verbal performance feedback. Each SP then recorded the feedback they gave the student.


다음으로, 학생들은 에바 외(2010년)에서 개정한 5개 질문 피드백 등급 양식을 사용하여 SP로부터 받은 피드백의 효과를 평가하였다. 양식에 대한 첫 4개의 질문은 학생들에게 효과적인 피드백(예: SP가 얼마나 건설적인 방식으로 긍정적인 피드백과 부정적인 피드백을 균형있게 사용했는가?) 마지막 질문은 학생들에게 SP가 피드백을 제공하는 데 있어 전반적인 효과에 대해 평가하도록 했다. 1은 "나쁘다"고, 5는 "뛰어난" 것이다. 피드백 점수의 전체적인 효과는 데이터 분석의 목적으로 사용되었다.

Next, students rated the effectiveness of the feedback received from the SP using a five-question feedback rating form adapted from Eva et al. (2010) The first four questions on the form primed the students to the qualities of effective feedback (e.g. “To what extent did the SP use a balance of positive and negative feedback in a constructive manner”?). The final question asked students to rate the overall effectiveness of the SP at providing feedback rated on a five-point Likert scale with 1 being “poor” and 5 being “excellent”. The overall effectiveness of feedback score was used for the purposes of data analysis.


그 후, 학생들은 LG 개발에 관한 서면 지침, SMART(특정, 측정 가능, 달성 가능, 현실적, 시간적)의 예시를 받고, 하나의 완성된 SMART LG를 작성하도록 지침을 받았다. (Li et al. 2010) 학생 LG의 프롬프트는 다음과 같다. "이번 SP 조우를 바탕으로, 당신의 임상 기술을 발전시킬 수 있는 SMART LG를 하나 만들어주십시오. 이 목표는 당신이 훌륭한 의사가 되기 위해 중요하다고 생각하는 것이어야 하며, 그 과정이 끝날 때까지 성취할 수 있어야 한다." 각각의 학생들은 후에 LG를 만들었다.

Following this, students were given written guidelines on LG development, examples of SMART (Specific, Measurable, Attainable, Realistic, and Timely) LG and instructions to complete one SMART LG. (Li et al. 2010) The prompt for the student LG was: “Based on this SP encounter, please create one SMART LG that will help you further your clinical skills development. This goal should be something that you consider important for you to be an excellent doctor and should be achievable by the end of the course”. Each student subsequently created a LG.


SA의 기여와 SP의 외부 피드백, 임상 기술 개발, 리콜 및 자체 생성 LG에 대한 평가 후 조사하였다.

We investigated the contributions of SA and external feedback from a SP, on the development, recall, and of clinical skills implementation self-generated LGs after assessments.


LG는 Bounds 외 (2013)에서 제안한 다음과 같은 범주로 분류되었다. 

    • (i) 학생의 SA만 해당, 

    • (ii) SP 피드백을 통해 확인된 SA, 

    • (iii) SP 피드백과 일치하지 않는 SA, 

    • (iv) SP 피드백만 또는 

    • (v) SA와 불일치하는 SP 피드백 

LGs were further categorized into the following categories proposed by Bounds et al. (2013): 

    • (i) student’s SA only, 

    • (ii) SA confirmed by SP feedback, 

    • (iii) SA in disagreement with SP feedback, 

    • (iv) SP feedback only, or 

    • (v) SP feedback in disagreement with SA. 


또한, 데이터 분석을 위해 두 개의 별도 그룹을 만들었다. 즉, SA와 관련된 총 LG와 피드백과 관련된 총 LG가 그것이다.

In addition, two separate groups were created for the purposes of data analysis: 

    • total LG associated with SA and 

    • total LG associated with feedback.


SA 그룹은 세 가지 범주로 분류되었다: 

    • SA only, 

    • 피드백에 의해 확인된 SA

    • 피드백과 불일치하는 SA. 

The SA group included three categories: SA only, SA confirmed by feedback, and SA in disagreement with feedback


피드백 그룹은 SP 피드백, 피드백을 통해 확인된 SA 피드백 및 SA와의 불일치 피드백에서만 LG로 구성되었다. 2주 후, 학생들은 LG를 상기시키고 다음 형태별 기술 평가 과정에서 배포된 서면 양식에 따라 이 목표를 달성하기 위해 취한 조치를 기술하도록 요청받았다.

The feedback group consisted of LGs from SP feedback only, from SA confirmed by feedback, and from feedback in disagreement with SA. Two weeks later, students were asked to recall their LG and describe any actions taken to achieve this goal on a written form distributed during the next formative skills assessment in their clinical skills course.


데이터 분석

Data analysis



결과

Results


LG의 출처

Source of LGs


모든 학생들이 LG를 썼다. 표 1의 결과는 LG 5개 카테고리의 참가자 비율이 같지 않다는 것을 보여준다. 가설로 세운 것처럼, 참가자들은 주로 자신의 SA를 기반으로 LG를 만들었다. SA(즉, SA만 해당, SP 피드백으로 확인된 SA, SP 피드백과 불일치한 SA)는 생성된 LG의 82.8%에 영향을 미쳤다.

All students wrote a LG. Results in Table 1 show that the percentage of participants across all five LG categories was not equal, as hypothesized, participants created LGs primarily based on their own SA with SA (i.e. SA only, SA confirmed by SP feedback, SA in disagreement with SP feedback) influencing LG generation 82.8% of the time.


학생들은 disconfirming 피드백을 받았을 때 SA를 기반으로 한 LG를 거의 만들지 않았다 (5.4%)

Students rarely generated LG based on SA when they received disconfirming feedback (5.4%);



LG생성에 대한 영향요인

Factors affecting the generation of LGs


SA는 학생 성적이나 전반적인 피드백 효과에 대한 인식에 관계없이 LG의 주요 원천으로 남아있다.

SA remained the predominant source of LGs irrespective of student performance or perception of overall effectiveness of feedback


그러나 이러한 요인과 관련하여 LG의 다른 공급원들 사이에 차이점이 지적되었다. 첫째, LG의 출처와 학생의 수행능력 사이에는 통계적으로 중요한 연관성이 있었다.

There were, however, differences noted amongst the other sources of LGs in association with these factors. First, there was a statistically significant association between student performance and source of LG,


[평균 성적(55/111, 49.5%)이나 높은 성적(12/25, 48%)보다] "성적이 저조한" 학생은 LG(10/32, 31.3%)에 피드백을 통합할 가능성이 적었다. 또 [평균(1.8%)이나 성적이 높은(0%) 학생보다] 성적이 좋지 않은 학생은 SP 피드백(21.9%)으로 disconfirmed된 SA를 기반으로 LG를 만들 가능성이 높았다.

Students who performed poorly were less likely to incorporate feedback into their LG (10/32, 31.3%) than students with average performance (55/111, 49.5%) or high performance (12/25, 48%). In addition, students who performed poorly were more likely to make LG based on SA that was disconfirmed by SP feedback (21.9%) than students with average (1.8%) or high (0%) performance.


둘째, [학생들의 수행능력에 대한 전반적 인식]과 [LG의 출처] 사이에 통계적으로 중요한 연관성이 있었다.

Second, there was a statistically significant association between student global perception of performance and source of LG,


[평균(35/71%, 49.3%), 우수(1/7, 14.3%)로 인식한 학생에 비해] 성적이 나쁘다고 "느낀" 학생은 LG(6/8, 75%)에 피드백을 통합할 가능성이 높았다.

Students who perceived they had performed poorly were more likely to incorporate feedback into their LG (6/8, 75%) compared to those that perceived their performance as average (35/71, 49.3%) or excellent (1/ 7, 14.3%).


셋째, 전반적인 피드백 효과와 LG의 출처 사이의 연관성은 통계적으로 유의하지 않았으며, 이는 피드백 효과가 LG의 출처에 영향을 미치지 않았음을 보여준다. 이는 표준화된 성격을 고려할 때 피드백의 variability가 매우 낮음을 반영할 수 있다.

Third, the association between perceived overall effectiveness of feedback and source of LG was not statistically significant, indicating that effectiveness of feedback did not influence source of LG. This may reflect the minimal variability of feedback given the standardized nature.


LG 회상 및 시행

LG recall and implementation


[SA를 기반으로 LG를 만든 학습자에 비해] SP 피드백만으로 LG를 만든 학습자는 LG를 회상할 가능성이 높았다.

Learners who created LG based solely on SP feedback (SP feedback only or SP feedback in disagreement with SA) were, compared to those who created LG based on SA, more likely to recall their LG,89.7 versus 67.6%


SP 피드백만으로 LG를 만든 학습자는 추가 변수를 제어한 결과 LG에 대한 리콜이 높았다.

After controlling for these additional variables, learners who created LG based solely on SP feedback had higher recall of their LG


SP 피드백만으로 LG를 만든 학습자가 LG를 구현할 가능성이 더 높았다.

Learners who created LG based solely on SP feedback were also more likely to implement their LG, 72.4 versus 48.9%


SP 피드백만으로 LG를 만든 학습자들은 LG를 구현할 가능성이 더 높았다.

learners who created LG based solely on SP feedback were more likely to implement their LGs


게다가, 피드백 품질은 통계적으로 유의미하게 LG전자의 구현가능성 증가와 연관되어 있다.

In addition, quality of feedback remained statistically significantly associated with higher odds of LG implementation,


고찰

Discussion


이 연구는 임상 기술 평가 후 초보 학습자가 ...

  • [(실제) 성과, 성과에 대한 전반적 인식 또는 피드백 효과에 대한 인식에 관계없이] 자신의 SA를 기반으로 LG를 가장 많이 만든다는 것을 보여주었다

  • 하지만, 그들의 LG를 피드백에 근거하는 학생들은 그들의 LG를 기억하고 행동할 가능성이 더 많았다

  • [피드백 효과에 대한 전반적 인식]은 학생들의 LG의 출처에 영향을 미치지 않았지만, 그들이 더 효과적인 피드백을 받았다고 인식하는 사람들은 그들의 LG에 대해 행동할 가능성이 더 높았다

  • 게다가, 자신들이 최고의 성적을 거뒀다고 믿는 학습자들은(즉, 높은 SA를 가지고 있었다) 그들의 LG에 대해 행동할 가능성이 더 높았다

이러한 결과는 비록 SA가 정확하지 않지만, 목표 설정과 후속 조치에서 필수적이라는 것을 시사한다.

This study demonstrated that following a clinical skills assessment, novice learners most often created LGs based on their own SAs regardless of performance, global perception of performance or perception of effectiveness of feedback. However, students who based their LGs on feedback were more likely to recall and take action on their LGs. While perception of overall effectiveness of feedback did not influence the source of students’ LGs, those who perceived they had received more effective feedback were more likely to act on their LGs. In addition, learners who believed they had performed the best (i.e. had a high SA) were more likely to act on their LGs. These findings suggest that although SAs are inaccurate, they are essential in goal setting and subsequent action.


이전 연구들은 보다 advanced 학습자들에게 LG 생성에 SA의 영향이 크다는 것을 보여주었다(Eva et al. 2010; Chang et al. 2011; Bounds et al. 2013). 바운스 외 연구진(2013년)은 표준화된 구술 보드 시나리오에 따라 응급의학 노인들에 의해 만들어진 LG의 출처를 조사했다. 우리의 연구와 일관되게, 이들 학습자들은 SA를 기반으로 LG의 대다수를 만들었고, 피드백을 통합한 LG에 대해 행동할 가능성이 더 높았다.

Prior studies have demonstrated the profound influence of SA in the generation of LGs in more advanced learners (Eva et al. 2010; Chang et al. 2011; Bounds et al. 2013). Bounds et al. (2013) examined the source of LGs made by senior emergency medicine residents following a standardized oral board scenario with feedback. Consistent with our study, these learners created the majority of LG based on SA and were more likely to act on LG that incorporated feedback.


에바 외(2010년)는 의대 2학년 때 LG 세대에게 외부 대 내부 요인이 미치는 영향을 조사했다. 이들은 LG 개발에서 SA가 차지하는 근본적인 역할도 입증했지만 LG의 구현은 평가하지 않았다.

Eva et al. (2010) investigated the influence of external versus internal factors on generation of LG in second year medical students. These authors also demonstrated the fundamental role SA plays in development of LG but did not evaluate the implementation of LG.


다른 의학 교육 연구자들은 LG의 구현에 기여하는 요소들을 탐구했다(Li et al..2010; Li et al. 2011; Lockspeiser et al. 2016). 개인적 성찰, 상충하는 요구, 환경적 부담의 어려움은 LG 달성에 있어 상당한 장벽으로 보인다. 성공적인 전략에는 학습자가 우선순위를 정하고 구체적이고, 측정 가능하고, 현실적인 LG의 개발이 포함된다(Li et al. 2010). LG 구현의 또 다른 필수 요소는 레지던트들이 LG를 개발하고 구현하도록 책임을지도록 하는 프로그램 서포트였다(Lockspeer et al. 2016). 

Other medical education researchers have explored factors that contribute to implementation of LGs (Li et al.2010; Li et al. 2011; Lockspeiser et al. 2016). Difficulty with personal reflection, competing demands, and environmental strain appear to be significant barriers in achieving LGs. Successful strategies include development of relevant LG that are prioritized by the learner and are specific, measurable, and realistic (Li et al. 2010). Another essential factor in implementing LGs was program support to help develop LG and hold residents accountable to implementation (Lockspeiser et al. 2016). 


이처럼 여러 문헌에서 점차 LG 개발의 복잡성과 후속 조치를 강조하고 있으며, LG의 실행은 어느 요인만으로는 예측할 수 없다는 것을 보여준다. 커터 외(2017년)에 의해 제안된 최근에 개발된 마스터 어댑티브 학습자 개념 체계는 이러한 연구 결과를 더 해독하는 데 도움이 될 수 있다. 이는 자기조절학습에 뿌리를 둔 4단계 과정(계획, 학습, 평가, 조정)으로, 자기주도학습자가 되기 위해 "의과대학생에게 필요한 기술과 프로세스의 육성을 위한 노력을 지도하기 위해 개발"되었다(White et al. 2014; Cutreet al. 2017)

This growing body of work highlights the complexities of LG development and subsequent action and demonstrates that implementation cannot be predicted by anyone factor alone. The recently developed Master Adaptive Learner conceptual framework proposed by Cutrer et al.(2017) may help to decipher these findings further. This is a four-phased process rooted in self-regulated learning: planning, learning, assessing, and adjusting that was “developed to guide efforts in fostering medical student development of the skills and processes necessary” to become self-directed learners (White et al. 2014; Cutreret al. 2017)


우리의 연구는 주로 임상 기술 강좌에서 진료를 받기 전의 의대생들 사이의 [계획 단계]에 초점을 맞추었다. 계획 단계는 학습 격차의 확인, 학습 기회 선택, 학습 자원의 식별 등으로 구성된다. 이 단계의 첫 번째 부분, [격차의 식별]에서 학습자는 SA를 수행한 다음 SP와의 피드백 세션을 수행했다. 그 후, 모든 학습자들은 LG를 만들었다. 그러나 LG를 구현하기 위해 행동한 학생은 일부에 불과했다. 이러한 행동action과 비행동inaction의 차이는 부분적으로 LG의 우선순위 결정과 관련한 서로 다른 결정differing decision으로 설명될 가능성이 있다.

Our study focused primarily on the planning stage amongst preclerkship medical students in a clinical skills course. The planning phase consists of identification of learning gaps, selecting an opportunity for learning, and identification of learning resources. In the first part of this phase, identification of gaps, learners performed a SA followed by a feedback session with a SP. Following this, all learners created a LG.However, only some took action to implement the LG. This difference in action and inaction is likely explained in part by differing decisions regarding prioritization of LGs.


예를 들어, 자신의 성적에 대한 전반적 인식이 가장 낮은 학생들은 실제 성과에 관계없이 목표를 달성할 가능성이 가장 낮았고, 반면 SA가 가장 높은 학생들은 목표를 달성할 가능성이 가장 높았다. 폭스 외 연구진(1989)이 기술한 바와 같이, 많은 노력을 요하는 목표는 완수될completed 가능성이 적다. 

For example, students with the lowest global perception of their performance were the least likely to follow through on their goals irrespective of actual performance while those with the highest SA were most likely to follow through on their goals. As Fox et al. (1989) described, goals requiring a large effort are less likely to be completed. 


(SA가 낮은) 학습자들은 이 과제를 극복할 수 없는 노력이 필요하다고 볼 수 있으며, 따라서 LG를 deprioritize할 수 있다.

These learners may view the task as requiring an insurmountable effort and therefore deprioritize this LG


반면, [더 큰 인지 부조화를 인지한 학생]들은 [피드백을 통합하지 않은 학습자들보다] LG에 대한 조치를 취했을 가능성이 더 높다. 개인이 인지 부조화를 깊게 느낄수록, 더 많은 동기 부여가 되어 그 학습 격차를 메우는 것을 우선시한다(Cordes 1984).

On the other hand, students who perceived a larger cognitive dissonance were also more likely to have taken action on their LGs than learners who did not incorporate feedback. The more deeply an individual feels cognitive dissonance, the more motivated an individual is to take action, thereby prioritizing filling that learning gap (Cordes 1984). 







 2018 Mar;40(3):296-301. doi: 10.1080/0142159X.2017.1406661. Epub 2017 Nov 27.

The impact of informed self-assessment on the development of medical students' learning goals.

Author information

1
a Department of Emergency Medicine , University of Michigan Medical School , Ann Arbor , MI , USA.
2
b Department of Pediatrics , University of Michigan Medical School , Ann Arbor , MI , USA.
3
c Department of Internal Medicine , University of Michigan Medical School , Ann Arbor , MI , USA.
4
d Department of Psychiatry , University of Michigan Medical School , Ann Arbor , MI , USA.
5
e Office of Medical Student Education , University of Michigan Medical School , Ann Arbor , MI , USA.
6
f Department of Learning Health Sciences , University of Michigan Medical School , Ann Arbor , MI , USA.

Abstract

PURPOSE:

This study investigates the contributions of self-assessment (SA) and external feedback on the development of learning goals (LG) and the influence on LG recall and implementation in medical students.

METHODS:

Following a standardized patient (SP) assessment, 168 pre-clinical medical students completed a SA, received SP feedback and created a LG. LG were categorized by source. Two weeks later, students recalled LG and described implementation. Chi-squared analyses were used to test the associations.

RESULTS:

SA influenced LG for 82.8% of students whereas SP feedback influenced LG for 45.9%. Students rarely generated LG based on SA when they received discordant feedback (5.4%), but sometimes incorporated feedback discordant from their SA into LG (14.9%). Students who created LG based on SP feedback were more likely to recall LG than those who created LG based on SA, 89.7 versus 67.6%, p < 0.05 and implement their LG, 72.4 versus 48.9%, χ2(1) = 5.3, p = 0.017. Students who reported receiving effective feedback were more likely to implement their LG than those reporting adequate feedback, 60.9 versus 37.9%, χ2(1) = 8.0, p = 0.01.

CONCLUSIONS:

SA is an essential part of goal setting and subsequent action. Perception of feedback plays a crucial role in LG implementation.

PMID:
 
29179635
 
DOI:
 
10.1080/0142159X.2017.1406661


피드백에 대한 성찰: 루프를 닫기(Med Teach, 2015)

Reflections on feedback: Closing the loop 

SUBHA RAMANI




최근, 나는 의학 교육의 피드백에 대해 많은 성찰을 한다. 사람들은 이것이 새로울 것이 없는 낡은 모자라고 생각할지도 모르지만, 그것은 사실과 거리가 멀다.

Lately, I have been reflecting a lot about feedback in medical education. One might think this is old hat with no new avenues for exploration, but that would be far from the truth.


의대생과 레지던트들은 교사들이 주는 피드백이 드물거나 효과적이지 않다고 보고한다(Bing-You & Trowbridge 2009; Delva et al. 2013). 피드백을 주기 위한 기법에 관한 많은 내용이 작성되었고 바로 이 주제에 대해 더 많은 교수진 개발 워크샵이 조직되고 있는데 왜 이러한 추세가 계속되는가(van de Ridder et al. 2008; Krackov 2011; Krackov & Pohl 2011).

Medical students and residents report that feedback given by their teachers is both infrequent and/or ineffective (Bing-You & Trowbridge 2009; Delva et al. 2013). Why does this trend continue when so much has been written about techniques for giving feedback and more faculty development workshops are being organized on this very topic (van de Ridder et al. 2008; Krackov 2011; Krackov & Pohl 2011).


내 초점은 피드백 루프를 탐색하는 것으로 바뀌었다. 이 루프는 피드백 대화가 계획되기 전에 이미 시작된다. 그것은 선생님과 학습자가 처음 만났을 때 시작된다.

My focus has turned to exploring the feedback loop; The loop starts well before the feedback conversation is planned. It begins when the teacher and learner meet for the first time,


학습 환경의 중요성

The importance of the learning environment


향후 피드백 교환을 위한 단계가 설정되는 도입부부터 긍정적인 학습 환경이 구축된다. 적절한 환경은 학습과 실수와 실수로부터의 학습을 conducive하고, 양 쪽의 한계를 솔직하게 인정하고, 모든 수준에서 배우려는 의지가 된다. 또한 교사들과 학습자들 사이에 신뢰를 구축한다. 신뢰는 피드백이 양방향 교환이 되도록 보장하며, 항상 전문적인 성장과 개선을 교환의 목표로 하여 장단점에 대한 논의를 포함한다(Krackov & Pohl 2011; Lombarts et al. 2014).

A positive learning environment is established at that introduction where the stage is set for future feedback exchange. A congenial environment is conducive to learning, making mistakes and learning from them, honest admission of limitations on both sides and a willingness to learn from all levels; it also establishes trust between teachers and learners. The trust ensures that feedback is a two-way exchange, includes discussion of strengths and weaknesses, always with professional growth and improvement as the goal of the exchange (Krackov & Pohl 2011; Lombarts et al. 2014).


학습 목표의 중요성

The importance of learning goals


모든 사람은 천재다. 그러나 나무에 오를 수 있는 능력으로 물고기를 판단한다면, 물고기는 자신이 바보라고 믿고 평생을 살 것이다. – 알버트 아인슈타인.

Everyone is a genius. But if you judge a fish on its ability to climb a tree, it will live its whole life believing it is stupid – Albert Einstein.


이 인용문은 우리가 의학 교육에서 학습자에게 피드백이 어떻게 제공되는지를 고려할 때 매우 적절하다. 일단 긍정적인 학습 분위기가 확립되면, 분명한 다음 단계는 학습 목표의 소통일 것이다. (창 외, 2011) 교육 리더는 로테이션의 목적과 목표를 가르치는 사람들에게 전달해야 하며, 교사는 이러한 목표를 학습자와 학습자가 그들 자신의 학습 목표를 설정하고 이를 교사에게 전달해야 하는 구체적인 행동으로서 전달해야 한다. 교사들과 학습자들은 함께, 성취한 목표와 아직 달성되지 않은 목표를 calibrate하기 위해 다양한 단계에서 그들의 성과를 교정할 수 있다.

This quote is very apt when we consider how feedback is provided to learners in medical education. Once a climate is positive learning established, the obvious next step would be communication of learning goals (Chang et al. 2011). Educational leaders need to communicate goals and objectives of the rotation or course to those who teach, teachers should communicate these goals as specific behaviours to learners and learners should establish their own learning goals and communicate these to their teachers. Together, teachers and learners can calibrate their performance at various stages to determine goals achieved and those yet to be achieved.


반성의 중요성

The importance of reflection


학생들에게 피드백을 받기 전에 [자신의 작업에 대해 생각하라]고 요청하면, 뇌에 있는 토양이 긁혀 피드백 씨앗이 정착하고 성장할 수 있는 장소가 된다 – Jan Chappuis

Asking students to think about their work before receiving feedback scratches up the soil in their brain so the feedback seeds have a place to settle in and grow – Jan Chappuis


강점과 약점에 대한 자기 성찰을 장려하는 것은 성찰적인 의사들을 훈련시키는데 필수적인 요소다(Cantillon & Sargeant 2008; Van Hell et al. 2009; Krackov & Pohl 2011; Chappuis 2014). 나는 피드백 교환의 중심에 자기 평가를 두는 펜들턴 접근방식을 좋아한다(Pendleton 1984). 나는 의학적 훈련생들이 overachiever로서, 자기자신에 대해서는 과도하게 엄한 경향이 있다는 것을 발견했다; 나는 종종 그들의 우울한 자기 평가를 반박하고 긍정적인 면을 부각시킨다. 따라서 자기 성찰로 대화를 시작하는 것은 잠재적으로 분노, 부정적인 감정, 시정 피드백에 대한 수용성의 부족을 최소화할 수 있다(Sargeant et al. 2008; Van Hell et al. 2009; Delva et al. 2013).

Encouraging self-reflection on strengths and weaknesses is an essential factor in training reflective practitioners (Cantillon & Sargeant 2008; van Hell et al. 2009; Krackov & Pohl 2011; Chappuis 2014). I like the Pendleton approach which places self-assessment at the centre of a feedback exchange (Pendleton 1984). I have found that medical trainees are overachievers and tend to be their own worst critics; frequently I end up refuting their gloomy self- appraisals expanding on and highlighting the positives. Thus, starting a conversation with self-reflection can potentially minimise anger, negative emotions and lack of receptivity to corrective feedback (Sargeant et al. 2008; van Hell et al. 2009; Delva et al. 2013).


계획의 중요성

The importance of plans


피드백이 학습 성과 측면에서 개선이 필요한 특정 장점과 분야를 열거하지 않고, 전문적 개선 계획을 기술하지 않는 경우는 아무런 소용이 없다(Boud & Molloy 2013; Watching 2014a, b). 이를 달성하기 위해 피드백은 특정한 행동, 세심한 관찰, 그리고 전문적인 발전의 동기를 부여하는 것을 목표로 하는 것이 가장 좋다. 이렇게 하면 피드백 루프가 완료된다.

Feedback serves no purpose if recipients do not list their specific strengths and areas that need improvement in terms of learning outcomes and describe professional improvement plans (Boud & Molloy 2013; Watling 2014a, b). To achieve this, feedback is best based on specific behaviours, close observation and aimed at motivating professional development. This completes the feedback loop.


그리고 마지막으로...

And finally...


피드백을 받는 사람들은 피드백 대화의 마지막에 다음 세 가지 질문에 답할 수 있어야 한다.

(1) 여기가 어디인가? – 개선을 위한 성능, 강점 및 영역의 보정

(2) 나는 어디에 있어야 하는가? – 달성해야 할 결과 - 지식, 기술 및 태도

(3) 거기까지 어떻게 가는가? – 반성, 자체 평가 및 실행 계획


Those receiving feedback, should be able to answer the following three questions at the end of a feedback conversation:

(1) Where am I? – Calibration of performance, strengths and areas for improvement

(2) Where do I need to be? – Outcomes to be achieved- knowledge, skills and attitudes

(3) How do I get there? – Reflection, self-assessment and action plan







 2016;38(2):206-7. doi: 10.3109/0142159X.2015.1044950. Epub 2015 Jun 1.

Reflections on feedbackClosing the loop.

Author information

1
a Brigham and Women's Hospital and Harvard Medical School , USA.
PMID:
 
26030380
 
DOI:
 
10.3109/0142159X.2015.1044950


성찰적 퍼포먼스 피드백의 촉진: 근거 및 이론기반 모델 R2C2 (Acad Med, 2015)

Facilitated Reflective Performance Feedback: Developing an Evidence- and Theory-Based Model That Builds Relationship, Explores Reactions and Content, and Coaches for Performance Change (R2C2)

Joan Sargeant, PhD, Jocelyn Lockyer, PhD, Karen Mann, PhD, Eric Holmboe, MD,

Ivan Silver, MD, MEd, FRCPC, Heather Armson, MD, Erik Driessen, PhD,

Tanya MacLeod, MSc, Wendy Yen, MA, Kathryn Ross, MSc, and Mary Power, MBA




피드백을 주고 받는 것은 이제 복잡한 과정으로 이해된다. 핵심요소에는 

  • 피드백 제공자와 수령자 간의 상호 작용, 

  • 수령자의 능동적 역할, 

  • 개선을 위한 수령자의 집중적인 계획이 포함된다.1–3

Giving and receiving feedback is now understood to be a complex process. Central elements include the 

  • interaction between the feedback provider and recipient, 

  • the recipient’s active role, and 

  • the recipient’s focused planning for improvement.1–3


피드백의 현재 관점은 학습자(또는 수신자), 교사(또는 제공자) 및 환경 간의 관계를 이해하기 위해 사회문화 렌즈를 사용한다.

Current perspectives of feedback use a sociocultural lens to understand the relationships among the learner (or recipient), the teacher (or provider), and the environment.


사회문화 렌즈를 사용하면 피드백 수용과 사용에 영향을 미치는 수많은 요인들이 조명된다. 이러한 요인으로는 

  • 피드백의 적시성과 구체성, 

  • 피드백의 성격이나 가치(긍정적, 부정적), 

  • 피드백 신뢰성에 대한 인식, 

  • 제공자-수령적 관계, 

  • 컨텍스트와 문화, 

  • 수령자의 개인적 목표, 

  • 수령자의 자기효능감 

...등이 있다.8–14 성과 피드백의 수령자가 항상 이를 수용하고 사용하는 것은 아니다. 그리고 중요한 것은 피드백이 수령자의 자기 인식을 혼란스럽게 할 경우 감정적인 반응을 불러일으킬 수 있다는 것이다.8–16

Using a sociocultural lens illuminates numerous factors that influence feedback acceptance and use. These factors include, 

  • feedback timeliness and specificity, 

  • the nature or valence (positive, negative) of the feedback, 

  • perceptions of feedback credibility, 

  • provider–recipient relationship, 

  • context and culture, 

  • the recipient’s personal goals, and 

  • the recipient’s self-efficacy.8–14 

Recipients of performance feedback do not always accept and use the feedback, and, importantly, the feedback can evoke emotional reactions if it disconfirms the recipient’s self-perceptions.8–16


피드백 대화의 촉진은 피드백의 수용과 사용에 긍정적인 영향을 미칠 수 있다.21,22 [피드백의 촉진]은 수신자를 적극적으로 [자신이 받은 피드백과 자신의 성과에 대한] 성찰적이고 의도적인 대화에 참여시켜서, 이 두가지를 더 잘 이해하게 만드는 것이다.

facilitation of the feedback conversation can have a positive effect on the acceptance and use of feedback.21,22 Facilitation of feedback means actively engaging recipients in a reflective, intentional conversation about their performance and the feedback they receive with the goal of increasing their understanding of both.


레지던트들은 피드백 촉진이 피드백을 명확하게 하고 이를 수용하고 사용할 수 있게 할 수 있다고 말했다. 피드백의 제공과 사용 사이에 격차가 존재하지만, 근거에 따르면, 촉진으로 피드백 사용을 향상시킬 수 있음을 시사한다.

residents reported that feedback facilitation can clarify the feedback and enable their acceptance and use of it.21–23 Although gaps exist between the provision and use of feedback, such evidence suggests that facilitation can enhance feedback use.


피드백 모델 개발: 이론적 전망과 증거

Developing a Feedback Model: Theoretical Perspectives and Evidence


세 가지 이론적 및 증거 기반 접근법

Three theoretical and evidence-based approaches


(1) 개인이 피드백 소유권, 목표 설정 및 조치 계획에 관여하는 휴머니스트 또는 사람 중심의 접근방식24 

(1) humanist or person-centered approaches which engage the individual in feedback ownership, goal setting, and action planning24; 


(2) 성과에 대한 자기 인식과 외부 피드백을 쉽게 통합할 수 있는 정보화된 자체 평가 접근법18 및 

(2) informed self-assessment approaches that facilitate integration of external feedback with self-perceptions of performance18; and 


(3) 행동 변화 과학, 피드백 사용과 변화에 대한 장벽을 평가하고 해결하기 위한 심리적 영역의 프레임워크.25

(3) the science of behavior change, a framework of psychological domains for assessing and addressing barriers to feedback use and change.25



휴머니스트 또는 사람 중심의 접근 방식

Humanist or person-centered approaches


휴머니즘과 사람 중심의 접근방식은 인지 및 행동 과학에서 출발한다. 목표는 개인의 자각과 관여를 향상시키는 것이다. 피드백을 받는 사람은 단지 정보를 받는 사람이 아니라 토론의 파트너다.24,26 촉진자는 FR이 [자신의 성과, 개인적 목표, 피드백의 통합, 개선 및 개발에 대한 계획]에 대해 받은 피드백에 대해 성찰하도록 안내한다.21,22,26 촉진자는 코칭을 사용하여 FR의 자기인식을 높이고, 자기주도성을 높인다. 촉진자는 FR의 개인적 발전과 목표 달성을 지원한다. 휴머니스트적인 기술인 코칭은 두 가지를 모두 포함한다. [변화목표를 식별하도록 촉진하는 것], [목표한 변화를 달성하기 위한 계획을 세우는 것]

Humanism and person-centered approaches draw on the cognitive and behavioral sciences; the goal is to enhance individual self-awareness and engagement. The feedback recipient is a partner in the discussion, not just a receiver of information.24,26 Facilitators guide the recipients’ reflection on the feedback they have received about their performance, their personal goals, their integration of the feedback, and their plans for improvement and development.21,22,26 Facilitators use coaching to increase the recipient’s self-awareness and self-direction; they support the recipient’s personal development and goal achievement. Coaching, a humanist technique, involves both facilitating the feedback recipient’s identification of goals for change and developing a plan to meet them.15,27–30


자체 평가 접근 방식 실행

Informed self-assessment approaches


두 번째 개념적 프레임워크인 "정보적 자체 평가"는 개인이 외부 및 내부 데이터(즉, 피드백)를 사용하여 자신의 성과를 평가할 수 있다는 것을 인정한다.14,18–20,31 외부 데이터는 비공식적인 구두 피드백에서 공식적인 직장 기반 평가에 이르는 다양한 형태를 취한다. 내부 자료에는 자신의 성과 감정 상태에 대한 자기 인식이 포함되어 있다. 외부 피드백은 정보에 입각한 자체 평가에 필수적이며, 반성은 피드백 수용에 종종 필수적이라는 것이 명백하다. 컨텍스트, 문화, 관계는 개인의 인식과 감정 상태와 함께 외부 피드백을 수용하고 사용하는 것에 모두 영향을 미친다.

The second conceptual framework, “informed self-assessment,” recognizes that individuals can use external and internal data (i.e., feedback) to generate an appraisal of their own performance.14,18–20,31 External data take many forms ranging from informal verbal feedback to formal workplace- based assessments. Internal data include self-perceptions of one’s performance and one’s emotional state. It is now evident that external feedback is essential to informed self-assessment, and reflection is frequently integral to feedback acceptance. Context, culture, and relationships, along with individual perceptions and emotional state, all influence the recipient’s acceptance and use of external feedback.


행동변화의 과학

The science of behavior change


마지막으로 행동 변화의 과학을 이해하는 것은 외부 피드백에 대응하여 개인의 변화와 개선을 촉진하고 지도하는 데 매우 중요하다. 지식, 기술, 동기부여, 자기효율성, 환경요소 및 사회적 영향을 포함한 여러 개인 및 사회적 요인들이 행동과 변화에 영향을 미친다.25,32 피드백 대화에서 이들을 체계적으로 탐구하면 성과 변화에 기여할 수 있다.

Finally, understanding the science of behavior change is critical to facilitating and coaching individual change and improvement in response to external feedback. Multiple individual and social factors influence behavior and change including knowledge, skills, motivation, self-efficacy, environmental elements, and social influences.25,32 Exploring these systematically in the feedback conversation can contribute to performance change.



방법

Method


설계.

Design


우리는 두 개의 인정된 연구 프레임워크에 의해 guide되는 반복적이고 다단계적이며 질적인 연구를 수행했다. 첫 번째는 복잡한 중재 연구를 위한 영국의료연구위원회(UK-MRC) 지침이다. 그 4단계는 

  • (1) 모델링 프로세스와 결과를 포함한 개발, 

  • (2) 타당성 시험, 

  • (3) 효과성 평가, 

  • (4) 지속 가능성이다.34

We undertook an iterative, multistage, qualitative study guided by two recognized research frameworks. The first is the UK Medical Research Council (UK-MRC) guideline for studying complex interventions. Its four stages are (1) development, including modeling process and outcomes; (2) feasibility testing; (3) evaluation of effectiveness; and (4) sustainability.34


연구를 guide하는 두 번째 관점은 "실제주의 평가"로, 개입 자체뿐만 아니라 개입이 일어나는 사회 시스템이나 맥락을 연구한다.35 현실주의 평가는 개입의 개별 요소들이 어떻게 작용하는지 그리고 그들이 관련된 맥락 안에서 어떻게 상호작용하는지를 이해하고자 한다. 이 질문은 다음과 같다. "무엇이 누구에게, 어떤 조건에서 효과가 있는가?"

The second perspective guiding the research is “realist evaluation,” which studies the social system or context in which an intervention occurs as well as the intervention itself.35 Realist evaluation seeks to understand how the individual components of an intervention work and how they interact with the people and within the context involved. It asks the question, “What works for whom and under what conditions?”


참여 인구 및 조직

Participating populations and organizations



연구 단계, 데이터 수집 및 분석

Research stages, data collection, and analysis



1단계: 모델링. 1단계에는 두 가지 하위 단계가 포함되었다. 첫째, 피드백 모델에 대한 구두 검토와 비평, 둘째, 모델을 사용하는 세션을 촉진하고, 그 다음에 설명 인터뷰가 있었다.

Stage 1: Modeling. Stage 1 involved two substeps: first, a verbal review and critique of the feedback model and, second, facilitated sessions using the model, followed by debriefing interviews.


2단계: 촉진자 준비(워크숍) 1단계에서 진행자 및 참가자의 개선 사항을 고려하고 통합한 후, 우리는 모두 20년 이상 실무에 종사하고 학생과 레지던트를 감독한 8명의 지원자를 추가로 모집했다.

Stage 2: Facilitator preparation (workshop). After considering and incorporating the refinements from the facilitators and participants in Stage 1, we recruited an additional eight volunteer physician facilitators who had all been in practice for over 20 years and had supervised students and residents.


3단계: 모델 타당성 시험(모델을 이용한 촉진된 피드백 세션 및 디브리핑 인터뷰) 우리는 최근 공식적인 성과 평가 보고서에 대한 신속한 피드백 토론에 참여하기 위해 8명의 의사 지원자를 모집했다.

Stage 3: Model feasibility testing (facilitated feedback sessions using the model and debriefing interviews). We recruited eight physician volunteers to participate in a facilitated feedback discussion about their recent formal performance assessment report.


분석. 모델과 각 단계와 효과성을 더 잘 이해하고 각 단계에 유용한 문구를 식별하기 위해 콘텐츠 및 주제 분석을 사용하여 피드백 세션의 녹취록과 브리핑 인터뷰를 분석했다.

Analysis. We analyzed the transcripts of the feedback sessions and debriefing interviews using content and thematic analysis36 to better understand the model, its phases, and their effectiveness, and to identify useful phrases for each phase.


4단계: 모델 다듬기. 모델을 다듬기 위해 전체 연구진은 정기적으로 전자우편과 텔레컨퍼런스를 통해 만나 분석 결과를 논의하고, 연구를 안내하는 이론적 관점에 비추어 결과를 질의하며, 각 단계에서 촉진자가 사용하는 유용한 문구들의 inventory를 확인하였다.

Stage 4: Model refinement. To refine the model, the full research team met regularly through e-mail and teleconference to discuss the results of the analyses, to query findings in light of the theoretical perspectives guiding the study, and to confirm the inventory of helpful phrases used by facilitators in each phase.


결과

Results


4단계는 (1) 관계 구축, (2) 반응 탐색, (3) 내용 탐색, (4) 성과 변경 코칭이다. 따라서, 우리는 그것을 R2C2 진행형 피드백 모델이라고 부른다. 표 2는 각 단계의 목표, 대표적인 촉진구 및 지침노트를 제공한다.

The four phases are 

(1) build rapport and relationship, 

(2) explore reactions, 

(3) explore content, and 

(4) coach for performance change. 

Hence, we refer to it as the R2C2 Facilitated Feedback Model. Table 2 provides the goal of each phase, representative facilitation phrases, and guiding notes.


일부 촉진자는 단계를 더 선형적으로 사용한다고 보고한 반면에, 다른 촉진자는 토론 내내 반복적이고 개방적인 방식으로 단계를 사용했다.

Whereas some facilitators reported using the phases more linearly, others used them in an iterative and open manner throughout the discussion.







1단계: 관계 구축

Phase 1: Build rapport and relationship


피드백 수령자는 [공식적으로 평가되는 것과 성과 보고서를 받는 것 둘 다] 민감하고 종종 위협적인 활동이라는 것에 일관되게 동의했다. 따라서, 피드백 대화는 사려깊음을 필요로 한다. 그들은 촉진자가 시간을 내어 

    • 수령인과의 관계를 구축하고, 

    • 수령인의 practice의 맥락과 과제challenge에 대해 배우고, 

    • 평가 과정에 대해 가질 수 있는 우려를 탐구해야 한다고 강조했다

촉진자와 FR들은 상호 존중과 신뢰를 구축하고 유지하는 것이 성과 평가에 관한 의미 있는 대화의 토대라고 강조했다.

Feedback recipients consistently shared that both being formally assessed and receiving performance reports are sensitive and often intimidating activities. Hence, the feedback conversation requires thoughtfulness. They stressed that the facilitator should take the time to 

    • build a relationship with the recipient, 

    • learn about his or her practice context and challenges, and 

    • explore any concerns he or she might have regarding the assessment process. 

Facilitators and recipients emphasized that building and maintaining mutual respect and trust was the foundation for meaningful conversation about performance assessment.


2단계: 성능 데이터에 대한 반응 탐색

Phase 2: Explore reactions to the performance data


[피드백 보고서 및 공개된 특정 항목에 대한] 수령자의 일반적인 반응을 탐색하니, 보고서가 그들의 성과에 대한 그들 자신의 견해(즉, 그들 자신의 평가)를 confirm하는지 여부를 드러내었다. 촉진자들은 다음과 같은 열린, 비판단적 질문을 함으로써 수신자가 피드백 데이터에 대한 개인적인, 심지어 감정적인 반응을 편하게 공유할 수 있었음을 알아내었다.

    • "보고서에 대한 초기 반응은 어땠나요?" 

    • "보고서에 깜짝 놀랄 만한 내용이 있었나?" 

    • "이 자료들은 당신이 스스로에 대해서 생각하고 있던 것과 어떻게 비교가 되는가?" 

수령자는 이 결과를 confirm하였고, 이러한 접근방식이 효과적이라고 보고하였다. 즉, 촉진자의 개방적이고 존중하는 질문과 성찰적인 청취가 정직한 반응을 공유할 수 있는 안전하고 존경스러운 환경을 조성한 것이다.

Exploring recipients’ general reactions to their feedback report and to specific items revealed whether the report confirmed their own views of their performance (i.e., their own self-assessments). Facilitators found that asking open, nonjudgmental questions, such as “What were your initial reactions to the report?” “Did anything surprise you in your report?” or “How do these data compare with how you thought you were doing?” enabled recipients to feel comfortable sharing personal, even emotional, reactions to the feedback data. Recipients confirmed this finding and reported that the approach was effective—that is, the facilitators’ open and respectful questions and reflective listening created a safe, respectful environment for sharing honest reactions.


3단계: 컨텐츠에 대한 이해도 탐색

Phase 3: Explore understanding of the content


이 단계에서 피드백 대화는 [리포트에 대한 FR의 반응을 요청하는 것]에서 [보고서의 내용을 확실히 이해하고, 그것이 변화와 학습에 제공할 수 있는 기회]로 전환된다. 촉진자는 명확하지 않을 수 있는 데이터를 명확히 하고, FR이 강점을 인식할 뿐만 아니라, 수행능력의 gap과 변화의 기회를 파악할 수 있도록 지도한다. 목표는 의사가 [피드백 데이터에서 발생하는 한 두 개의 구체적인 기회를 식별할 수 있도록 하는 것]이다.

At this phase, the feedback conversation transitions from soliciting the physician’s reactions to the report to ensuring a clear understanding of the report’s content and the opportunities it affords for change and learning. The facilitator seeks to clarify any data that might be unclear and to guide the physician in recognizing strengths, as well as performance gaps and opportunities for change. The goal is to enable physicians to identify one or two specific opportunities arising from the feedback data that are especially important to them and which they wish to address.


4단계: 성능 변경 코칭

Phase 4: Coach for performance change


4단계에서는 FR을 위한 [현실적인 목표개발]과 [목표를 향한 행동 계획]에 초점이 있다. 이 과정은 촉진자의 코칭을 통해서 enable된다. 이 단계에서 촉진자-수령자 대화는 다음을 다룬다. 

    • 피드백 수령자의 목표, 

    • 취해야 할 조치, 

    • 구현을 가능하게 하거나 방해할 요소 

    • 특히 장애요인에 대처하기 위한 구체적인 전략

In Phase 4, the focus is the physician’s development of realistic goals and an accompanying action plan to work toward the goals, enabled through the facilitator’s coaching. Facilitator–recipient dialogue at this phase concerns the nature of 

    • the feedback recipient’s goals, 

    • actions to be taken, 

    • factors that will enable and hinder implementation, and 

    • specific strategies to address these, especially the barriers.


촉진자는 코칭 역할에 대해 덜 준비되었다고 느낀다고 보고한 반면, 피드백 수령자는 이 활동이 그들이 변화할 수 있도록 하는 데 중심적이며 더 많은 관심을 필요로 한다고 보고하였다.

Whereas facilitators reported feeling least prepared for the coaching role, feedback recipients reported that this activity was central to enabling them to be able to change and required more attention.


요약하자면, 촉진된 피드백 세션의 전체적인 결과는 FR이 자신의 평가 데이터를 imposed threat이나 risk같이 부정적인 것으로 인식하지 않고, 개인적 기회로서 긍정적으로 인식할 수 있도록 하는 것으로 보였다. 촉진자 역할은 피드백 수령자가 자신의 성과 데이터에 대한 오너십을 향상시키는 것처럼 보였다. 촉진된 피드백은 피드백에서 확인된 practice change를 계획할 수 있도록 하는 것처럼 보였다.

In summary, an overall result of the facilitated feedback session appeared to be to enable physicians to recognize their performance assessment data positively as a personal opportunity and not negatively as an imposed threat or risk. The facilitator role seemed to enhance the feedback recipients’ ability to take ownership of their performance data; facilitated feedback seemed to empower physicians to plan to make the identified practice change(s).


내 생각에 평가받는 의사에는, 이런 것이 중요하다. [모든 것이 그 미팅을 통해 처리되는 한], 수세에 몰리지 않고 어느 정도 통제력을 느끼는 것, 그들이 토론하고 싶은 것에 대한 대화를 통제할 수 있다고 느끼는 것이 중요하다고 생각한다. 그래서 동료가 조언을 해주는 것 같은 분위기가 조성되어야 한다. 이것은 (뭔가 도움이 필요할지도 모르는 사람에게) 마치 같은 사무실에 있는 두 사람이 코칭이나 조언을 해주는 것과 같다. 협박의 분위기가 아니라 동료-대-동료의 상의consultation처럼 말이다. (2장, 수취인 2)

I think it’s important for the assessed doctor to feel somewhat in control and not on the defensive, and to feel that they can control the conversation about what they want to discuss, as long as everything gets addressed through the meeting. So there should be an atmosphere created such that it’s a peer giving advice, as if two people in the same office just coaching or giving advice to someone who maybe needs a little help with something. Like a colleague-to-colleague consultation and not an atmosphere of intimidation. (Site 2, Recipient 2)


고찰

Discussion


R2C2 진행형 피드백 모델 및 4단계

The R2C2 Facilitated Feedback Model and its four phases


1단계, 공감대 형성 및 관계 구축은 의사 수혜자에 대한 진정한 관심을 입증하고 존중과 신뢰를 조성하는 데 초점을 맞춘 휴머니즘적이고 사람 중심적인 접근방식에 초점을 맞췄다.24,26 참가자들은 opening question을 던지고, 서로를 알아가는 것이 피드백을 주고 받는 것을 더 쉽게 만든다고 지적했다.

Phase 1, building rapport and relationship, drew on humanistic and person-centered approaches that focus on demonstrating a genuine interest in the physician recipient and creating respect and trust.24,26 The participants indicated that asking opening questions and getting to know one another made giving and receiving feedback easier.


2단계에서는 반응을 탐구하면서 사람 중심의 접근법이 계속 대화를 유도했다. 구체적으로, 정보에 입각한 자체 평가의 개념은 보고서에 대한 피드백 수령자의 반응을 탐구하도록 유도하였다.18 수령자가 [자신의 반응과 견해가 중요하게 받아들여진다는 것]을 아는 것이 중요했다.

In Phase 2, exploring reactions, person- centered approaches continued to guide the conversation. Specifically, the concepts of informed self-assessment led to the exploration of the feedback recipients’ reactions to the report.18 It was important for recipients to know that their reactions and views were taken seriously.


3단계 내용 탐색은 또한 사람 중심의 접근법에 의해 유도되었는데, 이 접근법에서는 촉진자가 피드백 수령자에게 성과 보고서를 이해하는지, 아니면 질문이 있는지, 그리고 그들에게 어떤 데이터가 가장 중요한지 고려하도록 요청하였다.24,26 이 단계는 (정보에 입각한 자체 평가와 일관되게) 이 단계는 [피드백 수령자가 다루기를 꺼려할 수도 있는] 성과 격차와 그 격차에 영향을 미치는 요소를 고려하도록 장려했다.

Phase 3, exploring content, was also guided by person-centered approaches, in which facilitators asked feedback recipients to consider the performance report, whether they understood it or had any questions, and what data were of most importance to them.24,26 This phase, in keeping with informed self-assessment, encouraged feedback recipients to consider performance gaps that they may be reluctant to address, plus any factors influencing the gaps.


마지막으로, 4단계에서 퍼포먼스 변화를 위한 코칭, 촉진자는 목표 확인identifying실행 계획 수립에서 피드백 수령자의 코칭에 [사람 중심의 접근법]과 [행동 변화 원칙]을 성공적으로 적용했다.

Finally, in Phase 4, coaching for performance change, facilitators successfully applied person-centered approaches24,26 and principles of behavior change25 to coaching the feedback recipient in identifying goals and developing an action plan.


R2C2 피드백 프로세스의 핵심 목적은 의사가 자신의 성능 데이터에 대한 소유권과 사용에 대한 책임을 갖도록 하는 것이다.

The central intent of the R2C2 feedback process is to empower physicians to take ownership of their performance data and responsibility for their use.


성찰 및 코칭

Reflection and coaching


피드백 모델은 피드백의 효과에 영향을 미치는 요소들을 탐구하는 이전의 연구에 기초한다. 여기에는 관계와 문화, 피드백에 대한 감정적 반응, 자기 평가와 피드백 assimilation을 informing하는 촉진과 토론의 역할, 성찰과 코칭의 역할, 피드백 토론의 형식이 포함된다. 특히 성찰코칭이라는 두 가지 요소가 기여한 것은 더 많은 관심을 받을 만하다.

The feedback model builds on earlier research exploring factors influencing the effectiveness of feedback. These include relationships and culture,1,5,6 emotional reactions to feedback,22,38 the role of facilitation and discussion in informing self-assessment and feedback assimilation,22,39 the roles of reflection21,22,24 and coaching,29,30 and the format of the feedback discussion.21,28,40 The contributions of two elements in particular, reflection and coaching, deserve further attention.


명시적으로 명시되지는 않았지만, R2C2 모델의 각 단계는 [피드백 데이터에 대한 촉진된 성찰]과 [피드백을 받는 사람에게 그 데이터가 갖는 의미]를 통해 중재moderate된다. 이전의 연구자들은 이 기법을 사람 중심의 상담 접근에 효과적인 것으로 발견했다. 성찰은 학습의 전략으로서, "개인을 새로운 이해와 appreciation으로 이끌기 위한 목적의 활동으로서, 자신의 경험을 탐구하기 위해 참여하는 intellectual and affective 활동"이다R2C2 모델에 사용된 개방형 질문은 각 단계에서 단지 그러한 이해와 appreciation를 장려하기 위해 고안되었다. 즉, 

    • 개인의 practice와 맥락을 설명하고, 

    • 피드백에 대한 반응을 탐구하고, 

    • 피드백을 해석하며, 

    • 변화에 영향을 미치는 요소와 옵션들을 고려하는 것이다.

Although not explicitly stated, each phase of the R2C2 model is moderated through facilitated reflection on the feedback data and the data’s meaning to the feedback- receiving physician. Previous researchers have found this technique (facilitated reflection) to be effective in person- centered counseling approaches.24,26 Reflection is a strategy for learning, “an intellectual and affective activity in which individuals engage to explore their experiences in order to lead to new understandings and appreciations.”41(p19) The open-ended questions used in the R2C2 model are designed to encourage just such understandings and appreciation at each phase: 

    • explaining one’s practice and context, 

    • exploring reactions to the feedback,22,26 

    • interpreting the feedback,31,41,42 and 

    • considering options for and factors influencing change.15,43


퍼실리테이터들은 코칭이 의사들이 변화를 일으킬 수 있도록 하는 데 도움이 되지만 상대적으로 익숙하지 않은 기술이라고 언급했다. 경영진 코칭은 조직에서 리더십 향상과 기술 개발을 지원하기 위해 사용된다.27,29 자신의 개인 퍼포먼스 데이터를 바탕으로 한 의사들의 전문적 개발을 촉진하기 위한 코칭은 다음의 촉진을 포함한다.

    • 변화의 필요성을 인정

    • 관련 목표를 식별

    • 목표를 달성하기 위한 계획을 수립

Facilitator participants noted that coaching was a helpful and relatively unfamiliar skill to enable physicians to make changes. Executive coaching is used in organizations to support leadership improvement and skill development.27,29 Coaching to foster physicians’ professional development, based on their own personal performance data, involves facilitating 

    • their acknowledgment of a need for change, 

    • identification of related goals, and 

    • development of a plan to achieve the goals.15,44


중요한 것은, R2C2 피드백 모델에서, 코칭은 [관계 형성] 및 [피드백 수신자의 반응 탐색], 그리고 [자신의 퍼포먼스 피드백 데이터에 대한 이해] 뒤에 따라온다는 것이다. 피드백 받는 사람들은 코칭이 도움이 된다고 보고한 반면, 진행자들은 일반적으로 그것이 저절로 생기는 기술은 아니라고 보고했다. 코칭은 보다 전통적인 지시적인directive 가르침과 다르며, 촉진자들은 준비 워크숍에서 코칭 기술에 더 많은 관심을 요청했다.

Importantly, in the R2C2 Facilitated Feedback Model, coaching follows relationship building and exploring the recipient’s reaction to and understanding of his or her performance feedback data. Feedback recipients reported finding the coaching helpful, whereas facilitators generally reported that it was a skill that did not come naturally to them. Coaching differs from more traditional directive teaching, and facilitators requested more attention to coaching skills in their preparation workshop.


추가 연구를 위한 한계 및 영역

Limitations and areas for further research


연구 결과는 R2C2 진행형 피드백 모델의 실용적 사용과 지속가능성에 관한 질문을 제기한다. 모델은 피드백 세션에 대해 촉진자와 수령자 모두에게 시간 약속(약 45분)을 요구한다. 교수개발 워크숍(약 2시간)에도 참여한다. 워크숍에는 경험적 연습 시간이 포함되어야 하며 코칭 기법을 강조해야 한다.

study results raise questions regarding practical use and sustainability of the R2C2 Facilitated Feedback Model. The model requires a time commitment of facilitators and recipients alike (about 45 minutes) for the feedback session. Facilitators also participate in a faculty development workshop (about two hours). The workshop should include experiential practice time, and it should emphasize coaching techniques.


지속가능성에 대한 접근법(영국-MRC 지침의 4단계)에 inform하기 위해, 세션 및 교수개발의 다양한 모델을 엄격하게 탐구할 것을 제안한다. 예를 들어, 

To inform approaches to sustainability (Stage 4 of the UK-MRC guidelines34), we propose rigorously exploring varied models of providing the sessions and faculty development—for example, 

    • through professional and regulatory associations, 

    • during professional and continuing education conferences, 

    • through initiating peer feedback, and 

    • through coaching programs in which physicians identify a trusted colleague for the coaching.








 2015 Dec;90(12):1698-706. doi: 10.1097/ACM.0000000000000809.

Facilitated Reflective Performance FeedbackDeveloping an Evidence- and Theory-Based Model That Builds RelationshipExplores Reactions and Content, and Coaches for Performance Change (R2C2).

Author information

1
J. Sargeant is professor and head, Division of Medical Education, Faculty of Medicine, Dalhousie University, Halifax, Nova Scotia, Canada. J. Lockyer is professor, Department of Community Health Sciences, and senior associate dean, Education, Faculty of Medicine, University of Calgary, Calgary, Alberta, Canada. K. Mann is professor emeritus, Division of Medical Education, Faculty of Medicine, Dalhousie University, Halifax, Nova Scotia, Canada, and honorary professor and chair, Medical Education, Manchester Medical School, University of Manchester, Manchester, United Kingdom. E. Holmboe is senior vice president, Milestones Development and Evaluation, Accreditation Council for Graduate Medical Education, Chicago, Illinois, adjunct professor of medicine, Yale University, New Haven, Connecticut, and adjunct professor, Uniformed Services University of the Health Sciences, Bethesda, Maryland. I. Silver is vice president, Education, Centre for Addiction and Mental Health, and professor, Department of Psychiatry, Faculty of Medicine, University of Toronto, Toronto, Ontario, Canada. H. Armson is assistant dean, Continuing Professional Development, and associate professor, Department of Family Medicine, University of Calgary, Calgary, Alberta, Canada. E. Driessen is associate professor, Department of Educational Research and Development, Faculty of Health, Medicine and Life Sciences, Maastricht University, Maastricht, the Netherlands. T. MacLeod is research associate, Continuing Professional Development, Faculty of Medicine, Dalhousie University, Halifax, Nova Scotia, Canada. W. Yen is research associate, Research and Evaluation Department, College of Physicians and Surgeons of Ontario, Toronto, Ontario, Canada. K. Ross is research associate, Department of Evaluation, Research and Development, American Board of Internal Medicine, Philadelphia, Pennsylvania. M. Power is manager, Nova Scotia Physician Achievement Review Program, College of Physicians and Surgeons of Nova Scotia

Abstract

PURPOSE:

To develop and conduct feasibility testing of an evidence-based and theory-informed model for facilitating performance feedback for physicians so as to enhance their acceptance and use of the feedback.

METHOD:

To develop the feedback model (2011-2013), the authors drew on earlier research which highlights not only the factors that influence giving, receiving, accepting, and using feedback but also the theoretical perspectives which enable the understanding of these influences. The authors undertook an iterative, multistage, qualitative study guided by two recognized research frameworks: the UK Medical Research Council guidelines for studying complex interventions and realist evaluation. Using these frameworks, they conducted the research in four stages: (1) modeling, (2) facilitator preparation, (3) model feasibility testing, and (4) model refinement. They analyzed data, using content and thematic analysis, and used the findings from each stage to inform the subsequent stage.

RESULTS:

Findings support the facilitated feedback model, its four phases-build relationship, explore reactions, explore content, coach for performance change (R2C2)-and the theoretical perspectives informing them. The findings contribute to understanding elements that enhance recipients' engagement with, acceptance of, and productive use of feedback. Facilitators reported that the model made sense and the phases generally flowed logically. Recipients reported that the feedback process was helpful and that they appreciated the reflection stimulated by the model and the coaching.

CONCLUSIONS:

The theory- and evidence-based reflective R2C2 Facilitated Feedback Model appears stable and helpful for physicians in facilitating their reflection on and use of formal performance assessment feedback.

PMID:
 
26200584
 
DOI:
 
10.1097/ACM.0000000000000809


성장형 마음가짐을 동반한 피드백 문화를 만들기 위한 열두가지 팁(Med Teach, 2019)

Twelve tips to promote a feedback culture with a growth mind-set: Swinging the feedback pendulum from recipes to relationships

Subha Ramania , Karen D. K€oningsb, Shiphra Ginsburgc,d and Cees P. M. van der Vleutenb




도입

Introduction


피드백 교육 이니셔티브는 [제공자의, 기술 중심의 접근 방식]에서 [수신자의, 목표 및 영향 중심 접근 방식]으로 점차 전환되고 있다(Telio et al. 2015; Sarliment et al. 2015a; Bing-You et al. 2017b). 그러나, 교직원과 학습자는 피드백의 적절성과 품질에 대한 생각이 서로 같지 않을 수 있다. 교직원들이 의미 있고 구체적인 피드백을 제공했다고 생각하는 경우에도, 학습자는 교직원의 피드백이 모호하고 실행 불가능하다고 보고하는 경우가 자주 있다. (Bing-You and Trowbridge 2009; Anderson 2012; Ramani et al. 2017a, 2017a, 2017b)

Feedback training initiatives are gradually shifting away from provider technique-focused approaches to receiver goals and impact-focused approaches (Telio et al. 2015; Sargeant et al. 2015a; Bing-You et al. 2017b). Yet, teachers and learners may not agree on the adequacy and quality of feedback, with learners frequently reporting vague and non-actionable faculty feedback even when faculty believe they have provided meaningful and specific feedback (Bing-You and Trowbridge 2009; Anderson 2012; Ramani et al. 2017a, 2017b).


지난 10년 동안 의료 교육자들은 대화 내용에 대한 관계, 신뢰성에 대한 인식 및 제도적 학습 문화와 피드백에 대한 학습자의 수용성 등과 같은 사회문화적 요인의 영향에 더 큰 중점을 두기 시작했다(Mann et al. 2011; Sargeant et al. 2011a; Eva et al. 2012). 텔리오 외 연구진(2015, 2016)은 효과적인 피드백 상호작용을 위해서는 [학습자가 강력하게 참여하는 교사와 학습자 사이의 교육적 동맹]이 필요하다고 제안했다. Sarland 등(2015a, 2017a, 2017b)은 피드백 대화에 완전한 학습자 참여를 강조하는 R2C2 모델(관계, 반응, 콘텐츠 및 코칭)에 대해 설명했다.

Over the last decade, medical educators have begun to place greater emphasis on the influence of sociocultural factors, such as relationships, perceptions of credibility and institutional learning culture on the content of the conversation and learner receptivity to feedback (Mann et al. 2011; Sargeant et al. 2011a; Eva et al. 2012). Telio et al. (2015, 2016) proposed that effective feedback interactions require an educational alliance between teachers and learners with a strong learner involvement in the process. Sargeant et al. (2015a, 2017a, 2017b) described the R2C2 model (relationship, reaction, content and coaching) emphasizing full learner engagement in the feedback conversation.


예의바름이라는 기관적 문화는 학습과 동료적 활동에 도움이 되지만, 그것은 평가와 건설적인 피드백에 대한 솔직한 서술적 코멘트를 방해할 수 있다(Ginsburg 등, 2015; Ramani 등, 2017a).

While an institutional culture of politeness is conducive to learning and collegial work; it could hinder honest narrative comments on evaluations and constructive feedback (Ginsburg et al. 2015, 2016; Ramani et al. 2017a).


셰린이 설명한 3단계 조직문화는 의료교육의 피드백문화(Schein 2017)에 적용 가능하다. 

  • 가장 깊은 레벨은 조직이 외부세계에 의해 어떻게 보이는가에 대한 문서화되지 않은 가치와 가정을 가리킨다(여기서 일이 돌아가는 방식). 

  • 중간 수준은 문서화된 가치와 기대를 말하며, 사명문 및 커리큘럼 문서가 있다.

  • 가시적인 수준은 그 구성원들이 세계에 보여지는 이미지를 유지하기 위해 매일 어떻게 행동하느냐에 의해 특징지어진다. 

각 레벨은 [피드백 동맹과 관련이 있기에] 해부하고 이해해야 할 필요가 있다.

The three levels of organizational culture described by Schein are applicable to the feedback culture in medical education (Schein 2017). 

  • The deepest level refers to unwritten values and assumptions of how the organization is viewed by the outside world (that’s the way things are done here). 

  • The mid-level refers to written values and such expectations as mission statements and curricular documents. 

  • The visible level is characterized by how its members behave day to day to maintain the image presented to the world. 

Each level, as it relates to feedback alliances, would need to be dissected and understood


Figure 1


피드백 제공자

Feedback providers


팁 1 긍정적인 학습 환경을 구축하고 전문적인 역할 모델이 됨

Tip 1 Establish a positive learning climate and be a professional role-model


많은 교사들과 학습자들은 다가오는 피드백 대화에 두려움을 느끼며 반응한다. 피드백 제공자들은 [긍정적인 학습 환경을 능동적으로 설정하고, 목표와 관찰된 성과에 초점을 맞춘 빈번하고 형태적인 피드백 대화를 촉진할 것이라는 기대를 설정함으로써] 이러한 부정적인 측면을 완화시킬 수 있다. 업무 관계의 기간이 짧더라도, 적절한 학습 환경을 구축하면 더 생산적인 피드백 대화를 할 수 있다. 교사들이 모두를 존중하고, 여러 의견을 기꺼이 받아들이며, 자신의 한계와 오류를 인정할 준비가 되어 있다는 것을 증명하는 롤모델 역할을 할 때, 이것은 성장 촉진 피드백을 위한 장을 마련하게 될 것이고, 따라서 학습자들에게 더욱 수용될 것이다(Skeff and Mutha 1998; Srinivasan et al. 2011). 학습자로부터 피드백을 받는 교사들의 개방성을 강조하면서 양방향 피드백 대화의 개념도 논의될 수 있다.

Many teachers and learners react to upcoming feedback conversations with trepidation. Feedback providers can alleviate some of this negativity by proactively establishing a positive learning climate and setting expectations that they would facilitate frequent, formative feedback conversations that focus on goals and observed performance. Even if the duration of the working relationship is short, establishing a congenial learning environment could lead to more productive feedback conversations. When teachers serve as role-models in demonstrating respect for all, willingness to welcome multiple opinions, and readiness to admit own limitations and errors, this would set the stage for growth-enhancing feedback and thus more acceptable to learners (Skeff and Mutha 1998; Srinivasan et al. 2011). The concept of two-way feedback conversations can also be discussed, emphasizing teachers’ openness to receiving feedback from learners.


팁 2 성능 직접 관찰을 사용하여 피드백 데이터 생성

Tip 2 Use direct observation of performance to generate feedback data


피드백의 신뢰성에 대한 학습자의 인식은 피드백의 출처, 피드백 제공자와의 관계, 커뮤니케이션 방식, 자체 평가와의 일치 등 몇 가지 요인에 의해 영향을 받을 수 있다(Mann et al. 2011; Watling et al. 2013b; Watling 2014a). 성과에 대한 직접적인 관찰은 신뢰성의 중요한 결정요소로 보인다. 임상 학습자는 환자 또는 직원과의 상호작용 중에 교수진이 관찰하는 경우가 거의 없다(Kogan et al. 2012, 2017). 음악이나 스포츠의 코칭 문화와 달리, 임상 교사는 (특히 시니어 학습자가 데이터를 수집한 경우) 학습자를 직접 관찰하지 않고도 학습자 사례 프레젠테이션의 정확성을 받아들이는 경우가 많다(Walker et al. 2017; Watling et al., 2014). 임상 교사들이 학습자를 직접 그리고 자주 관찰하고 성과에 대한 구체적인 피드백을 제공함으로써 학습자의 신뢰성에 대한 인식을 높이는 것이 중요하다.

Learners’ perception of the credibility of feedback can be influenced by several factors including the source of feedback, their relationship with the feedback provider, the manner of its communication, and congruence with their own self-assessment (Mann et al. 2011; Watling et al. 2013b; Watling 2014a). Direct observation of performance appears to be one significant determinant of credibility. Clinical learners are rarely observed by faculty during their interactions with patients or staff (Kogan et al. 2012, 2017). In contrast to the coaching culture in music or sports, clinical teachers often accept the accuracy of learner case presentations without direct observation of data gathering, particularly from more senior learners (Walker et al. 2017; Watling et al., 2014, 2016). It is important that clinical teachers observe their learners directly and frequently, and provide specific feedback on performance, thus enhancing learners’ perception of its credibility.


팁 3 반성을 촉진하고 정보에 입각한 자체 평가

Tip 3 Facilitate reflection and informed self-assessment


학습자는 자기평가와 상충되는 피드백은 거부하는 경향이 있다.

Learners tend to reject feedback that conflicts with their self-assessment;


교육 전문가들은 정확한 자기 평가를 위해 복수 출처의 외부 데이터를 자기 성찰과 결합해야 한다고 주장해왔다(Eva and Regehr 2008; Sargeant et al. 2009, 2010; Mann et al. 2011). 또한, 대화를 자기평가로 시작하는 것은 교사들이 학습자의 장단점에 대한 통찰력(또는 부족)을 진단하고, 건설적인 피드백을 강화하는 출발점이 될 수 있다.

Educational experts have argued that external data from multiple sources should be combined with self-reflection for accurate self-appraisal (Eva and Regehr 2008; Sargeant et al. 2009, 2010; Mann et al. 2011). Additionally, initiating conversations with selfassessment can help teachers to diagnose learners’ insights (or lack of) into their strengths and weaknesses, and serve as a starting point for reinforcing as well as constructive feedback.


심리학자들에 의해 대인 커뮤니케이션에서 자아 인식을 향상시키기 위해 기술된 프레임워크인 Johari 창은 피드백에 대한 강력한 모델 역할을 할 수 있다(Luft talks 1969). 피드백에 이 모델 적용: 

    • 강점을 강화하고 자기 효율을 높이면 개방된 사분면이 확장될 수 있다. 

    • 학습 목표 지향 및 피드백을 촉진하는 것은 시각장애인 사분면을 다룰 수 있다. 

    • 교육 제휴와 신뢰 관계 구축은 숨겨진 사분면을 좁힐 수 있다. 

    • 자기발견 정신을 자극하면 미지의 사분면을 좁힐 수 있다(Ramani et al. 2017c).

The Johari window, a framework described by psychologists to enhance self-awareness in interpersonal communications, can serve as a robust model for feedback (Luft conversations 1969). Applying this model to feedback: 

    • reinforcing strengths and enhancing self-efficacy can expand the open quadrant; 

    • promoting a learning goal-orientation and feedback seeking can address the blind quadrant

    • building educational alliances and trusting relationships can narrow the hidden quadrant; and 

    • stimulating a spirit of self-discovery can shrink the unknown quadrant (Ramani et al. 2017c).


피드백 수신인용

For feedback recipients


팁 4 학습자 사이에 성장형 마인드셋을 구축하십시오.

Tip 4 Foster a growth mind-set among learners


Dweck는 고정형 마인드셋과 성장형 마인드셋이라는 두 가지 유형의 마인드셋을 설명했다(Dweck 1990, 2006). 성장 마인드셋은 학습자가 피드백을 찾고, 건설적인 피드백을 더 수용하며, 피드백을 일상적 성과에 통합할 수 있도록 한다. 교사들은 단순히 칭찬이나 판단(예: "뛰어난 직업, 형편없는 환자 의사소통")보다는 성과에 초점을 맞춘 언어를 사용하여 성장 마인드를 자극하는 데 중요한 역할을 한다. 기관은 건설적인 피드백을 정상화시키고, 모든 수준에서 전문적 개발을 우선시하며, 피드백을 받고 성과에 동화시키는 훈련을 제공함으로써 성장 마인드 설정을 장려해야 한다.

Two types of mind-sets have been described by Dweck, a fixed mind-set and a growth mind-set (Dweck 1990, 2006). A growth mind-set would allow learners to engage in feedback seeking, be more receptive to constructive feedback, and incorporate feedback into daily performance. Teachers have an important role to play in stimulating a growth mind-set by using language that focuses on performance rather than words that simply praise or judge (e.g. “excellent job, poor patient communication”). Institutions should encourage a growth mind-set by normalizing constructive feedback, prioritizing professional development at all levels, and providing training in receiving and assimilating feedback into performance.


팁 5 피드백 탐색 행동 권장

Tip 5 Encourage feedback seeking behavior


학습자는 현재 성과와 원하는 성과 사이의 차이를 보정하는 데 도움이 되는 능동적 피드백 탐색을 통해 개선이 필요한 자신의 강점과 영역에 대한 인식을 얻을 수 있다(Crommelinck 및 Anseel 2013). 

    • 성과 목표 지향성을 가진 전문가들은 좋은 인상을 주는 성과에 초점을 맞추고 있으며, 한계를 드러내고 이미지를 위협할 수 있는 피드백을 환영하지 않을 수 있다. 이러한 경향은 "게임을 한다playing the game"이라고 언급되었다(Gaunt 등, 2017). 

    • 반면에, 학습 목표 지향성을 가진 전문가들은 그들의 분야에서 숙달된 것을 성취하는데 초점을 맞추고 있으며, 그들이 성장하는데 도움이 되는 건설적인 피드백을 찾고 받아들일 가능성이 더 높다.

Learners can gain awareness of their strengths and areas needing improvement through active feedback-seeking which helps in calibrating the gap between their current performance and desired performance (Crommelinck and Anseel 2013). 

    • Professionals with a performance goal-orientation focus on performance that creates a good impression, and may not welcome feedback that might reveal limitations and threaten their image. This tendency has been referred to as “playing the game” (Gaunt et al. 2017). 

    • On the other hand, professionals with a learning goal-orientation focus on achieving mastery in their field, and are more likely to seek and accept constructive feedback that helps them grow.


팁 6 행동 변화에 대한 학습자 시작 작업 계획 촉진

Tip 6 Promote learner initiated action plans for behavior change


의학 학습자들은 교사로부터의 피드백이 종종 "행동할 수 없는" 경우가 많고 전형적인 피드백 대화는 성과 개선 계획으로 끝나지 않는다고 말한다(Bing-You and Trowbridge 2009; Anderson 2012; Ramani et al. 2017a, 2017b). 그러나 교사들이 구체적인 행동 계획을 권고한다 하더라도, 학습자들이 그들의 관행을 바꿀 것이라고 가정할 수는 없다. 성인 학습자가 다음과 같이 한다면 행동 변화가 더 쉽다.  

    • 자신의 학습 목표를 수립하고, 

    • 이를 교사와 소통하며

    • 이러한 목표의 관점에서 보정하고, 

    • 이러한 목표를 달성하기 위한 단계를 기술한다 

Medical learners state that feedback from their teachers is often not “actionable” and a typical feedback conversation does not conclude with a performance improvement plan (Bing-You and Trowbridge 2009; Anderson 2012; Ramani et al. 2017a, 2017b). Even if teachers recommend specific action plans, it cannot be assumed that learners would change their practice. Behavior change is more likely if adult learners 

    • formulate their own learning goals, 

    • communicate these to their teachers, 

    • calibrate where they are in terms of these goals and 

    • describe steps to achieve these goals. 


성인 학습자와 미래 성찰 실천자로서, 실행 계획은 성과 개선을 위해 학습자가 가장 잘 개시initiate할 수 있다.

As adult learners and future reflective practitioners, action plans are best initiated by learners for performance improvement.


피드백 관계

The feedback relationship


팁 7 교육 동맹 수립

Tip 7 Establish an educational alliance


피드백 대화는 단순한 일방적 교환보다는 복잡한 대인 커뮤니케이션이다.

Feedback conversations are complex interpersonal communications rather than simple one-way exchanges


    • 텔리오 외 연구진(2015, 2016)은 치료 제휴를 모델로 하여 피드백 대화를 구성하기 위한 "교육 동맹" 프레임워크를 제안했다. 학습자와 동맹을 맺는 것은 교육자가 학습자 성과에 대한 더 의미 있는 이해를 개발하고 구체적이고 유용한 피드백을 제공할 수 있게 할 것이다. 

    • 마찬가지로, Sarland 등(2015b, 2017b)은 R2C2 모델에 대해 관계 설정, 반응 탐색, 콘텐츠에 대한 이해 확인, 성장 코칭 등의 제안 단계를 설명했다. 

    • 빙유 외 연구진(2017b)은 탱고 춤을 은유로 삼아 정적 일방적인 피드백 레시피에서 역동적인 파트너십 기반의 대화로 나아갔다.

    • Using the therapeutic alliance as a model, Telio et al. (2015, 2016) proposed an “educational alliance” framework to construct feedback conversations. Forming an alliance with their learners would allow educators to develop a more meaningful understanding of the context of learner performance and provide specific and useful feedback. 

    • Similarly, Sargeant et al. (2015b, 2017b) described the R2C2 model with the following suggested steps: establish relationships, explore reaction, check understanding of the content and coach for growth. 

    • Bing-You et al. (2017b) used the tango dance as a metaphor to move from a static one-sided feedback recipe to a dynamic partnership-based conversation.


팁 8단계 교사 및 학습자가 행동 변화를 위한 학습 기회를 공동 창출하도록 권장

Tip 8 Encourage teachers and learners co-create learning opportunities for behavior change


학습자는 피드백을 동화시킬 수 있는 시간과 실천을 바꿀 기회를 가져야 한다.

Learners must have time to assimilate feedback and opportunities to change practice.


우리는 다음과 같이 구성된 피드백 대화를 위한 참여형 설계 루프를 제안한다. 

    • 학습자와 선생님에 의한 목표 설정

    • 교사들의 직접적인 성과 관찰

    • 자기성찰의 촉진을 포함하는 피드백 대화

    • 피드백과 변화 행동을 통합할 수 있는 학습/작업 기회 창출 

    • 새로운 성과에 대한 보고 

    • 교사와 학습자의 새로운 목표에 대한 논의를 통해 순환에 들어간다. 

이것은 그림 2에 묘사되어 있다.

We propose a participatory design loop for a feedback conversation comprising: 

    • establishment of goals by learners and teachers, 

    • direct observation of performance by teachers, 

    • feedback conversation that includes facilitated self-reflection, 

    • creating learning/work opportunities to incorporate feedback and change behavior, 

    • debriefing of new performance, and 

    • reentering the cycle through discussion of new goals by teachers and learners. This is depicted in Figure 2.



제도적 맥락

The institutional context


팁 9 학습자 자신의 효율성에 적절한 주의를 기울이십시오.

Tip 9 Ensure appropriate attention to learner self-efficacy


"체면"는 개인이 외부 세계에 투영하고자 하는 이미지를 묘사하는 데 사용되는 용어다(Brown and Levinson 1987). 체면은 긍정적이거나 부정적이다(팁 10 참조). 여기서 긍정적인 체면은 affirmation이나 자기효능감을 갖고자 하는 개인의 욕망이다. 의료 교사들은 종종 학습자의 감정을 상하게 하거나 자존심이 상할 것을 우려해 건설적인 피드백을 제공하는 것을 꺼린다(Sarling et al. 2008; Watchling et al. 2013a; Watchling 2014b; Bing-You et al. 2017a).

“Face” is a term used to describe the image individuals desire to project to the outside world (Brown and Levinson 1987). Face is further classified as positive and negative (see tip 10), where positive face is an individual’s desire for affirmation or self-efficacy. Medical teachers are often reluctant to provide constructive feedback, for fear of hurting learners’ feelings or damaging their self-esteem (Sargeant et al. 2008; Watling et al. 2013a; Watling 2014b; Bing-You et al. 2017a). 


학습자들은 건설적인 피드백이 실무의 변화에 더 도움이 된다고 보고하지만, 그들은 또한 긍정적인 피드백을 추구하는 경향이 있고, 종합적 작업장에 기초한 평가 환경에서 자신의 자아나 이미지를 손상시킬 수 있는 피드백을 회피하는 경향이 있다(Gaunt et al. 2017).

Though learners report that constructive feedback is more instrumental in changing practice, they too tend to seek positive feedback and avoid feedback that could damage their ego or image in summative workplace based assessment settings (Gaunt et al. 2017).


행동과의 대립이 행동 변화의 첫 단계로 식별되었기 때문에(van den Eertwegh et al. 2015), 형성적 평가 세팅은 학습자가 자아 비용(건설적 피드백에 따른 부정적인 느낌)자아(보강 피드백에 따른 자부심 증가)의 균형을 더 잘 맞출 수 있도록 도울 수 있다. 이는 결국 건설적인 피드백 수용을 높여줄 수 있다 (Gaunt 등, 2017). 건설적인 피드백은 [그것이 어떻게 표현되든 상관없이] 학습자들을 화나게 할 가능성이 있지만, 그들의 발전과 다음 단계로 나아가는 데는 필수적이다.

Since confrontation with one’s behavior has been identified as the first step to the change of behavior (van den Eertwegh et al. 2015), a formative assessment setting could help learners to better balance ego costs (negative feelings resulting benefits from constructive feedback) and ego (increased self-esteem resulting from reinforcing feedback) leading to increased acceptance of constructive feedback (Gaunt et al. 2017). Hearing constructive feedback will likely upset learners regardless of how it is phrased, but it is essential for their development and progression to the next level.



팁 10 감독과 자율성의 최적 균형 촉진

Tip 10 Promote optimal balance of supervision and autonomy


부정적 체면이라는 용어는 "자신의 행동이 다른 사람들에 의해 방해 받지 않고자 하는, 모든 유능한 성인 구성원의 요구"로 정의되었다(Brown and Levinson 1987). 임상 훈련의 핵심 목표는 학습자가 독립적인 실천으로 나아갈 수 있도록 돕는 것이지만, 환자의 안전과 품질에 대한 우려는 학습자의 감독을 필요로 한다(Ramani et al. 2017b; Watling et al. 2013a). 따라서 임상 교사들은 [감독과 자율성의 균형]을 맞추는 것이 중요하다. 

The term negative face has been defined as “the want of every competent adult member that his actions be unimpeded by others” (Brown and Levinson 1987). A key goal of clinical training is to help learners proceed towards independent practice, yet patient safety and quality concerns necessitate supervision of learners (Ramani et al. 2017b; Watling et al. 2013a). Thus, it is important for clinical teachers to balance supervision with autonomy. 


텐케이트 등 (2004)은 교사가 [학습자를 위한 완전한 외부 가이드]로부터 [공유된 가이드]로 나아가고, 최종적으로는 학습자가 독립적인 실천을 할 수 있는 경우에는 [완전한 학습자 내부의 가이드]로 이동할 수 있어야 한다고 묘사했다. 학습자의 필요와 목표는 훈련의 단계마다 다르기 때문에, 공유 지침shared guidance은 다음을 필요로 한다.

    • 학습자와의 지속적인 대화, 

    • 진행 상황 모니터링 및 

    • 학습 요구에 대한 교육의 적응을 

Ten Cate et al. (2004) described the concept of shared guidance where teachers can move along a spectrum of full external guidance of learners to shared guidance and finally full internal guidance when learners are capable of independent practice. Since needs and goals of learners are different at different stages of their training, shared guidance requires 

    • an ongoing dialog with learners, 

    • monitoring of their progress, and 

    • adapting teaching to their learning needs.


임상 학습은 교사, 동료 및 다학제 전문가를 포함한 팀과의 사회적 상호작용 중에 발생하므로, (자율적 학습보다는) 공동 규제co-regulated 학습 모델이 대학원 의학 교육에 더 도움이 될 수 있다.

clinical learning occurs during social interactions with a team including teachers, peers, and multidisciplinary professionals, therefore a model of co-regulated learning (rather than self-regulated learning) may better serve postgraduate medical education,


팁 11 지속적인 실천 개선 환경 구축

Tip 11 Establish a continuous practice improvement environment


의료 교육에서 피드백을 흔히 긍정 또는 부정적 피드백으로 구분하며, 재교육은 용어를 성능 향상 계획을 기술하는 데 사용한다. 따라서 건설적인 피드백은 (피드백 제공자와 수령자 모두에게) 부정적인 의미를 가질 수 있다.

In medical education, feedback is often referred to as positive or negative and the term remediation is used to describe performance improvement plans. Thus, constructive feedback can have negative connotations for both feedback providers and recipients.


플러스 델타 접근방식은 린 건설 연구소에서 기술한 형태별 평가 과정이다(http://leanconstruction.org/ media/learning_laboratory/Plus_Delta/Plus-Delta.pdf). 이 접근방식은 긍정적이거나 부정적인 판단 언어(좋은, 나쁜, 나쁜, 나쁜, 형편없는, 만족스럽지 못한, 그리고 만족스럽지 못한)보다는 개선 언어(여러분은 무엇을 바꾸거나 다르게 할 것인가, 당신의 관행을 어떻게 개선할 것인가)로 프레임되어있다.

The Plus Delta approach is a formative evaluation process described by the Lean Construction Institute (http://leanconstruction.org/media/learning_laboratory/Plus_Delta/Plus-Delta.pdf). This approach is framed in improvement language (what would you change or do differently, how would you improve your practice) rather than positive or negative judgmental language (good, bad, did well, did poorly, satisfactory, and unsatisfactory).


지속적인 연습 개선에서 언어 프레임language framed 접근법을 사용하는 교사들과 학습자들을 훈련시키는 것은 건설적인 피드백을 정상화할 수 있고, 따라서 학습자들이 더 수용 가능한 의미 있는 건설적인 피드백의 교환을 촉진할 수 있다.

Training teachers and learners in the use of language framed approach in a continuous practice improvement could normalize constructive feedback, thereby encouraging exchange of meaningful constructive feedback which is more acceptable to learners.


팁 12 전문성장을 강화하는 피드백 문화 강조

Tip 12 Emphasize a feedback culture that enhances professional growth


교육 기관은 학습 문화가 모든 수준에서 성장 촉진 피드백에 도움이 된다는 것을 명시적으로 확립하는 것이 중요하다. 그러한 문화는 다음을 강조할 것이다. 

    • 지속적인 형성적 피드백에 대한 명시적 지침 

    • 학습자와 교사의 강점과 개선 영역을 정규화하는 학습 환경 

    • 학습자와 교사 사이의 종적 및 신뢰 관계 

    • 성능 직접 관찰 

    • 교사 및 학습자 사이에서 원하는 피드백 

    • 목표 지향적이고 실행 가능한 피드백 대화에 대한 교육

It is important for institutions to explicitly establish a that learning culture is conducive to growth enhancing feedback at all levels. Such a culture would emphasize: 

    • explicit guidelines for ongoing formative feedback; 

    • a learning environment that normalizes strengths as well as areas for improvement among learners and teachers; 

    • longitudinal and trusting relationships between learners and teachers; 

    • direct observation of performance; 

    • feedback seeking among teachers and learners; and 

    • training in goal-directed and actionable feedback conversations.


(http://www.royalcollege.ca/rcsite/cbd/implementation/wbas/coaching-wbas-e).



결론

Conclusions


대화 중에 사용되는 언어는 컨텍스트, 학습자, 관계, 성능 수준 등에 따라 달라질 수 있으며, "만병통치약"은 없다.

Language used during conversations can vary based on the context, the learners, the relationship, level of performance, etc., there is no “one-size-fits-all”.





Sargeant J, Lockyer J, Mann K, Armson H, Warren A, Zetkulic M, Sokladaris S, Konings K, Ross K, Silver I, et al. 2017a. The R2C2 model in residency education: How does it improve feedback use? Acad Med. [accessed 2018 Jan 16]. https://doi.org/10.1097/ACM. 0000000000002131  XXX


Sargeant J, Lockyer J, Mann K, Holmboe E, Silver I, Armson H, Driessen E, Macleod T, Yen W, Ross K, et al. 2015a. Facilitated reflective performance feedback: developing an evidence- and theory-based model that builds relationship, explores reactions and content, and coaches for performance change (R2C2). Acad Med. 90:1698–1706.


Sargeant J, Mann K, Manos S, Epstein I, Warren A, Shearer C, Boudreau M. 2017b. R2C2 in action: testing an evidence-based model to facilitate feedback and coaching in residency. J Grad Med Educ. 9:165–170. 











 2019 Jun;41(6):625-631. doi: 10.1080/0142159X.2018.1432850. Epub 2018 Feb 7.

Twelve tips to promote a feedback culture with a growth mind-setSwinging the feedback pendulum from recipes to relationships.

Author information

1
a Department of Medicine , Brigham and Women's Hospital, Harvard Medical School , Boston , MA , USA.
2
b Department of Educational Development and Research, School of Health Professions Education, Faculty of Health, Medicine and Life Sciences , Maastricht University , Maastricht , the Netherlands.
3
c Department of Medicine , University of Toronto , Toronto , Canada.
4
d Wilson Centre for Research in Education, Faculty of Medicine , University of Toronto , Toronto , Canada.

Abstract

Feedback in medical education has traditionally showcased techniques and skills of giving feedback, and models used in staff development have focused on feedback providers (teachers) not receivers (learners). More recent definitions have questioned this approach, arguing that the impact of feedback lies in learner acceptance and assimilation of feedback with improvement in practice and professional growth. Over the last decade, research findings have emphasized that feedback conversations are complex interpersonal interactions influenced by a multitude of sociocultural factors. However, feedback culture is a concept that is challenging to define, thus strategies to enhance culture are difficult to pin down. In this twelve tips paper, we have attempted to define elements that constitute a feedback culture from four different perspectives and describe distinct strategies that can be used to foster a learning culture with a growth mind-set.

PMID:
 
29411668
 
DOI:
 
10.1080/0142159X.2018.1432850


알려지지 않은 것을 드러나게 하기: 전공의 교육에서 피드백의 문화에 자기인식이 미치는 영향(Med Teach, 2017)

Uncovering the unknown: A grounded theory study exploring the impact of self-awareness on the culture of feedback in residency education

Subha Ramania, Karen K€oningsb, Karen V. Mannc and Cees van der Vleutenb





도입

Introduction


성과에 대한 정확한 자기-보정은 다른 사람의 RIA와 ROA, 자기평가, 타인으로부터의 피드백을 필요로 한다(B 1995 1995; Eva and Regehr 2007, 2008).

Accurate self-calibration of performance requires reflection in and on action, self-assessment and feedback from others (Boud 1995; Eva and Regehr 2007, 2008).


아래에서는 피드백 대화에 자기 평가와 반성을 통합하기 위해 Johari 창을 사용하는 방법을 설명한다.

Below, we describe how the Johari window could be used to incorporate self-assessment and reflection into feedback conversations.


Johari 창은 1955년 Joseph Luft와 Harry Ingham에 의해 개발된 심리적 매트릭스로, 대인관계 중 자신의 행동, 감정, 상호작용 동기에 대한 인식과 이해를 가능하게 한다(Luft 1969). 또한 이 모델은 개인-환경 상호작용에도 적용되며 전문적 개발에서 자기 인식의 역할을 설명한다. 네 개의 사분면으로 나눠진다.

The Johari window is a psychological matrix developed by Joseph Luft and Harry Ingham in 1955, to enable awareness and understanding of own behaviors, feelings and interactions motivation during interpersonal (Luft 1969). The model also applies to person–environment interactions and explains the role of self-awareness in professional development. There are four quadrants,


우리는 이 매트릭스를 피드백, 피드백 찾기, 수용성 및 성과에 대한 영향의 문화에 대한 체계적인 질문을 유도하기 위한 효과적인 프레임워크로 제안한다.

we propose this matrix as an effective framework to guide systematic inquiries into the culture of feedback, feedback seeking, receptivity and impact on performance.


첫 번째 전략은 자기 평가인데, 전통적으로 자신의 능력을 평가하기 위한 unguided, self-generated 전략으로 간주된다. 그러나 전문가의 자기 평가 능력은 외부 관측치보다 정확하지 않은 것으로 보고되었으며, 최하의 수행자들은 자신의 오류에 대한 통찰력 부족으로 자신의 능력을 과대평가하는 경향이 있다(Kruger and Dunning 1999; Dunning et al. 2004; Eva and Regehr 2005; Davis 등, 2006; Ehrlinger et al. 2008; Sarlinger et al. 2008b. 이러한 부조화는 다른 사람의 데이터가 자기이미지와 상충되거나, 학습자가 출처가 신뢰성이 결여된 것으로 보는 경우 증폭된다(Sargeant et al. 2008a; Mann et al. 2011).

The first strategy is self-assessment, traditionally viewed as an unguided, self-generated strategy to assess one’s own abilities. However, professionals’ ability to self-assess has been reported as less accurate than external observations, with suboptimal performers tending to overestimate their competence due to lack of insight into their errors (Kruger and Dunning 1999; Dunning et al. 2004; Eva and Regehr 2005; Davis et al. 2006; Ehrlinger et al. 2008; Sargeant et al. 2008b). This incongruence is amplified when data from others conflict with impressions of self or if learners view the source as lacking credibility (Sargeant et al. 2008a; Mann et al. 2011).


자체 평가에 대한 보다 최근의 정의는 외부 출처의 데이터를 포함하는 것을 강하게 강조한다

  • 바우드(1995)는 자기 평가를 [자신의 성과에 대한 내외부의 데이터를 필요로 하는 과정]으로서, 이를 [표준과 비교하여 자신의 성과에 대한 판단을 내리는 과정]이라고 설명한다. 

  • Mann et al. (2011)은 내부 및 외부 데이터를 자체 평가에 통합하는 것으로 정의하면서 informed self-assessment라는 용어를 만들었다. 

  • 자기 주도적 평가 탐색이라는 용어는 자신의 현재 수준의 성과에 대한 형태적 평가와 종합적 평가를 외부로 보는 교육학적 활동을 말한다(Eva and Regehr 2008). 

이처럼 자기평가를 (개인 차원의 전략이 아니라) 상호작용적 전략으로 설명하는 것은 피드백 탐색, 수용, 그리고 행동에 대한 영향과 더 관련이 있다.

More recent definitions of self-assessment strongly emphasize inclusion of data from external sources. Boud (1995) describes self-assessment as a process that requires internal and external data about one’s performance and comparing these with a standard to make a judgment about one’s performance. Mann et al. (2011) coined the term informed self-assessment, defining it as the incorporation of internal and external data into self-appraisal. The term self-directed assessment seeking refers to the pedagogical activity of looking outward for formative and summative assessments of one’s current level of performance (Eva and Regehr 2008). These descriptions of self-assessment as an interactive rather than individual strategy are more relevant to feedback seeking, acceptance and impact on behavior.


두 번째 중요한 전략은 성찰인데, 이를 성찰적 학습과 성찰적 실천으로 분류할 수 있다

  • 성찰적 학은 지식, 기술 및 태도의 성장에 inform하기 위하여 자신의 경험과 성과에 대해 비판적으로 사고하고 분석하는 것을 말한다(Moon 2004; Chaffey et al. 2012; Hayton et al. 2015). 

  • 성찰적 실은, 의료 전문가의 핵심 신조로서, 지속적인 전문적 개발을 위해 자신의 행동을 성찰하는 능력이다(Sho€n 1983).

The second important strategy is reflection, which can be categorized as reflective learning and practice. 

  • Reflective learning refers to critical thinking and analysis of own experience and performance to inform growth in knowledge, skills and attitudes (Moon 2004; Chaffey et al. 2012; Hayton et al. 2015). 

  • Reflective practice, a core tenet for healthcare professionals, is the ability to reflect on one’s actions to enable continuing professional development (Scho€n 1983).


부드 외 연구진은 다음과 같이 말한다. "성찰은 사람들이 자신의 경험을 recapture하고, 생각하고, 숙고하고, 평가하는 중요한 인간의 활동이다. 성찰은 [경험을 가지고 work]하는 것이며, 이는 학습에 중요하다."(Boud et al. 1985).

Boud et al. state that: “Reflection is an important human activity in which people recapture their experience, think about it, mull it over and evaluate it. It is this working with experience that is important in learning” (Boud et al. 1985).


이전 연구의 주요 결과는 기관 문화가 피드백 교환의 내용과 신뢰성에 상당한 영향을 미친다는 것을 보여주었다. 전공의와 교직원들도 예의바름의 문화가 정직한 피드백, 특히 건설적인 피드백을 가로막는 장애물이었다고 보고했다.

Key results from the previous studies indicated that institutional culture had a significant impact on the content and credibility of feedback exchanges. Residents and faculty also reported that the institutional culture of politeness was a barrier to honest, especially constructive feedback.


방법

Methods


연구 세팅

Study setting


이 연구는 약 160명의 레지던트들이 참가한 대규모 도시 훈련 프로그램에서 실시되었다. 3~4년 훈련 트랙에 있는 레지던트들은 입원 및 외래환자 환경에서 일한다.

The study was conducted at a large, urban training program with approximately 160 residents. Residents on a 3- to 4-year training track work in inpatient and outpatient settings.


질적 접근 및 샘플링

Qualitative approach and sampling


가설 생성에 적합한 구성주의 근거이론 접근방식을 사용하여 임상 성과 및 전문 성장에 대한 피드백, 정보에 입각한 자체 평가 및 성찰에 대한 참여자의 관점을 탐구했다(Kennedy and Lingard 2006; Watling and Lingard 2012; Charmaz 2014). 자발적인 샘플링 전략을 사용하여, 우리는 포커스 그룹 토론을 위해 거주자와 교수진을 모집했다. 용도의 샘플링 전략은 대표적인 그룹을 대상으로 하며, 이 경우 병동이나 연속성 클리닉을 로테이션하는 레지던트와 대부분의 임상 감독과 가르침을 제공하는 제너럴리스트 교수진을 대상으로 한다(Creswell 및 Creswell 2013; Ramani and Mann 2016).

A constructivist grounded theory approach, appropriate for hypothesis generation, was used to explore participant views on the role of feedback seeking, informed self-assessment and reflection on clinical performance and professional growth (Kennedy and Lingard 2006; Watling and Lingard 2012; Charmaz 2014). Using a purposive sampling strategy, we recruited residents and faculty for focus group discussions. Purposive sampling strategies target representative groups, in this case residents who rotate on inpatient and continuity clinic settings and generalist faculty who provide most of the clinical supervision and teaching (Creswell and Creswell 2013; Ramani and Mann 2016).


자료 수집

Data collection


개방형 질문은 기관 피드백 문화에 대한 참여자의 의견, 피드백 추구, 수용성 및 양방향 피드백에 영향을 미치는 요소들을 탐구했다.

Open-ended questions explored participant opinions on the institutional feedback culture, factors impacting feedback seeking, receptivity and bidirectional feedback.


연구팀에서 사전에 논의된 개방형 질문을 대화의 계기로 삼았으며, 필요에 따라 명확화 및 추가 정밀화를 모색하였다(표 1).

Open-ended questions, discussed in advance by the research team, were used as triggers to initiate conversations, and clarification and further elaboration were sought as required (Table 1).



자료 분석

Data analysis


11개의 포커스 그룹 녹취록은 (Johari 창 모델을 사용하여) 새로운 프레임워크 기반 정성 분석을 위한 데이터를 제공했다. 개념적 주제 분석을 수행하여 조하리 창의 각 사분면에 대한 우리의 발견이 어떻게 정렬되거나 도전적인 구조와 일치하는지 탐구하였다. 자기평가나 성찰과 관련된 주제 및 프레임워크에 적용할 수 있는 주제만 식별되었다.

Eleven focus group transcripts provided data for a new framework-based qualitative analysis, using the Johari window model. A conceptual thematic analysis was carried out to explore how our findings aligned with or challenged constructs for each quadrant of the Johari window. Only themes related to self-assessment and reflection and applicable to the framework were identified.


성찰성을 보장하기 위해, 우리는 샘플링, 데이터 수집 및 분석에 대한 연구자들의 접근방식에 대한 연구자들의 영향을 반영하고 인정했다(Watt 2007; Watling and The Lingard 2012). 주임 저자(SR)는, 레지던트의 승진이나 졸업 결정이나 교직원의 직위에 대한 파워를 갖지 않는다. 그녀의 공동 연구자들은 비의사들로, 연구가 수행된 기관과 관련이 없으며, 건강 전문 교육 연구 전문가들이다. 이 팀은 또한 두 명의 연구 보조자를 포함했는데, 그 중 한 명은 포커스 그룹을 관찰하고 현장 메모를 했으며 다른 한 명은 독립적인 데이터 분석(LW, EP)에 참여했다. 각 녹취록을 검토하여 질문이 공개되고 광범위한 의견을 표현할 수 있도록 하였다.

To ensure reflexivity, we reflected on and acknowledged any influence of researchers in their approach to sampling, data collection and analysis (Watt 2007; Watling and The the Lingard 2012). lead author (SR), a faculty in Department of Medicine, is not responsible for promotion or graduation decisions for residents or in a position of power over faculty. Her collaborators are nonphysicians, not affiliated with the institution where the research was conducted, and experts in health professions education research (CV, KK). The team also included two research assistants, one of whom observed the focus groups and took field notes and the other participated in independent data analysis (LW, EP). Each transcript was reviewed to ensure that questions remained open-ended and allowed for expression of a wide range of opinions.



결과

Results


아는 것을 확인하기

Validating the known


"known"의 사분면은 레지던트 뿐만 아니라 그들의 감독하는 교직원과 동료들에게 알려진 역량 영역이나 행동을 가리킨다. 피드백 대화 중에 이러한 영역을 검토함으로써, FR의 자기평가가 정확하다는 것을 검증하고, 그들의 성과 개선 계획이 목표라는 것을 강화한다. 좋은 임상 습관을 강화하면 레지던트가 그러한 행동에 계속 관여할 수 있다.

The “known” quadrant refers to competency domains or behaviors known to the resident as well as their supervising faculty and peers. Reviewing these areas during feedback conversations validates the feedback receiver that their self-assessment is accurate and reinforces that their performance improvement plans are on target. Reinforcing good clinical habits can ensure that the resident will continue to engage in such behavior.


알려진 행동에는 강점과 결점이 포함되어 있지만, 교수진은 신뢰를 구축하고 지속적인 대화를 위한 기조를 정하는데 있어 긍정적인 피드백의 역할을 강조했다.

While known behaviors include strengths and deficiencies, faculty emphasized the role of positive feedback in establishing trust and setting the tone for ongoing conversations.


레지던트과 교직원들은 자기 평가의 정확성을 평가하고, 강점을 검증하고 강화하기 위해 알려진 행동을 논하는 것이 중요하다고 말했다.

Residents and faculty stated that discussing known behaviors is important to assess the accuracy of self-assessment, validate and reinforce strengths.


사각지대를 인정하기

Accepting the blind


"Blind" 사분면이란 레지던트에게 알려지지 않았지만 레지던트를 감독하는 교직원과 동료에게는 명확하게 보여지는 수행 영역을 말한다. 강점을 논하는 것은 쉬운 일이지만, Blind 영역에서 어려운 점은 underperformance에 대한 건설적인 피드백을 할 때이고, 이런 피드백은 수신자의 감정을 상하게 하지 않기 위해 종종 생략되곤 한다

The “blind” quadrant refers to areas of performance unknown to the resident but evident to supervising faculty and peers. It is likely easier to discuss strengths, but the challenge arises when the blind area involves constructive feedback on underperformance, often skipped to avoid hurting receivers’ feelings. 


    • 적극적인 피드백을 탐색에 참여하고

    • 피드백 대화를 학습 목표를 중심으로 진행하며, 

    • 성과 개선 계획을 촉진하는 것

...은 건설적인 의견을 수용하는 것을 더 맛있게 할 수 있다.

    • Engaging in active feedback seeking, 

    • grounding feedback conversations in learning goals and 

    • facilitating performance improvement plans 

...could make it more palatable to accept constructive comments.


교수진은 피드백을 추구하는 행동을 모델링하고 지속적인 전문적인 성장을 위해 그들 자신의 노력을 보여주면 레지던트들이 같은 일을 하도록 장려하고 건설적인 피드백을 수용하도록 향상시킬 수 있다고 밝혔다.

Faculty expressed that modeling feedback-seeking behavior and demonstrating their own efforts at ongoing professional growth could encourage residents to do the same and enhance their acceptance of constructive feedback.


항상 진솔하게 피드백을 요청하고, "나는 항상 주치의로서의 기술, 교육자로서의 능력 향상에 힘쓰고 있으니 솔직한 피드백을 부탁한다."(F)

I always ask for feedback genuinely and say, “I'm always striving to improve my attending skills, my teaching skills, so I’m asking you for honest feedback.” (F)


또한 피드백 훈련은 단순히 피드백을 주는 기술에만 집중하는 것이 아니라 그것을 받고 받아들이는 데도 초점을 맞춰야 한다는 의견이 제시되었다.

Additionally, it was suggested that feedback training should not just focus on the skills of giving, but also on receiving and accepting it.


[감정을 해소하는 전략이나, 특정 발언이 그 사람에 대한 공격이라는 암시를 피하기 위한 전략]을 다루는 과정에서 육아나 스포츠 코칭과 비교가 언급되었다.

The comparison to sports coaching as well as parenting was brought up as strategies to defuse emotion and avoid the implication that the comments were an attack on the person.


전반적으로, 참가자들은 Blind 사분면의 기술과 관련된 건설적인 피드백의 수용을 장려하기 위한 전략으로 다음을 지적했다.

    • 적극적인 피드백 탐색, 

    • 피드백 탐색 및 수신에 대한 훈련, 그리고 

    • 스포츠에서 볼 수 있는 것과 유사한 코칭 문화

Overall, participants indicated that strategies to encourage acceptance of constructive feedback pertaining to skills in the blind quadrant include 

    • active feedback seeking, 

    • training in feedback seeking and receiving, and 

    • a coaching culture similar to that seen in sports.


숨겼던 것을 공개하기

Disclosure of hidden


"Hidden" 사분면은 레지던트는 알고 있지만 교수진이나 동료에게 알려져 있지 않은 것으로 구성되며 따라서 레지던트가 자발적으로 공개해야만 드러난다. 매주 이뤄지는 피드백 교환에 대한 기대가 정기적으로 전달되었지만, 참가자들은 성과 목표는 거의 논의되지 않았으며, 이로 인해 대화는 스스로 밝힌 목표나, 레지던트가 느끼는 두려움, 한계를 중심으로 하지 않았다고 말했다.

The “hidden” quadrant comprises what is known to the resident but not to faculty or peers and therefore requires disclosure by the resident. While expectations about weekly feedback exchanges were communicated regularly, participants stated that performance goals were rarely discussed, resulting in conversations not guided by self-disclosed goals, fears or limitations.


로테이션 동안 집중하고자 하는 약점을 하나 또는 두 개 고른 인턴들이 있었다. 나는 그것이 전체 로테이션에 있어서 엄청나게 도움이 된다는 것을 발견했다.

I have had interns who pick one or two areas of weakness that they want to focus on for that rotation. I found that incredibly helpful in terms of the entire rotation.


같이 일하고 있는 사람에게 '저는 당신이 [내가 이 특정 영역에서 어떻게 하고 있는지]를 봐주면 좋겠습니다'는 식으로 물어보면 도움이 된다.(F)

It helps if you ask the person you’re working with, like ‘okay, I asked you to watch me in this particular domain and see how I’m doing’. (F)


피드백의 신뢰성은 피드백 제공자와 수신자 사이의 관계에 의해 영향을 받을 수 있다. 레지던트들은 자신이 신뢰하는 사람에게는 자신의 목표와 한계를 기꺼이 공개할 수 있으며, 이로서 더 집중적이고 실행 가능한 피드백이 가능해진다. 긍정적인 관계는 또한 더 많은 수행능력 관찰을 촉진할 수 있고, 피드백 데이터를 더 수용할 수 있게 할 수 있다.

The credibility of feedback could be influenced by the relationship between the feedback provider and receiver. Residents may be willing to disclose their goals, and limitations to those they trust, allowing for more focused and actionable feedback. Positive relationships can also stimulate more performance observation and make the feedback data more acceptable.


알 수 없었던 것을 발견하기

Uncovering the unknown


"unknown" 사분면은 레지던트는 물론 감독 교직원도 알 수 없는 영역을 가리킨다. 따라서, 동료들과 다른 직원들로부터 피드백을 구하는 것이 도움이 될 수 있다. 레지던트와 가까운 동료들은 교수는 언급하지 않았던 전혀 다른 기술에 대해 피드백을 제공했으며, 이들은 동료-피어 피드백 대화가 더 자주 이루어지기를 바란다고 말했다. 일부 입원 환자 주치의는 비의사 팀원의 피드백의 중요성을 강조했다. 왜냐하면, 이들은 의사-환자 상호 작용과 팀워크에 대해 다른 인식을 가지고 있기에 가치가 있기 때문이다.

The “unknown” quadrant refers to domains unknown to the resident as well as supervising faculty. Therefore, seeking feedback from peers and other staff could be helpful. Residents indicated that their near peers provided feedback on very different skills than faculty and wished peer–peer feedback conversations occurred more frequently. Some inpatient attendings emphasized the importance of feedback from nonphysician team members whose perceptions of physician–patient interactions and teamwork are likely to be different and valuable.


나는 인턴들이 교육, 런닝 라운드, 내가 얼마나 지지하는지, 그것이 우리와 어떻게 작용하는지에 대해 더 많은 피드백을 주기를 바란다. 이것은 주치의에게서 받을 수 있는 피드백과는 다르다 (R)

I wish interns give more feedback on the teaching, running rounds, how supportive I am, how it is working with us, the nitty-gritty. And it’s different feedback than I would get from an attending. (R)


피드백을 주는 기술에 대한 훈련은 레지던트와 교수 모두에 의해 중요하게 여겨졌다. 상호 전문성장의 목표를 강조하는 건설적인 피드백 제공을 위한 효과적인 기법이 제시되었다.

Training in the skills of giving feedback was considered important by residents and faculty. Effective techniques to providing constructive feed-back emphasizing the goal of mutual professional growth were suggested.



고찰

Discussion


우리의 조사 결과를 바탕으로, 우리는 Johari 창 모델이 교수와 레지던트 간의 피드백 대화, 그리고 기관의 피드백 이니셔티브 설계에 효과적으로 지침이 될 수 있다고 믿는다.

Based on our findings, we believe that the Johari window model can effectively guide feedback con-versations between faculty and residents, and the design of feedback initiatives by institutions


우리의 자료에 따르면 레지던트와 교수 모두 효과적인 피드백을 [단방향 하향식 과정이 아니라] [기관 문화에 의해 촉진되거나 억제되는 대인 커뮤니케이션]으로 보고 있다. 또한 임상 환경은 팀별로 환자 진료를 제공하기에, 최적의 기능을 위해서는 [자기 인식]과 [팀원들에 의한 자신의 행동에 대한 인식]이 모두 중요하다.

Our data suggest that both residents and faculty view effective feedback as an interpersonal communication facilitated or inhibited by the institutional culture, rather than a unidirectional top-down process.Additionally, the clinical environment is one where patient care is delivered by teams, where perception of self and perception of one’s behaviors by other team members are essential for optimal functioning. 


"open" 사분면은 개인 전문가와 팀이 가장 생산적이 될 수 있도록 한다. 성장 마인드를 자극하기 위해서는 강점과 결핍이 논의되어야 한다. 피드백 제공자의 경우, FR의 성찰을 장려하고, 자기평가를 하도록 요청함으로써 이 사분면을 확장시킬 수 있다. 그러나, 연구에 따르면 대부분의 사람들은 경험 재구성 시 긍정적인 데이터에 더 주의를 기울이기 때문에 부풀려진 자기 평가를 갖는 경향이 있다(Davis et al. 2006; Eva and Regehr 2008). 따라서, unguided reflection과 자기평가만으로는 피드백 수신자가 자신의 성능 수준을 부정확하게 보정할 수 있다(Kruger and Dunning 199).9). 자기성찰은 외부 소스의 성능 데이터를 통합할 때 더 정확하다(Boud 1995; 엡스타인 및 훈데르트 2002; 에바와 레게르 2008; 사리아 등). 2009; Mann et al. 2011; Sargeant et al. 2015).

The “open” quadrant enables individual professionals and teams to be most productive. Strengths as well as deficiencies need to be discussed to stimulate a growth mindset. For feedback providers, encouraging reflection and inviting self-assessment from receivers can expand this quadrant. However, research suggests that most people tend to possess inflated self-assessments as they pay more attention to positive data in reconstructing experiences (Davis et al. 2006; Eva and Regehr 2008); thus, unguided reflection and self-assessment can lead feedback receivers to inaccurately calibrate their performance level (Kruger and Dunning 1999). Self-reflection is more accurate when it incorporates performance data from external sources (Boud 1995; Epstein and Hundert 2002; Eva and Regehr 2008; Sargeant et al. 2009; Mann et al. 2011; Sargeant et al. 2015).


기관의 리더들은 [정보에 입각한 자기 평가]가 표준이 되는 문화를 촉진하고, 이러한 기술에 대한 훈련을 제공함으로써 개인과 팀을 위한 "open" 사분면을 넓히는 데 중요한 역할을 한다. 또한, supervisor는 학습자가 [각각의 단계마다 있어야 하는 곳에 대한 로드맵 세부사항]과 [성과에 대한 빈번하고 확실한 피드백]을 제시해야 한다.

Institutional leaders have a major role in promoting a culture where informed self-assessment becomes the norm, and providing training on these skills, thereby enlarging the “open” quadrant for individuals and teams.Additionally, faculty supervisors should present frequent,affirmative feedback on performance along with a roadmap detailing where learners need to be at different stages


"Blind" 사분면은 사람의 행동과 행동에 대한 무지에서 생긴다. 이 사분면을 축소하려면 다각적인 접근이 필요하다. 학습자는 [필요한 학습 목표를 달성하는 데 있어 수행의 적절성을 판단하기 위한 의식적인 노력]으로 정의되는 [능동적 피드백 탐색]을 통해 자기 인식의 사각지대를 깨달을 수 있다(Crommelinck 및 Anseel 2013). 피드백 탐색하는 행동은 유용한 정보에 대한 욕구에 의해서도 추진될 수 있고, 자신의 에고를 강화하거나, 긍정적 이미지를 유지하기 위해서 이뤄질 수도 있기 때문에, 어떤 목표를 지향하느냐에 의해 영향을 받을 수 있다(반데왈레 외). 2000; VandeWalle 외, 2001; Teunissen 외. 2009).

The “blind” quadrant arises out of ignorance about one’s behaviors and performance. A multipronged approach is needed to shrink this space. Learners can gain awareness of their blind spots through active feedback seeking, defined as one’s conscious efforts to determine adequacy of performance in attaining required learning goals (Crommelinck and Anseel 2013). Since feedback-seeking behavior can be driven by the desire for useful information, to enhance one’s ego, or maintain a positive image, it can be influenced by goal orientation of individuals (VandeWalle et al. 2000; VandeWalle et al. 2001; Teunissen et al. 2009).

  • performance goal orientation, 

  • learning goal orientation

기관들은 피드백 탐색 행동을 적극적으로 촉진하고, 훈련생과 직원들간의 성과목표지향보다는 학습목표지향을 강조하고, 이를 명확한 기대감으로 전달하며, 피드백을 탐색, 수용, 통합하는 전략을 포함하는 피드백 교육을 제공할 필요가 있다(Tunissen). (2013년 복)

Institutions need to establish a learning culture that actively promotes feedback-seeking behavior, emphasize learning goal rather than performance goal orientation among its trainees and staff, communicate these as clear expectations and provide feedback training that includes strategies for seeking, receiving and incorporating feedback (Teunissen and Bok 2013).


"Hidden" 사분면은 레지던트가 교수나 동료에게 드러내고 싶지 않아하는 정보, 감정, 두려움, 의제 또는 상황을 나타낸다. 얼마나 많은 것을 드러낼지는 상대방, 신뢰, 그리고 다른 사람들과의 관계에 달려있지만, 이 공간을 좁히기 위해서는 공개disclosure가 필요하다. 신속한 transition과 임상 의학의 짧은 로테이션으로 인한 많은 잠재적인 부정적인 영향들이 다음과 같이 설명되어 왔다. 

  • trainee와 faculty가 서로 신뢰할 수 있는 교육 또는 업무 관계를 수립할 수 없다

  • 서로의 동기, 헌신 또는 목표에 대한 지식이 부족하다

  • 과 평가와 피드백에 대한 기대의 공유된 정신적 모델이 부족하다

The “hidden” quadrant represents information, feelings, fears, agendas or context that a resident may hesitate to reveal to supervising faculty or peers. Disclosure is necessary to narrow this space, though how much to reveal would depend on the person, the trust and relationship with others. Many potential negative effects of rapid transitions and short rotations in clinical medicine have been described including 

  • inability of trainees and faculty to establish trusting educational or working relationships, 

  • lack of knowledge about each other’s motivation, commitment or goals, and 

  • lack of a shared mental model of performance appraisal and expectations for feedback (Bernabeo et al. 2011; Holmboe et al. 2011; Sargeant et al. 2011; Watling 2014a, 2014b; Sargeant et al. 2015; Dudek et al. 2016).


교육적 동맹 프레임워크는 학습자와 교사들 간의 지원적 관계를 촉진하여 학습자가 자신의 한계와 불안함을 쉽게 공개하고, 교사들은 두려움과 상황을 염두에 두고 학습자 목표에 피드백을 기초하는 의미 있는 피드백 교환을 초래할 수 있다(Telio et al. 2015; Telio et al.; Telm).io et al. 2016). 교육적 동맹을 구축하려면 학습자와 학습 과정에 대한 교사의 의지를 보여주는 대화가 필요하며, 공개가 가능한 관계를 만들어나감으로써 "hidden" 공간을 좁힐 수 있다. 교수진은 

  • 자신의 두려움과 한계를 인정하고, 

  • 학습자의 상황에 대한 이해를 보여주고, 

  • 목표 지향적인 피드백을 제공할 수 있다.

the educational alliance framework may promote supportive relationships between learners and teachers resulting in meaningful feedback exchanges where learners feel comfortable disclosing their limitations and insecurities, and teachers ground their feedback in learner goals while being mindful of their fears and the context (Telio et al. 2015; Telio et al. 2016). Establishing such an alliance necessitates conversations that demonstrate the teacher’s commitment to the learner and the learning process, and establish a relationship that permits disclosure, thus narrowing the “hidden” space. Faculty can 

  • acknowledge their own fears and limitations, 

  • demonstrate understanding of the learner’s context and 

  • provide goal-directed feedback.


마지막 사분면, "unknown" 공간은 레지던트, 감독자 또는 동료 모두 이러한 행동을 인식하지 못하기 때문에, [가장 다루기 어렵다].

The last quadrant, the “unknown” space, is the most challenging to address because neither the resident nor their supervisors or peers are aware of these behaviors.


이 영역을 축소하려면 다양한 접근방식이 필요하다. 여기에는 레지던트 스스로 새로운 자기 영역 발견할 준비가 되어야 하고, 서로 다른 supervisor가 직접 성과를 관찰해야 하며, 다중 소스 피드백이 필요하다. 간호사 또는 사례 관리자 및 환자와 같은 감독자, 동료 및 비의사 팀원은 모두 performance의 서로 다른 차원에 대해 인지하고 통찰력을 제공할 가능성이 있고, 이는 궁극적으로 레지던트의 전문적 개발을 강화한다(Sargeant et al. 2005; Ten Cate and Sargeant 2011; Holmboe and Ross 2012; Sar Sar 2015; Sargeant 2015; Van der Meulen et al. 2).017). 그러나 의사는 의사가 아닌 사람의 피드백에 대해 신뢰성이 떨어지고 덜 유익하다고 보는 경향이 있어서, 결국 피드백을 덜 수용하는 경향이 보이는 것으로 보고되었다(Sar et et al. 2005, 2007; Ten Cate and Sargeant 2011).

Shrinking this area would involve a multitude of approaches such as residents’ readiness to discover new areas of self, direct performance observation by different supervisors and multisource feedback. Supervisors, peers and nonphysician team members, such as nurses or case managers and patients, are all likely to perceive and provide insights on different dimensions of performance, thus enhancing professional development of residents (Sargeant et al. 2005; Ten Cate and Sargeant 2011; Holmboe and Ross 2012; Sargeant 2015; van der Meulen et al. 2017). However, it has also been reported that physicians tend to view nonphysician sources of feedback as less credible and beneficial, therefore less receptive to their feedback (Sargeant et al. 2005, 2007; Ten Cate and Sargeant 2011).


추가 연구를 위한 제안

Suggestions for further research


마지막으로, 우리는 Johari 프레임워크를 통합한 피드백 교육을 설계하고 그것이 교직원과 레지던트의 피드백 행동에 미치는 영향을 연구하는 것을 목표로 한다.

Finally, we aim to design feedback training incorporating the Johari framework and study its impact on feedback behaviors of faculty and residents.


결론들

Conclusions


  • 두려움와 한계을 드러내는 것을 정상화normalize시키고, 

  • 피드백 탐색을 장려하며, 

  • 비판단적 피드백을 주는 법을 훈련시키고, 

  • 종적 관계를 위한 기회를 제공하면, 

...궁극적으로 조하리 창의 "open" 사분면을 극대화할 수 있는 자아 발견을 촉진할 수 있다.

Normalizing disclosure of fears and limitations, encouraging feedback seeking, training in providing nonjudgmental feedback and providing opportunities for longitudinal relationships can enhance self-awareness and promote self-discovery which can ultimately maximize the “open” quadrant of the Johari window.



Ramani S. 2016. Reflections on feedback: closing the loop. Med Teach. 38:206–207.







 2017 Oct;39(10):1065-1073. doi: 10.1080/0142159X.2017.1353071. Epub 2017 Jul 25.

Uncovering the unknown: A grounded theory study exploring the impact of self-awareness on the culture of feedback in residency education.

Author information

1
a Department of Medicine , Brigham and Women's Hospital, Harvard Medical School , Boston , MA , USA.
2
b School of Health Professions Education , Maastricht University , Maastricht , The Netherlands.
3
c Dalhousie University , Halifax , Canada.

Abstract

AIM:

Self-assessment and reflection are essential for meaningful feedback. We aimed to explore whether the well-known Johari window model of self-awareness could guide feedback conversations between faculty and residents and enhance the institutional feedback culture.

METHODS:

We had previously explored perceptions of residents and faculty regarding sociocultural factors impacting feedback. We re-analyzed data targeting themes related to self-assessment, reflection, feedback seeking and acceptance, aiming to generate individual and institutional feedback strategies applicable to each quadrant of the window.

RESULTS:

We identified the following themes for each quadrant: (1) Behaviors known to self and others - Validating the known; (2) Behaviors unknown to self but known to others - Accepting the blind; (3) Behaviors known to self and unknown to others - Disclosure of hidden; and (4) Behaviors unknown to self and others - Uncovering the unknown. Normalizing self-disclosure of limitations, encouraging feedback seeking, training in nonjudgmental feedback and providing opportunities for longitudinal relationships could promote self-awareness, ultimately expanding the "open" quadrant of the Johari window.

CONCLUSIONS:

The Johari window, a model of self-awareness in interpersonal communications, could provide a robust framework for individuals to improve their feedback conversations and institutions to design feedback initiatives that enhance its quality and impact.

PMID:
 
28741446
 
DOI:
 
10.1080/0142159X.2017.1353071


사회문화적 렌즈를 통해 본 의미있는 피드백(Med Teach, 2019)

Meaningful feedback through a sociocultural lens

Subha Ramania,b , Karen D. K€oningsc , Shiphra Ginsburgd and Cees P. M. van der Vleutenc





도입

Introduction


한때는 static한 교사에서 학생으로의 독백으로서 피드백 대화는 역동적인 학습자 중심의 협력적 대화로 옮겨가고 있다.

The feedback conversation, once a static teacher to learner monologue, is shifting towards a dynamic learner-directed collaborative dialogue.


본 가이드의 목적

The purpose of this guide


이 가이드의 주된 목적은 교사 및 학습자가 의미 있고 실행 가능한 피드백 대화에 참여하고 기관이 성장 지향 피드백 문화를 확립하도록 돕는 것이다.

The primary aim of this guide is to assist teachers and learners in engaging in meaningful and actionable feedback conversations, and institutions in establishing a growth-oriented feedback culture.


조직문화 및 피드백

Organisational culture and feedback


문화는 피드백에 대한 사회문화적 관점의 중심이기 때문에, 우리는 Edgar Schein에 의해 묘사된 세가지 수준의 문화를 언급한다.

Since culture is central to sociocultural perspectives of feedback, we refer to the three levels of culture described by Edgar Schein,


(1) 가장 깊은 레벨은 암묵적 가치쓰여지지 않은 가정을 말하며, 

(2) 다음 레벨은 쓰여진 기대와 사명을, 

(3) 가장 눈에 띄는 레벨은 일상적인 행동을 묘사한다(Schein 2017).

(1) the deepest level refers to implicit values and unwritten assumptions, 

(2) the next level refers to written expectations and mission statements and 

(3) the most visible level depicts day to day behaviour (Schein 2017).


이러한 수준을 피드백 문화(그림 1)에 적용하면, 

    • 가치와 가정은 기관에서 일이 어떻게 이루어지고 새로운 구성원이 어떻게 적응하는지를 말하며, 

    • 다음 단계는 피드백에 대한 서면 기대를 나타내고, 

    • 가시적 수준은 교사와 학습자의 피드백 행동을 가리킨다. 

이상적으로, 이 세 가지 레벨은 서로 협력하여 작용하는데, 실제로는 서로 모순될 수 있다.

Applying these levels to feedback culture (Figure 1), 

    • values and assumptions refer to how things are done at the institution and how new members are acculturated, 

    • the next level refers to written expectations for feedback, and 

    • the visible level refers to feedback behaviours of teachers and learners. 

Ideally, the three levels would act in concert, in reality they may contradict each other.




사회문화적 현상으로서의 피드백

Feedback as a sociocultural phenomenon


피드백에 영향을 미치는 사회문화적 요인으로는 교사 특성, 교사-학습자 관계, 수행의 관찰, 학습자 자기효율성, 자율성, 피드백 탐색 및 수용, 피드백 데이터의 신뢰성 인식이 포함된다(Sargeant et al. 2007a; Delva et al. 2011; Mann et al. 2011; Eva et al. 20).12; Watling et al. 2013a, 2014a, 2014b, 2016; Ramani, et al. 2017a, 2017b, 2019). 자체 평가와 성찰은 피드백 추구와 수용의 핵심이다 (Sarar et al. 2008b, 2009).K€onings 등 2016). 의학 교육에서, 교수들은 우호적인 업무 관계를 유지하고 학습자의 자부심 및 자율성에 대한 위협을 주지 않기 위해 모호한 피드백 언어를 사용하는 것이 일반적이다(Sargeant et al. 2007, 2008a; Van de Ridder et al. 2015a, 2015b; Ramani et al. 2018).

Sociocultural factors that influence feedback include: teacher characteristics, teacher-learner relationships, observation of performance, learner self-efficacy, autonomy, feedback seeking and acceptance, and perceived credibility of feedback data (Sargeant et al. 2007, 2008a; Delva et al. 2011; Mann et al. 2011; Sargeant et al. 2011; Eva et al. 2012; Watling et al. 2013a, 2014; Watling 2014a, 2014b, 2016; Ramani, et al. 2017a, 2017b, 2019). Self-assessment and reflection are also central to feedback seeking and acceptance (Sargeant et al. 2008b, 2009;K€onings et al. 2016). In medical education, use of vague feedback language is common as faculty wish to avoid threats to friendly working relationships, learner self-esteem and autonomy (Sargeant et al. 2007, 2008a; van de Ridder et al. 2015a, 2015b; Ramani et al. 2018)


핵심 이론 원리를 피드백에 연결

Linking key theoretical principles to feedback


.사회문화론, 예의론, 자기결정론. (그림 2)

.Sociocultural theory, Politeness theory and Self-determination theory. (Figure 2).



사회문화이론 및 피드백

Sociocultural theory and feedback


이 이론은 학습이 상호작용, 협상, 협업을 통해 일어난다고 제안한다(Lave and Wenger 1991). 임상 학습은 팀원들이 다른 팀원들로부터 배우고 다른 팀원들과 함께 발전하는 커뮤니티에서 이루어지기 때문에, 기관들은 학습자 능력들 사이에서 팀워크 기술을 우선시해야 한다.

This theory proposes that learning occurs through interaction, negotiation, and collaboration (Lave and Wenger 1991). Since clinical learning occurs in a community where team members learn from and develop with others, institutions must prioritise teamwork skills among learner competencies.


이 이론을 피드백에 적용하면, 기관은 교사 및 학습자가 다음을 수행할 수 있는 명백한 기대를 확립할 수 있다. 

    • 학습 목표를 논한다 

    • 성찰에 참여한다. 

    • 관찰된 성과와 예상된 성과 사이의 gap을 보정한다

    • 형성적 피드백을 교환하여 gap을 좁힌다

성인 학습자들은 코치 역할을 하는 교사들과 이러한 대화를 이끌도록 장려되어야 한다.

Applying this theory to feedback, institutions can establish explicit expectations for teachers and learners to: 

    • discuss learning goals, 

    • engage in reflection, 

    • calibrate gaps between observed and expected performance and 

    • exchange formative feedback to narrow the gap. 

Adult learners should be encouraged to lead these conversations with teachers acting as coaches.


공손성 이론 및 피드백

Politeness theory and feedback


공손성 이론은 많은 대화가 듣는 사람이나 말하는 사람에게 잠재적인 체면face을 위협하는 행위라고 가정한다(Brown and Levinson 1987). 체면은 긍정적 체면(긍정적 이미지를 투영할 필요성)과 부정적 체면(폐를 끼치지 않고 행동할 자유)으로 분류된다. 임상 훈련에서 긍정적인 체면은 학습자의 자기효능감으로, 부정적인 체면은 학습자의 자율성으로 볼 수 있다.

Politeness theory assumes that many conversations are potential face-threatening acts to the hearer or speaker (Brown and Levinson 1987). Face is categorised as positive (the need to project a positive image) and negative (freedom to act without imposition). In clinical training, positive face can be viewed as learner self-efficacy and negative face as learner autonomy.


학습자들은 건설적인 피드백을 "부정적"이며 기대되는 공손함의 규범을 위반하는 것으로 볼 수 있다. 진정한 건설적 피드백은 학습자의 자기 효율성에 영향을 미치더라도 종적 성장을 위해 필수적이다. 그러나 임상 교사들은 주로 긍정적인 언어(좋은 직업, 함께 일하는 즐거움)를 사용하는 경향이 있다.

learners may view constructive feedback as “negative” and a breach of the norms of expected politeness. Honest constructive feedback is essential for longitudinal growth, even if affects learners’ self-efficacy. However, clinical teachers tend to use predominantly positive language (good job, pleasure to work with)


이러한 경향은 [자부심, 자기효율성 및 자율성에 대한 관심]이 [건설적인 서술과 피드백을 방해]하는 '예의바름'개념과 연계될 수 있다(Ginsburg honor et al. 2015, 2016; Ramani et al. 2017b, Ramani et al. 2019).

this tendency can be linked to politeness concepts where attention to self-esteem, self-efficacy and autonomy impedes constructive narratives and feedback (Ginsburg honest et al. 2015, 2016; Ramani et al. 2017b, Ramani et al. 2019).



자기결정론 및 피드백

Self-determination theory and feedback


자기결정론은 역량, 관련성, 자율성 등 내재적 동기를 강화하는 3가지 핵심 니즈를 제안한다(Ryan and Deci 2000, 2017). 학습 문화는 내적 동기부여를 강화할 방법을 적극적으로 모색해야 한다(Mann 2002년 4월 Ten Cate 등.은 피드백 대화 중 동기부여를 위한 세 가지 접근방식을 제안한다. 

    • 개인에서 맥락으로 전환

    • 지시적 메시지에서 자기조절로 전환 

    • 피드백 제공자에서 신자로 전환 (Ten Cate et al. 2011; Ten Cate 2013). 

Self-determination theory proposes three key needs that enhance intrinsic motivation: competence, relatedness and autonomy (Ryan and Deci 2000, 2017). the learning culture should actively seek to enhance intrinsic motivation (Mann 2002). Ten Cate et al. suggest three approaches to boost motivation during feedback conversations: 

    • shift the focus from the individual to the context; 

    • shift from instructional messages to self-regulation; and 

    • shift the focus from feedback provider to recipient (Ten Cate et al. 2011; Ten Cate 2013). 


[관찰된 데이터에 기초한 피드백과 facilitated reflection은] 교사와 학습자 사이의 종단적 관계가 촉진되는 맥락에서 발생해야 하며, 학습자는 [능력과 직무 복잡성]에 따라 자율성을 높여야 한다(Ramani, et al. 2018, 2019).

Feedback, grounded in observed data and facilitated reflection, should occur in a context where longitudinal relationships between teachers and learners are promoted, and learners are provided increasing autonomy based on ability and task complexity (Ramani, et al. 2018, 2019).


자기 자신과 다른 것의 균형: 성장 피드백을 향상시키는 모델

Balancing the self and the other: A model for growth enhancing feedback


피드백 신뢰성은 '자기'와 '기타'와 관련된 요인 사이에서 최적의 균형을 요구할 수 있다. 자신과 다른 사람들은 학습 문화뿐만 아니라 조직적 문화를 특징으로 하는 제도적 맥락에서 공존한다.

Feedback credibility may require an optimal balance of factors related to ‘self’ and ‘other’. The self and other co-exist in an institutional context which features an organisational culture as well as a learning culture.


자기 팩터 및 피드백(학습자)

Self-factors and feedback (learner)


그림 3

Figure 3.


마음가짐. Mind-set.


    • 고정형 마음가짐을 가진 사람들은 성공은 지성과 재능과 같은 선천적인 특성의 결과라고 믿고, 그들의 능력을 보여주는 데 초점을 맞추고 실패를 능력에 대한 부정적인 표현으로 인식한다. 그들은 건설적인 피드백을 얻거나 받는 것을 피할 것 같다. 

    • 성장형 마음가짐을 가진 사람들은 능력이 열심히 일하고 지속적인 학습을 통해 발전될 수 있다고 믿는다. 성장 마인드셋은 학습자가 건설적인 피드백을 찾고 받아들이도록 하고, 한계를 지식과 기술을 확장하는 기회로 공개하며, 실패를 능력에 대한 모욕이 아닌 학습 기회로 볼 수 있게 할 것이다.

    • People with a fixed mind-set believe that success is a result of innate traits such as intelligence and talent, focus on showcasing their skills and perceive failure as a negative statement of their abilities. They are likely to avoid seeking or accepting constructive feedback. 

    • Those with a growth mind-set believe that abilities can be developed through hard work and ongoing learning. A growth mind-set would allow learners to seek and accept constructive feedback; disclose limitations as opportunities to expand knowledge and skills; and view failure as a learning opportunity rather than an insult to their abilities.



목표 지향성. Goal-orientation.


[무력함을 보이는 개인]과 [어려운 과제에 직면했을 때 대처 전략을 사용하는 개인] 사이의 차이는 다음 두 가지 유형의 성취 목표에 기인한다. 

    • (1) 직무 관련 성취목표: 개인이 자신의 능력과 비교하여 능력을 개발하고자 하는 경우 

    • (2) 에고 관련 성취목표: 개인이 타인과 비교하여 역량 개발을 모색하는 경우

Differences between individuals who exhibit helplessness and those who use coping strategies when facing difficult tasks are attributed to two types of achievement goals: 

    • (1) task involvement, where individuals seek to develop competence relative to their abilities, and 

    • (2) ego involvement, where individuals seek to develop competence relative to others


학습 목표 지향성과 성과 목표 지향성 두 가지 유형이 제안되었다.

    • 과 목표 지향성을 가진 전문가들은 좋은 인상을 주는 데 초점을 맞출 수 있으며 한계를 드러내고 이미지를 위협할 수 있는 피드백을 반기지 않는다. 

    • 습 목표 지향성을 가진 전문가들은 자신의 분야에서 숙달력을 달성하는 데 초점을 맞추고 있으며 건설적인 피드백을 구하고 받아들일 가능성이 더 높다. 

Two types of goal orientation were proposed:learning goal-orientation and performance goal-orientation.

    • Professionals with a performance goal-orientation may focus on creating a good impression and not welcome feedback that could reveal limitations and threaten their image. 

    • Professionals with a learning goal-orientation focus on achieving mastery in their field and are more likely to seek and accept constructive feedback. 


자기 인식. Self-awareness.


자기인식은 여러가지 상황에서 자신의 행동을 분석하고 정확하게 평가할 수 있는 능력을 말한다. 자기인식 수준은 혼란(완전히 자기인식 없음)에서 메타 자기인식(남의 눈을 통해 어떻게 보이는지 까지를 인식)에 이르기까지 다양하다(Dweck 2017).

Self-awareness refers to one’s capacity for analysis and accurate appraisal of own behaviour in different situations. Levels of self-awareness range from confusion (complete unawareness of self) to meta self-awareness (aware of how one is seen through others’ eyes) (Dweck 2017).


심리학자 루프트와 인함(Ingham)에 의해 묘사된 조하리 창은 인간 상호작용에서 자아와 다른 사람들에 의한 행동의 인식의 다른 수준을 그린 2x2 모델이다(Luft 1969). 이 창문에는 네 개의 사분면이 있다. 

    • 자기와 다른 사람들에게 알려져 있다. 

    • 다른 사람에게는 알려져 있지만 자기에게는 알려지지 않은 맹목적인, 

    • 자기에게는 알려져 있지만 다른 사람에게는 알려져 있지 않은, 그리고 

    • 자기와 다른 사람들에게 알려지지 않은 것(그림 4). 

The Johari window, described by psychologists Luft and Ingham, is a two-by-two model that depicts different levels of awareness of behaviours by self and others in human interactions (Luft 1969). The window features four quadrants: 

    • open: known to self and others, 

    • blind: known to others but unknown to self, 

    • hidden: known to self but unknown to others, and 

    • unknown: unknown to self and others (Figure 4). 


자기인식을 강화하기 위해 학습자는 

    • 자신이 인식하지 못한 행동에 대해 학하기 위하여 피드백을 구하고, 

    • 교사가 상황에 따라 정확한 피드백을 제공할 수 있도록 자신의 한계를 공개해야 하며, 

    • 알려지지 않은 영역을 밝혀내기 위해, 복수의 출처의 성과 데이터를 수용함으로써 자아 발견에 나서야 한다(Ramani et al. 2017a).

To enhance self-awareness, learners should 

    • seek feedback to learn about behaviours that they are unaware of, 

    • disclose limitations for teachers to provide accurate feedback based on context, and 

    • engage in self discovery through acceptance of performance data from multiple sources to bring unknown areas to light (Ramani et al. 2017a).



자기효능. Self-efficacy.


학문적 환경에서 이전의 성과는 강력한 자기 능력의 원천이며, 도전을 극복하도록 동기를 부여해준다(Bandura 1977, 1997). 이러한 원칙을 피드백에 적용하면, 자기 효율성이 향상되는 학습자들은 더 자기인식을 잘 하게 되고, 내부적으로 도전에 직면할 동기가 생기며, 실천 과정에서 장애를 극복하고 성공할 수 있다. 따라서, 그들은 자기 평가, 피드백 찾기 및 수락, 성과에 대한 성찰 및 행동 변화에 더 많이 관여할 가능성이 높다.

In academic settings, previous accomplishments are a powerful source of selfefficacy and motivate professionals to overcome challenges (Bandura 1977, 1997). Applying these principles to feedback, learners with increased self-efficacy may become more self-aware, internally motivated to face challenges, overcome obstacles and succeed in their practice. Thus, they are more likely to engage in self-assessment, feedback seeking and acceptance, reflection on performance and committment to behaviour change.


자율성 Autonomy.


독립적으로 행동하고자 하는 욕구인 자율성은 내적 동기, 창조적 사고와 행동에 큰 영향을 미친다(Deci와 Ryan 1985, Ryan과 Deci 2017). 자율적인 업무 환경은 내재적 동기를 강화시키는 반면 상황을 통제하는 것은 동기부여를 저해할 수 있다. 긍정적인 피드백은 학습자에게 자기효능감을 줄 수 있지만, [구체적인 건설적 피드백과 개선을 위한 구체적인 실행 계획]이 independent practice에 중요하기 때문에, 모호한 긍정적인 평가만으로는 자율성을 서포트 해주지 못한다(Kusurkar et al 2011a, 2011b).

Autonomy, the desire to act independently, has a major impact on internal motivation, creative thinking and action (Deci and Ryan 1985; Ryan and Deci 2017). Autonomy-supportive work environments can enhance intrinsic motivation while controlling contexts could undermine motivation. Although positive feedback can affirm learners’ self-efficacy, vague positive comments alone are not autonomy-supportive since specific constructive feedback and concrete action plans for improvement are crucial for independent practice (Kusurkar et al 2011a, 2011b).


기타 요인- 교사와 기관

Other factors- The teacher and institution


그림 5는 학습 문화뿐만 아니라 조직적 문화로 구성된 제도적 맥락 안에 공존하는 자기 팩터와 기타 팩터의 제안된 모델이다.

Figure 5 is a proposed model of self-factors and other-factors co-existing within an institutional context composed of an organisational culture as well as a learning culture.



교사 관련 요인 

Teacher-related factors


자기 효능감을 고려한다. Addressing self-efficacy.


자기효능감은 도전을 직면하려는 내면적 동기와 의지를 증가시키며, 이전의 성취로부터 발생한다(Bandura 1977; Lane et al. 2004). 교사는 새로운 과제에 직면했을 때 학습자가 이전의 직업적 성공을 회상하고 장애물을 탐색하는 것을 도울 수 있다. 교사들은 그들이 성장하도록 돕기 위해 학습자들에게 도전할 뿐만 아니라 그들을 지원할 필요가 있다. 안내되지 않은unguided 자기 평가는 부정확한 경우가 많고(Reger, Eva 2006; Eva 및 Regehr 2008, 2011), 학습자는 자기 평가와 상충되는 피드백을 거부하기 때문에 (Mann 등, 2011), 교사들은 [다양한 출처의 데이터를 사용]하여 [원활한 성찰]과 [정보에 입각한 자기 평가]를 통해 학습자가 성과를 정확하게 calibration하도록 지도할 수 있다(B 1995 1995; Sargeant et al. 2008b, 2010).

Self-efficacy increases internal motivation and willingness to face challenges and arises from previous accomplishments (Bandura 1977; Lane et al. 2004). Teachers can help learners recall previous professional successes and navigate obstacles when faced with new tasks. Teachers need to support as well as challenge learners to help them grow (Daloz and Daloz 1999). Since unguided self-assessment is often inaccurate (Regehr and Eva 2006; Eva and Regehr 2008, 2011) and learners reject feedback that conflicts with their self-assessment (Mann et al. 2011), teachers can guide accurate calibration of learner performance through facilitated reflection and informed self-assessment using data from multiple sources (Boud 1995; Sargeant et al. 2008b, 2010).


요약하면, 선생님들은 다음을 통해 학습자의 자기 효능감을 향상시킬 수 있다. 

    • 이전의 성공 사례에 대해 토론

    • 성공에 기여한 행동 강화(지원) 

    • 그들에게 새롭고 복잡한 과제(문제)를 해결하도록 격려

    • 새로운 작업에서 성공하는 데 도움이 되는 건설적인 피드백을 제공

In summary, teachers can enhance learners’ self-efficacy by 

    • discussing previous successes, 

    • reinforcing behaviours that contributed to success (support), 

    • encouraging them to tackle new and complex tasks (challenge), and 

    • providing constructive feedback to assist them in succeeding at new tasks.


자율성 제공. Providing autonomy.


감독과 자율성의 균형을 맞추는 것은 대부분의 임상 교사들에게 어려운 일이지만, 안전한 환자 관리를 위해서는 이 균형이 필요하다. 지도guidance와 자기조절 사이의 균형을 최적화하는 것은, 교육과 학습 사이의 '건설적 마찰(Vermunt and Verloop 1999)'이라 불리며, 피드백과 성찰을 통해 발생할 수 있다. 교사와 학습자가 학습 환경을 공동 제작하는 [참여적 설계]는 학습자의 태도, 동기 부여, 헌신에 긍정적인 영향을 미친다(K€onings et al. 2005, 2014). 교사들은 학습 목표를 수립하고, 성찰을 촉진하며, 피드백을 제공하고, 다음 단계로 진행하기 위한 행동 계획을 시작하도록 요청함으로써 학습자가 학습의 소유권을 갖도록 권장할 수 있다. 

Balancing supervision and autonomy challenges most clinical teachers, but safe patient care mandates this balance. Optimising the balance between guidance and self-regulation, termed ‘constructive friction’ between teaching and learning (Vermunt and Verloop 1999), can occur through feedback and reflection. Participatory design, where teachers and learners co-create learning environments, has a positive effect on learner attitudes, motivation and committment (K€onings et al. 2005, 2014). Teachers can encourage learners to take ownership of their learning by asking them to formulate learning goals, facilitating reflection,providing feedback and asking them initiate action plans for progression to the next level. 


자기인식 향상 Enhancing self-awareness.


[자신의 행동을 정확하고 객관적으로 교정할 수 있는 능력]인 자기 인식은 개개인이 새로운 기술과 능력을 갖도록 돕는다(벤바삿과 바우말 2005). 자기인식은 내면(자기 내적 상태 인식)와 외면(타인에 대한 영향 인식)의 두 가지 측면을 갖는 역동적 과정이다(Goleman 1998).

Self-awareness, the ability to calibrate one’s behaviours accurately and objectively, helps individuals gain new skills and competencies (Benbassat and Baumal 2005). It is a dynamic process with two facets: internal (recognising own inner state), and external (recognising impact on others) (Goleman 1998).


임상 교육 환경에서 자기 인식은 독립적으로 발달하는 것이 아니라, 타인과 함께 발전한다(Rochat et al. 2012). 자신의 능력, 가정, 반응에 대한 인식은 학습자가 자신의 행동이 (환자를 포함한) 다른 사람들에 의해 어떻게 인식되는지 이해하는 데 도움이 될 수 있다.

In clinical education settings, self-awareness develops with others rather than independently (Rochat et al. 2012). Awareness of one’s abilities, assumptions and reactions can help learners understand how their actions are perceived by others including patients


교사들이 학습자의 자아 인식을 발전시키기 위해 사용할 수 있는 전략에는 다음이 포함된다. 

    • 피드백 대화를 위한 Johari 창 모델 적용(Luft 1969; Ramani et al. 2017a) 

    • 교육동맹을 발전

    • 퍼포먼스에 대한 성찰을 촉진

    • 어려운 임상 상황을 디브리핑, 

    • 자신의 도전과 한계를 공유하고 자아-발견을 촉진

Strategies that teachers can use to develop learner self-awareness include: 

    • application of the Johari window model to feedback conversations (Luft 1969; Ramani et al. 2017a), 

    • developing an educational alliance, 

    • facilitating reflection of performance, 

    • debriefing challenging clinical situations, 

    • sharing own challenges and limitations and stimulating self-discovery (Sargeant et al. 2008a, 2008b, 2010, 2015; Sargeant 2015; Telio et al. 2015, 2016; Armson et al. 2019).




기관 관련 요인

Institution-related factors


성장형 마음가짐 촉진. Fostering growth mind-set.


기관은 학습자가 도전과 좌절에 대처하고, 건설적인 피드백을 구하고 받고, 실패를 학습 기회로 다룰 수 있도록 성장 마인드셋을 육성할 수 있다. 임상 훈련에서 종적 관계가 결여되면 교육 동맹의 구축과 정직한 건설적 피드백을 교환이 어려워진다(Sargeant et al. 2008a, 2008b; Watling et al. 2013a; Watling 2014a; Van de Ridder et al. 2015a, 2015b). 그러므로 기관들은 [성장형 마인드셋]을 통해 교사-학습자 관계에 대한 기회를 창출하고, 건설적인 피드백의 교환을 정상화하며, 퍼포먼스 개선을 우선시해야 한다.

Institutions can foster a growth mind-set so that learners can cope with challenges and setbacks, seek and receive constructive feedback and treat failure as a learning opportunity. Lack of longitudinal relationships in clinical training makes it challenging to build educational alliances and exchange honest constructive feedback (Sargeant et al. 2008a, 2008b; Watling et al. 2013a; Watling 2014a; van de Ridder et al. 2015a, 2015b). Therefore, institutions should create opportunities for longitudinal teacher-learner relationships, normalise exchange of constructive feedback, and prioritise performance improvement through a growth mind-set.


학습 목표 지향성 촉진. Fostering learning goal-orientation.


학습 목표 지향을 가진 학습자는 피드백을 구하고 결함을 시정하고 미래의 성과를 개선하기 위한 수단으로 건설적인 피드백을 받아들일 수 있다(Sargeant et al. 2011; Van de Ridder et al. 2014). 성과목표지향은 '회피(실패를 회피성)와 증명(역량 증명)'으로 분류되며, 학습'의'평가를 학습을'위한' 평가보다 강조하는 총괄적 평가에서 더 자주 발생한다 (VandeWalle 및 Cummings 1997).

A learning goal orientation allows learners to seek feedback and accept constructive feedback as a means of correcting deficiencies and improving future performance (Sargeant et al. 2011; van de Ridder et al. 2014). Performance goal orientation, further categorised as avoidant (avoidance of failure) and prove (proving competence), dominates in a learning environment that emphasises summative assessments and ‘of’ rather than ‘for’ learning (VandeWalle and Cummings 1997).


기관은 [정기적인 성과 기반 피드백을 통해] 학습자가 자기이미지를 관리하는 것이 아니라, 업무의 역량과 숙달성에 초점을 맞추도록 할 수 있으며, 학습자 중심의 형성적평가 문화를 육성할 수 있다. 프로그램 평가는 그러한 평가 접근방식을 권장하는 모델이다(van der Vleuten et al. 2012). 교사, 학습자 및 기관장이 공동형성한 학습 환경은 사전 예방적 피드백 추구, 건설적 피드백 수용 및 양방향 피드백의 문화를 촉진할 수 있다(K€onings et al. 2005, 2014).

Institutions can foster a learner-centred formative assessment culture by focussing on competence and mastery of tasks rather than self-image, and through regular performance-based feedback. Programmatic assessment is a model that encourages such an approach to assessment(van der Vleuten et al. 2012). Learning environments co-created by teachers, learners and institutional leaders could promote a culture of proactive feedback seeking, acceptance of constructive feedback and bidirectional feedback (K€onings et al. 2005, 2014).





학습자, 교사 및 기관 피드백 요소의 균형을 최적화하는 실용적인 전략

Practical strategies to optimise the balance of learner, teacher and institutional feedback factors


피드백이 효과적인 것으로 간주되려면 학습자의 행동에 영향을 미쳐야 하지만(Delva et al. 2011, Moloy and Boud 2013, Boud 2015), 사회문화적 요인은 피드백 신뢰성과 수용성에 대한 인식에 강하게 영향을 미친다(Bing-You et al. 1997, Sarling et al. 2007, 2008a, 2011; Watchling 2014a, 2014b; Van de Rid et al. 2014).15a; 빙유 외 2017).

To be considered effective, feedback should impact learner behaviour (Delva et al. 2011; Molloy and Boud 2013; Boud 2015), but sociocultural factors strongly influence perceptions of feedback credibility and acceptance (Bing-You et al. 1997; Sargeant et al. 2007, 2008a, 2011; Watling et al. 2013a; Watling 2014a, 2014b; van de Ridder et al. 2014, 2015a; Bing-You et al. 2017).


이 섹션에서는 교사 및 학습자 전략이 결합되어 학습 환경의 공동 생성뿐만 아니라 교사-학습자 관계를 강조한다(그림 6).

in this section teacher and learner strategies are combined to highlight teacher-learner relationships as well as co-creation of the learning environment (Figure 6).



교사와 학습자가 의미 있는 피드백을 공동 작성하는 전략

Strategies for teachers and learners to co-create meaningful feedback


교육 동맹. Educational alliances.


교사 기법에 초점을 맞춘 피드백 이니셔티브는 교사와 학습자 사이의 피드백 적합성과 품질에 대한 인식의 불일치를 해결하지 못했다(Bing-You et al. 1997; Sender Liberman et al. 2005; Bing-You and Trowbridge 2009; BingYU et al. 2017). 새로운 모델은 학습자를 피드백 교환의 중심에 배치하고, 적합한 교사 학습자 교육 제휴를 강조한다(Telio et al. 2015, 2016).

Feedback initiatives that focus on teacher techniques have not resolved the mismatch of perceptions between teachers and learners on the adequacy and quality of feedback (Bing-You et al. 1997; Sender Liberman et al. 2005; Bing-You and Trowbridge 2009; BingYou et al. 2017). Newer models place learners at the heart of feedback exchanges and emphasise a congenial teacher learner educational alliance (Telio et al. 2015, 2016).


대인 커뮤니케이션 이론은 일대일 커뮤니케이션에서 네 가지 핵심 요소를 강조한다. 

    • 의사소통의 당사자,

    • 의사소통의 맥락, 

    • 당사자간의 관계 

    • 의사소통의 내용(Bylund et al. 2012).

Interpersonal communication theories emphasise four key elements in one-on-one communications: 

    • persons communicating, 

    • context of the communication, 

    • relationship between the persons and 

    • content of the communication (Bylund et al. 2012).


상자 3의 예시 대화

a sample conversation in Box 3:


    • 학습자와의 관계 설정

    • 학습자가 학습 목표를 논의하도록 권장

    • 자신의 강점과 해결할 과제에 대한 자체 평가 추진

    • 자기성찰 촉진

    • 강화 및 건설적 피드백 제공

    • 감정적 반응 인정 및 설명

    • 개선을 위한 실행 계획을 수립하도록 학습자를 가이드


    • Establish rapport with learners

    • Encourage learners to discuss their learning goals

    • Promote self-assessment on strengths and challenges

    • Facilitate self-reflection

    • Provide reinforcing and constructive feedback

    • Acknowledge and address emotional responses

    • Guide learners in formulating action plans for improvement


코칭은 [교육자가 학습자를 독립적 실무와 전문적 성장으로 이끌 수 있도록 돕기 위하여] 보건전문직 교수개발에 점점 더 많이 포함됨

Coaching, is increasingly included in faculty development for health professions educators to help them guide their learners towards independent practice and professional growth


R2C2 모델은 피드백 대화 중에 코칭 전략의 관계와 사용을 강하게 강조한다(Sargeant et al. 2015, 2017, 2018).

The R2C2 model strongly emphasises relationships and use of coaching strategies during feedback conversations (Sargeant et al. 2015, 2017, 2018).



학습자의 자존심과 자율성을 다루는 것. Addressing learner self-esteem and autonomy.


대부분의 대인관계를 (화자나 수령자의) '체면'에 대한 잠재적 위협으로 보는 예의 이론은 피드백 대화와 관련이 있다(Brown and Levinson 1987). 임상 교사는 학습자의 자기효율성(긍정적 체면)과 자율성을 위협하는 행동(부정적 체면)을 손상시킬 수 있는 언어를 피하는 경향이 있으며, 학습자는 학습자의 신뢰성이 의심되거나 정보가 자기평가와 상충될 경우 피드백을 거부한다(Sarar et al. 2005, 2007a; Watling et al. 2013a; Watling 2014b).

Politeness theory, which views most interpersonal interactions as a potential threat to the ‘face’ of the speaker or the recipient, is relevant to feedback conversations (Brown and Levinson 1987). Clinical teachers tend to avoid language that could damage learner self-efficacy (positive face) and actions that threaten their autonomy (negative face), while learners reject feedback if they doubt its credibility or the information conflicts with their self-assessment (Sargeant et al. 2005, 2007, 2008a; Watling et al. 2013a; Watling 2014b).


학습자들은 성과에 대한 직접적인 관찰이 피드백에 대한 신뢰성을 향상시킨다고 지적하지만, 동시에 직접 관찰이 자율성을 침해한다고 생각한다(Watling 2014a, Sargeant et al. 2008a, 2011a, Watling 2016, LaDonna et al. 2017, Ramani et al. 2018). 너무 이른 자율권 부여premature autonomy는 환자 진료의 오류를 초래할 수 있으므로, 교사들은 주어진 학습자에게 적합한 자율성과 관리감독의 균형을 맞출 필요가 있다.

Though learners indicate that direct observation of performance enhances credibility of feedback, they also view this as a potential infringement on autonomy (Watling 2014a, Sargeant et al. 2008a, 2011; Watling 2016, LaDonna et al. 2017; Ramani et al. 2018). Premature autonomy could lead to errors in patient care, therefore teachers need to balance supervision with autonomy appropriate for a given learner.


    • [퍼포먼스 향상 및 전문성 성장]에 포커스를 두어 건설적 피드백 제공

    • 이전 성공 사례의 강화를 활용하여 개선 분야 논의 유도

    • 피드백을 학습자 목표에 맞춤orient

    • 자율성은 점진적인 것임을 강조한다. 감독과 자율성의 균형은 학습자 수준과 능력에 따라 달라진다.

    • 교육 단계에 관계없이 학습자 성과를 지속적으로 직접 관찰해야 함


    • Focus constructive feedback on performance improvement and professional growth

    • Use reinforcement of previous successes to guide discussion of areas for improvement

    • Orient feedback towards learner goals

    • Emphasise that autonomy is developmental; balance of supervision and autonomy depends on learner level and abilities

    • Require ongoing direct observation of learner performance, regardless of stage of training


교사 및 학습자에 의한 학습의 공동 규제 Co-regulation of learning by teachers and learners


학습에 대한 공동규제는 (환자 치료라는 공동의 목표를 공유하는) 학습자와 감독자 교직원 사이의 상호의존성의 과정으로 설명된다(Rich 2017). 공동 규제 학습을 위한 참여적 설계 피드백 루프는 목표 지향 피드백, 자체 평가, 반사 및 실행 계획 개발로 구성된다(Ramani et al. 2019).

Co-regulated learning is described as a process of interdependency between learners and their supervising faculty as they share common patient care goals (Rich 2017). A participatory design feedback loop for co-regulated learning consists of goal-directed feedback, self-assessment, reflection, and development of action plans (Ramani et al. 2019).


4번 상자의 예시 대화

a sample conversation in Box 4:


    • 교사들:

학습자가 도전과 두려움을 공개할 수 있는 안전한 학습 환경 구축

강점과 결점이 존재한다는 것을 당연하게 함normalize

학습자의 임상 성과를 직접 관찰

Teachers:

Establish a safe learning environment where learners can disclose challenges and fears

Normalise presence of strengths as well as deficiencies

Perform direct observation of learner clinical performance


    • 학습자:

피드백 대화에 적극적으로 참여하고 시작할 수 있는 권한 부여empowered

교사의 지도에 따라 성과 개선 계획 수립

Learners:

Are empowered to actively engage in and initiate feedback conversations

Formulate performance improvement plans guided by teachers


    • 학습자와 교사들이 새로운 학습 기회를 공동 창출하여 실행 계획 구현

Learners and teachers co-create new learning opportunities to implement action plans



성장문화 정착을 위한 제도적 전략

Strategies for institutions to establish a culture of growth


유익한 피드백 문화를 확립한다. Establish a conducive feedback culture.


기관 문화는 피드백에 큰 영향을 미친다(Watling et al. 2014; Watching 2015; Ramani et al. 2017b). 전 세계 많은 의료 기관에서 학습 문화는 [종단적 교사-학습자 관계, 정기적인 성과 관찰, 구체적이지만 위협적이지 않은 건설적 피드백]을 촉진하지 않는다(Watling et al. 2013b, 2014; Watling 2014a; Ramani et al. 2017a, 2017b, 2018). 공손하고 nurturing하는 기관 문화가 있더라도, 이것이 꼭 정직하고 의미 있는 피드백 교환에 도움이 되는 것은 아니다(Ramani et al. 2017b, 2018).

institutional culture has a major impact on feedback (Watling et al. 2014; Watling 2015; Ramani et al. 2017b). The learning culture at many medical institutions worldwide does not promote longitudinal teacher-learner relationships, regular direct observation of performance and specific but non-threatening constructive feedback (Watling et al. 2013b, 2014; Watling 2014a; Ramani et al. 2017a, 2017b, 2018). Even a polite and nurturing institutional culture was not viewed as conducive to honest, meaningful feedback exchanges (Ramani et al. 2017b, 2018), thus the following strategies could be useful:


    • 동료와 비교하는 것이 아니라, 기대 성과와 비교하여 학습자를 보정하는 설계 평가 시스템

    • 관찰된 결과와 예상 결과 사이의 격차를 좁히는 데 피드백을 집중

    • 목표 및 실행 계획에 대한 정기적인 논의를 통해 교사-학습자의 종적 관계를 위한 기회 창출

    • 교사들이 교육 과정 내내 학습자 성과에 대한 스냅샷을 관찰할 수 있도록 함

    • 피드백 탐색 및 수용에 대한 교육 제공

Design assessment systems that calibrate learners against expected outcomes rather than peers

Focus feedback conversations on narrowing gaps between observed and expected outcomes

Create opportunities for teacher-learner longitudinal relationships with regular discussions of goals and action plans

Establish expectations for teachers to observe snapshots of learner performance throughout their training

Provide training in feedback seeking and acceptance


    • 모든 전문가가 장단점을 가지고 있음을 인정

    • 적절한 도움을 요청할 수 있는 의사willingness를 촉진

    • 휴머니즘과 환자에 대한 책임에 초점을 맞춤

    • 학습자가 전문적 성장의 ownership을 갖도록 empower

    • 협력적 양방향 피드백 활성화

    • 성찰적 실천에 초점

Acknowledge that all professionals have strengths and weaknesses

Promote willingness to seek help appropriately

Focus on humanism and accountability to patients

Empower learners to take ownership of their professional growth

Enable collaborative bidirectional feedback

Focus on reflective practice 


성장 마인드셋에 우선순위를 부여한다. Prioritise a growth mind-set.


[종합 평가와 동료와의 비교 문화]는 학습자들 사이에 고정형 사고방식을 심어 줄 수 있다(Harrison et al. 2016; Watling and Ginsburg 2019).

A culture of summative assessment and peer comparisons can instill a fixed mind-set among learners (Harrison et al. 2016; Watling and Ginsburg 2019).


    • 한계를 평가하기보다는 [학습을 위한 평가] 문화 조성

    • 지속적인 형성적 평가를 포함하고, 정보에 입각한 자체 평가를 권장하는 평가 시스템 설계

    • 학습자가 성과 격차 보정 및 개선 계획 수립에 대한 책임을 지도록 권장

    • 시험실적보다는 숙달된 학습을 강조

    • 지식 습득에 그치지 않고 평생학습과 비판적 사고 촉진

    • 학습자가 성과 목표보다는 학습 목표를 제시하는 기회 창출

Create a culture of assessment for learning rather than assessment of limitations

Design an assessment system which includes ongoing formative assessment and encourages informed self-assessment

Encourage learners to take responsibility for calibrating their performance gaps and formulating improvement plans

Emphasise mastery learning rather than exam performance

Promote lifelong learning and critical thinking rather than just acquisition of knowledge

Create opportunities where learners showcase learning goals rather than performance goals


교수개발

Faculty development.


교수진 개발 계획은 이 강력한 사회적 상호작용에 영향을 미치는 사회문화적 요인을 목표로 해야 한다.

faculty development initiatives should target sociocultural factors that influence this powerful social interaction.


  • 자신의 한계와 불안insecurities의 공개를 장려하는 안전한 학습 환경 구축

  • 라뽀 형성, 유익성 입증 및 학습자 성장 참여 등을 통해 피드백 탐색을 촉진

  • 학습자의 자기효능감을 address하고 학습자 수준과 능력에 적합한 자율성 제공

  • 학습자 성과에 대한 직접 관찰 및 정확한 보정 수행

  • 여러 소스의 평가 데이터를 사용하여 학습자와 함께 facilitated reflection과 informed 자기 평가(강도와 격차)에 참여

  • 자기효율성을 손상시키지 않고 관찰된 행동을 대상으로 특정 언어를 사용하여 건설적인 피드백을 교환

  • 피드백 대화 중 코칭 전략 적용

  • 교수 스스로 피드백 탐색 행동을 롤모델링, 한계 인정, 평생 학습을 통한 양방향 피드백 환경 구축


Establish a safe learning environment that encourages disclosure of limitations and insecurities

Facilitate feedback-seeking through rapport building, demonstration of beneficence and engagement in learner growth

Address learner self-efficacy and provide autonomy appropriate to learner level and ability

Perform direct observation and accurate calibration of learner performance

Engage in facilitated reflection and informed self-assessment (strengths and gaps) with learners using assessment data from multiple sources

Exchange constructive feedback using specific language targeting observed behaviours without damaging self-efficacy

Apply coaching strategies during feedback conversations

Establish a climate of bidirectional feedback through role-modelling of feedback seeking, admission of limitations and lifelong learning by teachers


Armson H, Lockyer JM, Zetkulic M, Konings KD, Sargeant J. 2019. Identifying coaching skills to improve feedback use in postgraduate medical education. Med Educ. 53(5):477–493.


Ramani S, Konings KD, Ginsburg S, van der Vleuten C. 2019. Twelve tips to promote a feedback culture with a growth mind-set: swinging the feedback pendulum from recipes to relationships. Med Teach. 41(6):625–631.


Gavriel J. 2016. Cognitive behavioural coaching principles and basic tools to support trainees. Educ Prim Care. 27(4):326–329. XXX


Ten Cate TJ, Snell L, Mann K, Vermunt J. 2004. Orienting teaching toward the learning process. Acad Med. 79(3):219–228.














 2019 Sep 24:1-11. doi: 10.1080/0142159X.2019.1656804. [Epub ahead of print]

Meaningful feedback through a sociocultural lens.

Author information

1
Department of Medicine, Brigham and Women's Hospital, Harvard Medical School , Boston , MA , USA.
2
Research and Scholarship, Harvard Macy Institute , Boston , MA , USA.
3
Department of Educational Development and Research and the School of Health Professions Education, Faculty of Health, Medicine and Life Sciences, Maastricht University , Maastricht , Netherlands.
4
Department of Medicine (Respirology) and Wilson Centre for Research in Education, University of Toronto , Toronto , Canada.

Abstract

This AMEE guide provides a framework and practical strategies for teachers, learners and institutions to promote meaningful feedback conversations that emphasise performance improvement and professional growth. Recommended strategies are based on recent feedback research and literature, which emphasise the sociocultural nature of these complex interactions. We use key concepts from three theories as the underpinnings of the recommended strategies: sociocultural, politeness and self-determination theories. We view the content and impact of feedback conversations through the perspective of learners, teachers and institutions, always focussing on learner growth. The guide emphasises the role of teachers in forming educational alliances with their learners, setting a safe learning climate, fostering self-awareness about their performance, engaging with learners in informed self-assessment and reflection, and co-creating the learning environment and learning opportunities with their learners. We highlight the role of institutions in enhancing the feedback culture by encouraging a growth mind-set and a learning goal-orientation. Practical advice is provided on techniques and strategies that can be used and applied by learners, teachers and institutions to effectively foster all these elements. Finally, we highlight throughout the critical importance of congruence between the three levels of culture: unwritten values, espoused values and day to day behaviours.

PMID:
 
31550434
 
DOI:
 
10.1080/0142159X.2019.1656804


F에 실패하기: 교수의 딜레마(Med Educ, 2019)

‘Failure to fail’: the teacher’s dilemma revisited

Marianne Mak-van der Vossen




'MUM effect'라는 용어는 사람들이 일반적으로 Uncommon Messages에 대해 Mum을 유지하는 것을 선호한다는 것을 나타내기 위해 사용된다. 의대생에게 낙제점fail을 주는 것은 정말 불쾌한 메시지다. 'MUM 효과'를 이해하는 것은 의료 교육자들이 보건 직업 교육에서 평가를 개선하는 데 도움이 될 수 있다.

The term ‘MUM effect’ is used to indicate that people generally prefer to keep Mum about Unpleasant Messages. Giving a fail to a medical student is such an unpleasant message. Understanding the ‘MUM effect’ could help medical educators to improve assessment in health professions education.


성적이 저조한 학생들에게 낙제를 주기를 망설여하는 경향은 의학교육에서 잘 알려져 있다.2,3 이러한 '실패 실패' 현상은 처음에 간호 교육에서 교사의 딜레마로 특징지어졌다4,5:

Reluctance to give a fail mark to underperforming students is well known in medical education.2,3 This ‘failure to fail’ phenomenon was initially characterised in nursing education as a teacher’s dilemma4,5:


실패하기를 꺼리는 더 최근에 기술된 이유는 

  • 기대에 대한 개념적 명확성의 결여

  • 판단의 주관성에 대한 우려, 

  • 학생의 평판을 해칠 염려, 

  • 적절한 교수진의 개발 부족, 

  • 재교육 과정과 그 결과에 대한 불확실성 등이다.6

More recently described reasons for reluctance to fail are 

  • a lack of conceptual clarity about expectations, 

  • concern over the subjectivity of one’s judgement, 

  • fear of harming a student’s reputation, 

  • lack of appropriate faculty development, and 

  • uncertainty about the remediation process and its outcomes.6


명분과 상관없이, 교육자들이 낙제시키기를 꺼리는 것은 유감스러운 일이다. 왜냐하면 학업성취도가 저조하다는 것이 제대로 밝혀지지identified 않은 학생들은 성적을 향상시키는 데 도움이 되는 도움을 받을 수 없기 때문이다.7 저조한 학업성취도는 종종 근본적인 개인적 또는 제도적 요인에 의해 발생한다. 그러한 원인을 해명하는 것은 낙오된 사람에 대한 지지를 이끌어내는 데 도움이 될 수 있다. 또한, underperformance와 환자 안전의 관계는 명확하다.

Regardless of the cause, educators’ reluctance to fail is unfortunate, because underperforming students who are not identified cannot be offered assistance that would help them improve their performance.7 Underperformance is often caused by underlying personal or institutional factors. Elucidating such causes could be of help in creating support for the lapsing person. Furthermore, the relation of underperformance and patient safety has been made clear.8


스카프 외 연구진은 교육자들이 관찰된 불만족스러운 작업에 대해 적절한 평가를 한 후에도 학생들에게 적절한 피드백이나 등급을 전달하지 못하는 경우가 많다는 것을 보여준다. 저자들은 'MUM 효과'를 극복하기 위한 다음과 같은 해결책을 제안한다: 

  • 프로그램적 평가 사용, 

  • 메시지가 학생에게 주는 이점에 초점을 둠

  • [바람직하지 않은 메시지 전달]을 직무의 일부로 한다. 

Scarff et al.1 indicate that educators often fail to communicate appropriate feedback or ratings to their students, even after having made a proper assessment about observed unsatisfactory work. the authors propose the following solutions to overcome ‘the MUM effect’: 

  • using programmatic assessment, 

  • focusing on the benefit of the message for the learner, and 

  • making delivery of undesirable messages part of the job. 


이에 더하여, 저자들은 바람직하지 않은undesirable 메시지를 전달하는 전문가인 특정 피드백 제공자를 설치할 것을 제안한다.

Furthermore, the authors propose to install specific feedback providers who are experts in delivering undesirable messages.


사람들이 왜 특정한 일에 참여하거나engage 참여하지 않기로 결정하는지를 더 일반적으로 고려하는 것으로 초점을 넓히는 동시에 가치가 있을 수 있다. 이 점에서 도움이 되는 것은 동기부여의 기대 가치-가치-비용 모델이다9, 

  • 과제 성공에 대한 기대치(내가 할 수 있는가?), 

  • 어떤 과제에 참여하는 것의 가치에 대한 인식(내가 그것을 하고 싶은가?) 및 

  • 어떤 과제에 참여하는 데 드는 비용(내가 그것을 하는 것을 방해하는 장벽이 있는가?)이다.

It may be worth simultaneously broadening the focus to more generally consider why people decide to engage or not engage in certain tasks. Helpful in this regard is the Expectancy-Value-Cost (EVC) model of motivation,9 which describes 

  • the expectancy of being successful in a task (Can I do it?), 

  • the perceived value of engaging in a task(Do I want to do it?) and 

  • the costs of engaging in a task (Are there barriers that prevent me from doing it?).


어떤 메시지가 불쾌하다면, 그것은 메세지의 효과는 교육자(기대)에게 보이지 않고, 교육자에게 개인적인 이익이 없으며(가치), 노력을 해야 하기 때문이다(비용). 우리의 연구는 기대, 가치, 비용이 모두 개인과 제도적 요인에 의해 영향을 받는다는 것을 보여주었다.10

A message is unpleasant because the effect of the message is not visible for the educator (Expectancy), there is no personal interest for the educator (Value), and she or he has to make the effort (Costs). Our research showed that expectancy, value and costs are all influenced by individual and institutional factors.10


EVC 모델은 'mumming'와 'failure to fail'에서 역할을 하는 제도적 요인에 대한 설명을 추가한다. 즉, 메시지는 다음의 경우에 불쾌하다

  • 기대치가 명확하지 않을 경우(예: 불합격 후 학생들에게 (추가) 기회가 없음) 

  • 가치와 규범이 공유되지 않는 경우(예: 규범이 (외부에서) 부과된imposed 경우) 

  • 반응responding 비용이 높은 경우(예: 성과를 관찰하고 평가하는 데 필요한 많은 시간)10

The EVC model adds an explanation for the institutional factors that play a role in ‘mumming’ and ‘failure to fail’: a message is unpleasant 

  • if expectations are not clear (e.g. no opportunities for students after failing), 

  • if values and norms are not shared (e.g. norms have been imposed), and 

  • if the costs of responding are high (e.g. much time needed to observe and evaluate performance).10


의료계는 오류가 필연적으로 발생할 수밖에 없고, 개인과 제도적 요인이 모두 오류를 만드는 데 역할을 한다는 점을 받아들였다. 이로써 의료 오류에 대하여 효과적인 '무-비난blame-free' 처리가 가능해졌고, 모든 이해관계자가 오류로부터 배워서, 궁극적으로 오류를 방지하는 상황이 가능해졌다.11

The medical community has accepted that errors inevitably will occur, and that both individual and institutional factors play a role in error making. This has enabled effective ‘blame-free’ handling of medical errors, to enable a situation in which all stakeholders learn from errors and ultimately prevent them from happening.11


학생들의 performance lapse는 필연적으로 발생하며, 그것이 학생들의 행복과 환자안전 모두에 영향을 미친다는 것을 인정해야 할 때가 왔다. 그 결과, 우리는 그 문제들에 대처하는 방법을 더 잘 배우고, 그것에 대한 개인적, 제도적 원인을 모두 논의하고, 그러한 원인을 서로 지원하여 수정할 필요가 있다.

the time has come to acknowledge that students’ performance lapses inevitably will occur, and that they influence both the individual’s well-being and patient safety. As a result, we need to enable a ‘blame-free’ handling of underperformance to better learn how to deal with the issues, discuss both personal and institutional causes for it, and support each other to modify such causes.


그렇다면 이것은 [우리가 원하지 않는 메시지를 전달하기 위해 특정한 피드백 제공자를 설치해야 한다는 것]을 의미하는가? 그렇지 않다. 그렇게 한다고 해서 [lapsing이 학습의 일부라는 것]을 인정하게 되지는 않을 것이다.12 성과를 평가하고 피드백을 제공하는 것은 의학 교육자의 업무에서 필수적인 부분이다. 모든 임상 교육자들은 그들의 학생들이 위험을 인식하도록 하기 위해, 만족스럽지 못한 성과를 기꺼이 논의하려고 해야하고, 논의할 수 있어야 한다.

Does this mean that we have to install specific feedback givers to communicate undesirable messages? No,that would not acknowledge that lapsing is part of learning.12 Assessing performance and providing feedback is an essential part of the task of a medical educator. All clinical educators must be willing and able to discuss unsatisfactory performance to make their students aware of the risk.


performance의 lapse는 의학에서 일상적인 일이다. 일단 의료 분야에서 일하는 모든 사람들에게 필연적으로 performance lapse가 발생하고, 그러한 과실에 대한 '불쾌한unpleasant' 메시지를 소통하는 것이 실수한 사람과 환자 돌봄에 도움이 된다는 것을 받아들이게 되면, 선생님의 딜레마는 사라질 것이다.

Lapses in performance are a part of daily life in medicine. Once we accept that performance lapses inevitably occur to everyone working in medicine, and that communicating an ‘unpleasant’ message about such lapses benefits both the lapsing person and future patient care, the teacher’s dilemma will have gone.


7 Ellaway RH, Chou CL, Kalet AL. Situating remediation: accommodating success and failure in medical education systems. Acad Med 2018;93 (3):391–8.




 2019 Feb;53(2):108-110. doi: 10.1111/medu.13772. Epub 2018 Dec 12.

'Failure to fail': the teacher's dilemma revisited.

Author information

1
Amsterdam University Centers, VUmc School of Medical Sciences, Research in Education, Amsterdam, the Netherlands.
PMID:
 
30548645
 
PMCID:
 
PMC6590308
 
DOI:
 
10.1111/medu.13772


인식, 문화, 신뢰: 의학교육에서 피드백 분해(Perspect Med Educ, 2014)

Cognition, culture, and credibility: deconstructing feedback in medical education

Christopher Watling





도입

Introduction


피드백이 학습의 필수적인 촉진자로 널리 지지되어 왔음에도 불구하고 [1–4] 학습자에게 그것이 어떻게 의미 있게 되는지에 대한 체계적인 조사는 거의 없었다.

Although feedback has been widely endorsed as an essential facilitator of learning [1–4], there have been few systematic investigations of how it becomes meaningful for learners.


방법

Methods


이 논문에 제시된 연구 질문은 [의학적 훈련의 특정 맥락 안에서 피드백이 학습자에게 어떻게, 왜 영향을 미치는지 또는 왜 영향을 미치지 않는지]를 조사하는 탐구적인 질문이다. 탐구 질문은 질적 접근법[5]이 유익할 수 있다. 따라서, 이 논문으로 구성된 다섯 가지 경험적 연구는 구성주의 기반 이론 방법론을 사용했다. 해석적 전통에 뿌리를 둔 구설주의 기반 이론은 사회적 또는 사회심리적 과정의 질적 연구에 엄격한 접근을 제공한다[6–8].

The research questions posed in this thesis are exploratory, examining how and why feedback may or may not impact learners within the specific context of medical training. Exploratory questions may benefit from a qualitative approach [5]; accordingly, the five empiric studies comprising this thesis used a constructivist grounded theory methodology. Rooted in an interpretive tradition, constructivist grounded theory offers a rigorous approach to the qualitative study of social or social psychological processes [6–8].


Results


1. 의료 학습자는 임상 작업에 참여함에 따라 자신의 발전을 형성하는 [다양한 정보 출처 또는 학습 단서]에 주의를 기울일 수 있다. 피드백은 롤모델, 임상 결과, 환자 또는 가족의 반응, 동료와의 비교를 비롯한 다른 학습 단서을과 학습자의 관심을 놓고 경쟁한다. 학습자의 관심을 얻고 영향력이 커지기 위해서는 피드백이 그 신뢰성에 대한 비판적 판단으로부터 살아남아야 한다[11].

1. As they participate in clinical work, medical learners can attend to a variety of sources of information, or learning cues, which shape their development. Feedback competes for learners’ attention with other learning cues, including role models, clinical outcomes, patient or family responses, and comparisons with peers. To win a learner’s attention and become influential, feedback must survive a critical judgement of its credibility [11].


2. 조절초점 이론은 의료 학습자의 피드백에 대한 반응, 특히 개인의 조절초점을 쉽게 결정할 수 있는 상황에 대한 통찰력을 제공한다. 그러나 실제 피드백 시나리오에는 조절초점 이론을 적용하는 데 어려움이 있다. 임상 업무는 촉진 초점 또는 예방 초점 중 하나만을 활성화한다고 분류기에 어려운 경우가 많다. 오히려 혼합된 조절초점이 존재하는 경우가 많다. 즉, 학습자는 의사로서의 커나가는 정체성을 둘러싼 열망(촉진 초점)과 안전한 직무 수행을 보장하고 오류를 방지하고자 하는 욕망(예방 초점) 모두를 가지기에, 피드백에 대한 반응을 예측하는 것이 복잡해질 수 있다. 또한 시간에 따라 조절 초점이 변화할 수 있는데, 이는 개인이 과제를 성찰하고 재구성하기 때문이다. 이러한 변화로 인해 학습자의 피드백에 대한 반응이 진화해나갈 수 있다. 마지막으로, 피드백의 출처나 내용에 대한 인식된 신뢰성과 같은 다른 요인들이 학습자의 응답을 지시하는 데 있어 조절초점보다 중요할 수 있다[12].

2. Regulatory focus theory provides some insight into medical learners’ responses to feedback, particularly in circumstances where an individual’s regulatory focus can be readily determined. There are challenges in applying regulatory focus theory to real feedback scenarios, however. Clinical tasks often resist classification as activating either promotion focus or prevention focus for learners. Rather, a mixed regulatory focus is often present; learners may be motivated simultaneously by their aspirations around their burgeoning identity as doctors (promotion focus) and their desire to ensure safe task performance and avoid errors (prevention focus), complicating efforts to predict their responses to feedback. Furthermore, regulatory focus can change over time, as individuals reflect on and reframe tasks; with this change, there can be an evolution of the learner’s response to the feedback they receive. Finally, other factors, such as the perceived credibility of the source or content of the feedback, may trump regulatory focus in directing learners’ responses [12].


3. 의학의 학습문화와 음악의 학습문화를 비교한 결과, 의학의 교육학적 접근방식을 뒷받침하는 '당연하게 여겨온 가정'을 알게 되었고, 가정과 학습에 대한 가치에 대해 의문을 제기할 수 있다. 의학은 실제 임상 경험에 몰입하는 것이 특징인 학습을 중요시하는 반면, 음악은 레슨lesson에 의한 학습을 중시하며, 체계적인 일대일 가르침과 개별적인 실천이 특징이다. 의학 학습자들이 역량을 목표로 하는 반면, 음악 학생들은 그 대신에 더 나은 퍼포먼스를 목표로 한다. 의학 학습자들은 교수자의 가르치는 능력보다 임상 기술을 더 중요시하는 반면, 음악에서는 교사의 교육 기술이 가장 중요하기에 그 반대이다. 자기평가는 양쪽 문화에서 학습자에게 어려운 일이지만, 의료 학습자는 자기평가를 자신이 개발할 수 있는 기술로 간주하는 반면, 음악 학생들은 외부 피드백이 항상 필요할 것이라는 것을 인식한다[13].

3. Our comparison of medicine’s learning culture with that of music revealed taken- for-granted assumptions that underpin medicine’s pedagogical approaches, and raised questions about these assumptions and their value for learning. While medicine values learning by doing, characterized by immersion in real clinical experiences, music values learning by lesson, characterized by methodical one- on-one instruction and individual practice. While medical learners aim for competence, music students aim instead for ever-better performance. While medical learners value their teachers for their clinical skills more than for their teachingabilities, the opposite is true in music, where teachers’ instructional skills are paramount. Self-assessment challenges learners in both cultures, but medical learners view self-assessment as a skill they can develop, while music students recognize that external feedback will always be required [13].


4. 학습자가 피드백이 의미 있는 것으로 인식되기 위해서는 [신뢰할 수 있고 건설적]인 특성을 필수적으로 여기지만, [신뢰할 수 있고 건설적]인 피드백에 정의는 분야에 따라 다르다. 학습 문화는 이러한 중요한 특성을 가진 피드백의 발생을 얼마나 효과적으로 지원하는지에 따라 상당히 다르다. 예를 들어, 교사 훈련과 음악의 학습 문화에서는, (의학에서는 이루어지지 않는 방식으로) 학습자의 성과를 직접적이고 철저하게 관찰할 수 있는 기회를 교육 과정에 포함하고 있다; 이러한 학습 문화는 직접적인 관찰에 그들의 피드백을 확고히 함으로써, 그 신뢰성을 강화한다[14].

4. Although learners across cultures identify credibility and constructiveness as essential for feedback to be perceived as meaningful, the very definitions of credibility and constructiveness are distinct to each learning culture. Learning cultures vary considerably in how effectively they support the occurrence of feedback possessing these critical characteristics. The learning cultures of both teacher training and music, for example, embed opportunities for direct, thorough observation of learner performance into their pedagogies in a way that medicine does not; by firmly grounding their feedback in direct observation, these learning cultures strengthen its credibility [14].


5. 학습과 피드백을 받는 경험에는 개별적으로나 문화적으로 정의된 요소가 모두 있다. 

  • 피드백을 향한 학습자의 동기 및 방향orientation과 (특정성, 적시성, 신뢰성 및 행동성 등) 피드백의 어떤 기본적 특성을 가치있게 여기는지에 성향여러 학습 맥락 전반에 걸쳐 안정적으로 나타난다. 

  • 학습 문화는 좋은 피드백이 발생하고 학습자가 어떻게 반응할지에 대한 조건과 기회를 형성함으로써 피드백의 영향을 조절한다[15].

5. There are both individually and culturally defined elements of the experience of learning and receiving feedback. 

  • Learners’ motivation and orientation toward feedback, as well as the value they place on certain fundamental feedback characteristics—specificity, timeliness, credibility, and actionability—appear stable across learning contexts. 

  • Learning culture modulates the impact of feedback by creating the conditions and opportunities for good feedback to occur and for learners to respond [15].


고찰

Discussion


우리는 학습에 대한 개인이나 사회문화적 관점 모두 피드백이 어떻게 경험되고 어떻게 의미를 획득하는지에 대한 복잡성을 설명하기에 충분치 않다고 주장한다. 대신 우리는 개별 학습자와 학습 문화를 모두 프로세스의 필수 요소와 불가분의 요소로 간주하는 피드백을 이해하기 위한 모델을 제안한다. 두 가지를 떼어놓고 설명할 수 없다.

We argue that neither the individual nor the sociocultural perspective on learning is fully adequate to account for the complexity of how feedback is experienced and how it acquires meaning. We propose instead a model for understanding feedback that considers both the individual learner and the learning culture as essential and inseparable elements of the process. Neither can be addressed in isolation.


피드백 반응response의 일부 개별적인 가변성은 불가피하지만, 학습 효과가 실현되기 위해서는 학습 문화 내에서 좋은 피드백이 제공되어야 한다. (종종 의학에서 일어나는 것처럼) 만약 어떤 문화가 일상적으로 좋은 피드백에 대한 장벽을 만든다면, 학습자들은 다른 곳에서 지침guidance을 구할 수 있고, 궁극적 결과는 그 문화 내에서 학습에 대한 피드백 가치가 점차적으로 감소하는 것이다. 그러나, 학습 문화는 의미 있는 피드백이 발생할 수 있는 지원과 기회 이상의 기여를 한다; 학습 문화의 규범과 가치는 신뢰할 수 있는 피드백으로 간주되는 것과 학습자의 관심을 요구하는 피드백을 형성한다. 따라서 학습 문화는 학습자에게 의미 있는 피드백의 정의를 형성한다.

Although some individual variability in feedback response is inevitable, good feedback must be available within a learning culture in order for its learning impact to be realized. If a culture routinely creates barriers to good feedback, as often occurs within medicine, learners may seek guidance elsewhere, with the end result being a gradual diminution in the value of feedback for learning within that culture. Learning culture contributes more than just the support and opportunity for meaningful feedback to occur, however; a learning culture’s norms and values also shape what counts as credible feedback and what feedback demands learners’ attention. Learning culture thus shapes the very definition of meaningful feedback for learners.


결론

Conclusion


이 논문의 중심적 기여는 피드백이 영향을 미치는 과정의 역사적으로 소외된 두 가지 요소 즉 학습자의 인식과 학습 문화로 의료 교육자들의 관심을 유도하는 것이다.

The central contribution of this thesis is to direct the attention of medical educators towards two historically marginalized elements of the process by which feedback achieves impact: learner perceptions and learning culture.





 2014 Apr;3(2):124-8. doi: 10.1007/s40037-014-0115-2.

Cognitionculture, and credibilitydeconstructing feedback in medical education.

Author information

1
Postgraduate Medical Education, Schulich School of Medicine and Dentistry, Western University, Medical Sciences Building Room M103, London, ON, N6A 5C1, Canada, chris.watling@schulich.uwo.ca.
PMID:
 
24604762
 
PMCID:
 
PMC3976480
 
DOI:
 
10.1007/s40037-014-0115-2


선한 판단을 수반한 디브리핑: 엄격한 피드백과 진정한 탐색의 조합(Anesthesiol Clin, 2007)

Debriefing with Good Judgment: Combining Rigorous Feedback with Genuine Inquiry

Jenny W. Rudolph, PhDa,c,*, Robert Simon, EdDc,d,e, Peter Rivard, PhDb, Ronald L. Dufresne, PhDf, Daniel B. Raemer, PhDc,d,e





자신의 실천에 대한 성찰은 경험적 학습 과정에서 중요한 단계다. 그것은 연습생들이 직접적인 경험으로부터 통찰력을 개발하고 이후 행동으로 통합하는 데 도움이 된다[1,2]. 시뮬레이션 사례에 참여하고 나서, 보고 또는 사후 조치 검토를 함으로써 의료 시뮬레이션을 사용하는 임상의사가 이러한 성찰을 수행할 수 있는 방법을 제공한다. 디브리핑은 훈련생이 정보와 감정 상태를 설명하고 분석하고 종합하여, 향후 유사한 상황에서 성과를 개선할 수 있도록 하는 것을 목표로 한다.

Reflection on one’s own practice is a crucial step in the experiential learning process. It helps trainees develop and integrate insights from direct experience into later action [1,2]. Subsequent to participating in a simulated case, debriefing or after-action review provides a way for clinicians using medical simulation to do this reflection. The goals are to allow trainees to explain, analyze, and synthesize information and emotional states to improve performance in similar situations in the future.


비판적 판단을 공유하는 것은 시뮬레이션과 보고에서 배우는 데 필수적인 부분이다. 하지만, 교사들은 종종 비판적인 생각과 감정에 목소리를 내는 것을 피한다. 왜냐하면 그들은 그들이 대립하는 것처럼 보이고 싶지 않기 때문이고 그들은 비판이 훈련생들의 감정을 상하게 하거나 방어하는 것으로 이어질 수 있다고 우려하기 때문이다. 비판적 판단은 많은 강사에게 딜레마를 안겨준다.

Sharing critical judgments is an essential part of learning in simulation and debriefing. Yet, instructors often avoid giving voice to critical thoughts and feelings because they do not want to seem confrontational and they worry that criticism might lead to hurt feelings or defensiveness on the part of the trainee. Voicing critical judgment poses a dilemma for many instructors


"심각한 성찰"이란 시뮬레이션 경험에 의해 야기된 임상 및 행동 딜레마와 혼란 영역을 표면화surface시키고, 이를 해결하는 데 도움이 되는 디브리핑 프로세스를 의미한다.

By ‘‘rigorous reflection’’ we mean a debriefing process that brings to the surface and helps resolve the clinical and behavioral dilemmas and areas of confusion raised by the simulation experience.


이 기사는 의료 시뮬레이션 연습에 대한 보고의 모델을 명시한다.

this article articulates a model of debriefing for medical simulation exercises.


이 작업에서 채택된 보고 모델은 세 가지 주요 구성요소를 가지고 있다. 

  • 첫 번째 구성요소는 인지과학 연구와 성찰적 실천에서 도출한 개념적 프레임워크로, 모의실험 중 연습생들의 행동을 지도할 때 두드러졌던 정신모델을 어떻게 조명할 것인지 강사에게 안내한다. 

  • 두 번째는 [연습생에 대한 호기심과 존중]과 [연습생 수행에 대한 명확한 평가판단]이라는 일견 모순되는 가치를 결합하는 근본적인 디브리핑 스탠스다. 

  • 세 번째 요소는 근본적인 입장을 취하는 (주장과 조사를 결합하는) 대화 방식이다.

The debriefing model adapted from this work has three primary components. 

  • The first component is a conceptual framework, drawn from research in cognitive science and on reflective practice, that guides the instructor on how to illuminate the mental models that were salient in guiding trainees’ actions during the simulation. 

  • The second is an underlying debriefing stance that unites the apparently contradictory values of curiosity about and respect for the trainee and the value of clear evaluative judgments about trainee performance. 

  • The third component is a way of talking (combining advocacy and inquiry) that enacts the underlying stance.


성찰적 실천: 방법 및 이론

Reflective practice: method and theory


성찰적 실천은 자신의 전문직업적 작업 관행과 그 바탕을 이루는 전용 가정들을 면밀히 조사하기 위해 사용되는 방법이다. 성찰적 실천은 종종 협력적 세팅[16]에서 이루어진다. 하버드 대학교와 매사추세츠 공과대학교의 연구원들은, 

  • 당연하게 여기는 가정과 정신적 일상을 면밀히 조사하는 것을 배운 성찰적 실무자들이 그들의 전문 기술을 스스로 수정하고 개선할 수 있다는 것을 발견했다. 

  • 그러나, 이러한 자기 검증에 능숙하지 못한 사람들은 확증하는 데이터를 봉쇄하거나 무시하는 경향이 있었고, 비효율적인 실행 습관을 유지했다[11,12,14,19,20].

Reflective practice is a method used to scrutinize one’s own professional work practices and the taken-for-granted assumptions that underlie them. It is often accomplished in a collaborative setting [16]: Researchers at Harvard University and the Massachusetts Institute of Technology found that reflective practitioners, who learned to scrutinize their taken-for-granted assumptions and mental routines, were able to self-correct and improve their professional skills. Those without skill in this self-scrutiny, however, tended to seal out or ignore disconfirming data and maintained ineffective habits of practice [11,12,14,19,20].


성찰적 실천의 기초가 되는 이론은 인지과학, 사회심리학, 그리고 인류학에 의존한다. 핵심적인 생각은 사람들이 외부 현실의 내부 이미지인 내부 인지 프레임을 통해 외부 자극을 이해한다는 것이다[20–25]. 이러한 이미지에 대한 용어는 무수히 많다:'Frame of reference','schemata','mental model' 등 이다. 사람들은 객관적 현실을 수동적으로 인지하지 않으며, 적극적으로 환경에 대해 필터링하고, 생성하고, 의미를 적용하는 방식으로 의미형성sense making에 관여한다[26–28].

The theory underlying reflective practice draws on cognitive science, social psychology, and anthropology. The central idea is that people make sense of external stimuli through internal cognitive frames, internal images of external reality [20–25]. Terms for these images are myriad: ‘‘frames of reference,’’ ‘‘schemata,’’ and ‘‘mental models,’’ to name a few. People do not passively perceive an objective reality, but engage in sensemaking by which they actively filter, create, and apply meaning to their environment [26–28].


그림 1은 프레임, 작용 및 시뮬레이션 결과 사이의 관계를 보여준다.

Fig. 1 shows the relationships among frames, actions, and simulation results.



이 틀들은 차례로 사람들이 취하는 행동을 형성한다. 임상적 틀과 사회적 혹은 대인관계적 틀은 모두 의학적 의사결정에 중요한 역할을 할 수 있다. 예를 들어, bag-mask 기구를 쉽게 사용할 수 없는 환경에서 무응답 환자를 관리하기 위해 호출되는 마취과 의사를 생각해 보자. 그들은 환자가 저산소혈증과 체포되는 동안 그들이 가장 친숙한 장치, 백 마스크, 그리고 치료를 지연시키는 것만을 사용하여 소생시킬 수 있다는 프레임을 가지고 있다. 이 모델은 [사람들의 행동]이란 그들이 직면하고 있는 상황을 어떻게 프레임하는지에 따른 불가피한 결과라고 암시한다.

These frames, in turn, shape the actions people take. Both clinical frames and social or interpersonal frames can play crucial roles in medical decision making. for example, consider an anesthesiologist who is called to manage an unresponsive patient in a setting where a bag-mask apparatus is not readily available. They hold the frame that they can only resuscitate using the device with which they are most familiar, a bag mask, and delay treatment while the patient descends into hypoxemia and arrest. The model suggests that people’s actions, are an inevitable result of how they frame the situation they face.


중요한 것은, 실수조차도 대개 의도적이며 합리적인 행동의 결과물이라는 것이다 [23,27,30]. 즉, 그 사람이 그 순간 상황을 어떻게 프레이밍 하는지를 고려한다면, 그 행동들은 완벽히 이치에 맞는다. 마취과 의사의 예를 계속하여, 강사는 수동적인 산소화를 고려하거나 입으로 하는 '구강 호흡'을 전달하는 대신, 환자가 불포화 되어 있는 동안 마취과 연수는 가차없이 백마스크 기구를 찾아다녔다는 사실에 놀랄지도 모른다. 그러나 이러한 행동들은 수련자가 백마스크 장치 없이는 기본적인 생명 유지도 이룰 수 없다고 믿고 있었고, 구강 호흡은 고려 대상이 아니었으며, passive oxygenation은 아예 배운 적도 없었음을 강사가 이해하게 되면, 완전히 말이 된다. 이처럼, 프레임을 표면화시키고, 그 프레임이 동작에 미치는 영향을 분석하고, 새로운 프레임과 동작을 만드는 것이 디브리핑 시간에 교육자가 할 일이다.

Importantly, even mistakes are usually the result of intentionally rational actions [23,27,30]. That is, the actions make perfect sense given how the person was framing the situation at that moment. Continuing the example of the anesthesiologist, the instructor may be surprised that instead of considering passive oxygenation or delivering a mouth-to-mask ‘‘rescue breath,’’ the anesthesiologist trainee searched relentlessly for a bag-mask apparatus while the patient desaturated. These actions make perfect sense, however, when the instructor understands that the trainee held the belief that basic life support cannot be achieved without a bag-mask device, mouth-tomouth was out of the question, and passive oxygenation is something that he has never learned. It is the instructor’s job during a debriefing to help the trainee 

  • bring these frames to the surface; 

  • analyze their impact on actions; and 

  • craft new frames and actions


성찰적 실천 모델에서 "결과"는 연습생이 취하는 행동에 의해 유발되는 것이다. 결과는 상태state이다. 연습생과 연습생들은 대개 원하는 결과가 무엇인지 암묵적으로 알고 있다. 강사와 연습자가 프레임-액션-결과 인과 시퀀스를 역순으로 탐색할 때 학습이 이루어진다.

‘‘Results,’’ in the reflective practice model, are seen to be prompted by the actions the trainee takes. Results are states . The instructor and the trainee usually have an implicit idea of what the desired results were. Learning occurs when instructor and trainee explore the frames-actions-results causal sequence in reverse.


토론 자세: 판단적 디브리핑에서 DWGJ로 이동

Debriefing stance: moving from judgmental debriefing to DWGJ


엄격한 보고 과정 중 가장 중요한 것은 강사들이 관찰한 시뮬레이션과 관련된 자신의 프레임을 식별하고 검사하는 것이다. 그 이유는 두 가지다. 

  • 첫째로, 강사들은 유사한 상황에서 [그들 자신이 사용했을지도 모르는 프레임과 행동]을 그들 자신의 경험으로부터 끌어낼 수 있어야 하고, 참가자에게 그것들을 공개할 수 있어야 한다. 

  • 둘째로, 강사들은 연습생들과 함께 연습하는 연습생들의 퍼포먼스에 대해 그들 [자신의 틀의 타당성을 기꺼이 시험해야] 한다. 

이것이 어떻게 작용하는지 설명하기 위해, 저자들은 강사가 판단, 비판단 및 DWGJ 접근방식을 사용할 때 강사의 기본 틀을 설명하고 대조하는 것으로 시작한다(표 1).

Crucial to the process of a rigorous debriefing is instructors’ learning to identify and examine their own frames related to the simulation they observed. The reasons for this are twofold. 

  • First, the instructors must be able to draw from their own experience the frames and actions they themselves might have deployed in a similar situation and to disclose these to the participant. 

  • Second, instructors have to be willing to test the validity of their own frames about the trainee’s performance with trainees. 

To explain how this works, the authors start by describing and contrasting instructors’ underlying frames when they are using judgmental, nonjudgmental, and DWGJ approaches (Table 1).




보고에 대한 판단적 접근법

The judgmental approach to debriefing


경멸에 찬 목소리로 한 무리의 학생들에게 '여기서 무슨 일이 일어났는지 누가 말해줄 수 있을까?' 또는 '패트의 큰 실수를 누가 말해줄 수 있을까?'라는 질문을 하는 강사의 목소리를 상상하거나 떠올려 보라. 이러한 판단적 접근법은 가혹한 비판에 얽매여 있건 아니면 좀더 부드럽게 적용되었건 간에, 진실truth는 오직 강사의 것이며, 실수는 trainee의 것이고, 연습자의 사고나 행동에 본질적인 실패가 있다고 가정한다.

Imagine or recall the instructor whose voice, dripping with disdain, inquires of a group of students, ‘‘Can anyone tell me what went wrong here?’’ or ‘‘Can anyone tell me Pat’s big mistake?’’ The judgmental approach, whether laced with harsh criticism or more gently applied, places truth solely in the possession of the instructor, error in the hands of the trainee, and presumes that there is an essential failure in the thinking or actions of the trainee.


특히 혹독한 비판을 포함하는 보고에 대한 판단적 접근은 심각한 비용을 초래할 수 있다. 굴욕, 동기부여, 혼란의 나중의 영역에 대한 문제 제기를 꺼려하거나, 전문 또는 임상 실험에서 재능 있는 훈련생들의 퇴실이다. 그러나 수치심과 비난적 접근은 중요한 덕목이 있다. 훈련생들은 강사가 중요하다고 믿는 것이 무엇인지에 대해 의심을 품지 않을 수 있다.

A judgmental approach to debriefing, especially one that includes harsh criticism, can have serious costs: humiliation, dampened motivation, reluctance to raise questions about later areas of confusion, or exit of talented trainees from the specialty or clinical practice altogether. But the shame and blame approach has an important virtue: the trainee is rarely left in doubt about what the instructor believes are the salient issues.


보고에 대한 비판단적 접근법

The nonjudgmental approach to debriefing


이러한 판단적 접근에서 벗어나고자 하는 강사들이 직면하고 있는 가장 큰 딜레마는 부정적인 감정과 방어, 사회적 체면을 보존하고 신뢰와 심리적 안전을 유지하는 동시에 어떻게 비판적 메시지를 전달할 것인가 하는 것이다.2 비판단적 접근법을 사용하는 강사들은 종종 샌드위치 접근법과 같은 보호적인 사회적 전략을 사용하거나, 중요한 통찰력을 걸러내거나, 또는 문제 주제를 완전히 피함으로써 딜레마를 해결한다[33,34]. 

The central dilemma facing instructors who want to move away from this judgmental approach is how to deliver a critical message while avoiding negative emotions and defensiveness, preserving social face, and maintaining trust and psychological safety.2 Instructors using a nonjudgmental approach often resolve the dilemma 

    • by using protective social strategies, such as the sandwich approach ; 

    • filtering out critical insights; or 

    • by avoiding the problem topic altogether [33,34]. 


또 다른 일반적인 방법은 침묵을 선택하고 비판적 생각이나 감정을 표현하지 않는 것이다. 침묵이나 비판단적인 접근을 선택하여 전문가적 비판을 모호하게 남겨두면, 훈련생들의 퍼포먼스와 관련한 중요한 통찰이나 느낌이 불확실하거나 표현되지 않은 채로 남게 된다. 이로써 수련의사와 그 조직은 업무 방식을 개선할 수 있는 인포메이션이 박탈당하게 된다[35]. 비판적 사고와 감정을 회피하는 것은 보고 내용을 안전하고 위협적이지 않은 주제로 국한 시키므로, 중요한 학습 영역을 건드리지 않는다[23]. 

Another common way is to choose silence and express no critical thoughts or feelings. When people choose silence or non-judgmental approaches that obscure their expert critique, important insights or feelings related to the trainee’s performance remain murky or un-expressed. This deprives the trainee clinicians, and their organizations, of in-formation that could improve how they work [35]. Avoiding critical thoughts and feelings also limits debriefings to safe-appearing, nonthreatening topics and leaves crucial areas of learning untouched [23]. 


많은 강사들은 선행 질문을 하고 훈련생을 강사가 갖고 있지만 명시적으로 말하는 것을 꺼리는 비판적 통찰력을 얻기 위해 친절한 어조로 소크라테스식 접근법을 사용해 왔다. 강사가 비판적 입장을 가지는 경우에, 판단을 위장하는 소크라테스식 질문은 역효과를 가져올 수 있다. 훈련생들은 그 질문에 혼란스러워하거나 강사가 설명해주지 않는 동기에 대해 의심을 가질 수 있다.

Many instructors, have used a Socratic approach in which leading questions are asked and a kind tone of voice is used to guide the trainee to the critical insight the instructor holds but is reluctant to state explicitly. when the instructor holds a critical judgment, Socratic questions that camouflage the judgment may backfire. The trainee may become confused by the question or (justifiably) suspicious about the instructor’s unexplained motives.


비판단적 접근은 심각한 약점을 가지고 있다. 비판단적으로 보이고 싶은 욕구에도 불구하고 얼굴 표정, 테너, 생도, 몸짓 언어 등 미묘한 신호에 의해 자신의 관점이 무엇인지 새어 나오는 경우가 많다. 무엇보다 가장 중요한 것은 그것이 비판단이 아니라는 것이다.

the nonjudgmental approach has serious weaknesses. Despite a desire to seem nonjudgmental, hints of one’s views often leak by subtle cues, such as facial expression, tenor, cadence, and body language. Furthermore and most importantly, it is not nonjudgmental.


비판단적 방식은 (판단적 방식과) 기본 가정이 동일하다. 내 말이 맞아; 난 완전한 그림을 가지고 있어; 내 일은 훈련생인 너에게 올바른 지식이나 행동을 전달하는 거야. 비판단적 접근방식은 실수는 토론의 대상이 아니며, 부끄러운 일이라는 것을 비언어적으로 전달한다. [36,37], 그렇기 때문에 강사가 비판단적 접근방식으로 얻고자 하는 바로 그 가치를 약화시킨다.

the underlying assumptions are the same: I’m right; I have the complete picture; my job is to hand off the correct knowledge or behavior to you, the trainee. the nonjudgmental approach conveys nonverbally that mistakes are not discussible, or possibly shameful [36,37], undermining the very values instructors aim to endorse with the nonjudgmental approach.


DWGJ 접근법

DWGJ approach


DWGJ 접근방식은 여러 가지 방법으로 보고의 초점을 바꾼다.3 

  • 첫째, 이것은 성인 학습자(강사 포함)가 강사에 의해 일방적으로 결정되거나 연습생과 협력하여 핵심 학습 목표를 향해 나아갈 수 있도록 심리적으로 안전한 컨텍스트를 만드는 데 초점을 맞추고 있다. 

  • 둘째, 보고의 초점은 수련자의 행동뿐만 아니라 수련자의 의미형성 체계(즉, 그들의 틀, 가정, 지식)까지 포함하도록 넓어진다. 

  • 셋째, 모의실험에 대한 사의 감지 시스템도 지형의 일부가 되어 질문의 대상이 된다(표 1 참조).

The DWGJ approach shifts the focus of debriefing in several ways.3 

  • First, it focuses on creating a psychologically safe context that enables adult learners (including the instructor) to move toward key learning objectives, determined either unilaterally by the instructor or collaboratively with the trainee. 

  • Second, the focus of the debriefing widens to include not only the trainee’s actions, but also the meaning-making systems of the trainee (ie, their frames, assumptions, and knowledge). 

  • Third, the instructor’s sense-making system about the simulation also becomes part of the debriefing terrain and open to question (see Table 1).


강사는 연습생과의 대화를 시작하기 위해 공유하는 상황에 대한 전문가의 견해를 가지고 있지만, 그것이 단 하나의 타당한 관점은 아닐 수도 있다의료 시뮬레이션 영역의 디브리핑에서 강사가 자신의 주요 우려사항를 설명하는 것은 중요하다.4 왜냐하면 특히 중요한 오류에 대해 직접적으로 언급하지 않음으로써, 나중에 실제 임상 환경으로 돌아올 때 임상 실수를 영구화하고 환자 안전을 저해할 수 있기 때문이다. 비판단적 접근방식과 대조적으로, DWGJ 접근방식에서 강사는 시뮬레이션에 대한 비판적 또는 수용적appreciative 통찰력을 명시적으로 공유한다. 그런 다음 이러한 통찰력을 다음 절과 상자 1에 설명된 대로 단계별로 연습생들과 함께 시험하고 탐구한다.

The instructor has an expert’s view of the situation that he or she shares to initiate dialog with the trainee, but it may not be the only valid view. Instructors’ stating their main concerns in a debriefing is especially important in the domain of health care simulation where being indirect about crucial errors can perpetuate clinical mistakes and undermine patient safety when the trainee returns to the real clinical environment.4 In this approach, in contrast to the nonjudgmental approach, the instructor shares critical or appreciative insights about the simulation explicitly. Then these insights are tested and explored with trainees step-by-step as illustrated in the next section and in Box 1.



이러한 '선한 판단' 접근방식전문가로서 강사의 의견을 중요시하는 동시에, 각 연습생들의 독특한 관점을 중요시하는 접근방식이다. 그 아이디어는 어떤 프레임이 연습생들의 행동을 유도하는지 알아내어, 실패와 성공을 학습자의 프레임으로 문제를 인식한다면, 독창적이며, 불가피하고, 논리적인 해결책이라고 이해하는 것이다

This ‘‘good judgment’’ approach is one that values the expert opinion of the instructors, while at the same time valuing the unique perspective of each trainee. The idea is to learn what frames drive trainee behaviors so that both their failures and successes can be understood as an ingenious, inevitable, and logical solution to the problem as perceived within their frames.


이것은 강사의 입장이 다른 세계관이나 틀과 결과적인 행동에 대해 궁금해 하는 인류학자의 자세와 같은 것이다.

This is where the instructor’s stance is like that of an anthropologist, curious about different worldviews or frames and about the resulting actions.


보고에 있어서 투명한 이야기: 옹호-선언을 통한 올바른 판단 접근법 제정

Transparent talk in debriefing: enacting the good judgment approach with advocacy-inquiry


DWGJ 프레임은 강사가 사용하는 화법에 의해 제정enact된다. 다른 모든 프레임, 정신적 모델, 또는 도식처럼, 좋은 판단 접근법의 토대가 되는 가치value는 눈에 보이지 않는다. 그것을 보는 유일한 방법은 행동으로 그 가치가 변화될 때뿐이며, 말하는 것speaking은 강사의 강력한 행동이다. 특히 효과적인 연설 스타일은 옹호와 탐구을 병행하는 것advocacy with inquiry이다. 

  • 옹호advocacy란 주장, 관찰 또는 진술인 반면, 

  • 탐구inquiry는 질문이다. 

이 두가지를 함께 짝을 지을 때, 강사는 대화적 과학자로서, 자신의 가설을 옹호하는 말로 말한 다음, 그 가설을 탐구하여 시험한다.

The DWGJ frames are enacted by the style of speaking used by the instructor. Like all frames, mental models, or schemata, the values underlying the good judgment approach are invisible; the only way to see them is when they are transformed into actions, and speaking is a powerful action for instructors. One particularly effective style of debriefing speech is to pair advocacy with inquiry. An advocacy is an assertion, observation, or statement, whereas an inquiry is a question. When pairing the two together, the instructor acts as a conversational scientist, stating in the advocacy his or her hypothesis, and then testing the hypothesis with an inquiry.


예를 들어, 강사가 '그래서, 데이먼, 나는 네가 활력징후가 악화되고 있을 때 백마스크 기구를 찾기 위해 환자로부터 물러났다는 것을 알아차렸다. 나는 환자에게 산소를 공급할 수 있는 다른 방법이 있을 것이라고 생각하고 있었(옹호). 그래서 궁금한데, 그 당시 상황을 어떻게 보셨습니까? (탐구)’’

For example, an instructor might say, ‘‘So, Damon, I noticed that you stepped away from the patient to find the bag-mask apparatus as the vital signs were deteriorating. I was thinking there possibly were alternatives means to oxygenate the patient (advocacy). So I’m curious: how were you seeing the situation at that time? (inquiry).’’


여기서 강사는 옹호+탐구를 이용하여 수련자의 행동을 guide했던 [보이지 않는 프레임]을 이끌어내고 있다. 이것은 강사들이 어떤 시나리오에서도 사용할 수 있는 일반적인 접근법이다. 단계 

  • (1) 관련 결과result를 알아차리고, 

  • (2) 결과result를 초래하는 것으로 보이는 행동action을 관찰하며, 

  • (3) 옹호-탐구 방법을 사용하여 결과를 만들어낸 프레임frame을 발견한다.

Here, the instructor is using advocacy plus inquiry to elicit the invisible frames that guided the trainee’s actions. This is the generic approach that instructors can use in any scenario: 

  • Step (1) notice a relevant result; 

  • step (2) observe what actions seemed to lead to the result; and 

  • step (3) use advocacy-inquiry to discover the frames that produced the results.


  • 이 말을 판단적 방식과 비교해보아라("데이몬, 그가 무절제하고 있다는 것을 알아차리는데 90초가 걸렸다니 믿을 수가 없어!") 또는 

  • 비판단적인 방식이라면 "내가 무슨 생각을 하고 있는지 맞춰봐라" 버전이다("그래서 데이먼, 당신이 백마스크 기구를 찾으러 갔을 때 이 환자의 포화상태는 어떠했는가?"

Compare this utterance with 

  • a judgmental version (‘‘Damon, I can’t believe it took you 90 seconds to notice that he was desaturating!’’) or 

  • a nonjudgmental ‘‘guess what I’m thinking’’ version (‘‘So, Damon, what was this patient’s saturation when you went to look for the bag-mask apparatus?’’)


옹호-탐구 방식은 강사의 관점과 우려를 명확하고 직접적으로 나타내며 데이몬이 잃어버린 장비를 찾는 데 주력했던 의미 형성 과정을 밝히기 시작했다.

The advocacy-inquiry utterance clearly and directly stated the instructor’s perspective and concerns, and set out to bring to light the meaning-making process that had Damon focused on finding missing equipment.


옹호-탐구 방식은 데이몬의 프레임을 표면화하는 데 도움이 된다. 그것은 또한 강사가 연습생들의 사고 과정을 배우도록 도와주고 더 깊은 가르침을 위한 지렛대를 제공한다. 분명히 말하면, 이 기술은 좋게 말하는 것이 아니다. 오히려 강사들의 생각, 판단, 감정을 정면이자 중심에 배치한다. 차이점은 강사의 견해를 [공개적 검증이 필요한 것]으로 처리함으로써(주장에 대한 견해를 말한 다음 질의로 다른 관점을 제시함으로써), 강사는 자신의 견해를 개방하고, 도전받게 하여, 자기자신을 학습에 노출시켜서 상호성을 증가시킨다는 것이다.

The advocacy-inquiry version helps surface Damon’s frames. It also helps the instructor learn about the trainees’ thought process and provides a lever for deeper teaching. To be clear, this technique is not about talking nicely. On the contrary, it places the instructors’ thoughts, judgments, and feelings front-and-center. The difference is that by treating the instructor’s views as requiring public testing (by saying their viewpoint in the advocacy and then inviting a different viewpoint with the inquiry), the instructor increases mutuality by opening his or her own views to challenge and making himself or herself vulnerable to learning.


표 2는 DWGJ 접근방식을 적용하는 방법의 예를 제공한다.

Table 2 provides an example of how to apply the DWGJ approach.




요약

Summary


DWGJ 접근법은 학습자가 방어적 자세를 갖거나, 강사의 비판적 판단을 추측하게 만들지 않고, 강사가 말하는 것을 학습자가 듣고 처리할 가능성을 높이기 위한 접근방식이다. DWGJ 명칭은 판단적 또는 비판단적 접근방식이 근본적으로 선한 판단이 아니라는 것을 의미하지는 않는다.

The DWGJ approach is designed to increase the chances that the trainee hears and processes what the instructor is saying without being defensive or trying to guess the instructor’s critical judgment. The DWGJ appellation is not meant to imply that the judgmental or nonjudgmental approach do not have good judgment as their basis.


우리는 접근방식의 긍정적인 특성을 강조하기 위해 "DWGJ"라는 유익한 이름을 선택했다. 이는 (흔히 판단 및 비판단 접근에서 발생하는) 잠재적 소음(오해 또는 방어성)을 줄이면서 훈련생에게 강사의 관점에 대한 명확한 신호를 제공하는 것이다.

We chose the salutary name ‘‘DWGJ’’ to highlight the positive attributes of the approach. These are, providing trainees with a clear signal about the instructor’s point of view while reducing the potential noise (misunderstandings or defensiveness) that too often is associated with the judgmental and nonjudgmental approaches.


판단적 접근법은 학생을 당황시키거나 굴욕시킬 상당한 위험을 내포하고 있으며, 비판단적 접근은 학습자에게 혼란스럽고 엇갈린 메시지를 보낼 수 있다. 두 가지 접근 방식은 모두 교사의 메시지와 연습생의 프레임의 명확성을 난독화obfuscate하거나 줄일 수 있다.

The judgmental approach poses a substantial risk of embarrassing or humiliating the student and the nonjudgmental approach may send confusing and mixed messages to the learner. Both approaches can obfuscate or reduce the clarity of the instructor’s message and the trainee’s frames.


DWGJ 접근방식은 두 가지 제약조건을 가지고 있다. 

  • 가장 중요한 것은 이 모델에서는 [trainee가 선의를 가지고 있으며, 옳은 일을 하려고 한다고 가정한다]는 것이다. 연습생이 고의적으로 태만하거나 악의적인 경우는 모델이 작동하지 않는다. 그러한 상황에서는 다른 기술들이 우월하다(카운셀링, 골 설정, 규율 등). 

  • 둘째로, 다음의 경우에는 이 접근법에 어려움을 느낄 수 있다. 권위자와 연장자에게 경의를 표하는defer 것이 가장 중요하게 여겨지는 문화에서 온 수련생들을 대할 때라든가, trainee가 강사의 견해와 모순되는 듯한 견해를 공개하는 것을 억제하는 경우이다이러한 맥락에서 DWGJ방법을 사용하기 위해서는 시뮬레이션 환경의 목표와 규범에 관한 명시적인 준비가 필요하며, 때로는 그것조차도 충분하지 않다.

The DWGJ approach has two constraints. 

  • The most important is that the model presumes that the trainee is operating with good will and is trying to do the right thing. In those rare cases where the trainee is willfully negligent or malevolent, the model does not work. In those circumstances, other techniques are superior (counseling, goal setting, discipline, and so forth). 

  • Second, instructors may find difficulty with this approach when dealing with trainees who come from cultures in which deferring to authority and elders is of paramount importance and inhibits their disclosing views that may seem to contradict those of the instructor. To support the method in this context, explicit preparation regarding the goals and norms of the simulation environment is required, and sometimes even that is not enough.


고충실도 임상 시뮬레이션의 열기와 드라마에 대한 설명에서는 주로 연습생들의 행동에 초점을 맞추기가 쉽다. 그러나 DWGJ 접근방식은 세 가지 추가 중요 분야를 강조한다. 

  • 첫째, 강사들이 인류학자 같은 질문을 하는 것이 필수적이며, 이것은 겉으로 드러나지 않는 감각 형성 과정, 인지 프레임, 그리고 수련자의 행동을 지배했던 감정들을 명확히 하는데 도움을 준다. 

  • 둘째, 강사들은 [자기 스스로에게는 보이지 않는 판단]과 [시나리오의 핵심 요소에 대한 우려]를 인식하고 명시적으로 서술하려고 노력해야 한다.

In debriefing the heat and drama of a high-fidelity clinical simulation, it is easy to focus primarily on trainees’ actions. The DWGJ approach, however, highlights three additional areas of importance. 

  • First, it is vital that instructors ask questions like those of an anthropologist, which help bring to the surface and clarify the invisible sense-making process, the cognitive frames, and the emotions that governed the trainee’s actions. 

  • Second, instructors work to become aware of, and explicitly narrate, their own invisible judgments and concerns about crucial elements of the scenario. 


그러나 강사는 자신의 판단이나 우려를 하나의 진리로 취급하는 대신, 같은 문제에 대한 수련자들의 견해에 대하여 자신의 견해를 검증test해야 한다. 이것은 강사들이 그들의 전문지식을 포기하거나, 그들의 판단을 겉보기 친절의 샌드위치로 위장하는 것을 의미하는 것이 아니라, 그들은 상황에 대한 관점을 가설hypothesis로서 말하고, 훈련생들의 관점을 정당화하고 탐구하기 위한 발판으로 사용하는 것이다. 훈련생들의 틀, 가정, 신념이 어떻게 그들이 취하는 행동을 촉진하는지를 이해함으로써, 강사들은 훈련생들에게 가장 중요한 문제와 그들의 교육목표를 일치시킬 수 있다. 마지막으로, DWGJ 접근법은 훈련생과 강사가 일반적인 임상 및 사회적 틀과 가정에서의 의도하지 않은 결과를 배우도록 돕는다.

But instead of treating their own judgments or concerns as the single truth, they test their views against the trainees’ view of the same issue. This does not mean that instructors relinquish their expertise, or disguise their judgments in a sandwich of niceties; rather, they state their view of the situation as a hypothesis and use that as a springboard to legitimize and explore the trainees’ view. By understanding how trainees’ frames, assumptions, and beliefs drive the actions they take, instructors can match their teaching objectives with problems that are most salient to the trainee. Finally, the DWGJ approach helps trainees and instructors learn of unintended consequences of common clinical and social frames and assumptions.







 2007 Jun;25(2):361-76.

Debriefing with good judgmentcombining rigorous feedback with genuine inquiry.

Author information

1
Department of Health Policy and Management, Boston University School of Public Health, 715 Albany Street, Boston, MA 02118-2526, USA. jrudolph@bu.edu

Abstract

Drawing on theory and empirical findings from a 35-year research program in the behavioral sciences on how to improve professional effectiveness through reflective practice, we develop a model of "debriefing with good judgment." The model specifies a rigorous reflection process that helps trainees surface and resolve pressing clinical and behavioral dilemmas raised by the simulation. Based on the authors' own experience using this approach in approximately 2000 debriefings, it was found that the "debriefing with good judgment" approach often sparks self-reflection and behavior change in trainees.

PMID:
 
17574196
 
DOI:
 
10.1016/j.anclin.2007.03.007


의학교육에서 문턱개념(MedEdPublish, 2018)

Threshold Concepts in Medical Education

Virginia Randall[1], Robert Brooks[1], Agnes Montgomery[1], Lauren McNally[1]




도입

Introduction


고등교육에서 문턱개념(TC)은 어떤 전문직에 특유한 사고와 추론의 방법을 설명하기 위해서 사용되며, 문턱개념을 통해 학습자는 전문직처럼 생각하고, 전문직이 될 수 있다. TC 프레임워크는 2003년에 처음 출판된 마이어와 랜드 (Meyer and Land, 2003; Land, Meyer and Smith, 2008; Meyer, Land and Baille, 2010; Land, Meyer and Flanagan, 2016)에 의해 개발되었다. 첫 번째 TC는 경제와 공학, 마이어와 랜드의 학문에 적용되었다. 그 이후 그들은 건축, 문학, 사회학, 회계학 등 다양한 직업에서 유럽, 호주, 뉴질랜드에서 연구되었다. 의료 교육에서 TC를 식별하는 소수의 연구가 최근 영국의 저자들로부터 나타났다(Neve, Wearn and Collett, 2016a; Barradeell and Peseta, 2017; Collett, Neve, Lloyd and Collett, 2017; Neve, Lloyd and Collett, 2017a).


Threshold concepts (TC) is a theoretical framework in higher education to describe ways of thinking and reasoning that are unique to a profession and enable the learner to "think like" and become a professional. The TC framework was developed by Meyer and Land (Meyer and Land, 2003; Land, Meyer and Smith, 2008; Meyer, Land and Baille, 2010; Land, Meyer and Flanagan, 2016) who first published in 2003. The first TCs applied to economics and engineering, the disciplines of Meyer and Land. Since then, they have been studied in Europe, Australia, and New Zealand in such diverse professions as architecture, literature, social studies, and accounting. A handful of studies identifying TCs in medical education have appeared recently from authors in the United Kingdom (Neve, Wearn and Collett, 2016a; Barradell and Peseta, 2017; Collett, Neve and Stephen, 2017; Neve, Lloyd and Collett, 2017b; Neve, Lloyd and Collett, 2017a).


TC에는 다음과 같은 네 가지 핵심 구성 요소가 있다.

There are four key components of a TC:




1. TC는 변혁적이다. 그것은 학습자가 자신을 개인/전문가로 보는 방식에 일대 변화를 일으킨다. (Mezirow, Taylor and associates, 2009). 학습자는 새로운 방식으로 자신을 식별하고, 새로운 방식으로 그들의 새로운 직업에 대한 그들의 역할을 인식하게 된다.

1. A TC is transformational; it causes an ontological shift in the way the learner views himself or herself as a person/professional (Mezirow, Taylor and associates, 2009). The learner comes to identify themselves in a new way, and appreciate their role in their new profession in a new way.


2. TC는 통합적이다; 그것은 전혀 관련이 없어 보이는 지식과 태도들의 조각들을 전체적으로 만들어낸다; 학습자가 깨달았을 때, "오, 그것이 그들이 말하고 있는 것이다" "이제 모든 것이 말이 된다!" 교직원으로서, 우리는 "전구가 켜졌다!"

2. A TC is integrative; it brings pieces of seemingly unrelated knowledge and attitudes into a whole, when the learner realizes, "Oh that is what they are talking about" "Now it all makes sense!" As faculty, we say "The light bulb went on!"


3. 변혁성, 통합성과 얽혀있는 것이 비가역성이다. 일단 TC를 수용하면, 그/그녀는 그것을 배우기 전으로 돌아갈 수 없다. 그것은 그들의 직업적 정체성이 되었다.

3. Intertwined with transformation and integration is irreversibility; once the learner embraces the TC, he/she cannot unlearn it. It has become their professional identity.


4. 네 번째 요소는 '성가심'이다. TC는 학습자가 자신과 자신의 역할을 경험하는 새로운 방법에 접근할 때 느끼는 불안감과 더불어 새로운 미지의 것에 대한 두려움이기도 하다. 우리는 환자에 대한 더 많은 관여와 책임을 원하지만 동시에 그들의 지식이 완전하지 않다는 것을 인식하고 실수를 할까봐 두려워하는 의대생을 본다. 의대생은 서로 상반되는 이 두 가지 존재 방식 사이에서 동요하며 화해하려고 애쓴다. 그들은 의학의 불확실성에 직면해 있다. 이 골치 아픈 단계에서 학생들은 불안해지고, 우울해지고, 냉소적이 될 수 있으며, 의대를 중퇴할 것을 고려할 수도 있다.

4. A fourth component is troublesomeness; a TC involves the angst that learners feel as they approach a new way of experiencing themselves and their role, coupled with their fear of this new unknown. We see medical students who want more involvement and responsibility for their patients, but who are simultaneously aware that their knowledge is not complete and are fearful of making a mistake. They oscillate between these two opposing ways of being and struggle to reconcile them. They are facing the uncertainty of medicine. During this troublesome phase, students may become anxious, depressed, cynical, and may contemplate dropping out of medical school.


TC의 또 다른 특징은 "한계성boundedness"이다. 즉, 얻은 지식은 해당 분야(연구에서, 의료에)에 특정된다. "담론적Discursive"이라는 말은, 어떤 분야의 언어를 사용하는 학생을 말하며, 실천공동체에 적응하는 것을 말한다. 종종 이것은 그들이 "스크립트"를 사용하고 있고 아직 이 새로운 언어의 자연스러운 사용으로 전환되지 않았다는 것을 학생이 알고 있을 때 모방하는 기간을 포함할 수 있다.

Other descriptors of TCs are "boundedness": the knowledge gained is specific to that discipline (in our study, to medical care). "Discursive" refers to the student using the language of the discipline, and fitting into the community of practice. Often times this may involve a period of mimicry, when the student is aware that they are using a "script" and has not yet transitioned to a natural use of this new language.


[자신의 TC 경험을 기억해내는 것은 숙련된 교수들에게 어렵지만], 이러한 개념이 우리 자신의 전문적 및 개인 식별에 눈에 보이지 않게 통합되어 있기 때문에 교수들에게도 중요하다(Meyer and Land, 2006). 교직원으로서, 우리는 TC의 juncture에서 막혀 있는 학생들을 찾아서, 그들의 우려에 귀를 기울이고, 비판단적인 토론을 제공하며, 무엇보다도 학생에 대한 경험이 일반적인 것임을 알려줌으로써 도울 수 있다. 의과대학 학생들이 겪는 고통은 부인할 수 없는 사실이며, 우리 모두는 우울증에 빠져 그들의 임상 시간에 의대를 떠나기로 결정하는 학생들을 보아왔다. 의대생의 고통을 다루는 방법에는 소규모 그룹 세션, 보호 환기 기회 및 반성의 지침이 포함된다(Egnew et al., 2018).

TCs are important for faculty to appreciate, although remembering one’s own TC experiences is difficult for experienced faculty, since these concepts have become invisibly integrated into our own professional and personal identify(Meyer and Land, 2006). As faculty, we can help students who may be stuck at a juncture with a TC by listening to their concerns, providing non-judgmental discussions, and most, of all, normalizing the experience for the student. That medical students suffer is undeniable, and we have all seen students who become depressed and decide to leave medical school in their clinical years. Ways of dealing with medical student suffering include: small group sessions, opportunities for protected venting, and guidance for reflection (Egnew et al., 2018).


방법

Methods


우리는 질적 분석의 기초 이론과 지속적인 비교 접근법을 사용하여 3학년 소아과 임상실습생의 성찰 에세이를 주제분석하였다. 성찰 에세이는 우리 의과대학의 커리큘럼 요건의 일상적인 부분이다. 그리고 학생들은 4년 동안 많은 것을 쓴다.

We used the grounded theory and constant comparison approaches of qualitative analysis to generate a thematic analysis of reflective essays from our 3rd year pediatric clerks. Reflective essays are a routine part of the curricular requirements at our medical school, and students write many during the course of the four years.


서면 반사에 대한 프롬프트:

Prompts for the written reflection are:


1. 임상실습 후 어떻게 변하셨습니까?

2. 의사처럼 생각할 수 있게 하는 의과대학 설립 이후 가장 중요한 개념은 무엇인가?

3. 당신이 경험한, 의사가 되는 과정에서 가장 어려운 개념은 무엇인가?

4. 당신이 올해 개발한 생명, 의학, 또는 학습에 대한 새로운 접근방법을 설명하라.

1. How have you changed since beginning your clerkship year?

2. What is the most important concept you have understood since beginning medical school that enables you to think like a physician?

3. What is the most difficult concept about being a physician you have encountered?

4. Describe any new approaches to life, medicine, or learning that you have developed this year.


에세이는 4학년 의대생 3명과 교직원 1명에게 할당되었다. 각 연구자 쌍은 동일한 과정을 따랐다. 독립적인 라인별 코딩, 해결될 때까지의 차이점 쌍 간의 논의. 연구자 3세트에 걸쳐 개발된 코드는 75개였다. 그 후 쌍 내에서 독립적인 주제 분석이 이루어졌으며, 완전한 합의가 이루어질 때까지 코드를 테마로 분류하고 주제를 논의하였다. 마지막에는 11개의 주제가 있었다. 4학년 의대생들은 TC 이론에 대해 대단한 통찰력을 보였으며, 많은 학생 투쟁으로 쉽게 자기 정체성을 드러냈다는 점에 주목할 필요가 있다. 그들은 교직원들에게 3학년 의대생의 관점을 설명할 수 있었고, 그래서 그룹들 사이에 이해를 만들어냈다.

The essays were divided among three 4th year medical students and a faculty member. Each pair of researchers followed the same process: independent line-by-line coding, then discussion between the pair of differences until resolution. There were 75 codes developed across the three sets of researchers. There followed independent thematic analysis within the pair, sorting the codes into themes and discussing the themes until there was complete agreement. At the end, there were 11 themes. It is worth noting that the 4th year medical students showed great insight into the theory of TCs, and readily self-identified with many student struggles. They were able to explain the point of view of the 3rd year clerks to the faculty member, and so created an understanding among the group.


결과/분석

Results/Analysis


학생들은 일반적으로 1-2페이지를 썼다. 많은 사람들이 TC와의 격렬한 감정적 만남을 묘사했다.

Students generally wrote 1-2 pages. Many were descriptive of intensely emotional encounters with TCs.


"똑똑해지는 것만으로는 충분하지 않아."

  • "지식이 전제조건이지만, 그 자체가 아니다."

  • "환자를 돌보는 것은 관련 지식을 아는 것뿐만 아니라 환자와의 인간관계를 발전시키는 것을 필요로 한다."

"Being smart isn’t enough"

  • "The knowledge is the prerequisite, but it isn’t the thing itself."

  • "Caring for patients requires not only knowing the relevant knowledge, but also developing a human relationship with the patient."

"환자에 관한 거야."

  • "처음부터 가끔, 그리고 언제, 내가 더 이상 내 자신을 당황하게 하거나 어떻게 일을 했는지에 대해 신경쓰지 않고, 오히려 환자를 치료하고 최선을 다하는 데 신경을 쓰기 시작하는 시점이 왔다."

  • "배고픔, 갈증, 수면, 깨끗한 집, 교육 등 내 욕구가 항상 환자의 요구에 뒤쳐질 것이라는 것"

"It’s about the patient"

  • "Sometime since the first day, and I am not sure when, there came a point where I no longer cared about embarrassing myself or how I did things, but rather I started caring abut treating the patient and doing the best I could."

  • "that my needs, whether it be hunger, thirst, sleep, a clean house or education, are always going to come second to my patient’s needs."

"인생은 공평하지 않아"

  • 그는 "젊은 시절이 있는 모든 것이 행복한 결말을 맺는 것은 아니다"라며 "삶이 항상 공평하지는 않다"고 말했다."

  • "이 경험은 나에게 어려웠다...이 여자애는 그날로 끝나는 게 아니라 인생을 시작하기로 되어 있었다. 나는 솔직히 다시 의학에 흥분하지 않는 데 시간이 좀 걸렸다."

"Life isn’t fair"

  • "It taught me that not everything with youth has a happy ending – life isn’t always fair."

  • "This experience was difficult for me…This girl was supposed to begin her life, not have it end that day. It honestly took me a little while t get excited about medicine again."

"때로는 정답이 없을 때도 있다."

  • "처음(혹은 아예) 정답이 없는 것은 매우 답답하다."

  • "치료 기준과 환자의 자율성 사이의 어려운 균형"

"Sometimes there isn’t a right answer"

  • "Not having the right answer the first time (or even at all) is very frustrating."

  • "The difficult balance between the standard of care and the patient’s autonomy."

"모든 사람을 구할 수는 없어."

  • "나는 네가 모든 사람을 구할 수 없고, 구원할 수 없다는 것을 배워야 했다. 심지어 가장 그럴 자격이 있는 사람일지라도... 이 경험은 때때로 나쁜 결과가 여러분이 할 수 있는 최선을 다했을 때도 일어난다는 것을 가르쳐 주었다."

"You can’t save everyone"

  • "I had to learn that you can’t and won’t save everyone, even those who deserve it the most…this experience taught me that sometimes bad outcomes occur even when you do the best you can."

"학습은 평생하는 것이다"

  • "나는 내가 배운 모든 것을 새로운 시각으로 다시 배우고 있다는 것을 알게 되었다."

  • "지금은 공부를 할 때, 꼭 해야 해서가 아니라 하고 싶어서 그러는 거야."

"Learning is lifelong"

  • "I find I am relearning everything I was taught with a new perspective."

  • "Now when I study, it’s because I want to, not because I have to."

"의학은 흑백은 아니고, 거의 항상 회색이다(모호하다)"

  • "의학적으로 '의학적으로' 있는 것은 많지 않다는 것을 이해하게 되었다.’"

  • "처음(또는 전혀) 정답이 없다는 개념은 여전히 매우 어렵다."

"Medicine isn’t black and white, but almost always grey"

  • "I have come to understand that not many things in medicine are ‘by the book.’"

  • "The concept of not having the right answer the first time (or at all), is still very difficult."



문턱 개념의 가장 중요한 주제는 "내가 의학이라고 생각했던 것과 현실 사이에 단절이 있다"는 것이다. 이 문턱개념은 이것과 이후의 모든 데이터 세트에서, 그리고 각 의대생 공동 투자자에 의해, 그들이 임상실습동안 마주친 것의 핵심으로 확인되었다.

The overarching theme of the threshold concepts is that "There is a disconnect between what I thought medicine was going to be and the reality." This threshold concept was identified in this and every subsequent data set, and by each medical student co-investigator as the core of what they encountered during their clerkship year.



고찰

Discussion


영국의 의대생들은 USUHS의 학생들과 확연히 다르다: 그들은 더 어리고, 의료 분야에서 경험이 적으며, 군대에 있지 않다. 그럼에도 불구하고, 이들 연구에서 드러난 TC들 사이에는 현저한 유사성이 있었다; 사실, 4개는 거의 동일하다. 표 1은 영국 연구와 우리의 연구에 기술된 TC 사이의 정합성을 보여준다.

The UK medical students are distinctly different from the students at USUHS: they are younger, have less experience in the health care sector, and are not in the military (as are the USUHS students.) Nonetheless, there was a striking similarity between the TCs that emerged from these studies; in fact, four are nearly identical. Table 1 demonstrates the convergence between the TCs described in the UK study and in our study.


두 연구 모두 불치의 증거를 발견하지 못했는데, 아마도 이러한 연구가 한 시점에 일어났기 때문일 것이다.

Neither study found evidence of irreversibility, probably because these studies occurred at a single time point.




결론

Conclusion


우리의 연구와 Collett와 Neve의 연구는 Barradell이 발견한 합성 주제, 특히 건강 과학("똑똑해지는 것은 충분하지 않다")과 실천 공동체로의 유입("Medicine is not black and white")을 설명한다. 이러한 TC는 환자를 돌보는 책임을 준비하는 데 보편적일 수 있다. 대안적으로, 이 TC들은 우리 사회와 매우 다른 의과대학 환경에서 다양할 수 있다.

Our work and that of Collett and Neve illustrate the synthetic themes found by Barradell, especially that of working anew with knowledge in the health sciences ("Being smart isn’t enough") and induction into the community of practice ("Medicine isn’t black and white"). These TCs may be universal to preparation for the responsibility of caring for patients. Alternately, these TCs may vary in medical school settings in societies very different from our own.





Threshold Concepts in Medical Education

Virginia Randall[1], Robert Brooks[1], Agnes Montgomery[1], Lauren McNally[1]

Institution: 1. Uniformed Services University of the Health Sciences
Corresponding Author: Dr Virginia Randall (virginia.randall@usuhs.edu)
Categories: Educational Theory, Students/Trainees, Teaching and Learning, Behavioural and Social Sciences, Research in Health Professions Education
Published Date: 21/08/2018

Abstract

Background - The theory of Threshold Concepts (TC) proposes that there are ideas necessary for a student to learn which enable them to think like a professional. Studies of TC in higher education have appeared since 2003. Studies in medical education are more recent.

Method - We studied TC using a qualitative analysis approach (grounded theory and constant comparison) to produce a thematic analysis of 135 de-identified reflective practice essays from students in the pediatric clerkship at our medical school.

Summary of results - Seven themes met our criteria for a threshold concept; transformative (ontological shift) and troublesome (causes angst). 2 TC in our students’ work were identical with those found by authors from the UK (“Medicine isn’t black and white,” and “Sometimes there isn’t a right answer,”) 4 TC were similar and 2 were distinct.

Discussion - Our findings suggest that there are some TC inherent (maybe essential) in personal and professional identify formation for a student moving from layperson to physician-hood, regardless of the setting of the medical school.

Keywords: Threshold Concepts; Medical student education; reflective practice


세대적 타자화: 밀레니얼 학습자에 대한 오해(Med Educ, 2019)

Generational ‘othering’: the myth of the Millennial learner

Joshua Jauregui,1 Bjorn Watsjold,1 Laura Welsh,2 Jonathan S Ilgen1 & Lynne Robins3





도입

INTRODUCTION


건강 직업 교육(HPE) 문헌에는 교육자가 수 세대에 걸친 학습자의 요구에 어떻게 실천을 적용해야 하는지에 대한 권고사항이 풍부하다. HPE 내에서 이러한 '세대'를 다룬 문헌의 확산은 비교적 최근에 일어났으며, X,1세대를 언급하였고, 더 최근에는 'Net Generation',2세대 'Generation Me',3,4세대 및 'Millenials'을 다루고 있다.1.5 특정 세대 코호트에 대한 교육적 권고사항을 제공하는 것은 [개인의 가치, 동기 및 행동이 그 개인의 생년월일을 기준으로 추정될 수 있다는 생각을 중심으로 한 일련의 믿음]세대 이론을 적용하는 것을 의미한다.

The health professions education (HPE) literature abounds with recommendations for how educators should adapt practices to the needs of generations of learners. This proliferation of generational literature within HPE has occurred relatively recently, with early references to Generation X,1 and, more recently, the ‘Net Generation’,2 ‘Generation Me’,3,4 and ‘Millennials’.1,5 Providing instructional recommendations for particular generational cohorts implies the application of generation theory, a set of beliefs centred around the idea that an individual’s values, motivations and behaviours can be surmised based upon that individual’s date of birth.


세대 이론이 만약 사실이라면, 이 이론은 학습자 중심의 교육에 대해서, 나이가 유사한 집단에 대해 도움이 될 만한 일반화를 제공한다따라서, 교수자와 서로 세대가 다른 학습자에게 주의를 기울여야 하는 권고안은 일반적으로 그들의 세대적 속성을 추정하고, 문제화한다. 예를 들면, 밀레니얼의 특성화(권한, 나르시시즘적, 기술에 끌리는)와 같은 것이다. 이렇게 문제화된 특성에 대해서 다른 세대의 구성원들이 특정 출생 코호트의 학습자들을 이해하고 효과적으로 도달할 수 있는 방법에 대한 지침을 제공한다.

If true, this theory provides generalisations about groups of individuals who are similarly aged in ways that might inform learner-centred instruction.1,3,4,6,7 Accordingly, recommendations to attend to different generations of learners generally problematise their supposed generational attributes – such as in the characterisation of Millennials as entitled, narcissistic and drawn to technology1,3,4,8 – and then provide guidance for how members of another generation can understand and effectively reach learners of a particular birth cohort.


세대이론의 기원

Origins of generation theory


세대의 개념은 가장 오래된 문헌에도 나온다: 그것은 부모에서 자녀로 지식의 소통을 포착하는 맥락에서 Ptahhotep(c. 2400 BCE)의 글에 나타나며, 사회의 원로들 사이에서 지혜가 성취되는 것이다.9 근대적 세대사상generational thinking은 18세기 후반과 19세기에 산업화와 민주화가 가족, 사회 계층 및 지리적 위치와의 전통적 유대관계를 약화시킴에 따라 일어난 사회변화의 유형에 대한 설명모델로 등장하였다.7,10 

The notion of generations can be traced to one of the oldest written texts: it appears in the writings of Ptahhotep (c. 2400 BCE) in the context of capturing the communication of knowledge from parent to child, and the achieving of wisdom amongst the elder members of a society.9 Modern generational thinking emerged in the late 18th and 19th centuries as an explanatory model for the types of social changes that took place as industrialisation and democratisation diminished traditional ties to family, social class and geographic location.7,10 


1928년 칼 만하임은 '세대의 문제'를 썼는데, 이 에세이는 '세대 이론'이라고 총칭할 수 있는 것에 대한 기초적 에세이로 남아 있다. 이 작품은 사회적, 문화적, 역사적 세대에 대한 현대적 사상을 공유한 경험이 그들의 세계관을 형성하고 그 행동들이 차례로 사회적 변화와 진보를 이끌어낸 비슷한 연령대의 개인들의 집단으로 검토했다.

In 1928, Karl Mannheim wrote ‘The Problem of Generations’, which remains a foundational essay for what can be collectively referred to as generation theory.11 This work examined contemporary ideas of social, cultural or historical generations as groups of similarly aged individuals whose shared experiences shaped their world views and whose actions, in turn, drove social change and progress.7,11 


현대 세대 이론의 많은 적용은 미국 역사에 근거하여 느슨하게 세대 스키마를 개발한 대중 사회학자 닐 하우와 윌리엄 스트라우스 덕분이다. 그들이 쉽게 알아볼 수 있는 세대 라벨의 틀에는 'GI 세대', '침묵 세대', '베이비 부머', 그리고 더 최근에는 '밀레니얼'이 포함된다. 다른 사람들에 의해 'X세대'와 'Cuspars'와 같은 추가적인 세대 라벨이 적용되었다.

Much of the modern application of generation theory can be attributed to Neil Howe and William Strauss, pop sociologists who developed a generational schema based loosely upon US history.7,12 Their easily recognisable framework of generational labels includes the ‘GI Generation’, the ‘Silent Generation’, ‘Baby Boomers’ and, more recently, ‘Millennials’.13,14 Additional generational labels, such as ‘Generation X’15 and ‘Cuspars’,5 have been applied by others.


세대이론은 '공유된 경험을 바탕으로 한 집단적 정체성을 강조함으로써 '전기와 역사의 격차를 메워라'는 찰스 라이트 밀스의 요구에 부응한다.16 한 세대 내의 멤버쉽은 더 넓은 사회문화적 틀에 사적인 경험, 인생 사건 및 기억을 앉힐 수 있는 수단을 제공하며, HPE의 학습자와 교육자 사이에 존재하는 나이 집단 간의 차이를 이해하는 틀을 제공하기 때문에 매력적이다.

Generation theory, for those with whom a generational identity resonates, answers Charles Wright Mills’ call to ‘fill[ing] the gap between biography and history’ by emphasising a collective identity based on shared experiences.16 Membership within a generation provides a means to situate private experiences, life events and memories in a wider sociocultural frame and is appealing because it offers a framework for understanding differences between age groups, such as those that exist between learners and educators in HPE.




방법

METHODS


이 검색은 반드시 비체계적이고 포괄적이지 않다. 그 대신, 그 목적은 세대간 코호트 간에 왜 알려진 차이가 존재할 수 있는지에 대한 새로운 사고방식을 자극하고, 우리의 교육과 연구 관행을 알리기 위해 세대 이론을 사용하는 잠재적인 결과를 이해하기 위한 수단으로 보건 직업 외부의 문맹으로부터 이끌어내기 위한 것이다.

this search is necessarily non-systematic and non-comprehensive. Instead, the intent is to stimulate new ways of thinking about why purported differences between generational cohorts might exist, and to draw from literatures outside the health professions as a means to understand the potential consequences of using generation theory to inform our instruction and research practices.


결과

RESULTS


건강 직업 문학에서 세대 이론이 어떻게 사용되어 왔는가.

How generation theory has been used within the health professions literature


세대차이에 대한 언급은 20세기 후반부터 HPE 문학에서 확산되기 시작했으며, 초기 연구에서는 X세대의 학습자들을 가르치는 방법에 대해 논의했고, 밀레니얼 학습자들을 다루는 더 최근의 연구가 이뤄졌다.6 우리가 찾은 연구의 대부분은 밀레니얼 학습자들을 다루고 있다. 

References to generational differences began to proliferate in the HPE literature in the late 20th century, with earlier work discussing how to teach learners from Generation X20 and more recent scholarship addressing Millennial learners.6 the vast majority of articles we found addressed Millennial learners,


'밀레니얼을 가르치는 방법'에 관한 HPE 문헌의 대부분은 일반적으로 밀레니얼 학습자의 특색을 나타내고 있다. 이들은 entitled하며, 빈번한 피드백과 칭찬이 필요하며, 테크놀로지-강화 학습에 적합하며, 멀티테스킹 학습에 적합한 짧은 집중도를 가지고 있는 것으로 묘사한다.14 밀레니얼 학습자의 믿음과 동기를 다루기 위해, 이들 연구에서는 강의시간을 줄이고,21 더 많은 온라인 학습 플랫폼을 통합하고,22 실습 학습 및 시뮬레이션 참여와 같은 핸즈온 교육 접근법 사용을 권고한다..1 교육자들은 또한 멘토링과 피드백을 할 때, 위계에 대한 이 세대의 반감을 고려하고, 더 비공식적인 업무 관계를 포용할 것을 권장한다.4,23 밀레니얼을 겨냥한 교수 채용 및 보유 전략도 이러한 개인들이 더 focused하고 기관의 목표보다 일과 삶의 균형을 추구한다는 개념과 일치시킬 것이 권장된다.

Much of the HPE literature concerning ‘how to teach Millennials’ generally characterise Millennial learners as entitled, in need of frequent feedback and praise, drawn towards technology-enhanced learning, and as having short attention spans suited to multi- tasking.1,4 To address the beliefs and motivations of Millennial learners, these articles suggest instructional approaches, such as shortening lectures,21 incorporating more online learning platforms,22 and engaging in more hands-on learning and simulations.1 Educators are also encouraged to tailor their mentoring and feedback so that they fit with this generation’s antipathy towards hierarchy and its embrace of more informal working relationships.4,23 Faculty recruitment and retention strategies targeting Millennials are recommended to align with the notion that these individuals are more focused on their work–life balance than on the goals of their institution.24


밀레니얼 학습자의 알려진 속성을 비판적으로 조사하는 문헌은 매우 적었다.31 예를 들어, 몇몇 그룹들은 밀레니얼 학습자가 선천적으로 기술에 능숙한 '디지털 원주민'이라는 개념에 이의를 제기했다.32 

We found a smaller body of HPE literature that critically examines the purported attributes of Millennial learners in the health professions.31 For example, several groups have disputed the notion that Millennial learners are ‘digital natives’ who are naturally facile with technology.32 


이 저자들은 대신 다음과 같이 제안한다. 기술에 대한 조기 노출과 편안함은 사회-경제적 지위와 같은 요인을 반영할 가능성이 높고, 기술에 대한 지속적인 강조가 오히려 점점 더 advantaged 학습자와 less advantaged 학습자 사이의 격차를 더 벌릴 수 있다고 경고한다다른 이들은 이 젊은 세대의 강점으로 추정되는 멀티태스킹이 실제로 시험 점수를 낮춘다고 강조하였다. 이 적은 문헌들은 일반적으로 맥락이 중요하다고 강조하며, 학습자의 행동은 개인의 인구 통계학으로 미리 결정되는 것보다 환경에 의해 더 많이 결정된다고 경고한다.

These authors suggest instead that early exposure to and comfort with technology are likely to reflect factors such as socio-economic status, and warn that continued emphasis on technology may widen the divide between more and less advantaged learners.33 Others have highlighted that multi-tasking, a supposed strength of this younger generation, actually lowers its test scores.33 This smaller body of literature generally cautions that context matters, and that learner behaviour is determined more by the environment than it is predetermined by the demographic of individuals.32,33


건강전문 문헌을 벗어난 세대이론의 비판

Critiques of generation theory outside the health professions literature


세대이론은 사회학문헌 7,10,16,17,34,35에서 널리 비판되어 왔으며, 이러한 비판들은 교육에 세대차이 구인을 적용하는 것에 대해 우려를 제기한다.

Generation theory has been widely criticised in the sociology literature7,10,16,17,34,35 and these critiques raise important concerns about the application of the generational differences construct in education.


세대적 고정관념을 만드는 것은 제한적이고 일반적이지 않은 데이터에 바탕을 두고 있다. 데이터와 무관하게, 전체 세대에 맞는 독특한 고정관념의 구축은 [많은 사람들 속에 내재된 내부 차이와 다양성을 무시]하며, 세대적 고정관념은 실제 차이나 행동가능한 차이보다 협소하고 특권적인 관점만을 반영할 수 있다.

the creation of generational stereotypes is based on limited, non-generalisable data; regardless of the data, the construction of a singular stereotype to fit an entire generation ignores the internal differences and diversity inherent in a large group of people, and the creation of generational stereotypes may reflect narrow or privileged perspectives more than real or actionable differences.


비즈니스 언론 34,36과 교육문학 17에서 최근 세대이론에 대한 검토가 있었지만, 1960년대와 1970년대 사회문학에 대한 광범위한 논쟁 끝에, 10,35,37–39 세대이론은 이후 2000년대까지 학술 연구에서 사라졌다. 현재 세대론의 부활은 '사회과학'이라기 보다는 '대중문화'로 보인다.34

There have been recent reviews of generation theory in the business press34,36 and education literature,17 but after extensive debate in the sociology literature in the 1960s and 1970s,10,35,37–39 generation theory disappeared from academic study until the 2000s. The current resurgence of generation theory appears to be ‘more popular culture than social science’.34


Strauss와 Howe가 밀레니얼에 대한 고정관념을 만들기 위해 사용하는 데이터 세트는 결과의 한계가 있다. 조사가 익명이 아니라 교사에 의해 관리되었기 때문에 잠재적으로 편향되어 있으며, 분석에는 응답의 45%를 차지하는 모든 '확실/모른' 답변이 포함되었다.17,40 리뷰어들은 또한 Strauss와 Howe의 주장이 종종 그들 자신의 데이터, 더 큰 데이터 세트의 주장 또는 다른 연구자들의 주장과 일치하지 않는다고 언급했다.3,17,40

The dataset used by Strauss and Howe to create the stereotype of Millennials are subject to the limitations of survey results and are potentially biased as the surveys were administered by teachers and not necessarily anonymous, and the analysis included the removal of all ‘not sure/don’t know’ answers, which comprised 45% of the responses.17,40 Reviewers have also noted that Strauss and Howe’s assertions often do not match their own data, those of larger datasets, or those of other researchers.3,17,40


세대 이론의 시작부터, 만하임은 각 연령 코호트에는 다수의 세대적 단위unit가 포함될 수 있다는 것을 인정했다. 그 단위들 자체로는 전체를 대표하지 못하며, 어떤 단위에 대해서는 일부 사람들만 접근권한이 있었을지도 모른다. 이러한 구분(세대를 시대정신이자, 결정론적이자, 메타피지컬 함의로 보는 것과 세대를 출생년도에 의해 정의되는 연령 코호트, 계보적 후예, 역사적으로 비슷한 삶의 단계와 경험을 공유하는 집단으로 보는 구분)이 세대이론을 사회학 연구로부터 분리시켰다.

Fromthe outset of generation theory, Mannheim conceded that each age cohort might contain several generational units, which themselves may not be representative of the whole, and membership of which may have been accessible to only a segment of society.11,16 This distinction, between 

    • generation as ‘zeitgeist’, with deterministic, ‘metaphysical’ connotations, and 

    • generation as ‘age cohort’, defined by birth year, genealogical descent, life stage or a shared experience of history, 

has separated generation theory fromsociological research.10,35,37


집단 간의 경계를 긋기 위한 시도를 하였으나, 인구통계학자 및 사회학자들은 [성별, 인종, 사회-경제적 지위 및 지리적 이유로 집단 내 차이가 집단 간 차이보다 더 큰 경우]가 많다는 점에 주목했다.

In attempting to draw boundaries between groups, demographers and sociologists have noted that within-group differences are often larger than between-group differences because of gender, race, socio-economic status and geography,


세대 간 구별을 뚜렷하게 만들기 위해서, 이러한 집단 내 차이를 뭉뚱그리는 것은 ageism의 초기 고찰에 있었고, 후에 다시 설명할 본질주의essentialism의 한 예이다.

Collapsing these within-group differences in favour of demonstrating clearer divides between age group has been acknowledged during these early discussions as a formof ageism41 and is an example of essentialism, to which we will return.


지식인들은 만하임 시대의 세대 집단의 중심에 자신들을 두었지만, 1960년대 이후, 세대는 주로 외부 당사자들에 의해 분류되어 왔다.10,11,16 

Intellectuals placed themselves at the centre of generational groups in Mannheim’s era, but since the 1960s, generations have largely been classified by outside parties.10,11,16 


로버트 월은 이 사실을 확인했다. '세대적 현상'은 그 자체가 역사의 특정 순간에 특정 사회의 특정 종류의 사회 집단의 반영일지도 모른다' 또한 한 세대를 정의하는 데 있어서, 세대를 정의하는 집단은 실재주의자substantialist의 오류를 범할 수 있다. 이는 '지식인들로 하여금 종종 자신의 미시적 특성을 전 세계로 확장하는 논리'를 말한다. 푸로넨은 부르디유의 사회 분류 이론을 세대 이론에 적용하여 한 집단이 다른 집단을 정의할 수 있을 때 생성된 권력의 차등, 그리고 우리가 다른 집단의 개념과 관련되는 그 정의에 편견을 가질 수 있는 동기를 식별하였다.16

Robert Wohl identified that ‘the generational phenomenon ... might itself be the reflection of a certain kind of social group in a certain kind of society at a certain moment in history’,10 and in defining a generation, the defining group may commit the substantialist fallacy, ‘the logic which often makes intellectuals extend to the whole world the characteristics of their own microcosm’.16 Purhonen applied Bourdieu’s theories of social classification to generation theory to identify the power differential created when one group is allowed to define another group, and the motivations that may bias that definition, which we relate to the concept of othering.16


고찰

DISCUSSION


우리는 세대간 차이점이 진정한 교육적 현상이라는 개념을 뒷받침할 만한 증거를 거의 찾지 못했다. 대신에 세대 집단을 특징짓는 행위, 즉 'entitled', ' 나르시시즘', '소시오패스', '기술지식', '팀워크 지향' 또는 '다양성 포용'으로 분류하는 행위들은, [라벨이 붙여진 집단]에 대한 것보다 이러한 [라벨을 적용할 힘을 가진 사람]들에 대해 더 많은 것을 말할 수 있다.

We found little evidence to support the notion of intergenerational differences as a true educational phenomenon. Instead, the act of characterising a generational group, whether as ‘entitled’, ‘narcissistic’, ‘sociopathic’, ‘tech savvy’, ‘teamwork-oriented’ or as ‘embracing diversity’, may say more about those who have the power to apply these labels than it does about the group that is being labelled. 


세대별 학습자, 그리고 더 최근에는 밀레니얼 학습자는 사회적으로 구성된 신화이다. 그리고 이 신화가 영속화되면서 권력의 차이와 고정관념은 둘 다 강화된다. 그 결과 개개인의 고유한 특성과 신념은 정당한 지위를 잃게 된다.

The generational learner, and more recently the Millennial learner, is a socially constructed myth, and it is the perpetuation of this myth that reinforces both power differentials and stereotypes, thereby delegitimising the unique attributes and beliefs of individuals.


따라서, 우리는 세대주의를 명확히 하기 위한 렌즈로서 본질주의타자화의 개념을 소개한다.

Thus, we introduce the concepts of essentialism and othering as a lens through which to clarify generationalism


우리는 HPE 문헌에서 이미 문화적 역량에서 문화적 겸손으로의 변화를 관찰했다. 세대주의도 마찬가지라면, 세대적 겸손이라는 새로운 개념의 채택을 주장한다. 우리는 [학습자의 배경과 무관하게, 모든 학습자들을 위한 실천 수단]으로 비판적 의식을 발전시키기 위하여 문화적 겸손의 권고를 반복한다.

We have observed the shift from cultural competency to cultural humility already in the HPE literature,42–44 and we view generationalism as a parallel construct and argue for the adoption of a novel concept of generational humility. We reiterate the recommendations of cultural humility towards developing critical consciousness as a means to practice for all learners, regardless of background.


본질주의는 겉으로 드러나는apparent 차이점에 근거하여 집단을 분류할 수 있다는 개념이며, 차이를 확인identify할 수 있는 한, 그 차이는 규정된 집단의 특성이 된다. 의학에서 본질주의는 범주화의 확장에서 발생한다. 범주화는 적절한 범주화(질병의 특징)부터 부적절한 범주화(외무나 출신지에 따른 사람의 범주화)까지 다양하다. 인종적 고정관념과 유사하게, 일단 연령 코호트에 대한 일련의 가정이 확립되면, 그 가정이 그 집단에 대한 우리의 인식을 형성하며, '생물학적이든 아니든, 특정 집단과 연관되어 발견되는 온갖 특징들이 그 집단에게 자연스러운 것으로 간주된다.'

Essentialism is the concept that groups can be categorised based upon apparent differences and, to the extent that differences are identifiable, that those differences are characteristic of the defined groups.45,46 In medicine, essentialism arises from the extension of categorisation from areas in which it may be appropriate, such as in characterising disease,46 to areas in which it may be inappropriate, such as in creating different categories of persons based on appearance or geographic origin, as in the social construct of race.45,47 Similar to racial stereotypes, once a set of assumptions about an age cohort is established, it can shape our perceptions of that group and ‘all manner of traits found in association with a particular group, whether biologic or not, are considered to be natural to that group’.45,48


특정 연령 코호트를 가장 효과적으로 가르치는 방법에 대한 [본질주의적인 가정]은 학습자의 개인적 강점과 도전에 대한 교사들의 호기심을 제한할 수 있다. 예를 들어, 밀레니얼 학습자가 테크놀로지에 능숙하다는 흔한 고정관념은 잘못된 결론으로 이어질 수 있다. 예를 들어, 이러한 학습자들이 정교한 소셜 미디어 문해력을 가지고 있다는 결론일 수도 있고, 각종 테크놀로지에 대한 접근성이 낮았던 disadvantaged 배경을 가진 학생에 대해서 낙인을 찍는 결론일 수도 있다.

 Essentialist assumptions about how to most effectively teach specific age cohorts may limit teachers’ curiosity about their learners’ individual strengths and challenges. For example, the commonstereotype that Millennial learners are technically savvy can lead to erroneous conclusions, such as that these learners also possess nuanced social media literacy, or stigmatisations, such as of students from disadvantaged backgrounds who may have had less access to emerging technologies. 


[세대 이론의 개념적 프레임워크에 결함이 있다는 점]과 [학습자의 세대에 기초하여 교육방식을 수정하는 것에 대한 근거 부족]을 고려한다면, 교육 방법 및 평가는 교육주제와 학습의 맥락을 목표로 해야 한다. 즉, 연령에 따라 학습자를 구분하여 여기서부터 추정한 요구나 욕구를 기반으로 해서는 안 된다. 예를 들어, 학생들이 테크놀로지 관련한 것을 선호할 것이라는 인식에 근거해서 무성찰적으로 새로운 교육 테크놀로지를 채택하는 대신, 우리는 그 테크놀로지가 의도한 학습 목표와 이러한 목표들이 달성되어야 하는 맥락과 일치하는지 평가해야 한다.

Given the flaws in the conceptual framework of generation theory and the paucity of empirical evidence for the modifying of teaching practices based on a learner’s generational label, educational methods and assessments should target the subject matter being taught and the context of learning, not the presumed needs or desires of learners in an age-based cohort. For example, instead of unreflectively adopting new educational technologies based on the perceived technology-related preferences of our learners, we ought to assess their alignment with intended learning objectives and the contexts in which these are to be achieved.


타자화는 본질주의와 상호작용하는 프로세스이다. 원래 de Beauvoir에 의해 '지배적인 남성 기준에 따라 여성을 차별화하는' 과정을 설명하기 위해 만들어진 타자화라는 용어는, 이제는 '어떻게 개인 간 구별짓기가 사회적 배제와 종속성의 뚜렷한 형태를 만들어내는가'를 설명하기 위해 더 광범위하게 적용되었다.50 지배적인 에이전트agent 그룹은 다른 대상target 그룹의 표상을 형성한다. 이 때의 기준은 에이전트 그룹의 규범적 기준과의 차이로 인한 대상 그룹의 특징이다. 이를 통해서 [사회적 공간에서 서로 다른 위치]를 반영하고자 하며, 이를 통해서 [에이전트 그룹과 대상 그룹의 대표적 구성원들 사이에 존재하는 정치적, 권력적 차이]를 강화하고자 한다.

Othering is a process that interacts with essentialism. Originally formulated by de Beauvoir to describe the process of ‘differentiating women according to a dominant male standard’,50 othering is now more broadly applied to describe how ‘interpersonal differentiation generates a distinct form of social exclusion and subordination’.50 The dominant agent group creates a representation of the other target group characterised by the latter’s differences from the agent group’s normative standard, to reflect different positions in social space and to reinforce political and power differentials between representative members of the agent and target groups.16,51–53


교수자와 학습자처럼, 만약 두 개의 그룹이 이미 분리된 사회적 공간에 존재하는 경우, 연령을 바탕으로 한 분리는 사회적 분리와 중첩되어서, 더 손쉽게 개별 학생의 정체성을 원초적인archetypal 정체성으로 응축하고, 교육와의 상대적인 지위를 더 쉽게 감소시키는 결과를 낳는다.54 교육자들은 연습생에 대한 관찰과 이들과의 상호작용을 해석할 때 [무의식적으로 세대적 가정에 의해 편향]될 수 있다. 예를 들어, 학습자가 일정 변경을 요청하거나 특정 개인 경력 기회를 옹호하는 경우, 이 요구를 세대적 고정관념을 사용하여 해석한다면, 그 자체의 장점이나 가치로 고려되기 보다는 밀레니얼 학습자의 'entitled된' 요구로 여겨져서 거부될 수 있다.

If two groups already exist in separate social spaces (e.g. HPE educators and their learners), this social separation, which overlaps a separation between age groups, readily allows for the students’ individual identities to be condensed into archetypal identities and diminishes their standing relative to that of the educators.54 Educators may be unconsciously biased by generational assumptions when they interpret their observations and interactions with trainees. For example, if learners request schedule changes or advocate for specific individual career opportunities, and those requests are interpreted using generational stereotypes, they may be denied as the ‘entitled’ demands of a Millennial learner rather than considered for their own merit or value.


최근의 문화적 역량에 관한 문헌은 HPE 공동체가 문화적 겸손의 틀을 향해 나아갈 것을 권고했다.42,43 이 저자들은 우리는 사회의 모든 구성원들에게 최상의 퀄리티 케어를 제공하기 위해 권력, 특권, 불평등의 사회적 역학을 이해하려고 하는 비판적 의식을 개발해야 한다고 주장한다

Recent literature on cultural competence challenged the HPE community to move towards a framework of cultural humility.42,43 These authors argue that we must develop a critical consciousness that seeks to understand the social dynamics of power, privilege and inequities in order to provide the best quality care for all members of society.


최근의 문헌 검토는 HPE 전체적으로 비판적 의식을 함양하는 것의 중요성을 강조한다. 이를 통해 '헬스케어 문화의 구성원들은 억압을 조장하는 검증되지 않은 가정과 싸울 수 있도록'해야 한다는 것이다.55 이를 위해서는 초점의 전환이 필요하다. 초점은 ['비인간적 고정관념'으로 저하될 수 있는 인식된 문화적 속성에 대한 지식, 기술, 태도를 가르치는 것]으로부터, [왜 이런 차이가 존재하는지 그리고 그것이 어떻게 치료에 영향을 미치는지에 대해 의문을 품게 하는 것]으로 전환되어야 한다.

Arecent literature review substantiates the importance of fostering critical consciousness within HPEas a whole so that ‘members of a healthcare culture can contend with unexamined assumptions that foster oppression’.55 This requires a shift in focus fromthe teaching of knowledge, skills and attitudes regarding perceived cultural attributes, which candegrade into ‘dehumanising stereotypes’, towards questioning why these differences may exist and howthey influence care.42,43,55


세대별 코호트에 기초한 HPE 학습자들의 고정관념을 피하기 위해, 우리는 [비판적 인식과 겸손을 바탕으로 학습자들의 고유한 요구, 가치 및 동기를 탐구하는 접근방식]인 세대적 겸손을 채택할 것을 제안한다. 세대차이의 렌즈를 통해 교육적 권고를 하게 되면, 이러한 차이를 자연스럽고 불변의 것으로 강화할 뿐이다. 대신 세대적 겸손은 교육자들이 그들의 교육적 관계에 내재된 권력의 차이에 대해 성찰을 유지하게 하고, 우리 학습자들의 인지, 감정적, 행동적 '재인간화'를 보다 깊이 생각하도록 요구할 것이다.43

To avoid stereotyping HPE learners based upontheir generational cohorts, we propose the adoption of generational humility, an approach in which critical consciousness and humility are used to explore the unique needs, values and motivations of learners. Formulating educational recommendations through the lens of generational differences reinforces these differences as natural and immutable. Generational humility would instead require educators to remain reflexive about the power differential inherent in their instructional relationships and to more deliberately consider the cognitive, affective and behavioural ‘rehumanisation’ of our learners.43



결론

CONCLUSIONS


밀레니얼 학습자를 포함하여, 세대적 원형generational archetype은 연령 코호트 내의 다양성을 무시하고, 연령층 간의 힘의 차이를 강화하는 신화다. 우리는 문화적 인식이 문화적 겸손에서 더 나아가 세대적 겸손을 포함하도록 확장할 것을 권고한다. 이를 위해서 우리는 HPE 공동체가 새로운 학습자 세대 내에 포함된 개개인을 형성하는 복잡하고 역동적인 사회적, 문화적, 역사적 상호작용을 이해함으로써 모든 학습자들을 위한 최선의 실천을 끊임없이 추구해야 한다. 세대차이에 기초한 교육적 권고안은 (그 차이가 긍정적이고 매력적인 속성이든, 부정적이고 낙인찍는 속성이든) 복잡하고 역동적인 사회적, 문화적, 역사적 영향과 개인의 요구를 무시하기 때문이다.

Generational archetypes, including the Millennial learner, are myths that minimise the diversity within age cohorts and reinforce power differentials between age groups. We recommend extending the cultural consciousness argument for cultural humility to include generational humility, in which we as an HPE community continue to seek best practices for all our learners through understanding the complex and dynamic social, cultural and historical interactions that shape the individuals in each new generation of learners. Educational recommendations based on generational differences, whether these are positive exoticised attributes or negative stigmatising ones, ignore these influences and individual needs.


REFERENCES






 2019 Sep 13. doi: 10.1111/medu.13795. [Epub ahead of print]

Generational 'othering': The myth of the Millennial learner.

Author information

1
Department of Emergency Medicine, University of Washington, Seattle, Washington.
2
Department of Emergency Medicine, Boston University, Boston, Massachusetts.
3
Department of Biomedical Informatics and Medical Education, University of Washington, Seattle, Washington.

Abstract

CONTEXT:

The health professions education (HPE) literature is replete with recommendations for how educators should adapt practices to the needs of generations of learners using generation theory to bridge perceived differences between learners and educators. Yet the evidence supporting the application of generation theory in HPE has not been critically examined. If unsubstantiated, these applications may perpetuate biases towards learners they are intended to support.

METHODS:

This paper critically reviews generation theory in the HPE literature, with particular focus on recent recommendations regarding "Millennial" learners. We used Google Scholar, MEDLINE, EBSCO, JSTOR and PsycINFO to search for articles pertaining to the origins and uses of generation theory within and outside HPE. This synthesis is presented as a preliminary understanding of how ideas of generation theory arose and permeated the HPE literature, and explores the effects of generation theory on education practices.

RESULTS:

In the HPE literature, the translation of generation theory into practice recommendations generally follows a pattern consistent with translations advanced in other literatures: broad generalisations drawn from limited data are used as evidence to support instructional approaches specifically designed for generational cohorts. Outside HPE, this application of generation theory has been criticised as a form of stereotyping that ignores the internal differences and diversity inherent in any large group of people. Accordingly, problematising the needs of generations such as "Millennial" learners in the HPE literature may perpetuate narrow or privileged assumptions by educators.

CONCLUSIONS:

Generational archetypes such as that of the "Millennial learner" are myths that perpetuate unfounded generalisations about cohorts, reinforce power differentials between age groups, and minimise the unique needs of individuals. To individualise and strengthen teaching practices in HPE, we recommend adopting "generational humility" as a means to more purposefully address the dynamic social, cultural and historical influences that shape individuals within each generation of learners.


학생과 교수의 근거기반의 학습전략에 대한 인식과 사용(Med Teach, 2019)

Awareness and usage of evidence-based learning strategies among health professions students and faculty

Felipe Pizaa,b , Jennifer Cohn Kesselheimb,c, Juliette Perzhinskyd, Joanna Drowose, Roni Gillisf, Khen Moscovicif, Theodora E. Danciug, Agnieszka Kosowskah and Holly Goodingb,i





도입

Introduction


효과적인 학습 기법에 관한 상당한 증거가 존재하지만(Ambrose et al. 2010; Gooding et al. 2017), 학생들은 비효과적인 학습 습관에 계속 의존한다(Brown et al. 2014). 수십 년 동안 인지 및 교육 심리학 분야에서 연구한 결과, 학습이 고달플 때 더 깊고 오래 지속된다는 것이 밝혀졌다(Dunlosky et al. 2013). 

While a significant body of evidence exists regarding effective learning techniques, (Ambrose et al. 2010; Gooding et al. 2017), students continue to rely on ineffective study habits (Brown et al. 2014). Decades of research in the fields of cognitive and educational psychology have found that learning is deeper and longer lasting when it is arduous (Dunlosky et al. 2013). 

  • 상연습(기억에서 사실을 불러오기), 

  • 간격두기(주기적 실행), 

  • 생성하기(해결법을 교육받기 전에 문제를 만들고 해결) 및 

  • 터리빙(공부하면서 여러 과목이나 주제를 섞는 것)

...은 각각 증거 기반 학습 전략으로서, 지식의 강하고 내구성이 높은 보유로 이어진다(Dunlosky et al. 2).013). 그러나 반복읽기, 강조표시 및 매스작업(크램밍)은 학부생들 사이에서 선호하는 학습전략이 되는 경우가 많다(Amlund et al. 1986).

  • Retrieval practice (recalling facts from memory), 

  • spacing (periodic practice), 

  • generation (creating and solving problems before being taught the solution), and 

  • interleaving (mixing multiple subjects or topics while studying) 

...are each evidence-based learning strategies that lead to stronger and more durable retention of knowledge (Dunlosky et al. 2013). However, rereading, highlighting and massed practice (cramming) are often the preferred study strategies among undergraduate students (Amlund et al. 1986).


이러한 인지과학 기반 학습 습관이 광범위한 학습자에게 가장 잘 작용한다는 강력한 증거에도 불구하고(Dunlosky et al. 2013), 많은 사람들은 개인이 시각, 글, 청각 또는 운동 감각과 같은 뚜렷한 학습 스타일을 가지고 있다고 계속 믿고 있다. 교수진이 학생들의 뚜렷한 학습 스타일에 맞춰야 한다는 생각은 교육 분야에서 더욱 집요한 신화 중 하나인데, 이는 교육 방식이 학생들이 선호하는 스타일과 일치할 때 더 잘 배운다는 과학적 증거가 거의 없기 때문이다(Riener and Willingham 2010).

Despite strong evidence that these cognitive sciencebased study habits work best for a broad range of learners (Dunlosky et al. 2013), many people also continue to believe that individuals have distinct learning styles such as visual, written, auditory or kinesthetic. The idea that faculty should match their teaching to students’ distinct learning styles is one of the more persistent myths in the field of education, as there is little scientific evidence that students learn better when the manner of instruction matches their preferred style (Riener and Willingham 2010).


메타인지(metacognition)는 자신의 학습에 대한 능동적인 모니터링을 수반하는 고차원적 의식 과정을 말한다(Flavell 1979). 메타인지는 학생들이 사용중인 학습 전략의 효과를 성찰할 수 있도록 하며, 더 높은 학업 성취도와 관련이 있다(Hartman 2001; Okoza et al. 2013; Jones and Idol2013). 이와는 대조적으로 효과적인 학습 전략에 대한 인식 부족은 고등학교(Maqsud 1997)와 대학생들(Hartwig와 Dunlosky 2012)의 학업성취도 저하와 관련이 있다. 증거 기반 학습 전략의 이해와 사용은 건강 직업 학생들에게 특히 중요하다. 왜냐하면 학생들이 독립적인 practice으로 전환되면 스스로 조절regulate하는 평생 학습에 참여해야 하기 때문이다(Cutreet al. 2017). 

Metacognition refers to a process of a higher-order consciousness that involves active monitoring of one’s own learning (Flavell 1979). Metacognition enables students to reflect upon the effectiveness of the learning strategies they use and is associated with higher academic achievement (Hartman 2001; Okoza et al. 2013; Jones and Idol2013). In contrast, a lack of awareness of effective learning strategies is associated with poor academic performance among both high school (Maqsud 1997) and college students (Hartwig and Dunlosky 2012). Understanding awareness and use of evidence-based learning strategies are especially important for health professions  students, as they will need to engage in self-regulated lifelong learning once they transition to independent practice (Cutreret al. 2017). 


효과적인 학습 전략에 대한 인식을 평가한 대부분의 발표된 연구는 모든 학습 기술, 지식 및 학습 조절를 포괄하는 Metacognition Awareness Inventory를 사용해 왔다. 

Most of the published studies that have assessed awareness of effective learning strategies have used the Metacognition Awareness Inventory, which encompasses all learning skills, knowledge and regulation of learning. 


Methods


Study design


Ethical approvals


Instrument


우리는 인지 심리학자들이 개발한 설문지를 사용하여 대학생들 사이의 증거 기반 연구 전략의 준수를 평가하였다(Kornell andBjork 2007; McCabe 2011; Hartwig and Dunlosky 2012; Morehead et al. 2016). 하버드 의대 의대생 10명과 교직원 10명을 대상으로 도구의 명확성, 흐름, 완료 시간 등을 평가하기 위해 설문조사를 시범 실시하였다. 우리는 우리 인구에 대한 사례 시나리오의 신뢰성을 개선하기 위해 의학 용어를 포함시키기 위해 사소한 언어 조정을 했다. 최종 평가 도구는 보충 부록 3(학생)과 보충 부록 4(교수)에 포함되어 있다. 설문 조사의 마지막에, 참가자들은 효과적인 학습 전략에 관한 유튜브 비디오를 보는 초대를 받았다. 설문 데이터는 Qualtrics Copyright (Provo, UT, USA, 2017)의 # 버전인 Qualtrics 소프트웨어를 사용하여 수집되었다. 

We used a questionnaire developed by cognitive psychologists to assess adherence to evidence-based study strategies among undergraduate college students (Kornell andBjork 2007; McCabe 2011; Hartwig and Dunlosky 2012; Morehead et al. 2016). We conducted pilot testing of the survey among 10 medical students and 10 faculty at Harvard Medical School to assess the instrument’s clarity,flow, and time for completion. We made minor language adjustments to include medical terminology to improve authenticity of case scenarios for our population. The final survey instruments are included in Supplementary Appendix 3 (students) and Supplementary Appendix 4 (faculty). At the end of the survey, participants received an invitation to view a YouTube video on effective study strategies. Survey data was collected using Qualtrics software,# Version [01/2018] of Qualtrics Copyright (Provo, UT,USA, 2017). 



Study population


Statistical analysis


근거 기반 학습 전략의 인식은 학습 스타일에 대한 한 가지 질문과 아래에 설명된 네 가지 사례 시나리오에 대한 가상의 응답을 사용하여 측정되었다. 우리는 표준 편차와 함께 비율과 수단을 이용한 서술적 통계를 제시한다.

Awareness of evidence-based learning strategies was measured using one question about learning styles and hypothetical responses to four case scenarios, described below. We present descriptive statistics using proportions and means with standard deviations.


설문지에는 생성, 검색, 인터리빙 및 간격과 같은 근거 기반 학습 전략에 대한 참가자의 인식을 평가하기 위한 4가지 사례 시나리오가 포함되었다(보조 부록 3 및 4). 참가자들은 각 사례 시나리오에 대해 1점의 비효과적인 것부터 7점의 효과적인 것까지 7 포인트 리커트 척도로 두 가지 가능한 연구 옵션을 평가했다. 각 시나리오에 대한 증거 기반 및 비증명 기반 반응에 주어진 평균 등급을 코헨의 d와 비교하기 위해 t-테스트를 사용했다. 그런 다음 각 사례 시나리오에 대해, 근거 기반 옵션이 비증거 기반 옵션보다 더 높은 등급이 부여된 경우 참가자의 반응을 0으로, 비증거 기반 옵션이 증거 기반 옵션보다 더 높은 등급이 부여된 경우 1로 코드화했다. 그런 다음, 우리는 키 제곱 테스트를 사용하여 증거 기반 옵션에 더 높은 점수를 주는 학생과 교직원의 비율을 정확하게 비교했다.

The questionnaire included 4 case scenarios to assess participants’ awareness of evidence-based learning strategies such as generation, retrieval, interleaving and spacing (Supplementary Appendices 3 and 4). Participants rated two possible study options for each case scenario on a seven-point Likert scale ranging from 1¼very ineffective to 7¼very effective. We used t-tests to compare the mean rating given to the evidence-based and non-evidence based responses for each scenario and report effect sizes with Cohen’s d. For each case scenario, we then coded participants’ responses as 0 if the evidence-based option was given a higher rating than the non-evidence-based option and 1 if the non-evidence-based option was given a higher rating than the evidence-based option. We then compared the proportion of students and faculty correctly giving a higher rating to the evidence-based option using a chi squared test.


결과

Results


참가자 특징

Participant characteristics


그림 1과 같이 전체 조사 응답률은 205/학생(679/2636, 26%)이 교수 1042, 20%보다 높았다.

As shown in Figure 1, the overall survey response rate was higher among (205/ students (679/2636, 26%) than faculty 1042, 20%).


학생 및 교직원 참가자의 특성은 표 1과 같다.

Characteristics of the student and faculty participants are shown in Table 1.





증거 기반 학습 기법에 대한 학생 및 교직원 인식

Student and faculty awareness of evidence-based study techniques


학생들은 증거 기반 연구 기법을 자주 사용하고 있다고 보고하지 않는다(표 2).

Students infrequently reported using evidence-based study techniques (Table 2).


대부분의 학생(558/679, 82%)이 자신의 학습 습관이 선생님에게 영향을 받지 않았다고 보고했지만(표 2) 대다수의 교직원들은 수업시간에 학습 기법을 논의하거나(125/205, 52%) 학생들에게 효과적인 학습 전략을 권고했다고 말했다(125/157, 80%)

Although most students (558/679, 82%) reported their study habits were not influenced by their teachers (Table 2), the majority of faculty said they discussed study techniques in class (106/205, 52%) or recommended effective study strategies for their students (125/157, 80%).


마지막으로, 교수진의 142/156명(91%)은 학생들이 서로 다른 학습 스타일을 가지고 있다는 믿음을 지지했고, 112/141명(79%)은 이러한 차이점을 수용하기 위해 그들이 가르치는 것을 보고하였다(표 3).

Finally, 142/156 (91%) of faculty endorsed the belief that students have different learning styles and 112/141 (79%) reported they teach to accommodate those differences (Table 3).


표 2와 3에서 보듯이, 학생들이 교사들의 학습 조언을 따랐다고 보고한 비율, 이전 자료로 돌아온 비율(공부할 때, 학생이 있을 때 또는 교수할 때, 교수할 때), 그리고 학습 스타일을 믿었던 비율과 관련하여 학생과 교직원들 사이에는 주목할 만한 차이가 있다. 

As shown in Tables 2 and 3, there are notable differences between students and faculty regarding 

    • following the proportion who reported students followed teachers’ study advice, 

    • the proportion who returned to earlier material (either when studying, if a student or when teaching, if faculty), and 

    • who believed in learning styles

이러한 차이는 교수진이 보고하는 것과 학생들이 하는 것과 각 그룹이 학습 스타일에 대해 인식하는 것 사이의 불일치를 나타낸다.

These differences depict a mismatch between what faculty are reporting and what students are doing and what each group perceives regarding learning styles.





증거 및 비증거 기반 사례 시나리오의 학생 및 교수들의 평가

Student and faculty ratings of evidence and non-evidence based case scenarios


교수진과 학생 모두 평균적으로 그러한 사례 시나리오의 경우 비증거 기반 옵션보다 생성, 검색 관행 및 간격의 증거 기반 옵션을 더 높게 평가했다(표 4). 그러나 인터리빙 시나리오의 경우 교직원 및 학생 모두 증거 기반 시나리오 옵션을 비증거 기반 옵션보다 낮게 평가했다.

Both faculty and students on average ranked the evidence-based options of generation, retrieval practice, and spacing higher than the non-evidence-based option for those case scenarios (Table 4). However, for the interleaving scenario, both faculty and students ranked the evidence-based scenario option lower than the non-evidence-based option.


대부분의 교직원과 학생들은 교직원과 교직원의 통계적 차이(v2 1⁄4 0.19, V 1⁄4 0.01) 없이 인터리빙 시나리오에서 비증명 기반 옵션에 더 높은 등급을 잘못 할당하였다.

The majority of both faculty and students incorrectly assigned a higher rating to the non-evidence-based option in the interleaving scenario, with no statistical difference between students and faculty (v2 ¼ 0.19, V ¼ 0.01) (Table 5).





고찰

Discussion


전반적으로 우리의 연구결과는 동일한 조사(Kornell and Bjork 2007; Hartwig 및 Dunlosky 2012; Morehead et al. 2016)를 사용하여 이전에 다른 학부에서의 교육에 나타난 패턴을 따르고 있으며, 몇 가지 주목할 만한 차이점도 있다. 비록 우리 표본의 43%의 학생들이 여전히 시험을 위해 벼락치기를 하고 있다고 보고했지만, 건강 직업 학생들은 타 전공의 대학 모집단에 비해 spacing 기술을 더 많이 사용하고 벼락치기의 사용은 더 낮다고 보고했다. 이것은 우리의 연구가 직접적으로 이것을 탐구하지는 않았지만, 건강관리 학생들은 시간이 흐르면서 공부하고 있는 정보를 유지하려는 의욕이 더 강하기 때문일 것이다. (대학생들에 비해 건강 직업 학생들 사이에서) 도표, 그림, 차트를 만드는 것은 더 인기가 있는 반면, 플래쉬카드를 사용하는 것은 덜 인기가 있다.

Overall our findings follow patterns previously shown in undergraduate general education using the same survey (Kornell and Bjork 2007; Hartwig and Dunlosky 2012; Morehead et al. 2016), with a few notable differences. Health professions students reported greater use of the spacing technique and lower use of cramming compared to the college populations, although 43% of students in our sample still reported cramming for tests. This may be because healthcare students are more motivated to retain the information they are studying over time, although our study did not explore this directly. Making diagrams, pictures or charts is more popular, while using flashcards is less popular, among health professions students compared to college students.


대학생들이 인지과학 조사의 원천인 경우가 많은 반면, 건강 직업 학생들은 이 영역에서 덜 연구되어 왔다. 학부 모집단을 위해 개발된 조사를 사용함으로써, 우리는 증거 기반 학습 기법의 인식과 사용이 전문 학교에의 진출과 함께 자연스럽게 개선되지 않는다는 것을 증명할 수 있다. 이것은 [효과적인 교육 전략을 갖도록 교사들에게 공식적인 훈련을 요구]하기 보다는 [교사들을 콘텐츠 전문가로 보는 교육 시스템]의 더 큰 문제를 보여주는 것일 수 있다. 본 연구의 교수진들은 학생들에 비해 증거 기반 교육 기법을 더 잘 사용한다고 보고한 반면, 그들은 계속해서 스스로 배우는 방식에 있어 spacing을 덜 활용하고, 반복읽기, 밑줄 긋기, 강조하기 등을 사용하며, 이는 증거 기반 교수에 대한 훈련이 부족하다는 점을 반영할 가능성이 높다. 우리 조사에 참여한 교직원의 절반 가량이 수업 시간에 학습 기법에 대해 토론하고 있다고 보고했지만, (학부생들의 설문조사에서와 비슷하게) 대다수의 학생들은 학습 방법에 관한 교직원의 지시에 따랐다고 보고하지 않았다. 효과적인 학습 전략에서 교수진의 전문지식이 부족한 점을 감안할 때 이것은 불합리하지 않을 수 있다.

While college students are often the source population in cognitive science investigations, health professions students have been less frequently studied in this domain. By using a survey developed for undergraduate populations, we are able to demonstrate that awareness and use of evidence-based learning techniques do not naturally improve with advancement to professional school. This may signal a larger problem of an educational system that views teachers as content experts, rather than requiring teachers to have formal training in effective educational strategies. While the faculty in our study reported better use of evidence-based educational techniques compared to the students, they continued to underutilize spacing in their own teaching and to endorse rereading, underlining, and highlighting, likely reflecting their lack of training in evidence-based teaching. While about half of the faculty in our survey reported discussing study techniques in class, the majority of the students did not report following faculty instruction regarding how to study, a similar pattern as seen in the surveys of undergraduate populations. This may not be unreasonable given the lack of faculty expertise in effective learning strategies.


마지막으로, 우리는 대학 교수진과 유사한 건강 직업 교수진에서 학습 스타일 이론에 대한 주목할 만한 지지를 발견했다(Morehead et al. 2016). 이러한 이론들은 1970년대에 널리 보급되었고, 종종 학생들이 선호하는 시각, 청각 또는 운동 기술과 같은 콘텐츠 표현 방식에 대한 교육을 주장했는데, 우리 조사와 대학 조사의 교수들도 이를 지지했다. 이것은 이 분야에서 교수개발에 대한 큰 필요성을 제시한다.

Finally, we found a notable endorsement of learning styles theory in health professions faculty, which is similar to college faculty (Morehead et al. 2016). These theories became popular and widespread in the 1970s and often advocated teaching to students’ preferred mode of content presentation such as visual, auditory or kinesthetic, which the faculty in our survey and the college surveys also endorsed. This presents a major need for faculty development in this area.


듀얼코딩이론은, 학습 스타일 이론보다 근거에 기반을 둔, 교육자를 위한 옵션이다(Sadoski 및 Pivio 2012). 이 이론에 따르면, 언어와 비언어적 내용은 각 채널의 정보에 대한 뚜렷한 표현을 만들어 내는 서로 다른 신경학적 경로를 사용하여 처리된다. 시각적 및 청각적 방법으로 정보를 제시하면 피질 영역에서 정보를 장기간 보존하고 코딩 및 통합할 수 있어 학생들이 단어 또는 이미지에서 개별적으로 또는 동시에 정보를 검색할 수 있다(Bruny e et al. 2008). 그런 다음, 교수진은 이중 코딩을 간격, 시험, 인터리빙을 포함한 다른 증거 기반 전략과 결합하여 학습을 강화할 수 있다.

Dual-coding theory is an option for reframing learning styles theories within a more evidencebased approach for educators (Sadoski and Paivio 2012). According to this theory, verbal and non-verbal contents are processed using different neurological pathways creating distinct representations for information in each channel. Presenting information in both visual and auditory ways enables coding and consolidation of information to long-term retention in cortex areas, permitting students to retrieve the information either from words or images individually or both simultaneously (Bruny e et al. 2008). Faculty can then combine dual-coding with other evidence-based strategies including spacing, testing, and interleaving to enhance learning.


학생 개개인의 선호와는 반대로, kinesthetic teaching은 교육내용(즉, 절차)에 적절하게 매칭될 수 있다. 

Kinesthetic teaching can be appropriately matched to the content to be taught (i.e.procedures) as opposed to individual student preferences. 


학생들의 결과를 교수 결과와 비교하는 것은 두 가지 목적에 도움이 된다. 첫째, 교수진은 자신이 모르는 것을 성공적으로 가르칠 수 없다(Dhaliwal 2013). 교수들이 갖고 있는 교육기술의 맹점을 파악함으로써 이 분야에서 추가적인 훈련을 추구할 수 있다. 둘째로, 교수진과 학습자 관점 모두에서 잘못된 인식을 확인하면 효과적인 해결책을 만들기 위해 지식 격차의 정도를 보다 정확하게 파악할 수 있다. 

Comparing student results with faculty results serves two purposes. First, faculty cannot successfully teach what they do not know themselves (Dhaliwal 2013). Identifying blind spots in teaching skills allows faculty to pursue additional training in this arena. Second, identifying misconceptions from both the faculty and learner perspectives paints a more accurate picture of the extent of the know-ledge gap in order to create effective solutions. 


이 연구는 인지과학의 이러한 핵심 교육 개념을 다루는 교직원 및 학생들을 위한 공식적인 커리큘럼의 개발 필요성을 뒷받침한다. 

This study supports the need to develop formal curricula for faculty teachers and students addressing these key educational concepts from the cognitive sciences. 


잠재적 및 실제적 영향

Potential and practical impact


우리는 증거 기반 학습 기법에 관한 높은 수준의 오해로 인해 메타인지 (인식)의 함양이 학생과 교직원 개발 프로그램의 핵심 요소가 될 것이라고 확신한다. 메타인지를 촉진하기 위해서는, 메타인지가 존재하며, 이는 인지와 다르고, 이것이 학업성취도를 높여준다는 것을 학습자들 사이에서 인식하게 하는 것에서 시작된다(Maqsud 1997; Hartman 2001, 2013; Hong et al. 2015). 교육 과정을 계획하는 동안, 교육자들은 학생들에게 내용 외에 더 높은 수준의 사고 절차를 배우고 촉진하는 방법을 가르치는 데 초점을 맞춰야 한다. 그런 다음, 교수진은 학생들이 인지 능력을 개발할 수 있도록 함으로써 의료 전문지식의 개발을 촉진할 수 있다(Hartman 2001, 2013).

We posit that fostering metacognitive awareness will be a key component in student and faculty development programs due to high levels of misconception regarding evidence-based learning techniques. Promoting metacognition begins with building awareness among learners that metacognition exists, differs from cognition, and increases academic success (Maqsud 1997; Hartman 2001, 2013; Hong et al. 2015). While planning a curriculum, health professions educators should focus on teaching students how to learn and facilitating higher-order thinking procedures in addition to content. Faculty can then foster the development of medical expertise by enabling their students to develop metacognitive capabilities (Hartman 2001, 2013).


예를 들어, 펜 주 의과대학은 학생들 사이에서 메타인지를 장려하는 동시에 효과적이고 효율적으로 배우기 위한 학습 전략을 제공하는 인지 능력 프로그램을 개발했다. (https://students.med.psu.edu/academics/cognitive-skills-program).

Penn State College of Medicine, for example, developed a cognitive skills program that promotes metacognition among their students while equipping them with study strategies to learn effectively and efficiently (https://students.med.psu.edu/academics/cognitive-skills-program).


효과적인 교육에 기여하는 주요 특징 중 하나는 [의료전문직 교육에서 다양한 목표를 달성하기 위한 다양한 교육 방법의 활용]과 [메타인지 원칙을 실용적인 프레임워크와 통합하는 것]이다(Steinert et al. 2016).

Among the key features that contribute to effective teaching are the integration of metacognition principles with practical frameworks and use of different instructional methods to achieve diverse goals in health profession education (Steinert et al. 2016).


의료 전문가로서 우리의 궁극적인 목표는 환자들의 관리와 안전을 개선하는 것이다. 메타인지 기술은 비판적 사고와 환자 문제 해결(Medina et al. 2017), 자기조절 평생 학습(Cutrer et al. 2017)에 중요하므로 이 과제에 도움이 될 수 있다(Zang 2002; Medina et al. 2017). 메타인지적 마인드를 갖는 것은 비판적 사고 인식을 높이고 보다 정확한 자기 평가를 촉진함으로써 의료 오류를 최소화할 수 있다(Medina et al. 2017).

Our ultimate goal as healthcare professionals is to improve care and safety for our patients. Metacognition may help this task as metacognitive skills are important for critical thinking and patient problem solving (Medina et al. 2017), as well as self-regulated lifelong learning (Cutrer et al. 2017), have argued that medical errors are partially a cognitive issue (Zhang 2002; Medina et al. 2017). Being metacognitively mindful can minimize medical errors by increasing critical thinking awareness and promoting more accurate self-assessment (Medina et al. 2017).


한계

Limitations



결론

Conclusions


[지금껏 과학이 효과적인 교육과 학습에 대해 밝힌 것]과 [우리의 보건전문직 학생들과 교수들이 보고하는 것] 사이에는 큰 차이가 있다.

There is a wide gap between what science has learned about effective teaching and learning and what our health professions students and faculty participants report doing.


우리의 연구결과는 교수진과 학생들이 교육과 학습에 대한 그들의 가정을 재검토하고 바람직한 어려움을 포용할 수 있는 기회를 제공한다. 건강전문 커리큘럼에서 인지과학을 활용해 의학 교육을 새로운 시대로 끌어올려야 할 때다.

Our findings present an opportunity for health professions faculty and students to reexamine their assumptions about teaching and learning and embrace desirable difficulties. It is time to bring medical education to a new era by using cognitive science in health professions curricula.


  • 메타인지: 자신의 인지 과정에 대한 개인의 인식이다. 학습에 관해서, 메타인지란 자신이 배운 것과 배우지 않은 것에 대해서 개개인이 얼마나 인식하는가로 해석될 수 있다. 자기 조절과 학습 지도에 있어 학습자에게 인식은 필수적이다(Seel 2012a).

  • 증거 기반 학습: 학습 성과를 도출하기 위해 경험적으로 입증된 접근법, 프로세스 및 전략의 종류를 설명한다(Seel 2012b).


  • Metacognition: Is an individual’s awareness of his or her own cognitive processes. With respect to learning, this can be interpreted as an individual’s awareness of what they have and have not learned. Metacognition is essential for learners in order to self-regulate and guide learning (Seel 2012a).

  • Evidence-based learning: Describes a class of approaches, processes, and strategies that have been empirically demonstrated to produce learning outcomes (Seel 2012b).













 2019 Aug 13:1-8. doi: 10.1080/0142159X.2019.1645950. [Epub ahead of print]

Awareness and usage of evidence-based learning strategies among health professions students and faculty.

Author information

1
a Critical Care Department, Hospital Israelita Albert Einstein , São Paulo , Brazil.
2
b Harvard Medical School, Harvard University , Boston , MA , USA.
3
c Dana-Farber/Boston Children's Cancer and Blood Disorders Center, Harvard Medical School, Harvard University , Boston , MA , USA.
4
d College of Medicine, Central Michigan University , Mount Pleasant , MI , USA.
5
e Charles E. Schmidt College of Medicine, Florida Atlantic University , Boca Raton , FL , USA.
6
f Moshe Prywes Center for Medical Education and Goldman School of Medicine, Faculty of Health Sciences, Ben-Gurion University of the Negev , Israel.
7
g School of Dentistry, University of Michigan , Ann Arbor , MI , USA.
8
h School of Medicine in Katowice, Medical University of Silesia , Katowice , Poland.
9
i Division of Adolescent and Young Adult Medicine, Boston Children's Hospital , Boston , MA , USA.

Abstract

Introduction: Learning is essential and life-long for faculty and students. Often students and teachers use ineffective learning strategies and are not aware of evidence-based strategiesMethods: A multicenter, international, cross-sectional, online survey-based assessment of awareness of evidence-based learning strategies among health professions students (n = 679) and faculty (n = 205). Results: Students endorsed many study habits which violate evidence-based principles, including studying whatever is due soonest (389/679, 57%), failing to return to course material once a course has ended (465/679, 68%), and re-reading underlined or highlighted notes (298.679, 44%). While the majority of faculty surveyed (125/157, 80%) reported recommending effective study strategies for their students, most students (558/679, 82%) said they did not study the way they do because of instruction from faculty. The majority of faculty (142/156, 91%) and students (347/661, 53%) believe students have different learning styles. Discussion: The results of this study demonstrate health professions students continue to use many ineffective study strategies, and both students and faculty hold misconceptions about evidence-based learning. While planning a curriculum, medical educators should focus on teaching students how to learn and use higher order thinking procedures in addition to teaching content.

PMID:
 
31407930
 
DOI:
 
10.1080/0142159X.2019.1645950


인게이지먼트와 프로그램적 평가를 통해서 본 멘토링의 복잡성(Med Educ, 2019)

The complexity of mentoring observed through engagement with programmatic assessment

Heeyoung Han1 & Ahreum Ju2




멘토링의 개념은 호머의 오디세이(Odyssey)까지 거슬러 올라가지만, 그 방식practice은 여전히 성숙하고 있다.1

The concept of mentoring can be traced back to Homer’s Odyssey, yet the practice is still maturing.1


직장 환경이 진화함에 따라, 멘토링이 제공되는 맥락이 다양해졌으며, 이렇게 맥락이 다양해짐에 따라 [멘토링과 그에 따른 교육적 영향을 확장하고 상세히 이해하기 위해서는] 다양한 상충되는 관점을 조사해야 할 필요가 생겼다.

as the workplace environment has evolved, mentoring has come to be situated in diverse contexts that require the investigation of various conflicting perspectives to extend and elaborate understanding of mentoring and its educational impacts.


몇몇 멘토들은 양육nurturing과 평가assessment라는 상반되는 패러다임 사이에서 역할 충돌을 경험했다. 이러한 갈등은 멘티들의 결함deficiencies에 초점을 맞춘 평가가 본질적으로 [권력 거리와 긴장을 유발]하기 때문에 다루기가 까다로우며, 이로 인해 학습자가 관계적 정신사회적 서포트에 대해 어떻게 인식하느냐에 영향을 미칠 것으로 예상할 수 있다. 관계적 정신사회적 서포트는 교육자가 학습자의 형성적 발달을 돕는 데 있어서 필수적이다.

some mentors experienced role conflicts between the different paradigms of nurturing and assessment. Such conflicts are challenging because assessment that focuses on mentees’ deficiencies inherently creates power distance and tensions, which can be expected to affect learners’ perceptions of relational psychosocial support, a key feature in the educator’s ability to help learners develop formatively.


멘토링이야말로 양육적 학습 환경을 조성하는 데 결정적이지만, 멘토링 관계의 역학을 세심하게 고려하지 않을 경우 바람직하지 않은 경험으로 이어질 수 있기 때문에, 멘토링 자체가 반드시 양육적인 것은 아니다.6–8 비효율적인 멘토링 관계로부터 아무런 혜택도 받지 못하고, 더 나쁘게는 갈등으로 끝날 수 있다.

although mentoring is crucial to the creation of a nurturing learning environment, mentoring practices themselves are not necessarily nurturing as they can lead to undesirable experiences if the dynamics of mentoring relationships are not carefully considered.6–8 It is possible to end up without any benefit or, worse, with conflict from an ineffective mentoring relationship.


멘토링이 [힘의 차이]라는 맥락에서 관계를 형성하는 것을 고려할 때, 멘토링의 복잡성은 조직의 다양성과 교차성intersectionality에 의해 발생한다. 여기에는 추가적으로 성별, 인종, 기관 문화가 작동하며, 이것들은 [멘토링이 실행되는 작업장 기반 평가(형식 또는 요약)의 유형과 무관하게] 영향을 미칠 것이다.

Given that mentoring involves relationship building in a context of power differences, complexities are introduced by diversity and intersectionality in an organisation, with gender, race and institutional culture being three additional factors that can be anticipated to have influence10–12 regardless of the type of workplace-based assessment (formative or summative) in which mentoring relationships are practised.


여성들은 멘토를 찾는 데 어려움을 겪는 경향이 있다. 9 아마도 그들이 남성 중심의 직장 문화에서 '라이징 스타'로 인식되는 경우는 거의 없기 때문일 것이다. 뮬러 외 연구원에 따르면, 남성 레지던트들은 여성 레지던트들보다 [응급의학과 의사로서 필요한 개인적 자질을 어떻게 개발해야 하는가]에 대해 일관된 피드백을 받는다. 반면 여성 레지던트들은 똑같이 개인적 자질이 없을 때에도 더 강한 비판을 받는 경향이 있다.

Women tend to have a harder time finding mentors,9 perhaps because they are rarely perceived as ‘rising stars’2 in male-dominated workplace cultures. According to Mueller et al.,13 male residents receive more consistent feedback on how to develop the personal qualities they need as physicians in emergency medicine than do female residents. Female residents, by contrast, tend to receive more strong criticism when they lack the very same personal qualities.


이것은 명백히 문제가 된다. 만약 멘토링이 개인의 발전에 도움이 된다면, 남성은 여성보다 더 높은 비율로 멘토링을 받기 때문에, 결국 더 높은 성과를 낼 가능성이 더 크다. 높은 성과는, 또 다시, 추가적인 preceptor들이 trainee들을 멘티로 맡게 됨으로서, 새로운 사이클이 시작되게 한다.

This creates an obviously problematic cycle. If mentoring helps one’s development, men, who receive mentoring at a greater rate than do women, have a greater chance of performing highly. High performance, in turn, makes it more likely that additional preceptors will take on trainees as mentees, causing the cycle to begin anew.


즉, 모든 겉보기에 긍정적인 자질에도 불구하고, 멘토링이, 성차별적 관행에서 자유롭거나 해로운 결과에 대해 면역이 되어있다고 가정할 수는 없다.

That is, we cannot assume that mentoring, for all its seemingly positive qualities, is free of gendered practice or immune to detrimental outcomes.


멘토링 관계에서의 멘토와 멘티의 인종(또는 성별)이 다른 경우, 멘토링이 효과적이려면 부정적 고정관념과 무의식적 편견과 같은 장벽을 극복해야 한다.15 예를 들어, 백인 남성 멘티들은 (아무것도 하지 않아도) 잠재력이 있는 것으로 암묵적으로 인식될 수 있지만, 일반적으로 유색인종 멘티 여성들을 성별이나 인종이 다른 멘토와 짝을 이룬 경우, 자신의 능력을 증명해야 한다. 그리고 이것은 그들이 경력개발을 더 더디게 만든다.

Cross-race (and cross-gender) pairing in mentoring relationships requires the overcoming of barriers such as negative stereotypes and unconscious biases for mentoring practice to be made effective.15 For example, whereas White male mentees may be implicitly perceived as having potential, women of colour mentees are generally required to demonstrate evidence of their ability in cross- gender and cross-race mentoring relationships, which leaves them on a slower track over the course of their careers.15


멘토의 상황과 멘티의 상황을 모두 고려한 이해를 바탕으로 [멘토링 관행 및 관계의 복잡성]에 대해 직면할 필요가 있다.

it is imperative to confront the complexity of mentoring practices and relationships with an understanding of the contexts in which both mentees and mentors are situated.


단순히 "다양한 성별, 인종, 다양한 민족을 포함하는 다양한 학생 모집단을 account for하기 위해 멘토링 관계가 어떻게 변화해야 하는가?"라는 질문을 던지는 것 만으로도 의도하지 않은 편견에 대한 인식을 높일 수 있고, 이는 프로그램 전체적으로 어떻게 관행을 변화시켜야 하는지를 밝혀줄 것이다.

Simply questioning how mentoring relationships need to change to account for different student populations that include heterogeneous performers and people of various genders, different races and diverse ethnicities might raise awareness of unintended biases that can shed light on how the programme as a whole needs to adapt its practices.


특정 모집단 내에서 개발된 멘토링의 단일한 고정된 모델만으로는 프로그램적 평가의 성공적인 사용을 보증하는 데 아무런 도움이 되지 않는다.

The consideration of a single fixed model of mentoring, developed within a certain population, does nothing to guarantee the successful use of programmatic assessment.





 2019 Jun;53(6):542-544. doi: 10.1111/medu.13906.

The complexity of mentoring observed through engagement with programmatic assessment.

Han H1Ju A2.

Author information

1
Department of Medical Education, Southern Illinois University School of Medicine, Springfield, Illinois, USA.
2
Department of Education Policy, Organization & Leadership, College of Education, University of Illinois at Urbana-Champaign, Urbana, Illinois, USA.
PMID:
 
31106889
 
DOI:
 
10.1111/medu.13906


의학교육에서 동기부여(AMEE Guide No. 115) (Med Teach, 2016)

Motivation in medical education 

Thierry Pelacciaa,b and Rolland Viauc




도입

Introduction


오랫동안, 실제로, 특히 헬스케어과 같은 매우 전문적인 분야에서 이 수준의 학생들은 동기부여되어있을 것으로 가정되었다. 따라서 보건과학과정의 동기 부여에 관한 구체적인 연구는 21세기 이후 증가하였지만 드물다(Kusurkar, Ten Cate, et al. 2011).

For a long time, it was indeed assumed that students at this level must be motivated, particularly in highly professional areas such as health sciences. Therefore, specific studies on motivation in health science courses are rare, although they have increased in number since the turn of the twenty-first century (Kusurkar, Ten Cate, et al. 2011).


이러한 연구의 대부분은 동기부여가 학습의 측면에서, 유익한 효과와 연관되어 있다는 것을 입증했다.

Most of these studies have demonstrated that motivation is linked to beneficial effects in terms of learning


비록 교사들이 학생들의 학습 동기에 대한 모든 책임을 지지 않지만, 여전히 교사의 책임은 매우 높다. (Viau 2009)

although teachers do not bear full responsibility for their students’ motivation to learn, their responsibility is very high (Viau 2009).


고등교육에서, 특히 의과대학에서, 학생들은 대부분 무언가를 배우는 것에 동기부여되어있기보다는 성공적으로 졸업하는 데 동기부여 되어있으며, 이것은 선발과정에서 더 강화된다 (Holland 2016).

Students in higher education are indeed mostly motivated to successfully graduate, especially in medical schools, where the selection process enhances such motivation, rather than the motivation to learn (Holland 2016).


동기부여란

What is motivation? 


매우 일반적으로 말해서, 그레이엄 & 웨이너(1996, 페이지 63)는 "동기motivation란 사람들이 왜 그렇게 생각하고 행동하는가에 대한 연구"라고 생각한다. 그러나 그 개념을 중심으로 만들어진 수십 개의 이론에 관해서는 동기에 대한 합의된 정의는 없다.

In a very general way, Graham & Weiner (1996, p. 63) consider that “motivation is the study of why people think and behave as they do”. However, there is no consensual definition of motivation, in regard to the dozens of theories that have been built around the concept.


이 중에서, 사회적 인지적 접근은 동기에 대한 연구에서 상당한 중요성을 얻었다. 사회인지적접근법에 따르면, 학습 동기는 개인 자신과 환경에 의해 결정된다. 더 정확히 말하면, 이것은 [자신의 학습 환경, 학습 행동, 그리고 환경 요인에 대한 학생들의 인식] 사이의 지속적인 상호 작용에서 비롯된다(Bandura 1997; Viau 2009).

Among these, the social cognitive approach has gained considerable importance in the study of motivation . According to this approach, motivation to learn is determined by both the individual himself and by the environment. More precisely, it results from constant interaction between a student’s perceptions of his learning environment, learning behavior, and environmental factors (Bandura 1997; Viau 2009).


사회적 인지적 접근법에 따르면, 동기라는 것은 단순히 어떤 활동을 수행할지 말지의 선택이 아니다. 동기라는 것은 일단 선택을 내린 이후에 한 사람이 취하는 행동의 방향, 강도, 지속성을 가리키는 것이다. 또한 동기란 시간, 상황 및 분야에 따라 달라지며, 성격 특성도 아니며, 항구한 개인 특성도 아니라는 사실을 강조한다(Murpy & Alexander 2000; Kusurkar et al. 2012). 이러한 이유로, 비아우(2009년)는 '동기' 보다는 '동기적 역동(motivational dynamic)'이라는 용어를 선호한다.

The social cognitive approach helps to understand that motivation does not simply refer to the choice of undertaking one activity or another, but also to the direction, intensity, and persistence of a person’s behavior once the choice has been made. It also highlights the fact that motivation varies according to the time, context and field within which it is studied, being neither a personality trait nor a permanent personal characteristic (Murphy & Alexander 2000; Kusurkar et al. 2012). This is why Viau (2009) prefers the term “motivational dynamic” rather than motivation.


우리는 다음과 같이 동기부여를 정의한다. 

"동기는 [학생]이 자신과 환경에 대해서 가지고 있는 인식에서 비롯되는 현상으로서, 동기에 따라 학생은 학습을 목적으로 하는 어떤 교육활동을 수행할지, 얼마나 꾸준히 할지, 얼마나 참여할지를 결정한다."(Viau 2009, p.12)

These considerations lead us to the following definition of motivation: 

“[Motivation is] a phenomenon originating in the perceptions that [the student] has of himself and his environment, which lead to his choosing to carry out the educational activity proposed, persevere and engage with it, with the aim of learning from it” (Viau 2009, p.12).


동기부여 역동 모델

“motivational dynamic model”,


imte_a_1248924_sm6170.docx



동기부여의 동적 모델

The motivational dynamic model


Figure 1 (available online as Supplemental Material).


교육 활동의 가치에 대한 인식("주관적 과제 가치"라고도 함)은 [학생이 자신이 추구하고 있는 목표에 기초해, 제시된 활동을 완료하는 데 대한 관심과 유용성에 대한 판단]으로 정의된다(Eccles et al. 1998; Viau 2009). 학생들이 주어진 교육 활동에 더 많은 가치를 둘수록, 그들은 더 인내하고, 더 깊이 있게 과제에 초점을 맞춘다(Pintrich & De Groot 1990; Pintrich & Schrauben 1992).

  • The perception of the value of an educational activity (also called “subjective task value”) is defined as the student’s judgment of the interest and usefulness of completing a proposed activity, based on the goals he is pursuing (Eccles et al. 1998; Viau 2009). The more value students place on the educational activities proposed, the more they persevere and focus on them in depth (Pintrich & De Groot 1990; Pintrich & Schrauben 1992).


인식된 자기효능("역량의 지각"이라고도 함)은 제안된 [교육 활동을 적절히 성공할 수 있는 능력에 대한 학생의 판단]이다(Viau 2009). 학생이 높은 수준의 자기효율을 가지고 있을수록, 높은 목표를 설정하고, 도전적인 활동을 선택하고, 노력을 조절하고, 인내하고, 스트레스와 불안을 관리하며, 결과적으로 높은 수준의 성과를 달성한다(Galand & Vanleede 2004). 

  • Perceived self-efficacy (sometimes called “perception of competence”) is the student’s judgment of his ability to adequately succeed with the proposed educational activity (Viau 2009). The more a student claims to have a high level of perceived self-efficacy, the more he sets high objectives, choses activities that challenge him,regulates his efforts, perseveres, manages his stress and anxiety, and consequently, achieves high levels of performance (Galand & Vanlede 2004). 


통제가능성에 대한 인식("통제권 인식"라고도 함)은 [학생이 활동의 진행에 대해 자신이 가지고 있다고 믿는 통제 정도]로 정의된다. (Viau 2009) 학생이 제안된 교육활동이 어떻게 진행될지에 대해 할 말이 있다고 느낄 경우에, 통제가능성에 대한 인식이 높다고 한다. 통제가능성에 대한 높은 수준의 인식을 가진 학생들은 그들의 학습에 더 헌신적이고 더 오래 인내한다. (Ryan & Deci 2000)

  • Perception of controllability (also called “perceived control”) is defined as the degree of control a student believes he has over the progress of an activity (Viau 2009). A student is said to have a high level of perception of controllability if he feels he has something to say about how the proposed educational activity will be carried out. Students with a high level of perception of controllability are more committed and persevere longer in their learning (Ryan & Deci 2000).


동기가 되는 동적 모델에 따르면, "동기부여된 학생"은

  • 주어진 교육 활동이 유용하거나 흥미롭다고 생각함(교육 활동의 가치에 대한 인식) 

  • 스스로 만족할 정도로 활동을 완료할 수 있다고 느낌(인식된 자기효능). 

  • 자신의 학습 활동의 진행에 책임이 있다고 느낌(관리가능성에 대한 인식).

According to the motivational dynamic model, a “motivated student” 

  • finds the proposed educational activities useful or interesting (perception of the value of an educational activity), 

  • feels capable of completing the activities to his own satisfaction (perceived self-efficacy) and 

  • has the impression of being responsible for the progress of his learning exercises (perception of controllability).



How can you act positively on your students’ motivation to learn?


Table 1 (available online as Supplemental Material),

Source of motivational dynamic

Strategy

Practical implementation

 The perception of the value of an educational activity

Explore students’ expectations and projects

-          Round table

-          Individual interview

-          Focus group interview

-          Written questionnaire

Explain the purpose of the material taught

-          Present the learning objectives

-          Link the learning objectives to the students’ future professional practice

-          Link what students learn in biomedical sciences to what they learn in clinical sciences, for example, by involving a fundamental scientist and a clinician in the preparation and presentation of the course

-          Promote early contact between students and patients

Promote activities that challenge students

-          Define learning objectives of average complexity, not too easy or too difficult to achieve

Make learning a problem-solving task

-          Create links between biomedical and clinical sciences

-          Associate learning tasks with the need to solve a problem

-          Start the lesson with a clinical case, an anecdote or iconographic resources

-          Prefer assessment in an authentic context

Strengthen the links between theory and practice

-          Create early (or simultaneous) clinical experience which matches the theoretical learning

Perceived self-efficacy

Promote success and motivating assessments, and support failure

-          Provide support in case of failure

-          Do not use marks as the only indicator of failure or success

-          Take the time to highlight what was learned before noting errors

-          Provide the student with realistic reassurance concerning the level of success he can achieve

-          Encourage progress and persistence

-          Make assessment criteria explicit

-          Do not multiply assessments used to provide marks

Take into account the consequences of vicarious experience

-          Provide feedback which is also intended to observers of a learning task, particularly when this has not "succeeded" for the pair being observed

Provide well-meaning feedback

-          Consider informal feedback to be important in motivating students

-          Do not focus feedback on penalising errors alone

-          Praise the positive aspects of an activity which was completed

-          Provide students with feedback targeting learning questions and not themselves personally

-          Use motivating semantics, based on opportunities for progress

Allow students to undertake teaching duties

-          Provide students engaged in PBL the opportunity to explain to the others what they have learned from their experience.

-          Promote the introduction of student tutoring by more advanced students

 

Perception of controllability (Perceived control)

Give the students opportunities to make significant choices for themselves

-          Let them choose a course

-          Let them choose a teaching unit

-          Let them choose a means of assessment, etc.



교육활동의 가치에 대한 인식 향상

Enhance the perception of the value of the activity


학생들의 기대치와 프로젝트 탐색

Explore students’ expectations and projects


학생들은 자신의 목표, 기대 및 프로젝트와 일치할 때, 주어진 활동에 더 많은 가치를 부여한다(Lens & Decruyenaere 1991; Pelacia et al. 2008). 따라서 학생들이 이러한 것들을 정의하도록 탐구하고 돕는 것이 중요하다(Kusurkar, Croiset, et al. 2011). 이 목표를 달성하기 위해, 당신은 라운드 테이블로 훈련 세션을 시작할 수 있다. 또한 참가자에게 질문하거나 서면 설문지를 제공하기 위해 개별 또는 포커스 그룹 인터뷰를 개최할 수 있다. 강의의 맥락에서, 예를 들어, 참가자들에게 "이 강좌에서 기대하는 것은 무엇인가"라는 질문을 하고, 그것에 대해 말하기 전에, 답을 공식화하고 적을 수 있는 충분한 시간을 줄 수 있다(McKeachie & Svinicki 2013).

Students give more value to a given activity if it is in line with their own objectives, expectations and projects (Lens & Decruyenaere 1991; Pelaccia et al. 2008). It is therefore important to explore and help students define these (Kusurkar, Croiset, et al. 2011). To achieve this goal, you may start the training session with a round table. You may also hold individual or focus group interviews to question participants or give them a written questionnaire. In the context of lectures, you could, for instance ask participants the question “What do you expect from this course” and give them enough time to formulate and write down their answer, before speaking about it (McKeachie & Svinicki 2013).


연결고리를 설정함으로써 학습 목표를 학생들의 기대와 프로젝트와 연관시키는 것이 중요할 것이다. 또는, 왜 학생들이 주어진 학습 순서의 맥락 안에서 그들의 기대를 충족시킬 수 없는지 설명하는 것도 유용하다.

it will be important to associate learning objectives with the students’ expectations and projects, by setting up links. Otherwise, it is useful to explain to the students why their expectations cannot be met within the context of the given learning sequence.


학습자료의 목적 설명

Explain the purpose of the material taught


특히 학습 목표를 정의함으로써 학습자료의 유익성과 유용성을 설명하는 시간을 갖는 것은 학생들이 주어진 활동에 가치를 부여하는데 도움이 된다(Hopstock 2007; Viau 2009). 이는 학생들이 강의, 특히 생물의학 과목과 관련된 강의에 대해 강의의 가치를 식별하는 데 어려움을 겪는 경우가 많기 때문에 더욱 중요하다(Barker & Olson 1997; Ten Cate et al. 2011; McKeachie & Svinicki 2013). 학습 목표는 이상적으로 학생들의 미래 직업 활동과 연결되어야 한다.

Taking the time to explain the benefit and usefulness of the course material, particularly by defining the learning objectives, helps students to give value to a given activity (Hopstock 2007; Viau 2009). This is all the more important as students often have difficulties in identifying the value of a course, particularly for lectures, especially those related to biomedical subjects (Barker & Olson 1997; Ten Cate et al. 2011; McKeachie & Svinicki 2013). The learning objectives should ideally be connected to the students’ future professional activity,


생물의학 과학에 관해서, 덴트와 하든(2001)이 "수직 통합"이라고 부르는 맥락에서, 이른바 "임상 지향적" 접근법과 환자와의 접촉의 조기 촉진(텐 케이트 등, 2011년)에서도 동기적 이득이 있다.

Concerning biomedical sciences, there is also a motivational benefit in a so-called “clinically oriented” approach and early promotion of contact with patients (Ten Cate et al. 2011), in a context of what Dent and Harden (2001) call “vertical integration”.



학생들에게 도전적인 활동을 장려한다.

Promote activities that challenge students


많은 교사들은 쉽게 성취할 수 있는 목표를 가지고 교습회를 시작하는 것이 학생들의 학습 동기를 증진시킬 수 있다고 잘못 믿고 있다. (Viau 2009) 

      • 그러나 너무 쉽게 성취할 수 있는 목표들은 지루함을 유발한다(Paris & Turner 1994) 

      • 반면에 너무 복잡한 목표는 활동은 포기하게 된다.

Many teachers wrongly believe that starting a teaching session with easily achievable objectives may enhance their students’ motivation to learn further (Viau 2009). Objectives that are too easy to achieve quickly lead to boredom with the activity (Paris & Turner 1994); on the other hand, objectives that are too complex can lead to the activity being abandoned.


학습을 문제 해결 작업으로 설정

Make learning a problem-solving task


학생들이 1학년 과목 내용을 실제 의료행위/의료현장과 연결시키는 과정에서 겪는 어려움은 동기부여의 감소를 가져온다(Barker & Olson 1997). PBL 접근법과 같이 주제를 문제 해결로 전환하게 되면, 학생들이 미래의 전문적인 활동을 바탕으로 문제를 이해하거나 해결함으로써 생물의학 과학과 임상 학문 사이의 연관성을 떠올려볼 기회를 준다. 이는 문제가 어느 정도 authenticity를 갖고 연관되어 있다면 활동의 가치에 대한 학생의 인식이 향상될 것이다.

The difficulties experienced by students in linking the content of first-year courses to the practice of medicine lead to a decrease of their motivation (Barker & Olson 1997). The fact of turning a topic into problem solving – such as in the PBL approach – give students the opportunity to draw links between biomedical sciences and clinical disciplines by understanding or even solving problems based on their future professional activity. This improves their perception of the value of the activity (Pedersen 2003), provided that the problem is linked to a certain degree of authenticity,


(활동의 가치를 높게 인식한다면) 평가 활동은 처벌이 아니라 학습 활동으로서 인식되기 때문에 학생들에 의해 높이 평가될 것이다. 왜냐하면 학생들은 자신이 수행한 학습 연습이 그들의 미래 직업과 관련된 문제를 해결하는 데 유용하다는 것을 알게 될 것이기 때문이다. (Viau 2009)

the assessment activities will be appreciated by the students other than as a source of penalization, because they will be aware that the learning exercises they have done have been useful in solving the problems linked to their future professional life (Viau 2009).



이론과 실천 사이의 연계성 강화

Strengthen the links between theory and practice


몇몇 연구는 링크 활동의 가치에 대한 인식에 영향을 보여주는 경향이 있는데, 초기 또는 진행 중인 훈련 중 건강 학생이 이론과 실습 사이에서 할 수 있다(Hopstock 2007; 펠라키아 등). 2009). 이러한 연결고리는 학생이 교육자료와 과련한 임상 활동에 대한 사전 지식을 가지고 있을 때 establish하기가 더 쉽다(위그필드 & 에클스 2000; 펠라키아 외). 2009). 따라서, 임상 로테이션과 대학 과목 사이의 연결은 동기부여와 관련하여 잠재적으로 흥미롭다(Cottin et al. 2002).

Several studies tend to show the influence on the perception of the value of the activity of links, which a health student during initial or on-going training can make between theory and practice (Hopstock 2007; Pelaccia et al. 2009). These links are easier to establish for students with prior knowledge of the clinical activity corresponding to the material being taught (Wigfield & Eccles 2000; Pelaccia et al. 2009). Therefore, making connections between clinical rotations and university courses is potentially interesting with respect to motivation (Cottin et al. 2002).


하지만, 많은 학생들이 스스로 이러한 연결을 만들지 못할 것이라는 점을 염두에 두는 것이 중요하다. 따라서, 교사들은 [강의 중에 가르치는 자료]와 [임상 환경에서 사용되는 자료] 사이의 연관성을 설명하기 위해 시간을 할애할 책임이 있다. 앞 단락에서 설명한 전략은 이 목적을 달성하는 데 도움이 될 것이다.

However, it is important to consider that many students will not manage to make this connection on their own. Therefore, teachers are also responsible for devoting time, during their lectures, to explain the links between the material being taught and its use in the clinical setting. The strategies described in the previous paragraphs should help you achieve this objective.



인식된 자기 효능감 향상

Enhance perceived self-efficacy


성공 촉진 및 평가 동기 부여, 실패 지원

Promote success and motivating assessments, and support failure


성공은 자기효능감을 높이는 반면, 실패는 감소시킨다(Bandura 1997; Holland 2016). 그러므로 교수는 나쁜 점수를 줌으로써 학습자를 벌주는 것에 만족하지 말고, 먼저 어떤 학습이 있었는지 강조한 다음, 그들이 어떻게 잘못되고 있는지를 인식하도록 돕는 것이 중요하다. 이 지원에는 특히 학생들에게 사용할 평가 기준을 명확히 하고 설명하는 것이 중요하다(Viau 2009).

Successes enhance perceived self-efficacy, whereas failures reduce it (Bandura 1997; Holland 2016). It is therefore important not to be contentwith penalizing learning by giving poor marks, but first to emphasize the learning that has taken place, and then helpthemto recognize where they are going wrong. This support notably includes the importance of clarifying and explaining the assessment criteria you will use to the students (Viau 2009).


점수와 코멘트를 최대한 빨리 연결시키고(Viau 2009), 학생의 성공 능력에 대해 안심시키는 것도 좋은 생각이다. 마지막으로, 학생의 학습 전략, 발전 정도, 끈기 등과 같은 학습의 다른 차원을 평가하는 것이 중요하다.

It is also a good idea to link the mark to comments as soon as possible (Viau 2009) and reassure the student about his ability to succeed, as soon as this is realistic. Finally, it is important to value other dimensions of the learning achieved, e.g. learning strategies used by the student, progress made and persistence (Mann 1999; Viau 2009; Kusurkar, Croiset, et al. 2011; Stegers-Jager et al. 2012).


대리 경험의 결과 고려

Take into account the consequences of vicarious experience


반두라(1997년)에 따르면, 누군가가 행동을 취하여 그가 수행하고 있는 임무에서 성공하거나 실패하는 것을 관찰하는 것은 관찰자의 동기부여에 영향을 미칠 가능성이 있다. 이 "대리적 경험"은 자신의 능력에 관해 자기진단을 내릴 수 있는 하나의 원천이며, 사회적 비교에 근거한 추론을 바탕으로 한다(반두라 1997).

According to Bandura (1997), observing someone take action and succeed or fail in the task he is carrying out is likely to influence the motivation of observers. This “Vicarious experience” is based on inferences made by the observers on the basis of social comparisons, which are the source of a type of self-diagnosis with respect to one’s own abilities (Bandura 1997).


선의의 피드백 제공

Provide well-meaning feedback


선의의 피드백은 단순히 최종 점수를 제공하는 것에 국한되지 않는다. 구두 설명은 실제로 학생의 자기효능감 인식을 높일 수 있는 지렛대 중 하나이다. 피드백을 바탕으로 학생을 동기부여하기 위해서는 학생들의 프레젠테이션의 긍정적인 요소들을 포함시키는 것이 중요하다. 물론 성과에 대해서는 현실적이어야 한다(반두라, 1997년). 이것은 종종 의식적인 노력을 필요로 할 것이다. 왜냐하면 자연적으로, 교사들은 종종 학생들이 저지른 실수에만 전적으로 집중하는 경향이 있기 때문이다.

Well-meaning feedback is not restricted to simply providing a final mark. Oral persuasion is indeed one of the levers likely to build a student’s perceived self-efficacy (Bandura 1997). To make such feedback motivating, it is important to include the positive elements of a student’s presentation while, of course, remaining realistic about the performance (Bandura, 1997). This will often require a conscious effort, because spontaneously, teachers often tend to focus exclusively on errors committed by students.


동기부여 피드백은 또한 건설적이어야 하며 위협적이지 않아야 한다. 즉, 학습 질문을 목표로 하고, 그 자신이 아닌, 훈련의 다음 단계에서 학생을 어떻게 도울 것인가에 초점을 맞추어야 한다(Van de Ridder et al. 2008; Kusurkar, Croiset et al. 2011). 이는 때로 학생을 판단하지 않고 학생이 조치를 수행한 상황을 의미하며, 복잡한 활동과 관련하여 코스 초기에 나타날 때와 같은 특정 실패를 표준화한다(Ten Cate et al. 2011). 사용된 단어 또한 중요하다. 예를 들어, "오류"보다는 "개선점"이라는 용어가 낫다.

Motivating feedback should also be constructive and not threatening, i.e. it must target learning questions and focus on how to help the student during the next stages of his training, and not on the person himself (Van de Ridder et al. 2008; Kusurkar, Croiset, et al. 2011). This will sometimes mean not judging the student but the context in which the action was carried out by the student, which will standardize certain failures, such as when they appear early in the course, with respect to complex activities (ten Cate et al. 2011). The words used are also important . For example, prefer the term “points for improvement” rather than “errors”


학생들이 교무 업무를 수행하도록 허용

Allow students to undertake teaching duties


학생들이 특정한 경우에 가르치는 역할teaching duties을 수행하도록 허용하는 것은 또한 자기효능감에 긍정적으로 작용할 수 있다.

Allowing students to undertake teaching duties on certain occasions can also act positively on their perceived self-efficacy.


관리 가능성에 대한 인식 향상

Enhance the perception of controllability


학생들의 관리 능력에 대한 인식에 긍정적으로 작용하기 위해 사용되는 주요 전략은 학생들에게 선택기회를 주는 것이다. 그러나 모든 선택이 동기부여를 하는 것은 아니다(Flowerday et al. 2004). 동기부여 선택은 학습자에게 목적적합하고 흥미롭거나 중요한 것으로 보이는 것(즉, 자기 실현의 기회를 제공하는 것)이며, 학생들의 능력과 양립하고 그들에게 그들을 안심시키고 중요한 학습을 촉진할 수 있는 감독을 제공하는 것이다(Katz & Assor 2007).

The main strategy used to act positively on your students’ perception of controllability is to give them the opportunity to make choices. However, not all choices are motivating (Flowerday et al. 2004). Motivating choices are those that appear to be relevant, interesting or important for the learners—i.e. that provide an opportunity for self-realization—while being compatible with the students’ abilities and providing them with supervision which will reassure them and promote significant learning (Katz & Assor 2007).


또한 이러한 선택에는 평가 방법과 시간을 선택하는 것과 같은 평가 유형도 포함될 수 있다(Reeve 등, 2003; Ten Cate 등, 2011).

The choices may also concern types of assessment, e.g. by choosing the methods and times of assessment (Reeve et al. 2003; Ten Cate et al. 2011).


Conclusion




 2017 Feb;39(2):136-140. doi: 10.1080/0142159X.2016.1248924. Epub 2016 Nov 21.

Motivation in medical education.

Author information

1
a Faculty of Medicine, Centre for Training and Research in Health Sciences Education (CFR-PS) , University of Strasbourg , Strasbourg , France.
2
b Prehospital Emergency Care Service (SAMU 67) , Strasbourg University Hospital , Strasbourg , France.
3
c Faculty of Education , University of Sherbrooke , Sherbrooke , Canada.

Abstract

Motivation is a concept which has fascinated researchers for many decades. The field of medical education has become interested in motivation recently, having always assumed that medical students must be motivated because of their commitment to highly specific training, leading to a very specific profession. However, motivation is a major determinant of the quality of learning and success, the lack of which may well explain why teachers sometimes observe medical students who are discouraged, have lost interest or abandon their studies, with a feeling of powerlessness or resignation. After describing the importance of motivation for learning in medicine, this Guide will define the concept of motivation, setting it within the context of a social cognitive approach. In the second part of this Guide, recommendations are made, based upon the so-called "motivational dynamic model", which provides a multitude of various strategies with positive effects on students' motivation to learn.

PMID:
 
27866457
 
DOI:
 
10.1080/0142159X.2016.1248924


다중역할 멘토링: 멘토의 개념화, 행동, 역할 충돌(Med Educ, 2019)

Multiple-role mentoring: mentors’ conceptualisations, enactments and role conflicts

Stephanie N E Meeuwissen, Renee E Stalmeijer & Marjan Govaerts






도입

INTRODUCTION


전통적으로 멘토링의 정의는 다음과 같은 것에 집중되어 왔다. 

Traditionally, definitions of mentoring have focused on: 


(i) 멘티 개발 및 학습 지원 

(i) supporting mentees’ development and learning, and 

(ii) 비판단적인 멘토-멘티 관계제공을 통한 멘티에게 안전한 환경 제공

(ii) providing a non-judgemental mentor–mentee relationship and thus a safe environment for mentees.



그러나, 멘토링의 정의와 개념은 종단적 평가와 프로그램적 평가(PA) 접근법의 도입에 따라 변화하고 있는 것으로 보인다.

However, definitions and conceptualisations of mentoring seem to be changing in light of the introduction of longitudinal and programmatic assessment (PA) approaches.7–9


프로그램적 평가는 시간이 지남에 따라 그리고 다양한 맥락에서 학생들의 역량 개발 평가를 지원하는 것을 목표로 한다.

Programmatic assessment aims to support the evaluation of students’ development of competence over time and across various contexts


PA에서는 포트폴리오를 사용하여 개별 학습 프로세스를 지도하고 학생 성과에 대한 종합 결정을 지원하는 것을 목적으로, 피드백과 학습 성과의 사이클cycles을 수집한다. 멘토의 역할은 PA의 성공에 필수적이다: 멘토는 의미 있고 성찰적 대화에 멘티를 참여시킬 수 있고, 상담과 후속 조치와 피드백 사용을 지원할 수 있다.10 따라서 멘토는 학생들을 지도하고 포트폴리오의 성과 데이터를 사용하여 그들의 역량 개발을 촉진해야 한다.

In PA, portfolios are used to collect cycles of feedback and learning outcomes aimed at guiding individual learning processes as well as supporting summative decisions about student performance.10,11 The role of the mentor is essential to the success of PA: mentors can engage mentees in meaningful, reflective dialogues, provide counselling and support the use of follow-up and feedback.10 The mentor is thus tasked with coaching students and stimulating the development of their competences through use of the performance data in the portfolio.12,13


그러나, PA에서 멘토는 동시에 학생들의 발전과 역량 수준에 대한 독립적인 평가 위원회에 조언을 제공한다. 이것은 멘토들이 코치와 평가자라는, 이중적이고 잠재적으로 충돌가능성이 있는 역할을 수행해야 하는 멘토링의 새로운 모델을 낳았다.

However, in PA, the mentor is also asked to advise an independent assessment committee on the students’ progress and competency level. This has resulted in new models of mentoring in which mentors have to fulfil the dual and potentially conflicting roles of coach and assessor.14,15


그러나 멘토링과 PA에 관한 문헌은 코치와 평가자의 역할을 분리하여 유지할 것을 일관되게 권고한다. 이러한 역할을 결합하는 것은 멘토, 멘토 및 멘토링 프로그램의 의도된 결과에 부정적인 영향을 미칠 수 있기 때문이다.

The literature on both mentoring and PA, however, consistently recommends keeping the roles of coach and assessor separate,1,10,16,17 as combining these roles may have a negative impact on the mentee, the mentor and the intended outcomes of the mentoring programme.18,19


예를 들어, 카발칸티와 데츠키는 멘티들이 성능 향상을 위한 중요한 기회를 놓칠 수 있다고 말했다. 마찬가지로, 버드베르크 피셔와 헤르타의 검토 결과들은 멘토들의 지속적인 점검checking이 학생들의 높은 불안감을 초래한다는 것을 보여주었다.

Cavalcanti and Detsky,19 for example, stated that mentees may miss important opportunities for improvement of performance. Similarly, findings from the review by Buddeberg Fischer and Herta18 indicated that constant checking by mentors resulted in high levels of student anxiety.


멘토 입장에서는 멘티의 성공여부에 대한 평가권한을 갖는데서 잠재적인 이해충돌이 생길 수 있다. 

  • 멘토들은 그들의 학생들을 '실패'하는 것을 꺼릴 수 있으며, 그 결과, 학생들의 성적에 대한 개인적인 판단과 학생들의 최종 평가 사이에 불일치를 야기할 수 있다.21 

  • 초기 연구에서도 멘토가 멘티를 평가함에 있어서 기밀 정보를 이야기해야 하거나, 나쁜 소식을 전해야만 하는 상황에서 강력한 관계에 대한 위협 문제를 겪는다고 밝혔다.

From the mentor’s perspective, potential conflicts of interest may arise from having a vested interest in the mentee’s success.19,20 

  • Like clinical supervisors, mentors may feel reluctant to ‘fail’ their students, potentially resulting in discrepancies between their personal judgements of students’ performance and the students’ final evaluation.21 

  • Earlier research also indicated a strong threat of relational problems  when mentors involved in their mentees’ assessments were forced to disclose confidential information or had to break bad news.22


평가자 역할을 맡아야 하는 것은 멘토 역할로 무엇을 해야하는지에 대한 불확실성과 혼란을 야기했다.23,24

Having to take on the assessor role caused uncertainty and confusion regarding the expectations of the mentor role.23,24


방법

METHODS


세팅

Setting


이 연구는 네덜란드 마스트리히트 대학의 MiM(Company Master in Medicine, MiM) 프로그램 내에서 수행되었다. 2013년, PA는 MiM 프로그램의 주요 특징 중 하나로 소개되었다. MiM은 3년간의 임상 회전으로 구성되며, 캐나다 의료 전문가 교육 지침(Canadian Medical Education Directives for Specialists, CanMEDS) 역할을 중요한 프레임워크로 사용하여 역량 기반 교육과 평가의 원칙에 따라 설계된다.25

The study was conducted within the undergraduate Master in Medicine (MiM) programme at Maastricht University, the Netherlands. In 2013, PA was introduced as one of the key features of the MiM programme. The MiM comprises 3 years of clinical rotations and is designed according to principles of competency-based education and assessment, using the Canadian Medical Education Directives for Specialists (CanMEDS) roles as its overarching framework.25


모든 학생들은 MiM 기간 동안 내과의사와 짝을 이룬다. 학생과 멘토는 매년 3~4차례 만날 것으로 예상된다. 멘토는 학생들을 지도하고 그들의 역량 개발을 지원하며 학생들의 진보와 성취 수준에 대한 독립적인 평가 위원회에 조언을 구한다. 이 조언은 포트폴리오의 성능 데이터에 대한 해석을 지원하는 세부 지침, 성능 표준(비료) 및 채점 논문에 기초해야 한다. 자세한 내용은 부록 S1을 참조하십시오.

Every student is paired with a physician-mentor for the duration of the MiM. Student and mentor are expected to meet three to four times each year. The mentor is asked to both coach students and support their development of competence, as well as advise an independent assessment committee on the student’s level of progress and achievement. This advice is to be based on detailed guidelines, performance standards (milestones) and scoring rubrics supporting interpretation of performance data in the portfolio. For more detailed information, please see Appendix S1.


방법

Methodology


우리는 CGT 접근법26,27을 사용하여 멘토들이 어떻게 멘토 역할을 개념화하고 제정하고 다중 역할 멘토링 시스템에서 발생가능한 역할 충돌을 경험하는지를 탐구했다.28 반복 프로세스에 따른 데이터 수집 및 분석.29 우리의 결과는 샘플링된 참여자와의 상호작용, 연구 프로세스 참여 및 결과 데이터에 대한 팀 해석 후 생성된다.27

We used a CGT approach26,27 to explore how mentors conceptualise and enact the mentor role and experience possible role conflicts in a multiple role mentoring system.28 Data collection and analysis followed an iterative process.29 Our results are constructed after interaction with the sampled participants, engagement in the research process and team interpretations of the resulting data.27


참여자

Participants


MiM 프로그램의 멘토는 다양성을 보장하기 위해 의도적으로 선별되었다.

Mentors in the MiM programme were purposively sampled to ensure variety


자료수집

Data collection


반구조적 인터뷰를 사용하여 멘토들의 

(i) 멘토 역할에 대한 개념화, 

(ii) 학생 진보의 코치 및 평가자로서의 구체적인 경험, 

(iii) 여러 역할을 결합한 경험 등을 탐구했다(최초 인터뷰 가이드의 부록 S2 참조). 

Using semi-structured interviews, we explored mentors’: 

(i) conceptualisation of the mentor role; 

(ii) specific experiences as a coach and assessor of student progress, and 

(iii) experiences of combining multiple roles (see Appendix S2 for the initial interview guide). 


인터뷰 가이드에는 다중역할 멘토링 시스템에서의 멘토링의 주요 개념27 및 프로그래밍 평가에 대한 정보를 제공하였다.10

The design of the interview guide was informed by key concepts27 of mentoring in a multiple-role mentoring system19,20,23 and programmatic assessment.10


본 연구의 목적상, '역할분쟁'을 "멘토들이 멘티의 학습과 개발에서 코치 역할과 더불어 학생의 발달과정에 대한 평가자로서의 역할에 대해 불편함을 표명한 상황으로 정의"했다.

For the purpose of this study, we defined ‘role conflict’ as situations in which mentors expressed feeling uncomfortable with their role as assessor of a student’s progress alongside being a student’s coach in learning and development.


자료 분석

Data analysis


SNEM과 MG는 처음 다섯 개의 성적표를 한 줄씩 독립적으로 읽고 코드화했다.30 오픈 코딩과 축 코딩을 사용하여 SNEM과 MG가 주제와 하위 테마를 생성하기 위해 초기 코드를 비교하고 논의하였다.31 인터뷰 11과 12의 분석과 코딩은 최종 코딩 계획을 확인했다. 분석의 마지막 단계에서 선택적 코딩을 위해 모든 기록을 함께 취하였다.31

SNEM and MG independently read and initially coded the first five transcripts line by line.30 Using open and axial coding, the initial codes were compared and discussed by SNEM and MG in order to generate themes and sub-themes.31 The analysis and coding of interviews 11 and 12 confirmed the final coding scheme. In the final phase of analysis, all transcripts were taken together for selective coding.31


반성성

Reflexivity


멘토들은 SNEM이 데이터 수집 당시에 이 멘토 시스템 내에서 멘토링도 받고 있다는 것을 알고 있었기 때문에 부정적인 경험을 공유하는 것을 꺼려했다고 주장할 수 있다. SNEM이 피드백 및 평가 시스템의 설계와 구현에 관여하지 않았기 때문에 멘토들도 더 개방적이고 정직하게 느낄 수 있었다. PA 시스템 시행 중에 직원들은 코치와 평가자의 이중 역할을 맡기를 꺼려하는 것처럼 보였다. 하지만, 멘토 제도가 어떻게 실제로 작동했는가에 대한 의문은 남아 있었다. 본 연구의 시작에서, 우리는 PA 내의 멘토링 프로그램의 이론적 목적과 잠재적인 함정에 대한 견해 외에 다른 견해를 갖지 않았다. 기존 문헌을 바탕으로 이 설정에서 갈등이 있었는지, 언제 일어났는지 궁금했다.

It could be argued that mentors felt reluctant to share negative experiences, because mentors knew SNEM was also receiving mentoring within this mentor system at the time of data collection. Mentors could have also felt more open and honest, as SNEM was not involved in the design and implementation of the feedback and assessment system. During the implementation of the PA system staff appeared reluctant to take on the dual role of coach and assessor. However, questions remained regarding how the mentor system worked in practice. At the start of this research, we held no views other than those on the theoretical aims and potential pitfalls of mentoring programmes within PA. Based on the existing literature, we were curious to discover if and when conflicts were experienced in this setting.



윤리적 고려

Ethical considerations


결과

RESULTS


(i) 권한부여적

(ii) 점검적

(iii) 지시적 ...이라는 세 가지 주요 멘토링 접근방식을 데이터에서 구성했다.

Three predominant mentoring approaches were constructed from the data: (i) empowering; (ii) checking, and (iii) directing.


이러한 결과의 요약은 표 2에 제시되어 있다.

A summary of these results is presented in Table 2.


권한부여적 접근

Empowering mentoring approach


이 접근방식을 가진 멘토들은, 협력관계에서 일하는 것과 유사하게 학습 방향에 대해 학생들과 공유된 이해와 합의를 발전시키는 것을 목표로 했다. 이러한 권한 부여 접근방식은 학생들의 발전을 위한 성찰전략으로 특징지어졌는데, 이는 모든 피드백, 시험 결과 및 개인적인 문제들이 학생에게 다시 mirrored back되었다는 것을 의미한다: 권한부여적 접근법을 가진 멘토들은 답을 제공하지 않고 대신 질문을 했다.

Mentors with this approach aimed to develop a shared understanding and agreement with students on the directions of their learning, similar to working in a partnership. This empowering approach was characterised by a reflective strategy for students’ development, meaning that all feedback, test results and personal issues were mirrored back to the student: mentors with an empowering approach did not provide answers, but asked questions instead.


하지만 나는 또한 그들이 어떻게 자신을 사람으로 보는지 물어본다. "누구세요?" "어떤 의사가 되고 싶으며 그런 생각을 해 본 적이 있는가?" (참여11)

But I also ask how they see themselves as a person. “Who are you?” “Do you have any idea what kind of doctor you want to be and have you ever thought about that?” (Participant 11)


권한부여적 멘토와 함께 학생들은 앞장서서 자신의 목표를 설정하도록 자극을 받았고, 이러한 목표를 달성하기 위한 전략을 선택하고, 그들 자신의 성과와 발전을 모니터했다.

With mentors who were empowering, students were in the lead and stimulated to set their own goals, select strategies to achieve these goals and monitor their own performance and development.


권한을 부여하던 멘토들은 평생학습능력의 개발을 촉진하고, 직업적 정체성의 발전을 위한 학생들의 개별학습 궤적을 지원함으로써 멘토들이 그들의 최대 잠재력에 도달하는 것을 목표로 삼았다.

Mentors who were empowering aimed for their mentees to reach their full potential through stimulating the development of lifelong learning skills and supporting students’ individual learning trajectories for the development of their professional identity.


학생들이 경험했던 문제적 상황을 토론할 때에도 권한을 부여하는 접근법이 유지되었다.

The empowering approach was maintained even in discussing problematic situations students had experienced.


권한부여 접근법을 가진 멘토들은 자신의 멘토링에서도 공유된 의사결정shared decision making 원칙을 적용하는 방법을 설명했으며, 이러한 접근방식은 자신의 전문적 배경(예: 일반 실무자[GP])의 핵심 가치에서 영향을 받았다.

Mentors with an empowering approach explained how they applied principles of shared decision making in their mentoring, sometimes influenced by the core values of their professional background (e.g. as a general practitioner [GP]).


나는 GP이기 때문에 항상 철저한 질문을 하고 맥락을 고려하려고 노력한다. 그리고 물론, GP로서, 나는 항상 환자들이 그들의 능력 안에서 문제를 스스로 해결하도록 노력해왔다. 그리고 그것이 이 학생들과 함께 하려고 노력하는 것이다. (참가자 5)

I am a GP, so I always try to ask thorough questions and consider the context. And of course, as a GP, I have always tried to let patients solve the issue themselves, within their competences. And that is what I also try to do with these students. (Participant 5)


이러한 멘토들은 학생들에게 학습 과정에 상당한 자율성을 주었지만, 그들은 사전에 제공된 성과 평가 지침을 따라 학생들의 발전을 모니터했다. 디지털 포트폴리오는 학생 개개인이 개인적으로나 전문적으로 최적으로 발전할 수 있도록 하는 정보를 위임받은 독특한 도구로 여겨졌다.

Although these mentors gave students a great degree of autonomy in their learning process, they monitored students’ development by following the offered guidelines for performance assessment. The digital portfolio was seen as a unique tool with entrusted information that empowered each student to develop optimally, both personally and professionally.


점검적 접근

Checking mentoring approach


주로 멘토링 접근법을 점검하는 우리의 표본에 있는 5명의 멘토들은 

(i) 학생들의 역량 개발을 모니터링하고 판단하고, 

(ii) 학생들이 이러한 문제에 초점을 맞추도록 하기 위한 지식, 기술 또는 전문적 개발의 취약점을 식별

...함으로써 그들의 역할을 수행했다. 

The five mentors in our sample with a predominantly checking mentoring approach fulfilled their role by: 

(i) monitoring and judging students’ competence development against performance standards, and 

(ii) identifying students’ problems, and weaknesses in knowledge, skills or professional development to ensure students focused on these issues. 


이러한 접근은 학생들이 졸업 시 요구되는 모든 성과 기준을 충족하도록 하는 것을 목표로 하는 멘토-멘티 관계를 이끌어 냈다.

This approach led to a mentor– mentee relationship that was largely instrumental, and aimed to ensure that students met all the required performance standards at graduation:


이 멘토들은 그냥 지나칠 수도 있었던 문제들을 탐지하는 데 책임을 느꼈다.

These mentors felt responsible for detecting problems that would otherwise go unnoticed.


점검적 멘토들은 학생들의 개인적personal 발전에 어떠한 개입도 언급하지 않았다. 오히려, 그들은 학생들이 그들의 학습과 발전에 대해 모든 책임을 지도록 요구하면서 관계에 있어서 일정한 거리를 두는 것처럼 보였다. 그들은 평가 시스템의 규칙과 지침이 그들의 멘토링 접근법을 결정했으며, 포트폴리오의 정보에 대한 평가 조언에 기초했다고 언급하였다.

Checking mentors did not mention any involvement in students’ personal development. Rather, they seemed to keep a relational distance, requiring students to take full responsibility for their learning and development. They stated that the rules and guidelines of the assessment system determined their mentoring approach and that they based their assessment advice on the information in the portfolio.



지시적 접근

Directing mentoring approach


멘토들에게 권한을 부여하는 것에 비해, 지시적 멘토들은 더 권위적으로 행동했고 어떤 조치를 취해야 하는지 그리고 상황을 어떻게 처리해야 하는지에 대해 학생들에게 가르치는 것처럼 보였다.

Compared to empowering mentors, the two directing mentors acted more authoritatively and seemed to instruct students on which steps to take and how to handle situations.


이 멘토들은 멘티학생에 대한 기득권을 가지고 있는 듯 보였고, [임상 환경에서의 개인적인 업무 경험]과 [훌륭한 의사는 무엇이고, 의사는 무엇을 할 수 있어야 하는지]에 대한 개인적인 신념을 바탕으로, 학생들이 '냉혹한 현실'에 대비하는 데 초점을 맞추고 있다.

These mentors seemed to have a vested personal interest in their student-mentees and focused on preparing their students for the ‘harsh reality’ of a physician’s working life, framed by their personal work experiences in the clinical setting and their personal beliefs on what constitutes a good doctor and what a doctor should be able to do.


이러한 멘토들은 학생들의 성과에 대해 책임을 느꼈고, 그들의 영역에서 의사가 되는 전문적인 기준을 유지하려는 근본적인 동기 부여를 느꼈다. 그들은 workplace based assessment and learning의 맥락에서 전략적인 행동을 인식하고 분명히 고려했다.

These mentors felt responsible for the performance of students, with the underlying motivation to maintain the professional standards of being a physician in their domain. They were aware of and explicitly took into account strategic behaviours in the context of workplace-based assessment and learning both from supervisors and students.


따라서 지시적 멘토들은 평가 시스템에 대해서 신뢰하지 못했고, 포트폴리오의 피드백이 유용하지 않다고 보았으며, 그들은 멘티가 실제 일상 실무에서 어떻게 하는지를 직접 보는 것을 선호하여, 자기 스스로가 멘토들의 성과에 대한 의견을 형성할 수 있도록 하였다.

Therefore, the directing mentors in our sample seemed to lack trust in the assessment system and the usefulness of feedback in the portfolio, and they preferred actually seeing their mentees function in daily practice, so that they themselves could form an opinion about the mentees’ performance.



역할갈등의 경험

Experiences of role conflicts


권한부여적, 점검적 멘토들은 평가자와 코치 역할의 조합을 학생의 발전에 가치를 더하는 것으로 인식하였으며, 두 역할을 수행함에 있어서 피드백과 평가 시스템에 의해 촉진되었다고 느꼈다. 접근방식은 다르지만, 권한부여적 멘토와 점검적 멘토들은 코칭 프로세스가 평가를 포함한다고 인식했다. 왜냐하면, 코칭이란 학생들과 정기적으로 피드백 대화를 나누며, 과거의 성과를 되돌아보고, 현재의 성과와 달성해야 할 목표 사이의 차이를 좁히기 위한 행동을 정의하는 것이기 때문이다

The predominantly empowering and checking mentors perceived the combination of the assessor and coach roles as added value for the student’s development and felt that they were facilitated by the feedback and assessment system in performing both roles. Although differing in approach, empowering and checking mentors perceived assessment to be embedded in the coaching process, as their coaching entailed having regular feedback conversations with the student, looking back on past performance and defining actions to close the gap between current performance and goals to be achieved.


전반적으로, 이 멘토들은 디지털 포트폴리오에 수집된 종합적인 증거에 기초하여 학생들의 성적을 평가하는데 자신감을 가졌다. 결론적으로, 코치와 평가자의 역할을 하는 것은 권한부여적, 점검적 멘토 역할에 필수적인 것으로 간주되었다.

Overall, these mentors felt confident in making judgements of students’ performance based on the aggregated evidence gathered in the digital portfolio. To conclude, the roles of coach and assessor were seen as integral to the mentor role of empowering and checking mentors:


역할 충돌은 주로 지시적 접근법을 가진 멘토들이 경험할 가능성이 더 높은 것 같았다. 이 멘토들은 평가자 역할이 전체적으로 불쾌한 경험이었다고 진술했다. 여기서 평가 조언을 제공하는 것은 코치로서의 그들의 업무를 방해하고 멘토-멘터리 관계에 거리감을 생기게 하는 것으로 보였다.

Role conflicts seemed more likely to be experienced by mentors with a predominantly directing approach. These mentors stated that the assessor role as a whole was an unpleasant experience, where providing assessment advice was seen as disrupting their task as coach and creating distance in the mentor–mentee relationship.


또한 이들 멘토들의 성과기준이 항상 평가시스템에서 정한 성과기준을 충족하지는 못했다. 대신에, 그들은 학생들에게 중요한 것에 대해 강한 개인적 신념을 가지고 있었다. 따라서 그들은 그들 자신의 경험을 토대로 하여 직접적인 관찰에 근거하여 판단을 내리는 것을 선호하였다. 그들은 학생들의 포트폴리오에 있는 '세컨더리 정보'를 신뢰하지 않았다. 그들은 이 정보를 신뢰성이 낮다고 생각했다.

Also, these mentors’ performance standards did not always meet the performance standards as prescribed by the assessment system. Instead, they had strong individual beliefs on what was important for students. They thus built upon their own experiences and preferred to make judgements based on direct observations. They mistrusted ‘second-hand information’ in the students’ portfolio, which they regarded as low in credibility:


나는 흰 코트를 입은[나의 멘티]를 본 적이 없다. 어쩌면 내 학생은 상황을 엉망으로 만들고 있고, 다른 모든 사람들은 "쟤 여기서 뭐하는거야?"라고 생각하고 있을 가능성이 있다. 그러나 나는 그것을 볼 수 없다. (3 참가)

I have never seen [my mentee] in a white coat. And it’s possible that he is making a mess of it and everyone else is thinking “wow what is he doing here in the hospital?” But I am not able to see that. (Participant 3)


지시적 멘토들은 부정적인 조언이 학생들의 발전과 미래 직업에 부정적인 영향을 미칠 수 있다고 걱정했다. 그들은 평가를 형성적 과정(학습에 대한 평가)이 아니라 총괄적 과정(학습의 평가)으로 계속 보았다. 이러한 멘토들의 책임감과 학생의 전문적 발전에 대한 vested interest을 가졌다는 사실은 평가행동으로도 드러났다. 이 멘토들은 포트폴리오에 관련된 개인 정보를 문서화하지 못하였다.

The directing mentors in our sample were worried that negative advice could have a negative impact on the student’s development and future career. they kept seeing assessment as a summative process (assessment of learning) and not a formative process (assessment for learning). These mentors’ feelings of responsibility and having a vested interest in students’ professional development were reflected in specific assessment behaviours such as failing to document relevant personal information in the portfolio:


솔직히 말하면, [나의 멘티 학생]에 대해, 나는 그의 포트폴리오에 그런 [부정적인 논평]을 넣지 않았다.[......]나는 이렇게 말했다. "너도 알다시피, 나는 지금 너에게 이것을 말하고 있는 중이다. 그리고 나는 이것을 너의 포트폴리오에 넣지 않을 것이다. 왜냐하면 그것은 너를 괴롭힐 수 있기 때문이다." 그래서 나는 가끔 그에게 개인적인 정보를 주고 싶다, "이것과 당신의 평가에 있어서, 그것에 대해 알아라." (참가자 3)

Let me be honest, with [my mentee student], I did not put that [negative comments] in his portfolio. [...]Isaid“you know, I am just telling you this now. And I am not going to put this in your portfolio, because it might haunt you”. So I like to give him some personal information now and then, “be aware of this and in your assessment, be aware of that”. (Participant 3)


이러한 멘토들은 경험 있는 역할 충돌에 대처하기 위해 다른 추가적인 메커니즘을 사용했다. 그들은 멘토로서 판단을 자제하고 학생들의 자기 평가에 크게 의존했다. 게다가,  포트폴리오에 있는 증거가 아니라, 멘티와의 미팅에서의 대화가 평가의 소스가 되었다. 그 후 포트폴리오를 다른 사람(즉, 포트폴리오 평가위원회)에게 넘겨 증거 검토를 직접 하지 않았다.

These mentors used different additional mechanisms to cope with their experienced role conflict. They refrained from making advisory judgements themselves and heavily relied on student’s self-assessment. In addition, the conversations during mentor meetings became a source of assessment rather than the evidence in the portfolio itself. The portfolio was then passed on to others (i.e. the portfolio assessment committee) to review the evidence instead of doing it themselves.


이 한 가지 사례로 나는 실제로 그 학생이 스스로 결정하도록 했다. 내가 그런 결정을 내려야 한다면, 나는 그것이 몹시 불편하다고 생각할 것이다. (참가자 3)

With this one case I actually let the student decide herself. If it would have been expected from me to make that decision, I would find that extremely uncomfortable. (Participant 3)


고찰

DISCUSSION


세 가지 멘토-멘터리 관계: 

  • 권한부여적 멘토는 멘티와 함께 협력적으로 일했고

  • 점검적 멘토는 도구적 멘토-멘토 관계를 발전시켰고

  • 지시적 멘토는 전형적으로 그들의 멘티들과 더 교수-중심적 관계를 발전시켰다.

three mentor–mentee relationships: 

  • empowering mentors worked together with their mentee-students in partnership, 

  • checking mentors developed more instrumental mentor– mentee relationships, whereas 

  • directing mentors typically developed more faculty-centred working relationships with their mentees.


역할 충돌의 정도는 멘토들이 선호하는 멘토링 접근법과 관련이 있는 것으로 보였다.

The extent to which a conflict of roles was experienced, seemed to be related to mentors’ preferred mentoring approach:


어떤 멘토링 접근법을 선택하느냐는 멘토들의 전문직업적 배경과 작업 환경과 관련이 있는 것처럼 보였다. 이와 같이 멘토가 멘토링을 어떻게 개념화하고 실천하는가는 현재 직업적인 환경에서의 요구나 환자에 대한 행동과 유사한 것으로 보였다.

The adopted mentoring approach seemed to be associated with mentors’ professional background and work setting. As such, mentors’ conceptualisations and enactments seemed to be either informed by demands of their current work settings or similar to their behaviour towards patients


또한, 학습에 대한 신념과 평가시스템에 대한 신뢰(또는 불신)이 멘토링 접근법에 영향을 미쳤다. 예를 들어, 지시적 멘토는 학습을 위한 평가 대신에 학습의 평가를 추구했다.

Additionally, deeply rooted beliefs about learning and (dis)trust in the assessment system influenced mentoring approaches. For example, the directing mentors in our sample seemed to be driven by assessment of learning, instead of assessment for learning priciples.9


Buell33은 네 가지 멘토링 모델(복제, 견습생, 양육 및 우정)을 설명했는데, 이는 우리 연구의 결과와 매우 유사하다(복제 – 지시, 견습생 – 점검, 육성 – 권한 부여). 

Buell33 described four different mentoring models (cloning, apprentice, nurturing and friendship), which to a large extent resemble findings from our study (

  • cloning – directive, 

  • apprentice – checking, 

  • nurturing – empowering). 


부엘의 연구 결과와 달리, 우리의 참가자들은 멘토-멘티 관계가 개인적인 우정으로 간주되는 접근방식인 우정 모델은 채택하지 않은 것 같다.33 오히려 이번 연구의 멘토들은 모두 전문적인 관계를 발전시키고 유지하는 것처럼 보였다.

Contrary to Buell’ s findings, our participants did not seem to adopt the friendship model – an approach in which the mentor–mentee relationship is considered to be a personal friendship.33 Rather, the mentors in our study all seemed to develop and maintain a professional relationship.


데카스트로 외 개별 멘토들이 의학의 맥락에서 특정 멘티-학생과의 관계에 따라 어떻게 다른 멘토 역할을 채택할 수 있는지를 강조하였다. 마찬가지로, Buell은 멘토-멘티의 상호작용 연속체에서 멘토링 모델이 발생할 수 있다고 설명했다. 멘티가 발전함에 따라 멘토의 역할이 진화한다는 것도 이미 언급되었다.

DeCastro et al.34 highlighted how individual mentors may adopt different mentoring roles depending on the relationship with the particular mentee-student in the context of medicine. Likewise, Buell33 described that mentoring models could take place in a continuum of mentor–mentee interactions. It has also been previously stated that as the mentee develops, the role of the mentor evolves.35


이전의 문헌과 대조적으로, 우리의 연구 결과는 다중 역할 멘토링 시스템이 반드시 역할 충돌과 연관되어 있는 것은 아니라는 것을 보여준다. Heeneman 등은 역할 충돌을 [멘토 경험의 부족, 멘티와의 개인적 관계, 학생들의 최종 수준에 대한 불확실성] 등과 연결시켰는데, 우리의 결과는 역할충돌에 대한 멘토링 접근법의 영향을 더 강조한다.

In contrast to previous literature,1,10,16,17 our findings show that multiple-role mentoring systems are not necessarily associated with role conflicts. Whereas Heeneman et al.15 linked role conflicts to a lack of mentor experience, the personal relationship with the mentee and uncertainties about students’ end level, our results point more towards the influence of the mentoring approach,


교수 중심의 관계에 가까운 지시적 멘토링 접근법이 멘토들의 부정적인 경험과 더 강하게 연관되어 있음을 확인했다. Arntfield 등도 학생-교육자 참여와 긍정적 경험의 피드-포워드를 위해서는 [학생의 주체성agency와 양방향적,순환적 프로세스]가 중요함을 강조했다. Student agency의 중요성은 학습과 평가 경험을 통제하는 것을 배우는 데 도움이 되기 때문에 학생 스스로 목소리를 높인다는 점에서 주목할 만하다.37

Our finding that a more directing mentoring approach, linked to a faculty-centred relationship, is more strongly related to the negative experiences of mentors, does align with the findings by Arntfield et al.36 They emphasised the importance of student agency and a ‘bi-directional and cyclical’ process to achieve student–mentor engagement and feedforward in positive experiences.36 It is noteworthy that the importance of student agency has been voiced by students themselves as it helps them learn to take control of their learning and assessment experience.37


우리의 연구에서 역할 충돌을 경험한 멘토들은 몇 가지 극복 메커니즘을 설명했다. 이러한 메커니즘은 부분적으로 프로그래밍 방식, 작업장 기반 평가에 대한 신뢰 부족과 '보조 정보'를 판단해야 하는 것과 관련이 있었다. 자신의 관찰에 의존하기를 원하는 평가자들은 이전에 Hawe38과 Driessen과 Shellle에 의해 보고되었다. 이전 연구에서는 평가자들이 어떻게 '직관적'이었는지 묘사했는데, 이들은 포트폴리오에 정보를 사용하는 대신에 그들의 '직감'을 신뢰하기를 원했다.

The mentors who experienced role conflict in our study described several coping mechanisms. These mechanisms were partly related to a lack of trust in programmatic, workplace-based assessment and having to judge ‘second-hand information’. Assessors wanting to rely on their own observations were previously reported by Hawe38 and Driessen and Scheele,39 who described how assessors were ‘intuitive’ and wanted to trust their ‘gut feelings’ instead of using the information in a portfolio.


우리의 연구는 멘토가 무엇이어야 하고 해야 하는가에 대한 개인적 믿음이 얼마나 뿌리 깊은지를 분명히 지적하며, 이러한 개인적 신념이 외부 지침과 성과 기준의 채택과 사용을 방해할 수 있음을 보여준다.

Our study, clearly points out how deeply rooted personal beliefs about what a mentor should be and do may hinder adoption and use of external guidelines and performance standards.


특히, 우리의 연구는 멘토링에 대한 오래된 정의와 개념(멘티의 발달을 지원하고, 비판단적 멘토-멘터리 관계 제공)에서 벗어나는 것이 문제가 없다는 것을 보여준다. 

  • 멘토링의 기존 정의는 비판단적 측면을 강조하고 있지만, 

  • 권한부여적 멘토링과 검검적 멘토링에서 평가 프로세스란 학습/발전/목표달성을 가이드하기 위하여 수행하는 개인의 교육적 계획-수행-학습-행동 사이클에 부합하며, 지속적인 형성평가에서 얻은 정보를 활용한다.

Notably, our study shows that moving on from the old definition and conceptualisation of mentoring – supporting mentees’ development and providing a non-judgmental mentor–mentee relationship1,6 – does not need to be problematic. 

  • Whereas the old definition of mentoring incorporated the nonjudgemental aspect,6,10 

  • both the empowering and checking mentoring approaches are in line with current developments in medical education in which the assessment process is seen as a personal educational plan-do-study-act cycle, informed by ongoing formative assessments to guide learning, improvement and goal achievement.41


한계

Limitations


첫째, 우리의 연구 결과를 다른 국가 환경으로의 이전 가능성에 대해 거의 언급할 수 없다. 그러나 데이터를 수집한 문맥에 대한 두꺼운 설명을 제공함으로써 전송 가능성을 높이기 위한 모든 시도가 이루어졌다.

First, Little can be said about transferabilityof our findings to other (inter)national settings  . However, every attempt was made to augment transferability by providing a thick description29 ofthe context in which data were collected.


함의

Implications for practice and future research


PA 시스템의 멘토에 대한 교수진 개발은 PA를 뒷받침하는 핵심 원칙, 포트폴리오 기반 학습 및 평가뿐만 아니라 멘토 신념과 멘토링 접근방식, 멘토-멘티 관계 및 멘토링의 잠재적 결과 사이의 상호작용에 어떻게 영향을 미칠 수 있는지에 초점을 맞추어야 한다.

Faculty development for mentors in PA systems should not only focus on key principles underpinning PA, portfolio-based learning and assessment, but also on mentor beliefs and how these may affect the interaction between mentoring approach, mentor–mentee relationships and potential outcome of mentorships.



결론

CONCLUSIONS


자율적인 멘토링 접근법을 통해 평가가 학습 과정과 통합되는 학습 문화를 개발할 수 있다.

With the empowering mentoring approach, a learning culture can be developed in which assessment is integrated with the learning process.


부록 S1. 멘토에 대한 요구사항 및 멘토 판단 위치.

Appendix S1. Requirements for mentors and positioning of mentor judgements.







 2019 Jun;53(6):605-615. doi: 10.1111/medu.13811. Epub 2019 Feb 5.

Multiple-role mentoring: mentors' conceptualisationsenactments and role conflicts.

Author information

1
Department of Educational Development and Research, Faculty of Health, Medicine and Life Sciences, Maastricht University, Maastricht, the Netherlands.

Abstract

INTRODUCTION:

Outcome-based approaches to education and the inherent emphasis on programmatic assessment in particular, require models of mentoring in which mentors fulfil dual roles: coach and assessor. Fulfilling multiple roles could result in role confusion or even roleconflicts, both of which may affect mentoring processes and outcomes. In this study, we explored how mentors conceptualise and enact their role in a multiple-role mentoring system and to what extent they experience role conflicts.

METHODS:

We conducted a constructivist grounded theory study at one undergraduate medical school. A purposive sample of 12 physician-mentors active in a programmatic assessment system was interviewed. Data analysis followed stages of open, axial and selective coding through which themes were constructed.

RESULTS:

Three predominant mentoring approaches were constructed: (i) empowering (a reflective and holistic approach to student development); (ii) checking (an observant approach to check whether formal requirements are met), and (iii) directing (an authoritative approach to guide students' professional development). Each approach encompassed a corresponding type of mentor-mentee relationship: (i) partnership; (ii) instrumental, and (iii) faculty-centred. Furthermore, mentors' strategies, focus, agency provided to students and perception of the assessment system characterised mentoring approaches and relationships. Role conflicts were mainly experienced by mentors with a directing mentoring approach. They used various coping mechanisms, including deviation from assessment guidelines.

CONCLUSIONS:

In multiple-role mentoring in the context of programmatic assessment, mentors adopted certain predominant mentoringapproaches, which were characterised by different strategies for mentoring and resulted in different mentor-mentee relationships. Multiple-role mentoring does not necessarily result in role conflict. Mentors who do experience role conflict seem to favour the directing approach, which is most at odds with key principles of competency-based education and programmatic assessment. These findings build upon existing mentoring literature and offer practical suggestions for faculty development regarding approaches to mentoring in programmatic assessment systems.

PMID:
 
30723949
 
DOI:
 
10.1111/medu.13811


언제 조언자, 코치, 멘토가 되어야할지 선택하기(J Grad Med Educ, 2018)

Choosing When to Advise, Coach, or Mentor

Karen Marcdante, MD (@KMarcdante)

Deborah Simpson, PhD (@debsimpson3)





현 상황

The Challenge


학습자는 간단한 조언을 구할 의도였는데, 교수진은 지속적인 성능 향상의 필요성을 느꼈을 수 있다. 반대로, 학습자는 멘토링을 원했는데, 교직원은 한 가지 조언만 제공할 계획이었을 수 있다. 이처럼 잘못 조정된 기대는 교수진과 훈련생에게 좌절감을 줄 수 있다(교수에게 좌절이란 trainee가 조언을 듣지 않거나 가이드를 무시할 때도 일어난다)

While a learner may be looking for a quick piece of advice, faculty may see a need for ongoing performance improvement. Alternatively, the learner may want mentoring, while the faculty member plans to offer a single bit of advice. Misaligned expectations can be frustrating for faculty (when a trainee does not appear to listen or act on the guidance offered) and for a trainee


알려진 것

What Is Known


교수진과 연수생들은 다양한 형태의 지도guidance를 둘러싼 공통적이고 명확한 용어가 부족하다.

Faculty and trainees lack common, clear terminology surrounding various forms of guidance.


조언, 코칭, 멘토링은 지도guidance 제공에 대한 3가지 접근방식이다.

Advising, coaching, and mentoring are 3 distinct approaches to providing guidance.


각 접근방식은 특정한 목적을 가지고 있으며, 서로 다른 전문지식을 필요로 하며, 다른 전략을 사용한다.

Each approach has a specific purpose, requires different (but overlapping) expertise, and uses different strategies.


  • 은 학습자가 따를 수도 있고 따르지 않을 수도 있는 특정 사건에 대한 전략을 제공한다. 

  • 멘토링은 학습자의 커리어의 많은 측면을 형성하는 데 도움이 되는, 경험적 지혜가 제공되는 장기적인 관계를 의미한다. 

  • 코칭비지시적인nondirective 질문을 통해 학습자가 목표를 확인하고 해결책을 개발하는 데 도움을 준다는 것을 의미한다.

  • An advisor offers strategies about a specific event, which the learner may or may not follow. 

  • Mentoring implies a long-term relationship in which experiential wisdom is offered to help build the many aspects of a learner’s career. 

  • Explicitly stating that your role is to ‘‘coach’’ means that you will help the learner identify a goal and develop solutions through nondirective questions.


오늘 바로 할 수 있는 일

How You Can Start TODAY


Guidance의 연속성을 이해하고 조언과 멘토링을 구분한다.

Understand the continuum of guidance and differentiate advising from coaching from mentoring.


테이블

TABLE


학습자가 어떤 guidance를 원하는지 명확하게 묻고, 접근 방식을 조정하고, 무엇을 제공할 것인지 설명하십시오. 학습자가 늘 자기가 무엇을 원하는지(또는 무엇을 필요로 하는지)를 알고 있는 것은 아니다. 단기 및 장기 목표를 명확히 하는 것은 올바른 전략을 선택하고 기대에 더 잘 부응하는 데 도움이 될 수 있다. 전략이 선택되면 학습자와 대화에서 무엇을 다루고 무엇을 다루지 않을지 토론하십시오.

Explicitly ask the learner what guidance he or she is seeking, align your approach, and explain what you will be providing. Learners do not always know what they want (or need). Clarifying short- and long-term goals can help you select the right strategy and better meet expectations. Once a strategy is selected, discuss with the learner what will and what will not be covered in your interactions.


상호작용을 즐기세요. '전구에 불이 들어오는 것'을 지켜보는 것, 즉 발전과정과 커리어 개발을 지켜는 것은 많은 사람들이 의학교육을 하는 이유이다.

Enjoy the interaction. Watching the ‘‘light bulb go on,’’ seeing progress being made and careers developing, is why many are involved in medical education.


상호 작용 중 역할 전환에 주의하십시오. 코칭에서 학습자는 질문을 통해 자기만의 계획을 개발한다. 만약 그 영역에 대한 전문지식을 가지고 있다면, 멘토 역할로 전환하는 것은 어렵지 않다. 어떤 사람들은 "coaching with wisdom"이라고 부른다. 예를 들어, "나는 지금 너를 멘토링하고 있다."라는 말로서 역할이 바뀌었다는 것을 매우 명확히 하라. 그리고 나서 필요하다면 적절할 때 다시 코칭으로 전환하라.

Be cautious about switching roles during an interaction. In coaching, the learner develops an individualized plan through questioning. When you have expertise in the area, it is easy to switch into the mentor role. Some call that ‘‘coaching with wisdom.’’ Be very clear that you have switched roles by saying, for example, ‘‘I am mentoring you now.’’ Then switch back to coaching when appropriate.


더 나은 조언자/멘토/피드백이 되는데 필요한 피드백을 요청하면서 마무리하라. 역할을 얼마나 잘 수행했는지 반성하고 학습자에게 직접 물어보십시오. 당신의 기법이 얼마나 유용했는지에 대한 피드백을 받는다면, 다양한 guidance 전략의 신중한 실행이 가능해질 것이며, 당신이 이 역할을 수행하는데 있어 개선하는데 도움을 준다.

Close each interaction by soliciting feedback to help you improve your use of all strategies. Reflect on how well you stayed in the role, and ask the learner directly. Feedback on the usefulness of your technique allows for deliberate practice of the various guidance strategies and helps you improve in carrying out this role.


장기적으로 할 수 있는 일

What You Can Do LONG TERM


소속된 프로그램에서 조언, 코칭, 멘토링이라는 용어가 어떻게 사용되고 있는지 검토하십시오. 동료와 정의를 공유하십시오. 특히 학습자와 대화할 때 모든 조언자, 멘토, 코치라는 용어의 개념을 명확하게 하라.

Review your program’s use of the terms advising, coaching, and mentoring. Share the definitions with colleagues. Make sure that all advisors, mentors, and coaches clarify terms, especially when talking with learners.


다양한 환경에서 다양한 수준의 학습자와 주제/초점을 가진 다양한 기술을 의도적으로 연습할 수 있는 기회를 모색한다. 도움을 요청받는다면, 어떤 전략을 사용할지 생각해보라. 적절한 용어를 고르라. 조언인가 코칭인가 멘토링인가? 

  • 조언: '너는 이렇게 해야 한다'

  • 코칭: '무엇을 할 수 있겠는가?'

  • 멘토링: '내 경험에 따르면, 이렇게 하는 것이 좋더라'

Seek out opportunities to deliberately practice different skills with different levels of learners in different settings and with different topics/focus. When asked for your input, think about what strategy to use. Select words that differentiate advising versus coaching versus mentoring (eg, 

  • advising, ‘‘You should . . .’’

  • coaching, ‘‘What could you do?’’

  • mentoring,‘‘In my experience, this has worked.’’


교수, 학생에게 guidance의 범위에 걸친 차이에 대해 교육한다. 각 방법의 주요 특징을 정의하고 역할극 중에 역할을 명시적으로 정의하는 연습을 한다. 학습자가 자주 찾는 사람에게 효과가 있는 기술과 그렇지 않은 기술을 공유하도록 격려한다. 저부담 환경에서 피드백을 통해 연습할 수 있는 기회를 활용하십시오.

Provide training to faculty and learners on the differences across the spectrum of guidance. Define key features of each method and practice explicitly defining roles during role-plays. Encourage those who are frequently sought out by learners to share techniques that work and those that do not. Take advantage of opportunities to practice with feedback in a low-stakes environment.





 2018 Apr;10(2):227-228. doi: 10.4300/JGME-D-18-00111.1.

Choosing When to AdviseCoach, or Mentor.

PMID:
 
29686766
 
PMCID:
 
PMC5901806
 
DOI:
 
10.4300/JGME-D-18-00111.1


졸업후의학교육의 교수학습에 교육이론의 실용적 가치(J Grad Med Educ, 2018)

The Practical Value of Educational Theory for Learning and Teaching in Graduate Medical Education

Anthony R. Artino Jr, PhD

Abigail Konopasky, PhD



"좋은 이론만큼 실용적인 것은 없다." —커트 르윈, 19431.

‘‘There is nothing as practical as a good theory.’’ —Kurt Lewin, 19431


'좋은 선생님이 되기 위해 교육 이론에 대해 알 필요는 없다. 가르치는 것은 과학보다 예술이고, 그래서 정말로 필요로 하는 것은 내용 지식, 연습, 약간의 타고난 재능뿐이며, 나는 거의 모든 것을 아무에게나 가르칠 수 있다."

‘‘I don’t need to know anything about educational theory to be a good teacher. Teaching is more art than science, and so all I really need is content knowledge, practice, and a little bit of natural talent, and I can teach anyone almost anything.’’


우리의 관점에서, 좋은 선생님이 되기 위해 필요한 것에 대한 이 철학은 근시안적이다.

From our perspective, this philosophy on what it takes to be a good teacher is shortsighted.


GME에서 교육이론의 실용적 가치

The Practical Value of Educational Theory in GME


무엇보다도, 교육 이론은 어떤 동기부여가 된 교사들이 효과적인 교육을 할 수 있는 틀을 제공한다. 교육 이론은 교수-학습과 관련된 기본 메커니즘에 대한 설명을 제공한다.2 이론은 우리에게 왜 그리고 어떤 상황에서 특정한 학습 전략과 교수법이 효과가 있는 반면, 다른 이론들은 그렇지 않은지를 말해준다.

First and foremost, educational theory provides a framework from which any motivated teacher can build effective instruction. Educational theory provides explanations about the underlying mechanisms involved in learning and teaching.2 Such theories tell us why and under what circumstances certain learning strategies and teaching methods work, while others do not.


자기결정이론은 좋은 예다. 심리적인 안녕을 얻기 위해서는 능력, 자율성, 관련성 등 3가지 본질적인 심리적 욕구가 충족되어야 함을 시사한다.3

Self-determination theory is a good example. It suggests that 3 essential psychological needs must be met in order to achieve psychological wellness: competence, autonomy, and relatedness.3


둘째, 교육 이론은 교육자들이 전통과 의례가 아니라 경험적으로 시험된 원칙에 근거한 교육을 설계하는 데 도움을 줄 수 있다.5 일상 생활에서 교직원들은 종종 레지던트에게 동료들에게 그들이 가르치는 방식을 가르친다. 비록 이것이 성공하는 경우가 있을지라도, 그것은 근거가 있어서라기보다는 그저 운이 좋았기 때문이다.

Second, educational theory can help educators design instruction grounded in principles that have been empirically tested, rather than being the result of tradition and ritual.5 In everyday practice, faculty often teach residents and fellows the way they were taught. Although this may be successful, its effectiveness is based more on luck than evidence.


간단히 말해서 이론과 연구는 교육적 실천과 시너지 효과를 낼 수 있다.  이론과 실천은 서로에게 도움이 되며 서로를 향상시킨다. 이러한 시너지의 결과는 "연구 증거와 실험된 이론에 기초한 과학적으로 유효한 교육 방법의 집합"이다. 약간의 독창성으로 숙련된 교사들은 '근거와 이론에 기반한 방법'을 그들 자신의 맥락에 맞게 조정할 수 있다.

In short, theory and research can work in synergy with educational practice—each informing and enhancing the other. The consequence of such synergy is ‘‘a set of scientifically valid methods of instruction based on research evidence and tested theory.’’6 With a little ingenuity, skilled teachers can then adapt these methods to their own context. 


교육의 예술이 작동하는 곳은 바로 이론을 실천으로 번역하는 순간이다.

It is here, in the translation from theory to practice, where the art of teaching, operates.7



교육이론의 한계

Limitations to Educational Theory


교육이론에도 한계가 있다. 가장 중요한 것은 "모든" 맥락에서 배움과 가르침을 충분히 설명하는 단일 통일된 교육 이론이 없다는 것이다. 그 대신 교사는 다른 이론적 진영이나 관점으로 조직된 이론들의 집합체 속에 갇혀 있는데, 이것은 일반적으로 하나의 상황에서 학습과 가르침의 측면을 설명하지만, 다른 상황에서는 적용가능하거나 유용하지 않을 수도 있다.

Educational theory also has limitations. The most important may be the lack of a single unified education theory that fully explains learning and teaching in all contexts. Instead, the teacher is stuck with a collection of theories organized into different theoretical camps or perspectives, which generally explain aspects of learning and teaching in certain situations, but may not be applicable or useful to others.


예를 들어, 행동주의는 어떻게 교실에서 학생들이 특정한 보상과 처벌에 반응할 것으로 예상할 수 있는지를 설명하지만, 임상에서 병동 회진 동안 굴욕과 수치심과 같은 가혹한 교육 관행에 의해 학습자 웰빙이 어떻게 부정적인 영향을 받는지에 대해서는 거의 설명하지 못한다.8

For example, behaviorism explains how students in a classroom can be expected to respond to particular rewards and punishments,2 but it does little to explain how learner well-being is negatively affected by harsh teaching practices like humiliation and shaming during bedside rounds.8


이러한 한계를 염두에 둔다면, 교육 이론을 지도map에 대한 우리의 사고 방식에 비유해서 생각하는 것이 도움이 된다. 지도map는 어떤 영역을 추상적으로 나타낸 것이지만, 어떤 단 하나의 지도도 실제의 물리적 지형을 완전하고 정확하게 나타낼 수 없다.2 대신에, 주어진 지도는 단지 그것이 묘사하는 지역의 단순화된 버전일 뿐이고, 특정한 측면을 부각시키고 다른 것을 모호하게 한다. 지도는 영토가 아니라고 말하는 이유이다.

With these limitations in mind, it is helpful to think about educational theories similarly to how we think about maps. A map is an abstract representation of the territory it depicts, but no singular map can completely and accurately represent the actual physical terrain.2 Instead, a given map is just a simplified version of the territory it depicts, highlighting certain aspects and obscuring others. Thus, it has been said that the map is not the territory.9


교육 이론은 비슷한 방식으로 작동한다. 각 이론은 학습과 가르침에 관련된 기본 메커니즘을 추상적으로 설명하지만, 어떤 단 하나의 이론도 완전하고 정확하게 설명하지는 않는다. 따라서 교사는, [숙고적deliberate 실천과 다양한 상황에 걸친 체계적인 실험을 통해] 다양한 교육 이론에 친숙해지고 적용하는 법을 배우는 것이 유익하다.

Educational theories function much in the same way. Each theory is an abstraction that attempts to explain the underlying mechanisms involved in learning and teaching, but no singular theory does so fully and accurately. As a teacher, it is beneficial to become familiar with, and learn to apply, different educational theories through deliberate practice10 and systematic experimentation across diverse situations.


변화하는 지형을 반영하기 위해 지도를 업데이트하는 방식과 마찬가지로, 교육 이론도 현대 연구와 새로운 발견을 반영하도록 업데이트하고 조정할 필요가 있다. 예를 들어, 2000년대 초에 연구자들은 새로운 정보를 다룰 때 학습자가 작업기억 용량이 제한된다는 전제 하에 모델인 인지 부하 이론을 연구하고 있었다.11 

Similar to the way maps are updated to reflect changing terrain, educational theories also need to be updated and adjusted to reflect contemporary research and new discoveries. For example, in the early 2000s, researchers were studying cognitive load theory, a model based on the premise that learners have a limited working memory capacity when dealing with new information.11 


연구자들은 인지부하를 줄이기 위해 고안된 교육방법이 초심자에게는 매우 효과적이었지만, 더 경험이 많은 학습자에게는 오히려 그 효능을 상실하고 심지어 학습에 부정적인 결과까지 가져온다는 것을 발견했다.12 전문지식 역전 효과expertise reversal effect로 알려진 이 생각은 교육에도 직접적인 영향을 끼쳤다.12학습자의 전문성 수준에 맞게 방법을 조정해야 한다.

The researchers found that instructional methods designed to reduce cognitive load were highly effective for novices, but often lost their efficacy, and even had negative consequences for learning, when employed with more experienced learners.12 This idea, known as the expertise reversal effect, has direct implications for instruction, as it suggests that methods should be tailored to the learner’s expertise level.


GME의 학습과 교육에 교육이론의 적용

Applying Educational Theory to Learning and Teaching in GME


GME와 관련된 학습 이론

Learning Theories Relevant to GME


사회적 인지 이론은 20세기 후반 행동주의의 한계와 행동주의가 자극에 대한 학습자의 반응에만 초점을 맞추는 것에 대응하여 개발되었다.2 사회적 인지 이론은 학습자가 자신의 신념, 목표, 태도 및 가치를 가진 능동적인 참여자(또는 대리자)라고 여긴다. 따라서, 학습자는 강사instructor의 적절한 지원을 받아 자신의 학습 과정을 계획하고, 감시하고, 성찰할 수 있어야 한다.15 

Social cognitive theory was developed in the second half of the 20th century in response to the limitations of behaviorism and its sole focus on learner responses to different stimuli.2 Social cognitive theory posits that learners are active participants (or agents) with their own beliefs, goals, attitudes, and values. Thus, learners, with appropriate support from instructors, should be able to plan, monitor, and reflect on their own learning processes.15 


예를 들어, 고품질의 환자 기록을 어떻게 써야하는지에 대해 고심하는 인턴은 자신의 병력청취에 능력을 향상시키기 위한 목표를 설정하고, 멘토와 함께 자신의 발전과정을 되돌아볼 기회를 제공받을 수 있다. 사회적 인지이론의 중심적 아이디어는 자기효능감으로서, 이것은 특정 분야에서 자신이 얼마나 능력있는지에 대한 학습자의 신념이다.16 긍정적인 자기효능감 신념은 학습 내용을 숙달하고 동료들의 성과를 관찰하는 능동적인 경험을 쌓으며 발전한다. 

For instance, an intern struggling with how to take a high-quality patient history could be offered opportunities to set goals for improving his or her history taking, and then reflect on his or her progress with a mentor. A central idea in social cognitive theory is self-efficacy, or a learner’s beliefs about how capable he or she can be in a certain area.16 Positive self-efficacy beliefs develop from active experiences mastering learning content and vicarious experiences observing peers’ achievements. 


따라서 임상의는 숙련도를 쌓기 위해 기술을 연습하고, 유사한 숙달 경험을 가진 동료를 관찰하는 과정에서 새로운 기술에 대한 자기효능감을 구축할 수 있다.

Thus, a clinician could build self-efficacy in a new skill with hands-on experiences practicing the skill to attain mastery and through observation of peers having a similar mastery experience.


상황학습이론은 능동적 참여에 뿌리를 둔 또 다른 이론이다. 그러나, 상황학습이론은 사회문화적 전통에서 출발했는데, 여기서 배움이란 사회적 관행과 맥락이 학습 자체와 얽혀 있는 것으로 본다.17 상황학습이론에서 학습은 "목표와 활동을 공유하는 개인의 집단"인 실천공동체에서 발생한다(예: 평가 연구에 관심 있는 임상의 집단이 실천공동체를 형성할 수 있다).

Situated learning theory is another theory rooted in active participation. However, situated learning theory emerges from sociocultural tradition, where social practices and contexts of learning are viewed as intertwined with the learning itself.17 In situated learning theory, learning happens in communities of practice—groups of individuals who share goals and activities (eg, a group of clinicians who are interested in assessment research might form a community of practice). 


실천공동체에 새로 들어온 학습자는 주변부에서 중심부로 이동하기 위해 공동체의 다른 사람들과 활발하고 열정적으로 참여해야 하는 초보자이다. 예를 들어, 2레지던트는 거주자는 의료 교육 연구 논문에서 주치의와 함께 작업할 수 있으며, 주치의는 이 논문의 나머지 초안을 작성한다. 이러한 공유된 실천을 통해 레지던트는 의학교육연구공동체의 중심부에 더 가까이 다가간다.

Learners new to a community of practice are viewed as novices who must actively and authentically engage with others in the community to move from the periphery to the center. For instance, a second-year resident might work with an attending on a medical education research paper, with the resident framing the introduction and the attending writing the rest of the paper’s first draft. Through this shared practice, the resident moves closer to the center of the medical education research community of practice.



GME에서 교육이론

Instructional Theory in GME


많은 교육적 이론은 많은 학습 이론들을 보완한다. 이것들은 특정 학습 이론에 기술된 원리에 근거하여 학습 환경을 구조화하는 방법에 대한 모델이다. 예를 들어, 인지 부하 이론에 대한 연구를 지시 설계를 위한 실용적인 모델로 변환하기 위해 4요소 교육설계(4C/ID) 모델을 명시적으로 만들었다. 일부 교사들이 초보 학습자를 단순하고 진정성없는inauthentic 않은 과제(예: 학습자에게 객관식 시험 제공)에만 참여시키려는 경향에 대한 반발로서, 4C/ID는 교수자가 보다 복잡하고 authentic한 과제에 학습자를 참여시키도록 권장한다.

Complementing the plethora of learning theories are a number of instructional theories. These are models for how to structure learning environments based on principles articulated in particular learning theories. For example, the 4-component instructional design (4C/ID) model was explicitly created to translate research on cognitive load theory into a practical model for designing instruction.19 Partly in reaction to teachers’ tendencies to engage novice learners exclusively in simple, inauthentic tasks (eg, giving learners a multiple-choice test), 4C/ID encourages instructors to engage learners in more complex and authentic tasks. 


레지던트들이 클리닉이나 병동 등에서 매일 authentic한 업무에 종사하는 GME에서는 이 아이디어가 완전히 새로운 것은 아니지만, 4C/ID는 교수에게 이러한 복잡성을 통해 학습자를 더 잘 지원하고 인지 과부하를 방지할 수 있는 전략을 제공한다. 예를 들어, 인턴은 필요할 때 감독자로부터 "적시에" 지원을 받아 복잡한 환자 기록을 가져가도록 요청받을 수 있다(예: 레지던트가 막혀있는 것 같으면 질환의 onset에 대해 물어볼 수 있는 단서를 제공한다). 

While this idea is not completely novel in GME, where residents engage daily in authentic tasks in clinics or inpatient units, 4C/ID provides instructors with strategies to better support learners through this complexity and prevent cognitive overload. For example, an intern might be asked to take a complex patient history, with ‘‘just-in-time’’ support from a supervisor when needed (eg, offering a cue to ask about the onset of illness if the resident seems stuck). 


더 오래되었지만 여전히 관련 있는 교육 이론은 Gagne의 9가지 교육 사건이다.14 이 모델은 교사들에게 다른 종류의 결과(예: 지식 합성과 같은 인지 결과 또는 봉합과 같은 운동 결과)를 지원하기 위한 학습 조건을 만들기 위한 구체적인 구조를 제공한다. 기억과 지식의 보유에 대한 인지 처리 이론과 동기에 대한 사회적 인지 아이디어를 사용하여, 9가지 '사건 시퀀스'는 강사가 지원 학습 환경을 조성하도록 가이드해준다. 

An older but still relevant instructional theory is Gagn´e’s 9 events of instruction.14 This model gives teachers a concrete structure for creating learning conditions to support different kinds of outcomes (eg, a cognitive outcome like knowledge synthesis or a motor outcome like suturing). Using cognitive processing theories of memory and knowledge retention, and social cognitive ideas about motivation, the 9 distinct sequenced ‘‘events’’ guide instructors in creating supportive learning environments. 


이러한 원칙은 처음에는 교실 학습을 위해 개발되었지만, GME 맥락에서 귀중한 지침을 제공할 수 있다. 명확한 학습 목표를 제공하고 새로운 개념과 현존하는 지식 사이에 명시적인 연결을 만드는 것은 대부분의 교육적 맥락에서 학습을 개선할 수 있는 유용한 관행이다.14

While these principles were initially developed for classroom learning, they can offer valuable guidance in GME contexts. Providing clear learning objectives and making explicit connections between new concepts and existing knowledge are helpful practices that can improve learning in most educational contexts.14



요약

Summary


좋은 이론만큼 실용적인 것이 없는 시대다.

It is during those times that there is nothing as practical as a good theory.1







 2018 Dec;10(6):609-613. doi: 10.4300/JGME-D-18-00825.1.

The Practical Value of Educational Theory for Learning and Teaching in Graduate Medical Education.

PMID:
 
30619514
 
PMCID:
 
PMC6314359
 [Available on 2019-12-01]
 
DOI:
 
10.4300/JGME-D-18-00825.1


문턱개념이란 무엇이고, 의학교육에 어떻게 활용될 수 있는가? (Med Teach, 2016)

What are threshold concepts and how can they inform medical education?

HILARY NEVE1, ANDY WEARN2 & TRACEY COLLETT1

1Plymouth University Peninsula Schools of Medicine and Dentistry, UK, 2The University of Auckland, New Zealand



배경

Background


성찰적 실천은 일종의 문턱개념으로 확인되었다(Joyce 2012). 고등교육에 대한 광범위한 연구 후 마이어와 랜드(2003)에 의해 처음으로 기술된 문턱개념은 다양하고 다양한 분야의 교육자들에 의해 수용되어, 점차 연구가 증가하게 되었다. 의학 교육에서는 거의 논의되지 않지만, 문턱개념은 의학교육자들에게 중요한 새로운 관점을 제공할 수 있는 잠재력을 가지고 있다.

Reflective practice has been identified as a likely threshold concept (Joyce 2012). First described by Meyer and Land (2003) following extensive research into higher education, threshold concepts have been embraced by educators in many and diverse disciplines, leading to a growing body of research. While rarely discussed in medical education, they have the potential to offer medical educators important new perspectives.


목적

Aims


문턱개념이란 무엇인가?

What is a threshold concept?


배움이란 핵심 개념의 블록을 형성하는 관점으로 볼 수 있으며, 종종 학습 성과로 요약된다. 핵심 개념은 학습을 위해 이해가 필요한 것으로서, 올바르게 구성되어야 한다.

Learning is often thought of in terms of building blocks of core concepts, which are often encapsulated in our learning outcomes. Core concepts have to be understood for learning to progress and must be correctly constructed.


문턱개념은 훨씬 더 많은 것을 할 수 있다. 그것들은 주제에 대해 질적으로 다른 관점을 야기하며, 주제의 숙달 [sic]을 이루는 것이다. 중요하게도, 문턱개념은 일종의 포털과 같은 것으로 묘사되어 왔다: "어떤 것에 대해 이전에 접근하기 어려웠던 새로운 사고 방식을 열어준다."

Threshold concepts, it is proposed, do much more. They lead to a qualitatively different view of subject matter and are central to achieving mastery [sic] of a subject. Importantly, they have been described as being like a portal: ‘‘opening up a new and previously inaccessible way of thinking about something’’ (Meyer & Land 2003).


Meyer and Land(2003)에 따르면, 문턱개념은 변혁적이다. 즉, 일단 이해되면 주제에 대한 학습자의 인식이나 그들이 세상을 보는 방법에 상당한 변화를 가져올 수 있다. 이전에 숨겨져 있던 연결이 시야에 들어오기 때문에 통합적이다(통합은 학습자를 위한 '아하!' 순간과 종종 연관된다). 더욱이 학생들이 포털을 향해 여행할 때, 말 그대로 문턱에 서 있는(라틴어로 문턱을 뜻하는 limina) 문턱 단계liminal phase(Cousin 2006)를 통과한다고 한다. 학생들이 인지적으로 또는 감정적으로 그들의 떠오르는 지식과 씨름하기 때문에 이 절박함은 종종 쉽지 않은troublesome 일이다. 일단 새로운 이해를 습득하게 되면, 습득 전으로 되돌릴 수 없으며, 잊혀지지 않는다. 각 문턱개념은 다른 개념 영역(Meyer & Land 2003)과 구분되어 제한된다bounded, demarcated.

According to Meyer and Land (2003), threshold concepts are transformative, in that once understood, they can result in a significant shift in a learner’ perception of a subject or how they see the world. They are integrative inasmuch as connections that were previously hidden come into view (integration is often associated with an ‘‘Aha!’’ moment for learners). Moreover, as students journey towards the portal, they are said to pass through a liminal phase, (Cousin 2006), literally standing on the threshold (limina meaning threshold in Latin). This liminality is frequently troublesome (Perkins 2006) as students wrestle cognitively and/or emotionally with their emerging knowledge. New understandings, once learned, are usually irreversible and unlikely to be forgotten. It is noted that each threshold concept is bounded, demarcated from other conceptual areas (Meyer & Land 2003).


첫 번째 문턱 개념 중 일부는 과학 과목에서 확인되었다. 예를 들어, 물리학의 중력(Meyer &Land 2003)과 생물학의 진화(Ross et al. 2010)와 같은 것이다.

Some of the first threshold concepts were identified in science subjects. For example, gravity in physics (Meyer &Land 2003) and evolution in biology (Ross et al. 2010)



의학교육에서 문턱개념

Threshold concepts in medicine


의학 교육자'는 그 자체가 교수학습과 관련된 문턱개념들과 씨름해야 할지도 모른다. 서비스 사용자의 목소리를 표현하고 평가하는 것(Kirwan 2014), 학습에 대한 학생 참여 유지(Noonan 2012), 사고 과정이 정답보다 더 중요하다는 것(Mills & Wilson 2014)등이 다른 분야에서 문턱개념으로 제안되었다. 외과적 훈련의 맥락에서, Kneebone(2009)은 시뮬레이션을 통해 임계 개념을 배우는 데 진정성authenticity이 중요하다고 제안했다.

Medical educators’ may themselves have to grapple with threshold concepts related to teaching and learning. Representing and valuing the voice of service users (Kirwan 2014), sustaining student engagement in learning (Noonan 2012) and thinking processes are more important than correct answers (Mills & Wilson 2014) have been proposed as threshold concepts for teachers in other disciplines. In the context of surgical training, Kneebone (2009) has suggested authenticity is important for learning threshold concepts through simulation.


문턱개념이 교육자와 학습자에게 어떻게 도움이 되는가?

How might threshold concepts assist medical educators – and learners?


문턱개념을 확인함으로써, 우리는 변혁적이고 개념적으로 어려운 아이디어들을 중심으로 학생들의 학습을 더 집중하고 구조화 할 수 있다. 예를 들어, 항상성도 하나의 문턱개념(Meek & Jamieson 2012)이며, 항상성을 가르치고 배우는 것은 어려운 일일 가능성이 있다. 항상성이라는 개념을 이해하기 위해서 교사들은 학생들이 이 개념에 대해서 thinking with 하게 만들어야 한다. thinking about만으로는 부족한다. 어떻게 학생들로 하여금 생리학, 생화학, 항상성이 깨짐으로서 생기는 여러 임상질환을 이해하게 할 것인가?

By identifying threshold concepts, we can better focus and structure student learning around these transformative and conceptually difficult ideas. For example, homoeostasis has been identified as a threshold concept (Meek & Jamieson 2012) and is likely to be problematic to teach and learn. Unless adequate time is provided and unless teachers engage students in thinking with, rather than about, the concept, how will students properly comprehend subjects, such as physiology, biochemistry or the many clinical disorders, that are underpinned by disrupted homoeostasis?


교사들은 학습자들이 새롭고 중요한 개념을 이해하는데 시간을 들일 수 있다는 것을 이해할 필요가 있다. 학습자가 문턱단계liminal phase에 있는 동안 "오래된old 이해와 창발적emergent 이해 사이를 오갈 수 있다"(Cousin 2006). 이 단계에서 학생들이 평가를 통과하더라도 부분적으로만 이해했을 수 있다.

Teachers need to understand that learners, can take time to get to grips with new and important concepts. While in the liminal phase, learners ‘‘may oscillate between old and emergent understandings’’ (Cousin 2006). In this stage, students may pass assessments, while only having a partial understanding of a subject.


문턱개념은 또한 우리가 전문직 정체성 형성 중에 마주친 역할의 혼동을 이해하는 데 도움을 줄 수 있다. 예를 들어, 학생들은 종종 불확실성을 감수하는 것에 대해 걱정을 표현하고, 불확실성이 임상에 필수적이라는 것을 받아들이기 위해 분투한다. 불확실성은 의학을 포함한 많은 분야에서 문턱개념으로 확인되었다(Collett et al. 언론에서).

Threshold concepts may also help us understand some of the role confusion encountered during professional identity formation. For example, students often express anxiety about working with uncertainty and struggle to accept that this is integral to clinical practice. Uncertainty has been identified as a threshold concept in many disciplines, including medicine (Collett et al. In press).


여러 학문을 해독함으로써 교사들은 학생들이 어려운 개념을 이해할 때 발생하는 중요한 병목현상을 식별하고 이를 명시적인 단계로 세분하는 데 도움을 줄 수 있다. 이러한 병목현상은 때때로 교사 자신의 가정과 암묵적인 지식과 관련이 있다: 전문가로서 교사들에게 어떤 개념은 스스로는 그 존재조차 알지 못할 정도로 자연스럽고 명백하기 때문에 간과할 수 있다. 성찰적실천도 그러한 예 중 하나이다. 그것은 경험이 풍부한 실무자들에게는 암묵적인 과정일 수 있지만, 학습자에겐 그 성격과 학습에서의 그것의 역할이 분명하지 않을 수 있다(Mann 등). 2009).

The process of decoding the disciplines (Middendorf & Pace 2004) can help teachers identify crucial bottlenecks in student understanding of troublesome concepts and break these down into explicit steps. These bottlenecks sometimes relate to teachers’ own assumptions and tacit knowledge: expert teachers may skip over steps that have become so innate and obvious that they are unaware of their existence. Reflective practice is one such example; it may be a tacit process for experienced practitioners, but its nature and its role in learning may not be apparent to learners (Mann et al. 2009).


결론

Conclusions


문턱개념이라는 프레임워크는 교육에 대한 우리의 전통적인 생각들에 대해 의문을 제기해야 한다. 예를 들어, 각 학습자가 transformation되는 과정이 복잡하고 다양하다는 사실은 선형적 학습모델에 도전한다(Land et al. 2005).

The threshold concept framework challenges us to question some of our traditional ideas about education. For example, the complexity and variation in each learner’s transformation (multiple attempts at understanding, experimentation and reflection on integration) challenges the linear learning outcome model (Land et al. 2005)


어쩌면 문턱개념 자체가 문턱개념일 수 있다! 경험에 따르면 이론을 이해하는 것은 그 자체로 귀찮을 수 있다는 것이다: 문턱개념의 개념은 처음에는 추상적이고 이해하기 어려워 보일 수 있다. 그러나 그것은 우리가 우리의 교육적인 일을 보고 실천하는 방식을 변화시킬 것이다.

It may be that threshold concepts are themselves a threshold concept! Our own experience is that getting to grips with the theory can be troublesome: the notion of threshold concepts can initially seem abstract and hard to understand. Yet it has transformed the way we see and practice our educational work.





 2016 Aug;38(8):850-3. doi: 10.3109/0142159X.2015.1112889. Epub 2015 Nov 26.

What are threshold concepts and how can they inform medical education?

Author information

1
a Plymouth University Peninsula Schools of Medicine and Dentistry , UK .
2
b The University of Auckland , New Zealand.

Abstract

The notion of "threshold concepts" is being widely applied and researched in many disciplines but is rarely discussed within medical education. This article is written by three medical educators who regularly draw on threshold concept theory in their work. They explore here the nature of threshold concepts and describe how the theory can offer medical educators new perspectives in terms of how they design curricula, approach teaching and support learners.

PMID:
 
26609736
 
DOI:
 
10.3109/0142159X.2015.1112889


학습의 정의 확장: 개인에서 사회에서 시스템으로 (Med Educ, 2019)

Expanding the definition of learning: from self to social to system

Douglas P Larsen




연구자들과 교육자들은 단순히 더 나은 교육보다는 효과적인 학습에 더 초점을 맞춘다.

researchers and educators focus more on effective learning rather than simply better teaching.


이런 것들이 발달했다.

문제 기반 학습,

팀 기반 학습,

검색 관행에 기반한 도구

경험 기반 임상 교육

마스터 학습자 등의 개념

자기조절학습(SRL)

problem-based learning,

team-based learning,

Tools based on retrieval practice

experience-based clinical education

Concepts such as the master learner

self-regulated learning (SRL)


규제 기관은 평생 학습을 촉진하기 위해 의과대학에 대한 인증 기준을 만들 때, 이러한 개인 중심적 사이클을 명문화하고, 의사들을 위해서 MOC 유지를 요구한다. 7,8 개개인의 SRL 스킬을 향상시키기 위하여 [학습 목표 및 개별화된 학습 계획 수립]과 같은 교육 개입은 이제 흔한 것이 되었다. 그러나, 학습과 학습자에게 초점을 맞추는 것이 항상 기대했던 대로 되는 것은 아니다.

Regulatory bodies codify the elements of this individual- centred cycle in accreditation standards for medical schools and maintenance of certification requirements for practising physicians as they attempt to promote lifelong learning.7,8 Education interventions such as learning goals and individualised learning plans, aimed at individual SRL skill development, have become common. However, this focus on learning and learners does not always play out as anticipated.


직장에서의 학습은 종종 단순히 한 명 학습자의 인지 과정 내가 아니라 [감독자와 연습생 사이의 상호작용]을 통해 발생한다.

learning in the workplace often occurs through the interactions between supervisors and trainees rather than simply within the cognitive processes of the learner.


관계적 주체성개개인이 자기행동의 목표를 공동-창조하기 위해 서로의 자원을 끌어모을 때 발생한다.14

relational agency occurs as individuals draw on the resources of each other to co-create the object of their actions.14


한 연구는 '직장에서의 학습은 의사소통적 관행에 있다'는 것을 보여주었다.

The present study adds to a growing body of research demonstrating that ‘learning resides in the communicative practices of workplaces’,


실제로, 몇몇 조사에서는 임상실습 학생 학습 목표이든 전공의의 개별화된 학습 계획이든, 표면적으로는 SRL을 위한 교육 개입으로 보이는 것들이 성공적으로 실행되기 위해서는 협력적인 대화가 필수적이라는 것을 발견했다.16–18 Berkhout 외 연구진 19에 따르면 직장에서의 학습경험이 늘어날수록, 학습자가 SRL을 추구할 때 점점 감독자와 더 협력적인 모습을 보였다.

Indeed, several investigations have found collaborative dialogue to be essential for successful implementation of ostensibly SRL educational interventions whether they be clerkship student learning goals or resident individualised learning plans.16–18 Berkhout et al.19 showed that as learners became more experienced in workplace learning, their pursuit of SRL became increasingly collaborative with supervisors.


수술실의 위계가 어떻게 포함적 또는 배제적 어포던스를 만들어 내는가? Nieboer 등은 수술실에서 레지던트들은 적극적인 참여를 통해서 임상적 의사결정 논의를 시작할initiate하도록 empower해준다는 것을 발견했지만, 이와 달리 외과 임상실습 학생에 대한 연구에서 따르면, 임상실습학생들은 학습 목표에 대한 대화를 시작할 책임은 감독관에게 있다고 생각하며, 종종 감독관들은 이에 대한 관심을 표명하지 않았다고 하였다.

How does the hierarchy of the operating room produce affordances that both include and exclude? Nieboer et al. found that residents’ active participation in the work of the operating room easily empowered them to initiate discussions around their decisions, but another study of medical students on a surgical clerkship found that students often perceived supervisors as responsible for initiating conversations about their learning goals and that supervisors often did not express an interest.21


단순히 [개별 학습 프로세스]에서 [학습을 위한 사회적 상호 작용]을 강조하는 것에는 그 이상의 무언가가 존재한다. 오히려, 우리는 실천과 관련된 전체 체계entire system of practice를 고려해야 한다.22: 사람, 공간, 계층, 사회적 규범, 노동의 분할, 공식적인 규칙과 규정, 기술, 담론 등. 이 모든 것이 상호작용하여 학습을 이루기 때문이다.

additional opportunities exist beyond simply shifting from individual learning processes to an emphasis on social interactions for learning. Rather, we must consider the entire system of practice: people, spaces, hierarchies, social norms, divisions of labour, formal rules and regulations, technologies, as well as discourses, which all interact to create learning.22


문화-역사 활동 이론(CHAT), 배우-네트워크 이론(ANT), 복잡성 이론, 시스템 엔지니어링과 같은 프레임워크는 [학습을 이끄는 연결과 상호작용]을 인식, 분석, 최적화할 수 있는 렌즈를 제공한다.22,25 이 렌즈를 통해 본다면, 학습 시스템은 [참가자와 개별 요소가 어떻게 변화하는지] 뿐만 아니라 [시스템 자체가 어떻게 변화하는지]를 보여준다. 이러한 시스템 자체의 변화 과정은 확장적 학습expansive learning이라고 일컬어 왔다.

Frameworks such as cultural-historical activity theory (CHAT), actor-network theory (ANT), complexity theory and systems engineering, among others, provide lenses to recognise, analyse and optimise the connections and interactions that lead to learning.22,25 When seen through these lenses, a learning system describes not only how the participants and elements change but also how the system itself changes.22,25 This process of system change has been referred to as expansive learning.28


시스템 관점에서 학습을 보는 것은 중요하다. 시험이나 역량으로 측정되는 [사실이나 기술의 축적]과 같은 말로는 설명은 학습을 적절히 설명해주지 않는다. 그보다, 학습이란 것은 [시스템을 구성하는 모든 요소들의 상호 작용으로부터 나타나는 실천의 변화]이며, 왜냐하면 시스템 그 자체가 그러한 실천의 변화로부터 변화하기 때문이다.

To see learning from a systems perspective has significant consequences. Learning can no longer be adequately described as the accumulation of facts or skills measured on tests or through competencies. Rather, learning will be seen as changes in practice that emerge from the interplay of all of the elements of a system as the system itself changes from those practices.


교육자들은 학습자가 실천 시스템systems of practice에 온전히 참여할 수 있도록 교육과정을 설계해야 하며, 학습자가 참여를 통해서 자기자신과 자신의 실천이 어떻게 변화되는지 인식하도록 도와주어야 한다.

Educators will design curricula that enable full participation by trainees in the systems of practice and help learners to recognise how they and their practices are changed by that participation.


  • 교수들은 (더 이상 자신을 콘텐츠 제공자가 아니라) 실천의 공동창출 과정에서 스스로도 변화하는 시스템의 참여자로 인식해야 할 것이다.

Faculty members will not see themselves as content deliverers but as participants that are also changed through the co- creation of practice.


  • 환자들은 교육과 헬스케어 관행 양쪽의 공동 창조자co-creator가 될 것이다.

Patients will be co-creators of both educational and health care practices.


  • 규제기관은 [학습과 역량의 정의]를 확장하여 [실천의 복잡성을 인정하는 디자인 기준]을 포함할 것이다.

Regulatory bodies will expand their definitions of learning and competence and design standards that account for the complexities of practice.


  • 연구자들은 시스템의 요소들 사이의 복잡한 관계를 풀기 위해 더 미묘한 질문을 할 것이다.

Researchers will ask more nuanced questions to untangle the complex relationships between elements of systems.





 2019 Jun;53(6):539-542. doi: 10.1111/medu.13893. Epub 2019 Apr 26.

Expanding the definition of learning: from self to social to system.

Author information

1
Neurology, Washington University in St. Louis, St. Louis, Missouri, USA.

PMID:
 
31025769
 
DOI:
 
10.1111/medu.13893


학습이론과 교육개입: 층화분석을 통한 의미있는 영향력근거 만들기(Acad Med, 2019)

Learning Theory and Educational Intervention: Producing Meaningful Evidence of Impact Through Layered Analysis

Anna T. Cianciolo, PhD, and Glenn Regehr, PhD




최근 수십 년 동안, 의학교육의 혁신은 증가해 왔다.

In recent decades, innovations in medical education have multiplied,


그러나 혁신을 구현하는 것은 어렵다. 잘 확립된 방법에 기초한 혁신도 [필요한 조직 변경의 범위 때문에] 예상대로 구현되거나 시간이 경과함에 따라 지속되지 않을 수 있다.7

Implementing innovations, however, is difficult; Even innovations based on well-established methods may not be implemented as expected, or sustained over time, because of the extent of organizational change needed.7


"효과가 있었는가?"에 대한 평가연구는 의학 교육 저널에서 가장 많이 인용되는 연구결과들 중 하나이다.11 그러나, 바람직한 결과를 입증하는 것조차 쉽지는 않다. 체계적인 검토와 메타 분석은 일반적으로 학업 성과의 큰 차이를 보여주지 않는다.10 또한 혁신의 긍정적인 효과가 피상적이고 과도하며, 12 또는 심지어 잠재적으로 오해의 소지가 있다고 보고되는 것도 드문 일이 아니다.13,14 

“Did it work?” evaluations are among the most cited publications in medical education journals.11 However, demonstrating desired outcomes also is difficult. systematic reviews and meta-analyses commonly show no significant difference in academic performance.10 In addition, it is not uncommon for the positive reported effects of innovations to be superficial and transient,12 or even potentially misleading.13,14 


의료 교육 커리큘럼 실험에서 세 가지 광범위한 비평이 있다.

  • 하나는 무원칙한 실험 설계와 실행이다.15 

  • 또 다른 하나는 지배적인 인지, 사회, 기술 또는 기타 학습 조건에 대한 부적절한 설명이다. 이 경우에 학습자가 달라지면 근본적으로 다른 경험을 할 수 있고 따라서 일관되지 않은 결과를 얻을 수 있다.10

Three broad critiques have been leveled at curricular experiments in medical education. 

  • One is the use of unprincipled experimental design and conduct.15 

  • Another is inadequate description of the prevailing cognitive, social, technological, or other learning conditions,12,15 which could produce fundamentally different experiences for different learners and therefore inconsistent results.10


아마도 비판 중 가장 많은 것은, 어떤 혁신이 교육적으로 작동해야만하는 개념적 기전에 대한 설명이 취약하다는 것이다. 이렇게 이론적 틀의 없을 경우, 인터벤션을 설계하고 시험하고 성과 척도를 선택하는 능력을 제한한다.12–14

Perhaps the broadest of these critiques is weak articulation of the conceptual mechanisms by which an innovation should work educationally. This lack of theoretical framing limits our ability to design and test interventions and to select outcome measures.12–14


이러한 비평은 [커리큘럼 실험]이 미래의 설계와 평가에 더 도움이 되려면, [주어진 맥락에서의 학습에 대한 이해를 심화시키기 위해 더 많은 것을 해야 한다]는 것을 의미한다.9 다른 식으로 말하자면, 일관성 있고 의미 있는 영향의 증거를 얻으려면, "그것이 작동했다"는 결론을 내리기 전에 "그것"이 무엇인지에 대한 면밀한 조사가 있어야 한다.

These critiques imply that curricular experiments must do more to deepen understanding of learning in context so that they can better inform future design and evaluation.9 Put another way, to produce coherent, meaningful evidence of impact, conclusions that “it worked” should not precede scrutiny of what “it” was.


"의도적인 개입은 무엇인가?", "실제로 그런 개입이 있긴 했는가?"

“What is the intended intervention?” and “Did that intervention, in fact, occur?”


개입의 층위

Layers of Intervention


"의도한 개입이란 무엇인가?"라는 질문은 개입이 정체성을 갖는다고 본다. 개입이란 것은 [변형에도 불구하고 자신의 본질을 유지하는 분류 가능한 실체]다. 개입의 본질은 개입의 기저에 깔린 교육 철학이다: 이 교육철학은 서로 다른 교육 철학을 바탕으로 한 다양한 개입을 구별해주는 개념적 학습 조건이다.

The question “What is the intended intervention?” involves seeing an intervention as having an identity; it is a classifiable entity that retains its essence despite being adapted.16 This essence is the intervention’s underlying educational philosophy: the conceptual learning conditions that distinguish the intervention from those based on different educational philosophies.17


바르피오와 동료들은 "테크닉이나 원칙이 새로운 맥락으로 변형될 때, 변화에도 불구하고 여전히 그 혁신의 철학을 지지하는지를 분석하는 것이 중요하다"고 지적했다. 그들의 훈계는 '정체성'을 교육 혁신의 핵심 관심사로 만든다; 정체성이 핵심이 된다는 것은 [개념적 학습 조건의 유지]를 우선시하고, [개념적 학습 조건을 보존하기 위해 원칙과 기법의 공들인 적응]을 요구한다.

Varpio and colleagues17(p365) noted, “When techniques and/or principles are modified to a new context, it is important to analyze if the modifications still uphold the innovation’s philosophy.” Their admonition makes identity a core concern in educational innovation; it prioritizes the maintenance of conceptual learning conditions and calls for conscientious adaptation of principles and techniques in order to preserve them.


교육 실험의 관점에서 보면, "적절하게 스케일링되지 않으면, 개입은 완전히 다른 것이 될 수 있다. 잘못 된 개입은 이론이 주어진 규모에서 작동하는지 여부에 대한 검증이 아니라, 그냥 처리되어야 할 워크시트나, 통과해야만 하는 수업'에 그칠 수 있다."18(p23). 즉, "적절하게 스케일링되지 않으면", 개입은 그 본질을 잃고, 혁신가는 개입의 표면적 특성만 집행하게 되어서, 당초에 촉진하려던 개념적 학습 조건은 무시하게 된다.

Put in terms of educational experimentation, “if scaled improperly … interventions could become something different entirely—not a test of whether the theory works at scale, but a worksheet to be handed out or a lesson to ‘get through.’”18(p23) That is, when “scaled improperly,” an intervention’s essence is lost; the innovator has focused on the implementation of surface-level characteristics without attending to the conceptual learning conditions they were designed to foster.16


"실행implementation"이란 특정한 교육적 개입의 기능에 영향을 미치는 복잡한 요인 집합으로 구성된다.12,16 예를 들어, 커리큘럼에 서로 상충하는 철학을 구체화하는 혁신을 포함시키면, 더 작은 수준에서 이뤄지는 개입에서 학습자의 참여에 영향을 줄 수도 있고, 개념적 학습 조건을 수립하려는 노력에 방해가 될 수도 있다.19"의도한 개입은 무엇인가?"를 확인하기 위해서는 혁신자들은 [underlying theory]와 [광범위한 상황 요인]을 모두 설명해주는 개입의 정체성을 밝혀야 한다.

implementation comprises a complex set of factors that affect an educational intervention’s function.12,16 For example, embedding an innovation in a larger curriculum that embodies competing philosophies can affect learner engagement with the local intervention and negate efforts to establish the conceptual learning conditions of interest.19 Asking, “What is the intended intervention?” requires innovators to represent their intervention’s identity in a way that accounts both for underlying theory and a broad range of contextual factors.20


실행이 [개입의 본질을 보존하는지 여부]를 결정하는 것은 실험의 기본 요소인 조작 점검manipulation check과 유사하다.20

Determining whether implementation preserves a intervention’s essence has parallels in a basic element of experimentation: the manipulation check.20


  • 조작에 따라 결과가 예상대로 나온다면예상 (개입은 "작동"), Manipulation checks는 이 효과가 [이론에 기반한 예상했던 주장의 결과인지] 또는 [부수적이고 잘 이해되지 않는 조건에 따른 결과인지]를 실험자들이 구별하는 데 도움이 된다. 

  • 반대로 예상대로 결과를 얻지 못할 경우(개입은 "작동"하지 않음)Manipulation checks은 [이것이 이론의 실패]인지 아니면 [원하는 개념적 조건을 만드는 표면적 조작의 실패인지]를 실험자가 알 수 있도록 도와준다. 

Manipulation checks는 이런 방식으로 개입의 정체를 추궁한다.

  • When a manipulation has the anticipated outcome (the intervention “works”), manipulation checks help experimenters discern whether this effect enables plausible claims about theory or instead must be attributed to incidental, poorly understood conditions. 

  • When a manipulation fails to have the expected outcome (the intervention does not “work”), the manipulation check helps the experimenter know whether this was a failure of theory or the failure of surface manipulations to create the desired conceptual conditions. 

In this way, manipulation checks interrogate an intervention’s identity.



그림 1

Figure 1



(1) 교육철학은 가장 아래층으로, 즉 개입의 본질이다. 근본적 레이어란, 개입이 그것을 설계한 사람이 주장하는 것이 되기 위해 유지되어야 하는 맥락-비의존적인, 이상적인 학습 조건의 진술이다.

Educational philosophy is shown at the bottom of the figure as the foundational layer, the essence of an intervention. It is a context- independent, idealized statement of the learning conditions that must hold for the intervention to be what its designer claims it is.


(2) 근본적 레이어의 위에는 개입의 구조적 측면을 반영하는 레이어가 존재한다. 이 레이어는 맥락에 따라 조정될 수 있지만, 그럼에도 불구하고 개념적 학습 조건을 확립하기 위해서는, 다소 일반화되고 비교적 안정적인 접근방식이어야 한다.

Above this is the layer of principles, reflecting the structural aspects of the intervention, which may be adjusted adaptively to context, but nevertheless represent somewhat generalized and relatively stable approaches to establishing conceptual learning conditions.


(3) 개입의 가장 상층에는 가장 맥락에 민감한 것들이 존재하고, 이것은 주로 테크닉으로 구성된다. 이 테크닉은 맥락적 요인에 따라 달라지며, 국지정 상황에 따라 맞춤형으로 개입이 달라질 수 있다.

The top layer of the intervention, the most context sensitive, comprises techniques, which account for contextual factors and allow the intervention to be tailored to a local setting.


레이어를 명확하게 나누지 못하는 것은, 실제적으로 레이어 간 명확한 경계가 없음을 반영한다.

The lack of clear delineations between layers realistically reflects the absence of clear boundaries between them.


개입을 여러 층으로 보고, 맥락적 영향을 받는 정도가 각각 다르다고 본다. 

Viewing interventions as layered, with each layer differing in sensitivity to contextual influence


즉, 개입의 표면 특성을 바꾼다고 해서, 개입의 정체성을 위협하지는 않는다.

That is, adaptations of an intervention’s surface characteristics do not necessarily threaten its identity;


반대로, 테크닉을 유지한다고 해서 반드시 교육철학이 보존되는 것도 아니다.16

Conversely, as Yeager and Walton18 noted, retention of techniques across contexts does not necessarily imply preservation of educational philosophy.16


작업 예제: PBL

A Worked Example: PBL


PBL의 교육철학은 집중적으로 조사되었다.25 그 현재 형태는 발달하는 의사들이 맥락-특이적인 풍부한 지식을 구성해야 하고, 이 지식구축은 본질적으로 동기 부여되고 자기조절되는 학습과정을 통해 사회적 맥락 안에서 일어난다는 생각을 반영한다.26,27 그럼에도 불구하고 실제로 무엇이 PBL 튜토리얼 동안 진행되며, 이것이 지식 구축 프로세스와 부합하는지 여부는 대부분 검증되지 않은 채로 남아 있다.28–30

PBL’s educational philosophy has been investigated intensively.25 Its current form reflects the idea that developing physicians must construct a wealth of context-specific knowledge and that this knowledge construction occurs within a social context via an intrinsically motivated and self-regulated learning process.26,27 Nevertheless, what actually goes on during PBL tutorials, and whether it resembles this knowledge construction process, remains largely unexamined.28–30


그림 2.

Figure 2.



PBL의 교육 철학: 이론적 학습 조건, PBL을 강의와 구분하는 구성주의적 학습 철학, 32 일대일 과외와 유사한 사회적 학습 활동을 반영하는 학습 조건, 33,34

PBL’s educational philosophy: the theoretical learning conditions, consistent with a constructivist philosophy of learning, that distinguish PBL from lecture,32 that reflect social learning activities akin to those of one-on-one tutoring,33,34


이러한 조건은 주제, 학습 도구 및 교육 자료뿐만 아니라 학습자 및 학습자-교수자 간 상호 작용으로 구성된다.35,36

These conditions comprise learner interaction with one another and the facilitator, as well as with the subject matter, learning tools, and instructional materials.35,36


PBL의 원칙(그림 2, 중간 계층)는 PBL의 사회적, 인지적 학습 조건을 확립하는 그룹 구성원들 사이의 문제 중심적이고 긍정적인 상호의존성을 촉진하기 위해 생각되는 구조적 요소들이다.35,44

The principles of PBL (Figure 2, middle layer) are structural elements thought to foster the problem-driven, positive interdependence among group members that establish PBL’s social and cognitive learning conditions.35,44


  • 첫 번째 원칙은, 실세계 관련 문제(예: 임상 사례)의 사용은 학습자의 본질적인 관심을 자극하여 교육 도구 및 자료에 참여하게 한다.36,45

One principle, the use of relevant, real-world problems (e.g., clinical cases), motivates learners’ intrinsic interest to engage with instructional tools and materials.36,45

  • 두 번째 원칙, 즉 PBL은 학습자 주도적이라는 원칙은, 학습자가 자신의 문제 해결 목표를 정의하고, 개인적으로 지식 격차를 식별하고, 이를 해결하기 위한 접근법을 만들 기회를 제공한다.36,45

A second principle, that PBL is learner led, affords learners the opportunity to define their own problem-solving goals, to personally identify their knowledge gaps, and to create an approach to addressing them.36,45

  • 세 번째 원칙인 문제 해결은 협력적 노력이며, 학습자가 문제 해결을 그룹화하여 집단적인 해결책에 도달했을 때 성공을 달성하는 데 자신의 이해와 사고 능력을 기여할 수 있도록 한다.35,44,45

A third principle, that problem solving is a collaborative effort, enables learners to contribute their own understanding and thinking skills to group problem solving, achieving success when a collective solution is reached.35,44,45


이러한 원칙을 고수하면 PBL의 인지적, 사회적 학습 조건이 촉진되지만, 구조적 특성이 다르더라도 교육 철학이 보존되는 등의 적응이 필요할 수 있다.

Adherence to these principles promotes PBL’s cognitive and social learning conditions, but context may call for adaptations such that educational philosophy is preserved even if structural characteristics differ.


비록 임상 사례들이 새로운 의대생들에게 일정정도의 미스터리와 충분한 참여 기회를 제공하지만, 문제 해결은 일반적으로 기초 과학 지식의 획득이라는 궁극적인 목표를 가지고, 비교적 구조화된 가설-연역적 과정을 적용하는 것을 포함한다. 그러나 가설연역적 조사 과정은 [학습자들이 정보를 수집하는 쉽고 더 만족스러운 방법이 있다는 것을 발견함]에 따라 매력을 잃는다.47

although clinical cases offer new medical students a degree of mystery and ample opportunity for engagement, problem solving commonly involves applying a relatively structured, hypothetico- deductive process with the ultimate goal of acquiring basic science knowledge.21,46 This investigative process loses its appeal as learners discover that there are less arduous, more satisfying ways to gather information.47


짧은 시간 내에 많은 양의 지식을 습득해야 한다는 학생들의 우려를 고려한다면, 학생 참여를 촉진하기 위해 튜터의 강화된 가이드가 필요할 수 있다.

increased tutor guidance may be needed to stimulate students’ engagement in light of their concerns about acquiring a high volume of knowledge in a short time.


적어도 어떤 조건에서는, PBL의 특징인 문제 중심적이고 협력적인 탐구를 유지하기 위해 다소 더 많은 튜터 주도적 상호작용이 바람직할 수 있다. 이러한 예들은 개입 설계가 맥락을 고려해야 한다는 것을 보여줄 뿐만 아니라, 맥락이 특정 개입의 원칙을 어떻게 의도하지 않은 방식으로 구현하도록 영향을 미치는지를 보여준다.

at least under some conditions, a somewhat more tutor-led interaction may be desirable to maintain the problem- centered, collaborative inquiry definitive of PBL.25,47 These examples not only show that intervention design must take context into account; they also illustrate how context may influence the implementation of principles in unintended ways.


맥락적 요인의 결과로, 교육 테크닉마다 큰 차이가 생길 수 있는데, 그렇지만 각각의 variation이 모두 PBL의 원칙을 반영할 수 있다. 다양한 variation이 존재한다는 것은 그룹 규모와 구성, 튜터 콘텐츠 전문성, 문제 형식과 양식과 같은 주제에 대한 논의가 활발하다는 점에서 잘 드러난다. 

As a result of contextual factors, there may be wide variability in instructional techniques, with each variation reflecting a principled approach to PBL. This is illustrated by the lively discussion in the literature on such topics as group size and composition,36,44,48 tutor content expertise,49 and problem format and modality.36,50,51


이것들과 관련된 많은 질문들에 대한 답은 "그때마다 다르다"이다.

The answer to these and many related questions is “It depends,”


실행과정에서 설계자의 목표가 전복될 수 있는 방법은 많다. 예를 들어 

  • 사례집과 중복되는 강의를 동시에 하는 것(예: "하이브리드 PBL"19), 

  • 사례 토론 전에 학습 목표를 주는 것,24 

  • 고부담 개별 시험을 보는 것 등은 모두 본질적으로 동기 부여되고 자기 주도적인 학습을 탈선시킬 수 있다. 

There are numerous ways that implementation can subvert the designers’ goals. 

  • For example, holding concurrent lectures that duplicate case material (i.e., “hybrid PBL”19), 

  • distributing learning objectives prior to case discussion,24 and 

  • high-stakes individual testing47 can all derail intrinsically motivated and self-directed learning. 


그러므로 실제로 실행된 개입이 원칙에 부합하고, 의도된 교육 철학을 담았다고 합리적으로 주장하기 위해서는, 맥락에서 기술에 대한 신중하고 직접적인 분석이 필요하다.

Therefore, careful, direct analysis of technique in context is needed to plausibly argue that a given implementation is principled and has achieved the intended educational philosophy.


레이어드 분석

Layered Analysis


교육적 조작 점검의 형식을 제공하는 형태로서, 계층 분석layered analysis은 (결과와는 상관없이) 교육자의 주장을 뒷받침한다. 

특정한 방법으로 개입했다

그들의 혁신이 기초적인 교육 철학에 의해 정의된 학습의 조건을 설정했다.

Providing a form of educational manipulation check,20 layered analysis supports an educator’s claim that they intervened in a specific way, that their innovation set the conditions for learning as defined by underlying educational philosophy, regardless of what happened as a result. 


계층적 분석은 이론화된 대로 개입이 일어났을지에 대한 문제를 예견한다. 이 근본적인 질문에 대한 대답이 긍정적일 경우에만 결과 성취가 개입과 관련이 있다고 볼 수 있다.

Layered analysis foregrounds the question of whether the intervention occurred as theorized, with the understanding that outcomes achievement becomes relevant only if the answer to this fundamental question is positive.


Figure 3.


PBL 튜토리얼 그룹을 직접 관찰한 결과, PBL의 교육 철학이 항상 고려되는 것은 아니며 상호 작용이 일련의 미니-강의 또는 질의응답 훈련과 더 유사할 수 있다는 것을 보여주었다.24 이러한 의도하지 않은 학습 조건의 존재는 완전히 다른 교육 철학이 명목상의 PBL 개입에서 작용하고 있음을 시사한다. 그리고 이로부터 교육 혁신에 대한 흥미로운 세 번째 분석 질문이 던져진다. "어떤 개입이 실제로 일어났는가?"

Direct observation of PBL tutor groups has shown that PBL’s educational philosophy is not always taken up by its participants and that interaction can more closely resemble a sequence of mini-lectures or question-and- response drills.24 The presence of these unintended learning conditions suggests that an entirely different educational philosophy may be at play in a nominally PBL intervention,19 which raises an intriguing, third analytical question to ask of educational innovations: “What intervention actually occurred?”


PBL의 교육 철학이 보존되지 않은 경우, 혁신의 원칙과 기술에 대한 의문이 제기될 수 있다. 특히, 이러한 의문은 실제 실행된 것이 맥락에 insensitive하고, PBL의 사회적, 인지적 학습 조건의 확립을 무심코 뒤엎었을 때 던져질 수 있다. 

If PBL’s educational philosophy has not been preserved, questions may be raised regarding the innovation’s principles and technique—specifically, where implementation may have been insensitive to context and inadvertently subverted the establishment of PBL’s social and cognitive learning conditions. 


그림 3의 중간은 [협력적, 문제 중심 및 학습자 주도의 문제 해결을 보장하는 것과 관련이 있는] '기술'과 '맥락 요인' 사이의 적합도 조사를 위한 예제 질문을 제공한다. 학습자의 팀워크 기술, 개인교사의 촉진 기술, 협업 업무 특성, 기관 평가 문화와 같은 것.

The middle of Figure 3 offers example questions for probing the degree of fit between technique and contextual factors relevant to ensuring collaborative, problem-focused, and learner-led problem solving, such as learners’ teamwork skill, tutors’ facilitation skill, collaborative task characteristics, and institutional assessment culture.28,47,52–55


예를 들어, 기술-맥락 적합성이 성공적으로 실행되었다면, 인구학적으로 다양한 튜토리얼 그룹의 모든 구성원들 사이에서 동등한 참여로 나타날 수 있다. 그 이유는, 

  • 아마도 숙련된 튜터가 심리적으로 안전하고 포괄적인 맥락을 효과적으로 설정하고 유지했기 때문일 수 있다.

  • 또는 문제해결 과제가 모든 그룹 구성원을 포함하는 역할과 책임을 신중하게 설명했기 때문일 수도 있다. 

  • 또는 그룹 멤버들은 그들 자신의 역동성을 감시하고 촉진할 수 있는 팀워크 기술이 잘 발달되어 있었기 때문이었을 수 있다.

As an example, successful technique– context fit may be revealed in equal participation among all members of a demographically diverse tutorial group, 

  • perhaps because a skillful tutor effectively set and maintained a psychologically safe and inclusive context; 

  • or because the problem-solving task had carefully delineated roles and responsibilities inclusive of all group members; 

  • or because group members had well-developed teamwork skills that allowed them to monitor and facilitate their own dynamic.44


결어

Concluding Remarks


증거 기반 교육 관행의 진전은 긍정적인 결과 보고서의 부족이 아니라, 어떤 개입이 시도되었는지, 어떤 상황에서 시도되었는지, 그 결과가 다른 환경에서 교육 문제 해결을 의미하는지에 대한 부정확함에 의해 제한된다.

Advancements in evidence- based educational practice are limited not by a lack of positive outcome reports but, rather, by imprecision regarding what interventions were tried, under what circumstances, and what the results mean for educational problem solving in other settings.


실험 조작 검사experimental manipulation check처럼,19 

  • 혁신의 개념적 기반을 신중하게 고려하면서 

  • 맥락을 충분히 설명하고

  • 개입이 실행될 때 교육철학이 보존되었음을 보여준다면,

의도된 개입이 이론화된 대로 발생했다는 주장을 지지하고 결과를 논리적 결과로 해석하는 데 신뢰성을 부여한다.

Like conducting an experimental manipulation check,19 

  • carefully considering an innovation’s conceptual basis, 

  • accounting for context, and 

  • demonstrating the preservation of educational philosophy in implementation 

...supports the argument that the intended intervention occurred as theorized17,18 and lends credibility to interpreting outcomes as the logical result.


계층 분석Layered analysis은 구현된 후에 개입에 대해 검토하는 방법일 뿐만 아니라, 사전에 개입 설계에 대해 접근하는 방식을 제공한다.

Layered analysis offers an approach not only to examining an intervention after it has been implemented but also for designing interventions proactively.


그것은 주어진 기술혁신이 맥락에서 어떻게 작용한다고 생각하는지에 대한 접근법을 제공함으로써 설계자들이 [개입의 "활성 성분active ingredients"을 개발]하기 위해서 묵시적, 인과적 모델을 명시적이고 목적적으로 만들도록 유도한다. 이와 같이 계층 분석을 사용하는 것은 실험에서 하는 것과 일치하지만, 단순성에 대한 위협이라기 보다는 맥락을 이론 구현하는 데 필요한 하나의 자원으로 취급한다는 것에 차이가 있다.10

It provides an approach to specifying how a given innovation is thought to work in context, prompting designers to make their implicit causal models explicit and purposefully to develop the intervention’s “active ingredients.” In this way, using layered analysis is consistent with the experimental tradition, yet it differs by treating context as a resource for implementing theory rather than as a threat to simplicity.10


중요한 것은, 교육 혁신을 개념화하기 위해 계층화된 분석을 채택하는 것 만으로 원하는 결과의 생산을 보장하지 않을 것이다.

Importantly, adopting layered analysis for conceptualizing educational innovations will not guarantee the production of desired outcomes.


혁신의 층위를 나눠서 보면 실행된 혁신이 얼마나 애초의 교육 철학에서 멀어져 왔는지 알 수 있다.

Viewing innovations from a layered perspective can reveal just how far an implementation has drifted away from the educational philosophy it was assumed to invoke.


어떤 혁신의 기저에 깔린 이론을 분석하는 것은 시간이 많이 걸리고 지적으로 어려운 일이다. 여러 제약조건이 많지만 그 중에서도 긍정적인 결과를 빨리 만들어 실용적인 것에만 관심이 있는 간결하게 전달하라는 압력은 커리큘럼 평가에 적용되는 정밀한 조사를 제한할 가능성이 있다.

Analyzing the theory of an innovation is time-consuming and intellectually challenging. Among many other constraints, pressure to produce positive outcomes quickly and communicate them succinctly to a practically minded audience likely limits the scrutiny applied to curricular evaluation.


더욱이, PBL의 작업 예제가 보여주듯이, 면밀한 검토는 [개입의 철학]과 [개입이 구현된 상황] 사이의 해결되지 않은 긴장이 혁신자의 목표를 전복시켰다고 밝혀낼 수도 있다. 그것은 심지어 교육철학이나 원칙 자체의 과소평가나 불일치를 드러낼 수도 있다.

Moreover, as the worked example of PBL demonstrates, close examination may reveal that unresolved tensions between an intervention’s philosophy and the context in which it was implemented subverted the innovator’s goals. It may even reveal underspecification or inconsistencies of educational philosophy or principles themselves.


layered analysis는 학문적인 행동의 과정이며, 주어진 맥락에서 이뤄지는 학습을 더 깊이 이해하려는 노력이다.

layered analysis, is a scholarly course of action, an endeavor for those seeking to deepen understanding of learning in context.






20 Horsley T, Regehr G. When are two interventions the same? Implications for reporting guidelines in education. Med Educ. 2018;52:141–143. XXX









 2019 Jun;94(6):789-794. doi: 10.1097/ACM.0000000000002591.

Learning Theory and Educational InterventionProducing Meaningful Evidence of ImpactThrough Layered Analysis.

Author information

1
A.T. Cianciolo is associate professor of medical education, Southern Illinois University School of Medicine, Springfield, Illinois; ORCID: https://orcid.org/0000-0001-5948-9304. G. Regehr is professor, Department of Surgery, and associate director, Center for Health Education Scholarship, Faculty of Medicine, University of British Columbia, Vancouver, British Columbia, Canada; ORCID: https://orcid.org/0000-0002-3144-331X.

Abstract

Like evidence-based medicine, evidence-based education seeks to produce sound evidence of impact that can be used to intervene successfully in the future. The function of educational innovations, however, is much less well understood than the physical mechanisms of action of medical treatments. This makes production, interpretation, and use of educational impact evidence difficult. Critiques of medical education experiments highlight a need for such studies to do a better job of deepening understanding of learning in context; conclusions that "it worked" often precede scrutiny of what "it" was. The authors unpack the problem of representing educational innovation in a conceptually meaningful way. The more fundamental questions of "What is the intended intervention?" and "Did that intervention, in fact, occur?" are proposed as an alternative to the ubiquitous evaluative question of "Did it work?" The authors excavate the layers of intervention-techniques at the surface, principle in the middle, and philosophy at the core-and propose layered analysis as a way of examining an innovation's intended function in context. The authors then use problem-based learning to illustrate how layered analysis can promote meaningfulunderstanding of impact through specification of what was tried, under what circumstances, and what happened as a result. Layered analysisshould support innovation design and evaluation by illuminating what principled adaptation of educational technique to local context could look like. It also promotes theory development by enabling more precise description of the learning conditions at work in a given implementation and how they may evolve with broader adoption.

PMID:
 
30640265
 
DOI:
 
10.1097/ACM.0000000000002591


취약성과 신뢰성 사이의 긴장 포용하기: 보건전문직 교육에서의 지적 솔직함(Med Educ, 2018)

Embracing the tension between vulnerability and credibility: ‘intellectual candour’ in health professions education

Elizabeth Molloy1 & Margaret Bearman2





도입

INTRODUCTION


의료는 어려운 작업 환경이다. 의사의 실수는 재앙적인 상황을 초래할 수 있고, 부족함과 실수를 감추려는 싸움은 심각한 스트레스를 야기할 수 있다.12

Health care is a challenging working environment. Practitioner errors can lead to catastrophic situations and, the fight to hide deficits and mistakes may cause serious stress.1,2


사회적 지위를 유지하려고 하다보면, 의료전문가들이 자신의 취약성을 표현하는 것을 어렵게 만들 수 있다. 이처럼 신뢰성을 촉진하고 취약성을 숨겨야만하는 고질적 욕구endemic needs는 학습과 실천의 퀄리티에 장애가 된다.

seeking to maintain social status can make it difficult for health professionals to express vulnerability. This endemic need to promote credibility and hide vulnerabilities can, interfere with learning and quality of practice.


교사들은 건강 전문가들이 [스스로를 취약하게 만드는 것]과 [신뢰도를 유지하는 것] 사이의 균형을 어떻게 협상하는지를 모델링하는 데 역할을 한다.

teachers have a role to play in modelling how health professionals negotiate the balance between making themselves vulnerable and maintaining credibility.


우리는 이 긴장을 협상하고 포용하는 데 도움이 되는 방법으로 '지적 솔직성'(학습자와 교사들의 사고 과정, 딜레마, 실패를 노출하는 것)을 제공한다. 지적 노출의 이 민첩한 형태, 또는 우리가 이전에 '지적 노출streaking'라고 불렀던 이것은 자신의 학습과 타인의 학습을 목적으로 하는 브레인스토밍, 즉흥성, 공개가 특징이다.

we offer ‘intellectual candour’ (or the exposure of learners and teachers’ ‘thought processes, dilemmas or failures’3) as a way to help negotiate, and embrace, this tension. This nimble form of intellectual exposure, or what we have previously termed ‘intellectual streaking’,3 is characterised by brainstorming, improvisation and disclosure for the purpose of one’s own learning and the learning of others.


학생 성찰에 대한 연구는 학생들이 평가의 목적을 달성하기 위해 내러티브를 조작하는 것을 시인했음을 나타낸다.7 이런 이야기는 익숙하다.

studies of student reflections indicate students have admitted to fabricating narratives to achieve the objectives of assessment.7 This is familiar territory.


앞서 나는 여러 가지 조건이 걷잡을 수 없는 상황에서 가벼운(그러나 이해할 수 차례) 실수를 저질렀다. 이제, 나는 나의 가벼운 실수로부터 포괄적으로 배웠고, 이 귀중한 학습 루프는 내가 다시 비슷한 실수를 할 것 같지 않다는 것을 의미한다. 지금은 모든 것이 잘 되고, 성찰적인 연습 기회는 매우 유용했다, 고맙다.

Earlier, I committed a mild (but understandable) mistake, amidst a number of conditions out of my control. Now, I have comprehensively learned from my mild mistake, and this valuable learning loop means I am unlikely to make a similar mistake again. Things are now OK, and the reflective practice opportunity was very useful, thank you.


이 이야기로부터 억양, 어조, 그리고 '영웅적 서사'를 통해 즉각적 보여준다. 이러한 방식의 성찰은 취약성을 숨기고, 적절한 수준의 신뢰성을 유지하게 해준다. 이것은 별로 놀라운 일이 아니다.

These stories are immediately recognisable via their cadence, tone and ‘hero narrative’ . This type of reflection illustrates both a hiding of vulnerability and the maintenance of an appropriate level of credibility.This is hardly surprising.


성찰은 학습자가 자신의 퍼포먼스에 대한 판단을 내림에 있어서 위험을 감수할 것을 요구한다. 이러한 판단을 감독관에게 전달할 때, 감독관은 종종 평가자과 서포터/개발자의 역할을 겸하고 있기 때문에 위험이 더 커진다. 이전의 연구들은 학습자는 자기 노출의 과정에서 겁을 먹게 되고 걱정하게 되며, 종종 경쟁이 극심하고 고부담 산업에서의 자기고양적 태도와는 정반대라는 것을 발견하게 된다.8,9 이러한 위험은 현실일 수 있다: 보건 직업 교육 문화 내에서 오류가능성와 취약성은 바람직한 특성으로 여겨지지 않는다.10

Reflection demands that learners take risks in making judgements about their own performance. There is further risk in communicating these judgements to supervisors, who often adopt a combined role of assessor and supporter/developer. Previous studies indicate that learners find these processes of self-exposure daunting, fraught and, at times, the opposite of self-serving in a competitive and high-stakes industry.8,9 These risks may be real: fallibility and vulnerability are not viewed as desirable characteristics within the health professions education culture.10


학생들은 주로 이런 것을 경험한다.

  • 사전 준비된 슬라이드로 현장 전문가의 강의, 

  • 학습자에게 자신의 실습 적자에 대해 '모두 반성하라'고 요청했음에도 불구하고 스스로 취약성을 거의 보이지 않는 임상 전문가들과의 피드백 대화, 

  • 그리고 환자와의 잘 짜여진 커뮤니케이션 스크립트를 가진 숙련된 임상의의 관찰을 들 수 있다.

Students have: 

  • lectures by experts in the field with pre-prepared slides; 

  • feedback conversations with clinical experts who rarely exhibit vulnerability themselves despite asking learners to ‘reveal all’ about their own practice deficit, and 

  • observations of experienced clinicians with well-crafted and well- rehearsed communication scripts with patients.


학생들은, 자신의 취약점에 대해서 이야기하는 의사는 거의 보기 힘들다. 이야기를 하더라도, 그것은 이미 다 지나간 옛날 이야기일 뿐이다.

we have not observed a great deal of reciprocal vulnerability, with the exception of clinicians talking about deficits in their own practice that are firmly rooted in the past.11,12


왜 선생님들은 취약점을 더 즉각적, 규칙적으로 밝히지 않는가? 그 이유는 자신의 어려움을 노출하는 것이 청중과 잘 맞지 않을 때, 선생님의 신뢰성에 위협이 될 수 있기 때문일지도 모른다. 이러한 지위의 감소는 결과적으로 교육동맹에 있어서 교수와 학생 사이의 신뢰를 손상시킬 수 있다.13

why don’t teachers reveal vulnerability more readily and more regularly? The answer may be that when the exposure of struggle does not sit well with the audience, there may be a compromising of the teacher’s credibility. This reduction in status can, in turn, compromise trust between parties in the educational partnership.13



학습 상황에서 취약성을 드러내는 것의 가치와 위험

THE VALUE AND RISKS OF SHOWING VULNERABILITY IN LEARNING ENCOUNTERS


의료전문직 교육환경에의 학습자들은 그들의 지위와 역할의 성격 그 자체로 취약성을 갖는다. 게다가, 임상 환경은 매우 큰 위험을 가진, 도전적이고 스트레스가 많은 작업장이다. 감독관은 일을 안전하고 잘 처리하기 위하여 동료들(학생 포함)에게 의지한다. 그러므로 모든 의사들은 상당한 취약성을 가지고 있다. 왜냐하면 직장에서 받는 사회적 압력을 피할 수 없고, 무엇보다 자신의 행동이 환자에게 잠재적으로 위험을 야기할 수 있기 때문이다. 이런 어려움에도 불구하고, 배움은 벌어져야 한다.

Learners in a health professional environment have vulnerabilities simply by the nature of their status and role. Moreover, clinical environments are challenging and stressful workplaces with very high stakes. Supervisors, rely on their colleagues (including students) to get the work done safely and well. All practitioners therefore have significant vulnerabilities because of the potential harms of their own actions as well as the inevitable social pressures of the workplace . Against this backdrop, learning needs to occur.


배우기 위해서는 사물을 다르게 상상할 수 있는 가능성에 마음을 열어야 한다. 이것은 나이든 것에 새로운 사실을 추가하는 점진적 학습incremental learning 이상의 것으로, 일레일리스는 이를 '축적적' 학습assimilative learning이라고 부른다.17 이것은 변화로서의 배움이다. 앎 또는 존재함의 다른 방법을 깨닫는 것이고, 자신이 세상을 이해하는 방법에 대한 인신론적 또는 존재론적 변화이다. 변혁적 학습이론에 따르면 '혼란스러운 딜레마'는 배움의 촉매 역할을 하다14

In order to learn, we have to open ourselves up to the possibility of imagining things differently. This is more than the incremental learning of adding new facts to older ones, what Illeris calls ‘assimilative’ learning.17 Rather, this is learning as transformation; different ways of knowing or being; profound epistemic or ontic changes in how we make sense of the world.18,19 Transformative learning theory suggests that ‘disorienting dilemmas’ act as catalysts for learning.14


학생들은 '대단한 수준의 혼란'을 경험하다. 그들이 보고 듣고 경험했던 것을 설명하기에는, 그들이 가진 frame of reference가 불충분하다는 것을 깨닫는다. 20 Berger21에서 언급했듯이, 그러한 변화는 사람들이 전에 알지 못했던 것들을 보게 하고, 선택가능했음을 몰랐던 것을 선택하게 할 수 있다. 그렇지만.....관점의 변화는 무언가의 상실을 동반한다. 즉, 기존에 갖고 있던 관점에 대한 만족을 잃는 것이다.

student experience ‘a significant level of disruption ... where their frame of reference is shown to be inadequate to explain what they have seen, heard or experienced’.20 As Berger21 notes, ‘such a change can lead people to see things they had not noticed before and to have choices they didn’t realise they had. But ...this change of perspective comes with a loss – a loss of satisfaction with earlier perspectives’.


즉, 학습자는 반드시 방향감각 상실 또는 훨씬 더 심하게 말하면 '재앙적 해체'의 가능성에 자신을 노출시켜야 한다.22 따라서 심화된 학습은 자기와 세계에 대한 감각에 큰 지장을 줄 수 있다. 이런 혼란을 겪으려면 취약해질 필요가 있다.

In other words, learners must necessarily expose themselves to the possibility of disorientation or, far worse, ‘catastrophic disorganisation’.22 Hence, profound learning may be highly disruptive to a sense of self and the world. In order to experience such disruptions, we need to be vulnerable.


혼란스러운 딜레마와 관련된 취약성이 반드시 만천하에 공개되어야 하는 것은 아니다. 그러나, 우리는 교사들과 동료들이 종종 성찰을 하는데, 또는 새로운 의미를 만드는 데 핵심적인 역할을 한다고 제안한다.

Disorienting dilemmas and associated vulnerabilities do not need to be made public. However, we suggest that teachers and peers often have a key role to play in helping us to reflect and make new meanings.


게다가, 스스로의 딜레마와 불확실성에 대해 말함으로써, 우리는 스스로를 학습에 프라이밍 시킨다. 다른 사람들과 자기자신에게 "앎에 대한 새로운 방식"에 개방적일 준비가 되었다고 신호를 보내는 것이다. 즉, 이것은 '자신을 취약하게 만드는 것'이기도 하다.

Moreover, by voicing our dilemmas and uncertainties, we prime ourselves for learning. We flag to others and ourselves that we are open to new ways of knowing or, as commonly said, we ‘make ourselves vulnerable’.


'자신을 취약하게 만드는 것'은 학습에 도움이 될 수 있지만, 그것은 우리가 완벽하지 못하다는 것을 보여준다. 특히 의료 종사자들과 학생들이 이미 상당한 취약성을 가지고 있는 임상 작업장에서 우리의 가장 취약한 지점을 드러내는 것은 위험한 짓이다. 어떤 상황에서는, '자신을 취약하게 만드는 것'이 착취, 법적 위협, 괴롭힘으로 이어질 수 있다. 우리의 직업적 지위가 크게 손상될 수도 있으며, 심지어 직업적 기회를 잃을 수도 있다. 즉, '자신을 취약하게 만드는' 사람은 누구나 신용을 잃을 가능성이 생기는 것이다.

‘Making yourself vulnerable’ may be good for learning but it exposes us to be less than perfect. Revealing our underbellies is a risky business, particularly in clinical workplaces where health care practitioners and students already have significant vulnerabilities. In some situations, ‘making yourself vulnerable’ might lead to exploitation, legal threat, bullying . Our professional status may be significantly damaged, andwe may lose career opportunities. In other words, anyone who ‘makes themselves vulnerable’ also opens themselves up to losing credibility.



학생-교사 관계에서 신뢰성의 역할

THE ROLE OF CREDIBILITY IN THE STUDENT– TEACHER RELATIONSHIP


신용은 모호한 못한 개념이다. 어떤 사람이 믿을만 할 때, 우리는 그들을 '믿는다'. 그러나, 어떤 사람의 신용에 대한 인식은, 우리 자신의 내적 감정과 일치하지 않을 수도 있다. 예를 들어, 당신이 의사인데 당신의 환자에게 '나는 당신에게 무슨 문제가 있는지 몰라'라고 말하더라도, 당장은 신용의 상실을 느끼지 못할 수도 있다. 그러나, 당신은 모르겠지만, 그 말을 들은 환자는 '의사라면 환자가 무엇이 잘못되었는지 알아야 한다'고 생각할 수도 있다.

Credibility is a slippery concept. If a person is credible, we ‘believe’ in them. However, our perception of that person’s credibility may not align with their own internal feelings. For example, if you are a doctor and you say to your patient, ‘I’m afraid I don’t know what’s wrong with you’, you may not feel any immediate loss of credibility. However, unbeknownst to you, the patient may think ‘a doctor should know what’s wrong’.


당신이 신뢰를 잃었는지 알 수 있는 유일한 방법은 환자가 자신의 우려를 표명하는 것이다. 아마도 그 환자는 다음 진료 예약을 잡지 않음으로써 그것을 알려줄 것이다. 따라서 신뢰성은 [다른 사람들이 당신에게 투자하는 속성]이고, 신뢰에 대한 당신의 느낌은 다른 사람이 당신에게 어떤 투자를 하는지를 인식하는 것에서 비롯된다. 이것은 영원한 딜레마다: 우리는 항상 다른 사람들에 의해 우리에게 주어지는 속성을 유지하려고 한다.

The only way you will know if you have lost credibility is if the patient expresses his or her concerns, possibly by not returning for a follow-up appointment. So credibility is an attribute that others invest in you, and your feelings of credibility arise from your perceptions of that external investment. This is the eternal dilemma: we are seeking to maintain an attribute that is always given to us by other people.


신뢰성을 유지하려는 것은 인간 존재의 핵심 특징이다. 고프만의 '체면' 개념이 있다: 자기가 갖고 있는 '자신의 이미지로서, 사회적으로 승인된 속성의 관점에서 묘사됨'.16 우리는 여러 체면을 가지고 있으며, 그것들은 사회적 상호작용을 통해 상호적으로 구성되고 보존된다. 즉, '체면'은 자신의 신뢰도에 대한 자기 스스로의 표상이다. 이것은 사회적 상황에서 일관된 행동과 생각의 패턴을 수행enact함으로써 드러난다.

Seeking to maintain credibility is a key feature of human existence. from Goffman’s notion of ‘face’: one’s own ‘image of self, delineated in terms of approved social attributes’.16 We have multiple faces, and they are mutually constructed and preserved through social interactions. In other words, ‘face’ is one’s own representation of one’s own credibility, through enacting a coherent pattern of actions and thoughts in social situations.


자신을 취약하게 만들 때의 긴장감은 분명하다. 그것은 우리 모두는 심지어 풍부한 학습기회를 앞에 두고서도, 가능한 최대로 '체면을 살리기' 위해 노력한다는 것이다.

Already, the tension with making oneself vulnerable is clear: we all try to ‘save face’ whenever possible, even at the expense of a rich learning encounter.


고프만은 모든 상호작용도 의식ritual적인 형식를 가지고 있다고 제안한다.16 예를 들어 환자가 일반의사를 방문할 때 환자와 의사를 특정한 방법으로 배치하는 의식적인 상호작용이 있다. 의사가 환자에게 '어디가 불편해서 오셨나요?'라고 묻는 것은 신뢰성을 유지하고자 하는 강한 의례적인 상호 작용이다. 예를 들어 의사가 환자에게 '하이파이브'로 인사를 한다면, 의식이 깨지면서 의사의 신빙성도 의심받을 수도 있다.

Goffman suggests any interaction has a ritual form, in which individuals mutually construct each other’s positions.16 For example, when a patient visits a general practitioner, there are ritual interactions that position the patient and doctor in certain ways. ‘How can I help you today?’ is a strongly ritualised interaction that seeks to maintain perceptions of credibility. If the ritual is broken, for example with a ‘high-five’ greeting, the credibility of the doctor might be called into question.


마찬가지로, 로테이선 종료시에 오고가는 피드백에서, 교육자들은 종종 '봐라, 너는 올바른 길을 가고 있습니다'와 같은 문구로 학생들에 대한 평가를 시작한다. 이러한 말로 피드백을 수행함으로써 교육자를 퍼포먼스의 진단자의 지위를 갖게 한다: "안심하라, 나는 너를 주의깊게 지켜봤다, 나는 충분한 지식을 가지고 있으며, 내 생각으로는 당신은 대부분의 기준을 충족시키고 있다." 

Likewise, in end-of- rotation feedback encounters in the workplace, educators often begin their appraisal of students with phrases like ‘Look, you are on the right track’.9The opener positions the educator as the diagnostician of performance: rest assured, I’ve watched you carefully, I’m a knowledgeable other, and in my opinion you are meeting most of the standards of work. 


의식은 또한 자신의 수행에 대한 이러한 논의를 통해 [학습자가 해체되지dismantled 않을 것]임을 명시함으로써 학습자의 체면을 보호한다. 학습자도 마찬가지로 그 사람을 전문가로 인정함으로써, 의식ritual적인 반응을 보이는 교육자의 체면을 지켜준다.

The ritual also protects the face of the learner by indicating that he or she is not going to be dismantled through this discussion of his or her performance. The learner likewise constructs the face of the educator with equally ritualised responses by positioning that person as the expert.9 


우리의 사회적 구조와 위치, 지위를 반영하는 이러한 의식화된 상호작용은 신뢰 방정식의 일부에 불과하다. 학습자들은 이토록 정중하게 체면을 보호하기 위한 행동을 하지만, 정작 피드백을 주의 깊게 들은 다음 무시해 버릴 수 있다. 직장 훈련에서 신뢰성 판단은 '누가' 피드백을 전달했는지에 기초한다. 다시 말해서, 학습자가 피드백을 어떻게 자기개발에 사용하는지(또는 사용할지 여부)에는 신뢰성이 중요하다.23

These ritualised interactions, which reflect our social structures, our positions and status, are just part of the credibility equation. Learners can politely enact these face-saving rituals, listen attentively to instructions for improvement and then ignore them. In workplace training, credibility judgements are based on who has delivered the feedback; in other words, credibility is critical in how (and if) the learner integrates the feedback into his or her development.23


요컨대, 신뢰성은 교수-학습에 중요한 요소다. 

  • 교사에게 있어, 자신의 신뢰도를 증진시키는 것은 학습자들이 받아들일 수 있는 교육적 상호작용을 만드는 데 중요하다. 

  • 마찬가지로, 학습자에게 있어서, 선생님과 의미 있는 대화를 하고, 시험을 통과하고, 커리어에 걸쳐 진전을 이루고싶다면, 자신의 신뢰도를 증진시키는 것이 중요하다.

In short, credibility is a significant factor in learning and teaching. For the teacher, promoting his or her own credibility is an important part of making a teaching exchange acceptable to learners. Likewise, for the learner, promoting his or her own credibility is important if he or she wishes to engage in a meaningful dialogue with the teacher, pass examinations and progress through a career.


우리는 사회적 지위에 따라 신뢰도의 구조가 바뀌는 것을 본다. 의대 1학년 학생은 바보 같은 질문을 할 것으로 예상할 수 있다; 새로운 컨설턴트는 전문가로 보이기 위해 더 열심히 노력해야 한다.

We see constructions of credibility shift with social status. A first-year medical student can be expected to ask silly questions; a new consultant has to strive harder to appear expert.


일반적으로 다른 사람들이 당신을 신뢰하게 할 수준의 '자격'을 확립하는 데 까지는 많은 시간과 에너지와 노력이 필요하다. 그렇기에, 교사들은 자신의 신뢰도를 유지하기 위해 학습자에게 답을 '제공'해야 하고, 적합한 과제를 '골라야 한다'고 느낄 수 있다.

It typically takes a lot of time, energy and work to establish credentials to a point where others will invest you with credibility. As a consequence of this, teachers may feel they must ‘provide’ the answers and ‘pick’ the right tasks for learners in order to maintain their own credibility.


또 다른 예를 들어, 임상 교육자들은 종종 학습자에게 피드백을 전달하기 위해 열심히 코멘트 작성하는데, 이 때 학습자-교사 관계의 안정성을 유지하기 위해 칭찬과 비판의 균형을 맞춘다그러나 그들은 [학습자의 관점을 이해하는 데], 그리고 [성과에 대한 관점이 얼마나 align되었는지]를 이해하는데에는 거의 에너지를 소비하지 않는다. 신뢰도를 높이려는 이 세련된 공연은 일종의 독백이며, 이것은 '자신을 취약하게 만드는 것'과는 거리가 멀어 보인다.

To give another example, clinical educators often painstakingly craft comments to deliver feedback to the learner, a balance of praise and criticism to maintain stability in the learner–teacher relationship, and yet they expend little energy in attempting to understand the learner’s perspective and the extent to which viewpoints about the performance align.24,25 This polished performance, or monologue, which seeks to boost credibility, seems a long way from ‘making oneself vulnerable’.


지적 솔직함: 신뢰-취약 난제의 해결책?

INTELLECTUAL CANDOUR: A SOLUTION TO THE CREDIBILITY–VULNERABILITY CONUNDRUM?


학습자들은 자신을 노출시키고 신뢰도가 낮아질 위험을 기꺼이 감수해야 한다. 교사들도 마찬가지이다. 교사는 현실 세계의 문제들과 씨름하는 것의 뉘앙스와 복잡성을 보여주려면, 자신의 취약성을 드러내는 방식을 모델링할 필요가 있지만, 이 역시 교사의 신뢰도를 떨어뜨릴 위험이 있다.

Learners must be willing to take the risk of exposing themselves and lowering their credibility. Teachers likewise need to model these exposures in order to demonstrate the nuances and complexities of grappling with real-world problems, but, similarly, risk lowering their credibility.


우리는 지적 솔직함이 이러한 상반된 긴장cross-currents을 관리하는 데 도움이 되는 유용한 방법이라고 제안하고 있다. 그렇다면 지적 솔직함은 무엇일까? 지적 솔직함이란 진정으로 복잡한 문제나 상황에 대한 사고를 말로 풀어내는 것이다. 그것은 즉흥적인 성격을 가지고 있다. 잘 다듬어지지 않았으며, 창발적인 대화이며, 주저하며 실패의 가능성이 가득하다. 그것은 관대하다기 보다는, 폭로적이고, 짧은 목격이다.

We are proposing that intellectual candour is a useful way to help manage these cross-currents.3 So what is intellectual candour? It is the verbalisation of thinking with respect to a genuinely complex problem or situation. It has an improvisational quality. It is not polished, and the discourse is emergent, hesitant and fraught with the possibility of failure. It is a short glimpse, revelatory rather than indulgent.


이것은 'think aloud'와 비슷하다. 이 때 말하는 사람은 상황에 대한 반응을 설명하거나 정당화하거나, 완벽에 대한 요구 없이, 아이디어를 논의하면서 '브레인스토밍'과 유사하게 말하게 된다. 다만 지적 솔직함과 think-aloud의 차이는 지적 솔직함의 목적은 자신과 타인의 배움이라는 것이다.

This is speaking as thinking, similar to ‘thinking aloud’, where the speaker might explain or justify a response to a situation, or ‘brainstorming’, where ideas are discussed without a demand for perfection. What distinguishes intellectual candour from thinking aloud is that the purpose is learning, both for self and others.



그렇다면 지적 솔직함이 어떻게 배움을 촉진시킬 수 있을까?

So how can intellectual candour promote learning?


다음을 배운다: 

(i) 겸손 

(ii) 성찰 기술, 

(iii) 대화를 통해 아이디어와 생각을 만들어 내는 방법 

to learn: 

(i) humility; 

(ii) reflection-in- action skills, and 

(iii) how to formulate and generate ideas and thoughts through dialogue. 


게다가 교사들이 자신의 마음과 꾸밈없는 진실을 밝힐 때, 그들은 학생들에게 취약해지는 방법을 모델링하는 것이다. 그렇게 함으로써, 그들은 배움은 평생의 과정임을 보여준다.

Moreover, when teachers reveal their minds and unvarnished truths, they are modelling how to be vulnerable. By doing so, they illustrate that learning is lifelong


이런 유형의 'candour'는 취약성과 신뢰도 사이의 긴장을 조화해주지 않는다. 또한 우리가 그것을 원하는 것도 아니다. [취약성을 드러내는 것]과 [신뢰도를 추구하는 것] 사이의 균형점은 항상 변화하고 있으며, 때문에 그 어떤 학습자와 전문가도 그것을 숙달했다고 주장할 수는 없다.

This type of ‘candour’ does not reconcile the tension between vulnerability and credibility and nor do we want it to. The balance point between expressing vulnerability and seeking credibility is ever changing, and, for that reason, learners and experts alike can never claim to have it mastered.


자신감을 키우려고 한다면, 취약한 사고를 드러내는 것이 항상 합리적인 요구는 아닐지도 모른다. 그래서 '선생이 먼저 나서야 한다'고 제안하는 것이다. 사회적 자본이 많은 사람들이 배움에 마음을 열 준비가 되어 있을 때, 그리고 거기에 무언가를 상실할 가능성이 수반될 때, 그것은 겸손과 가능성의 분위기를 만들어낸다. 학생들은 자신들이 선생님과 같은 팀에 있다는 생각을 엿볼 수 있을 것이다.

Revealing faltering thinking, when you are trying to build your own confidence, may not always be a sensible call. This is why we suggest that ‘teachers go first’. When those with high social capital are prepared to open themselves to learning, and the concomitant possibility of loss, it creates an atmosphere of humility and possibility. Students may get a glimpse of the notion that they are on the same team as the teacher.




Box 1 당신이 지적으로 솔직하다는 것을 나타낼 수 있는 문장들

Box 1 Phrases that may indicate you are being intellectually candid


아직 이 일을 제대로 파악하지 못했는데....

잘 모르겠는데...

나도 오락가락 해...

아직 잘 이해가 가지 않지만, 내가 생각하고 있는 것은....

내 생각에 이 문제는....

나도 여전히 고생하고 있는 문제가 있는데....

내가 끝까지 얘기할 동안만 조금 참고 들어주면 좋겠는데...


I haven’t quite got a grip on this yet. . .

I’m not sure. . .

I’m in two minds. . .

I don’t quite understand this yet, but what I’m thinking is. . .

When I look at this problem. . .

What I still struggle with in my own practice is. . ..

Bear with me, while I talk this through. . .


교사가 지적 솔직함을 모델링하는 것의 장점

POTENTIAL BENEFITS OF TEACHERS MODELLING INTELLECTUAL CANDOUR


우리는 특히 교사들이 학생들의 학습을 촉진하기 위해 지적 솔직함을 사용할 수 있다고 제안한다. 여기에 위험이 없는 것이 아니다. 비록 겸손은 지위가 높은 사람을 매력적으로 보이게 하는 속성이지만, '자신을 취약하게 만드는 것'은 신용을 높일 수 있는 동시에 그것을 떨어뜨릴 수도 있다.

We particularly suggest that teachers can use intellectual candour to promote student learning. This is not risk-free. Although humility is a very appealing attribute in someone of high status and ‘making oneself vulnerable’ can also boost credibility as well as diminish it, there is always a potential fall in credibility.


상호 취약성을 통한 신뢰 구축

Building trust through reciprocal vulnerability


연구에 따르면, 배움의 핵심 부분으로서 신뢰가 필요하다. Beitat의 주석에 따르면, 

'개인적 신뢰는 동적이고 자발적인 과정으로, 

인지적 요소와 감정적 요소를 모두 포함한다.

이 요소가 미래 결과에 대한 긍정적인 기대를 형성하기에, 

관련된 위험에 수반될 수 있는 취약성을 수용함으로써 협력을 가능하게 한다.'29 

research suggests the need for trust as a key part of learning. As Beitat notes: ‘Interpersonal trust is a dynamic and voluntary process, involving both cognitive and affective elements that shape a positive expectation about a future outcome and thus enables cooperation by accepting vulnerability posed by associated risks.’29 


우리는 지적 솔직함에는 신뢰가 필요하고, 신뢰는 구축될 수 있다고 제안한다.

We suggest that intellectual candour requires trust and can build trust.


자신의 지위가 높고 전문가인 교사가 교육적 상호작용 내에서 자신의 문제, 불확실성, 내면의 생각을 드러내면 "내가 내 것을 보여주겠다, (만약 괜찮다면) 너도 네 것을 보여줄 수 있다"라는 메시지를 전달한다. 이를 통해 학습자들은 관점을 바꾸고, 자신의 주장을 다른 사람의 주장과 그들 자신의 검증에 내어놓고, 다른 사람들의 생각을 받아들이고, 이전에 고려하지 않았던 개념들 사이의 관계를 발전시킬 가능성이 더 높을 수 있다.

If a teacher, who by his or her position is higher status and more expert, reveals problems, uncertainties and inner thoughts within educational interactions, there is a sense of I’ll show you mine and (if you like) you can show me yours. Learners may therefore be more likely to 

    • shift viewpoints, 

    • subject their own arguments to others’ and their own scrutiny, 

    • take on others’ ideas and 

    • develop links between concepts that they had not previously considered.


다른 말로 하면, '신뢰는, 우리가 다른 사람에게 나 자신의 신뢰를 내어놓을 위험을 무릅쓰지 않고는 결코 자라지 않는다'는 것이다.30

In other words, ‘trust never grows without our taking the risk of placing our trust in others’.30


상호 취약성의 장점은 직장에서 언어적 피드백 대화를 관찰하는 두 가지 연구에 의해 제시된다.9,12 이 두 연구에서 학습자가 자신의 결함을 스스로 평가함으로써 스스로를 취약하게 만든 대화는 두 가지 특색 있는 특성을 보였다.

The merits of reciprocal vulnerability are suggested by two observational studies of verbal feedback conversations in the workplace.9,12 In both these studies, the conversations in which learners allowed themselves to be vulnerable by self-evaluating their own deficits exhibited two distinctive qualities.


    • 첫째, 교육자들은 자신의 성과에 대한 학습자의 평가를 정당하게 받아들였으며, 학습자가 반응할 수 있는 충분한 공간이 제공되었다. 

    • 둘째로, 교육자들은 '신뢰의 분위기'를 조성하는데 도움을 주었다. 이는 물론 관찰된 '피드백 이벤트' 이전에 구축된 것일 수 있지만, 이 분위기의 독특한 특성은 상호적 취약성이었다. 즉, 교육자들은 스스로를 완벽하다고 표현하지 않았고, 실제로 그들 자신의 딜레마와 어려움을 학습자와 공유할 준비가 되어 있었다.

    • Firstly, the educators extended legitimate invitations for learners’ evaluations of their own performances, including providing enough space for responses. 

    • Secondly, the educators helped to build an atmosphere of trust. This may of course have been building prior to the observed ‘feedback event’; however, the distinguishing characteristic of this atmosphere was reciprocal vulnerability. In other words, educators did not present themselves as perfect and were prepared to share their own dilemmas and struggles in practice.



오류를 인정하는 문화 구축

Building a culture that acknowledges fallibility


우리는 교사들의 지적 솔직함이 학습자들이 지적 위험을 감수할 수 있는 분위기를 만들 수 있다고 본다. 사람들은 자신이 충분히 안전하다고 느낄 때, 덜 정제된 자신의 모습을 드러낼 준비를 하며, 여기에 '미완성된' 사고 과정과 행동이 따라온다.

We have suggested that teachers’ intellectual candour can create an atmosphere that allows learners to take intellectual risks. When people feel safe enough, they are also prepared to reveal their less polished selves and accompanying ‘unvarnished’ thought processes and actions.


빌렛31은 '... 실수를 하는 것은 우리가 무언가를 배우는 방식의 핵심이다. 실수는 일을 통해, 그리고 직장 생활 전반에 걸쳐 일어난다'라고 지적한다.

As Billett31 notes, ‘... making errors is central to how we learn, and come to perform in and through work and across working life’.


솔직한 교류를 장려함으로써 학습자들은 완벽주의를 연마하기보다는, 오류를 인정하는 문화로 사회화될 가능성이 높다. 도입부에서 언급했듯이, 의료실무는 오류가 재앙적인 상황으로 이어질 수 있는 고부담high-stake 환경이다. 동시에, 오류는 불가피하게 발생하며 이러한 오류를 숨기거나 과소평가하려는 시도는 심각한 결과를 낳는다.1

By encouraging candid exchanges, learners are more likely to be socialised into a culture that acknowledges fallibility rather than honouring perfectionism. As mentioned in the introduction, health care practice is a high-stakes environment in which errors can lead to catastrophic situations. At the same time, errors are inevitably made and the attempts to hide or downplay these errors lead to serious consequences.1


임상 교육자들이 지적 솔직성을 실천enact한다면, 오류의 정상화normalize를 임상 실습의 일부로서 만들 수 있을 것이다. 이것은 학습자들이 복잡하고 전체적인 실천적 관점을 세우는 것을 도와줌으로써 완벽주의 문화의 위험을 완화시킬 수 있다.

We argue that clinical educators who enact the features of intellectual candour can help normalise fallibility as part of clinical practice. This may mitigate the perils of a culture of perfectionism by helping learners build a complex, holistic view of practice


현장에서의 텐션 포용하기

EMBRACING THE TENSION IN PRACTICE


우리는 지적 솔직함을 이론적 용어로 신뢰성과 취약성 사이의 동적인 균형을 관리하는 것으로 묘사해 왔다. 이 개념들이 어떻게 실천에 옮깁니까?

We have described intellectual candour as managing the dynamic balance between credibility and vulnerability in theoretical terms. How do these notions translate to practice?


지적 솔직함의 특징

Table 1 Features of intellectual candour


즉흥적인 

Improvisational

당신은 바로 '지금 그 순간'에 있는 것이다. 이것은 학습자 또는 교사가 최소한의 사전 설명만으로 반응하고 충동을 완화한다는 것을 의미한다. 즉흥연주에 관한 문헌을 읽는 것이 도움이 될 수 있다. 그렇다면 이것이 어떻게 작동하는지 그리고 어떻게 끝나는지에 대한 더 많은 이해를 쌓을 수 있다.

You are ‘in the moment’. This means the learner or teacher is responsive and working off impulse, with minimal pre-scripting. It may be helpful to read the literature on improvisation34,35 to build further understanding of how this works and to what ends


목적적인 

Purposeful 


또한 학습과 학습의 촉진의도가 동시에 존재하여, 이것이 당신이 하고 있는 일이라는 의식도 있다. 이것은 목적이 없거나 허황된 비방이 아니며, 이러한 focused intention은 연극적 즉흥연주의 원칙이 단지 지금까지 도달했을 뿐이라는 것을 의미한다. '솔직한 노출'의 목적은 명확하고 의식적이어야 하며 효과(즉각 및 후속)에 주의할 필요가 있다. 즉, '그 순간에 투자'하는 동안, 자신의 행동의 목적과 효과에 대해 계속 책임을 지는 것이다. 지적 솔직함은 하고싶은대로 하는 것이나 관심끌기와는 맞지 않는다.

At the same time, there is a conscious awareness that this is what you are doing, with the intention of both learning and promoting learning. This is not purposeless or spurious verbiage and this focused intention means that the principles of theatrical improvisation only reach so far. The purpose of the ‘candid exposure’ needs to be clear and conscious and be watchful for the effects (immediate and downstream). This means that while you are ‘invested in the moment’, you continue to be accountable for the purpose and effect of your actions. Intellectual candour as indulgence or attention seeking does not fit here


대화적

Dialogic 


그것은 대화적인 만남의 일부분이다. 비록 더 많은 청중들에게 지적으로 솔직해지는 것은 가능하지만 (서면으로 반성하거나 컨퍼런스 프리젠테이션의 불확실성을 공개하는 등), 그것은 아마 다른 사람들과 함께 일할 때 가장 효과적이고 즉흥적일 것이다. 이런 형태의 공동 작업을 통해 신뢰는 더욱 쉽게 확립될 수 있다. 이것은 상호작용을 초래할 수 있다. 취약성은 취약성을 유발하여 변형 학습의 가능성을 제공한다.

It is part of a dialogic encounter. Although it is possible to be intellectually candid for a broader audience (such as in written reflections or by revealing uncertainties in a conference presentation), we think it is most effective and most likely to be improvisational when working with others. Through this form of joint work, trust can be more easily established. This may lead to reciprocation. Vulnerability invites vulnerability, which offers the promise of transformative learning


긴장에서 오는 스릴

Thrill of the tension


선을 걷는 '흔들림'을 느끼는 것, 즉, 자신을 취약하게 만들고 신뢰를 잃을 가능성을 통해 자신의 길을 협상할 때 스트레스나 긴장의 느낌이 들 가능성이 있다. 균형점은 맥락, 상호작용의 특성, 이해관계자의 '역할' 또는 외부적으로 할당된 위치에 따라 정기적으로 변화한다. 게다가, 당신은 결코 당신 자신의 신뢰도를 알 수 없다 – 다른 사람들이 생각하고 느끼는 것에 대한 단서만 판단할 수 있다.

Feeling the ‘thrill’ of walking the line; that is, there is likely to be a sensation of stress or tension as you make yourself vulnerable and negotiate your way through the possibility of losing credibility. The balance point shifts regularly depending on context, the nature of the interactions, and the ‘roles’ or externally assigned positions of the stakeholders. Moreover, you can never know your own credibility – you can only judge cues as to what others are thinking and feeling


감정적 차원

Emotional dimensions


감정적 결과에 대한 감정적 준비가 필요하다. 아마도 즐거움이나 두려움 같은 일종의 감정을 느낄 수 있을 것이다. 지적 솔직함이 의도대로 흐르지 않으면, 당신은 부끄러움을 느낄지도 모른다. 때로는 청중도 이 수치심을 물려받을 수 있다.

An emotional preparation for an emotional consequence. You are likely to feel emotion – possibly exhilaration or fear. When your intellectual candour does not hit the mark, you may feel shame. At times, the audience can inherit this shame too



표 2 목적적 지적 솔직함을 촉진하기 위한 교육 전략

Table 2 Educational strategies to promote purposeful intellectual candour


접근 방식 및 예제

Strategic approach and Examples of how these may be enacted in the classroom or health workplace


1 정서적 위험을 낮추는 교수법

1 Pedagogies that lower the emotional risk


일부 교육학자는 학습자와 교사들에게 '상대적으로 안전하고 제한된 환경'에서 생각하는 법을 배울 기회를 제공한다. 한 가지 비유는 줄타기를 낮은 줄 위에서 또는 심지어 바닥에서 하는 것을 배우는 것이다. 동료 학습 활동은 이것의 한 예인데, 동료 학습 상황에서는 학생들이 같은 수준에 있기 때문에 신뢰 관계가 더 낮다. 크리스천과 벨의 연구에서 알 수 있듯이, 이것은 불안감을 줄일 수 있고 학습자들은 '불확실성의 영역'을 공개할 준비가 될 것이다.36

Some pedagogies afford learners and teachers a chance to learn to think in a ‘relatively safe’ and confined environment; an immediate analogy is learning to walk the tightrope on a low wire or even on the floor. Peer-learning activities are one example of this; in peer-learning situations, the credibility stakes are lower as students are at the same level. As Christiansen and Bell’s study indicates, this may reduce anxiety and learners will be prepared to ‘disclose areas of uncertainty’.36


더 높은 수준에서 보자면, 피드백 영역의 또 다른 전략은, (과제에 대한 '학습자의 성과'를 강조하기보다) 배울 때 작업work 표준을 강조하는 대화를 하는 것이다. 이 전략은 존슨과 몰로이의 피드백 대화 검토에서 분명했다. 표준을 가지고 보정calibration하게 되면 학습자의 정서적 위험이 낮아진다 (정보가 학습자의 '자신'을 위협할 가능성이 적음). 그리고 또한 교사들에게도 '암묵적 기준'에 존재할 수 있는 불확실성들을 드러내어 줄여줄 기회를 제공한다.

Another strategy from the feedback domain, with a higher level of challenge, is to emphasise standards of work in learning conversations, rather than cutting straight to the learner’s performance on the task. This strategy was evident in Johnson and Molloy’s examination of feedback conversations.12 Calibration against a standard takes away from the learner’s emotional risk (information is less likely to threaten the learner’s ‘self’) and also affords teachers an opportunity to reveal some of the uncertainties that may exist around tacit standards of practice. 


시뮬레이션 기반 학습은 직무의 기술적 요구를 유지하면서 감정적 이해관계를 낮출 수 있는 교육 접근법의 또 다른 예다.

Simulation-based learning is another example of a teaching approach in which emotional stakes can be lowered (although research shows us not as much as we think), while maintaining technical demands of the task


2 공동작업을 제공하는 활동

2 Activities that offer joint work


'참여'에 대한 비유는 학습에 대한 유용한 비유다. 벵거38은 '실천 공동체'의 핵심 부분으로서 '공동 작업'을 강조한다. 우리는 공동 작업이 '생산적 취약성'을 촉진하기 위한 매우 유용한 전략이라고 제안한다. 양측이 모두 조우에 대한 통제력을 완전히 상실해야만 서로 반응하고 대화하며 서로의 일에 기반을 둘 수 있다. 예를 들어, 두 명의 방사선사가 실시간으로 협력하여 나머지 부서에 대한 프레젠테이션을 준비할 때, 그들은 공유된 일련의 학습 성과에 의해 인도되지만, 동료들을 위한 이러한 학습 성과를 달성하기 위해 자료나 활동을 생성하는 방법에 대한 개념은 매우 다를 수 있다. 

Learning as ‘participation’ is a well-understood metaphor in education37 and Wenger38 emphasises ‘joint work’ as a key part of ‘communities of practice’. We suggest joint work is a very useful strategy to promote ‘productive vulnerability’. Both parties can only be responsive, dialogic and build on each other’s work if they authentically lose some control over the encounter. For example, when two radiologists work together in real time to prepare a presentation for the rest of the department, they are guided by a shared set of learning outcomes, but may have very different notions of how to generate material or activities to achieve these learning outcomes for their colleagues. 


공동구성co-construction은 본질적으로 사람들이 무언가를 얻기 위해 무언가를 잃도록 요구한다. 영역을 포기하고, 영역을 넓히거나, 이전에 구상되지 않은 새로운 영역을 건설하는 과정에서, 양 당사자는 즉흥적인 시간 속에 있으며, 실제real(혹은 최소한 제약된) 시간을 보내고 있다. 이는 [학습 성과가 아니라] 환자 치료가 핵심이 되는 직장 학습에서는 달성하기가 더 어렵다. 이러한 공동구성을 위한 조건을 만드는 감독자나 교사들은 종종 'arrived' 정체성(경험자, 폐쇄적)이 아닌 'becoming' 정체성(겸손함, 개방성)의 구현을 통해 이를 성취한다.39 

Co-construction by its nature requires that people lose something in order to gain something. Giving up turf, gaining turf or building brand new turf not previously conceived, both parties are improvising and in real (or at least constrained) time. This is more challenging to achieve in workplace learning in which the primary currency is patient care, not learning outcomes. Supervisors or teachers who create these conditions for co-construction often achieve this through embodiment of a ‘becoming’ identity (humility, openness), rather than an ‘arrived’ identity (expert, closed).39 


평가 또는 피드백 대화에서 학습자와 교사는 최근에 발생한 임상적 조우에 대한 shared reconstruction을 만들고 개선을 위해 초점을 맞출 영역에 대한 각자의 의견을 제공하는 경우가 많다. 이러한 대화가 잘 진행되면, 양 당사자는 사건의 흐름, 개선이 필요한 사항과 이유에 대해 '진단'을 내리고, 새로운 지식을 실천에 옮기기 위한 계획의 공동개발을 공동으로 구성한다.39 경청, 협상, 타협은 모두 이 공동작업의 핵심 원칙이다.

In assessment or feedback conversations, learners and teachers often create a shared reconstruction of the clinical encounter that just occurred and provide their respective opinions about areas to focus on for improvement. When these conversations go well, both parties co-construct the flow of events, the ‘diagnosis’ of what needs improvement and why, and the shared development of a plan to incorporate the new knowledge into action.39 Listening, negotiation and compromise are all central tenets of this practice of joint work


3 현상유지를 방해하는 의례들

3 Rituals that disrupt the status quo


고프만이 말한 것처럼, 만약 대부분의 의식들이 상태 보존에 관한 것이라면, 16 우리가 현 상태를 순간적으로 뒤집는 의식들을 세운다면 어떤 일이 일어날지 생각해 볼 가치가 있다. 스노든은 지식경영에 있어서 '집착된 사고'를 교란시키는 의식의 역할을 제안한다.40,41 

If most rituals are about preserving status, as proposed by Goffman,16 it is worth considering what happens if we set up rituals that momentarily invert the status quo. Snowden proposes the role of ritual in disrupting ‘entrained thinking’ in knowledge management.40,41 


예를 들어, 그는 코끼리 귀와 코끼리 코를 가진 우스꽝스러운 모자의 사용. 추측을 해서는 안 된다는 팀의 합의에 따라, 추측을 하다 붙잡힌 첫 번째 사람은 다른 사람이 비슷한 실수를 저지르기 전까지는 모자를 써야 했다. 현명한 사전 계획은 그 그룹의 가장 선임된 구성원이 첫 번째 가정을 했다는 것을 의미했다. 유머는 긴장과 비판을 분산시켰기 때문에 매우 중요했다.41. 

For example, he describes a ritualised disruption ‘involving the use of a comical hat with elephant ears and an elephant trunk. . . Following agreement by the team that assumptions must not be made, the first person caught making an assumption had to wear the hat until someone else was caught in a similar mistake. Judicious advance planning meant that the most senior member of the group made the first assumption. . . Humour was critical as it diffused tension and criticism’.41 


놀이playfulness는 표준 교습 의식을 통해 보다 유동적인 즉흥적인 공간으로 탈바꿈할 수 있는 기회를 제공한다. 마찬가지로 여러 '환자'를 시간 경과에 따른 변경 사항과 함께 시뮬레이션된 '병력 청취' 역할극으로 바꾸면 의식 붕괴가 발생한다. 물론 이런 게임과 같은 조건 하에서 학습이 계속될 수 있지만, 아무도 '일반적'으로 행동할 것을 기대할 수 없기 때문에, 이러한 의식ritual은 지위status의 개념을 제거한다

Playfulness affords the opportunity to break through the standard teaching rituals into a more fluid improvisational space. Similarly, swapping multiple ‘patients’ into simulated ‘history taking’ role-play with set timed changes42 achieves a ritual disruption. The ritual removes notions of status, as no-one could possibly be expected to act ‘normally’, although, of course, learning can continue under these game-like conditions. 


교실이나 회의장에서의 의식에 대한 또 다른 흥미로운 방해는 부드럽고 큐브 모양이고 던질 수 있는 마이크를 사용하는 것이다. 수업 참가자들이 발표자나 군중에게 질문을 전달하기 위해 스탠딩 마이크까지 걸어가는 대신, 그들은 빠른 속도로 휴대용 마이크에 던져진다. 마이크 핑퐁은 동료들간의 비공식적이고 자발적인 교류를 촉진한다. 이러한 파괴적인 의식은 설정하는데 시간이 걸리고, 위험을 수반하는 경험적 학습의 만남과 마찬가지로, 그룹 보고를 따를 수 있다.

Another exciting disruption to rituals in classroom or in conference settings is the use of soft, cube-shaped and throwable microphones. Instead of class participants walking up to a standing microphone to deliver a question to the presenter or crowd, they are thrown, at speed, the portable microphone. Microphone ping-pong promotes informal and spontaneous exchanges between colleagues. These disruptive rituals take time to set up and, like any experiential learning encounter that carries risk, may be followed by a group debriefing


학습자와 교사가 취약점을 동시에 해결하는 비생산적인 방법 중 하나는 '가짜 취약성'이다. 이것은 (학습자들이 자서전과 소설을 교환할 정도로 '게임'을 하는) 직장 내 중대 사건에 대한 성찰적 글에서 볼 수 있다.7 다른 예로, 우리는 또한 이미 오래 전에 다 지나간 과실에 대해 고백을 하는 경험많은 교사나 전문 임상의를 본다.

One unproductive way that learners and teachers alike resolve the vulnerability–credibility tension is by ‘faking vulnerability’. We see this in reflective writing about critical incidents in the workplace, in which learners ‘play the game’ to the extent that they exchange fiction for autobiography.7 We also see examples of experienced teachers or expert clinicians making a oft-used confession of a fault that happened a long time ago.


가짜 솔직함의 다른 특징은...

  • 실수를 결국 해피엔딩으로 공유하는 것일지도 모른다. 

  • 또는 문제에 거의 관여하지 않았던 실수를 공유하는 것이다(예: 다른 사람의 불충분함을 선언).

  • 또는 주제에서 벗어난 어려움이나 부족함을 공개하는 것이다(프로페셔널한 자아를 뛰어난 것으로 제시하면서, 주말에 하는 목공 일은 잘 못한다고 이야기한다).

Other characteristics of faux candour may be sharing mistakes with a happy ending, sharing mistakes in which you had little involvement in the problem (e.g. declaring others’ inadequacies), or disclosing struggles or deficits that are off topic (being prepared to discuss your below-average carpentry job at the weekend while presenting your professional self as outstanding).


가짜 취약성은 모든 사람의 신용을 그대로 유지하게 한다. 그러나 정당한 위험 감수와는 같은 분위기를 만들어내지 못하며, 많은 경우에 신뢰와 배움이 가능하지 않다. "지적 포토샵"으로 눈속임을 할 수도 있지만, 신뢰성의 상실할 위험을 감수하는 '자신을 취약하게 만드는 것'과는 다르다.

Presenting a faux sense of vulnerability allows everyone’s credibility to remain intact. However, it does not create the same atmosphere as legitimate risk taking, and we suggest that, in many cases, trust and learning are not enabled. Intellectual photoshop may look the part, but it is not the same as ‘making oneself vulnerable’ with its real attendant risks of loss of credibility.


지적 솔직함의 한계

LIMITATIONS OF INTELLECTUAL CANDOUR


우리는 취약성과 신뢰성의 줄타기를 관리하는 것이 실제로는 매우 어려운 일임을 인정한다. 상사의 눈에 신뢰를 얻으려고 애쓰는 형편없는 학습자는 상층부에 앉아 있고 상사와 대화하는 동안 더 많은 지적 위험을 감당할 수 있는 '신용'을 가지고 있는 동료와 비교하여 그의 지적 솔직함에 더 절제되는 것을 선택할 수 있다.

we acknowledge that managing the vulnerability and credibility tightrope is challenging to do in practice. A poorly performing learner who is striving for credibility in the eyes of his supervisor may rightfully choose to be more restrained in his intellectual candour, compared with a peer who is sitting at the top of the class and has ‘credit’ to afford more intellectual risks during conversations with supervisors.


지적 솔직함의 부정적인 영향

The negative impacts of intellectual candour


교육자들에게 있어, 학습자들이 모호함과 단점에 대해 알 필요가 없는 경우가 있는데, 이런 것들이 물을 흐리게 하고 곧 같은 직장에서 동료가 될 사람들의 눈에 잠재적으로 교사의 신용을 떨어뜨릴 수 있기 때문이다. 더구나 때로는 (실제로는 아무것도 없었음에도 불구하고) 신뢰의 상실감이 압도적일 수 있다.

For educators, there are times when learners do not need to know about ambiguities and shortcomings as these may muddy the waters and potentially discredit the teacher in the eyes of those who will shortly become colleagues in the same workplace. Moreover, sometimes the feelings of loss of credibility (even though, in reality, there has been none) can be overwhelming.


전문성(교사, 학생, 임상의)의 핵심 측면 중 하나는 의사소통적 만남에서 적절한 경계를 이해하고 제정하는 것이다. 다른 사람들에게 '모든 것을 감수해라'는 것은, 노출에 대한 명확한 목적이 없고, 지적 솔직함의 효과를 추적할 수 있는 성찰적 또는 평가적 능력이 없다면, 이는 그저 전문적 행동의 위반breach이 될 가능성이 높다.

One of the key facets of professionalism (of teacher, of student, of clinician) is an understanding and enactment of appropriate boundaries in communicative encounters. ‘Bearing all’ to others, without a clear purpose for the exposure and without the reflective or evaluative capacities to trace the effects of intellectual candour, would likely constitute a breach of professional conduct.


CONCLUSIONS






 2019 Jan;53(1):32-41. doi: 10.1111/medu.13649. Epub 2018 Sep 7.

Embracing the tension between vulnerability and credibility: 'intellectual candour' in healthprofessions education.

Author information

1
Department of Medical Education, University of Melbourne, Melbourne, Victoria, Australia.
2
Centre for Research in Assessment and Digital Learning, Deakin University, Geelong, Victoria, Australia.

Abstract

The tension between expressing vulnerability and seeking credibility creates challenges for learning and teaching. This is particularly true in health care, in which practitioners are regarded as highly credible and making errors can often lead to dire consequences and blame. From a transformative learning perspective, expressing vulnerability may help individuals to access different ways of knowing. By contrast, from a sociological perspective, seeking to maintain credibility results in ritualised interactions and these ritualised encounters can reinforce credibility. One means of embracing this tension between expressing vulnerability and appearing credible is 'intellectual candour', an improvisational expression of doubts, thoughts and problems with the dual purpose of learning and promoting others' learning. Educators' revelations of inner struggles are proposed as a means of inviting reciprocal vulnerability. This builds trust and a platform for learning, particularly of the transformative nature. It also allows modelling of how to balance the vulnerability-credibility tension, which may provide a template for professional practice.

PMID:
 
30192024
 
DOI:
 
10.1111/medu.13649


철학이 문화를 만나는 곳: 코치는 스스로의 역할을 어떻게 개념화하는가(Med Educ, 2019)

Where philosophy meets culture: exploring how coaches conceptualise their roles

Christopher J Watling1 & Kori A LaDonna2



도입

INTRODUCTION


의학교육에서 급증하는 코칭의 인기는 부분적으로 직관적으로 이해될 수 있는 정도에서 근거가 될 수 있다. 스포츠, 음악, 경영 코칭의 세계와 비교한 결과, 의학 교육자들은 코칭이 학습을 효과적으로 촉진할 수 있다는 생각을 받아들이기 시작했다. 코칭은 역량 바탕 의학 교육CBME 접근방식에 필요한 촉진자로 권장되어 왔다.

Coaching’s surging popularity in medical education may be grounded, in part, in the extent to which it just seems to make intuitive sense. Spurred on by comparisons to the worlds of sports, music and executive coaching,1–3 medical educators have begun embracing the idea that coaching may effectively foster learning. Coaching has been touted as a necessary facilitator of competency-based medical education approaches4,5


만약 우리가 비판 없이 코칭을 받아들인다면, 우리는 그것이 우리가 희망하는 만큼 효과적으로 우리의 교육적 필요를 충족시키지 못할지도 모른다.

If we embrace coaching uncritically, we may find that it fails to serve our distinct educational needs as effectively as we may hope.


증가하는 문헌은 코칭 접근법이 의학교육의 특정 영역에서 가치가 있을 수 있다는 것을 암시한다.

A growing literature suggests that coaching approaches may be valuable in certain domains of medical education.


Lovell의 최근 의학 교육 코칭에 대한 리뷰는 코칭이 기술 습득을 향상시키고, 수술 오류를 줄이고, 시험 점수를 향상시키며, 고군분투하는 학생들을 식별한다는 것을 보여주는 좋은 품질 데이터를 보여주었다.2 그러나 발표된 연구결과가 코칭에서 "무엇을 하는지does"를 탐구했지만, 의학 교육에서 코칭이 "무엇인지is"를 밝히는 경향이 없었다. Lovell은, 사실, 의학 교육에서의 코칭에 대한 검토가 정의의 혼란 때문에 특히 어려웠다고 언급했다; 코칭이라는 용어는 종종 티칭이나 멘토링과 교환하여 사용되었다.2

Lovell’s recent review of coaching in medical education revealed good quality data showing that coaching improves technical skills acquisition, reduces surgical error, improves examination scores and identifies struggling students.2 But as published work explores what coaching does, it has not tended to illuminate what coaching is in medical education. Lovell, in fact, noted that a review of coaching in medical education was particularly challenging because of definitional fuzziness; the term coaching was often used interchangeably with teaching and mentoring.2


피드백과 코칭의 불분명한 관계는 문제를 더 복잡하게 만든다.

Coaching’s unclear relationship with feedback complicates matters further.


피드백 관행practice의 상태에 대한 의학 분야의 만성적인 좌절이 코칭의 새로운 인기에 기여했을 수 있다. 그러나 실제로 일부 현대적 피드백 모델은 피드백 과제를 극복하고 학습자에게 더 유용하게 만들기 위해 관계, 솔직한 대화 및 학습자 안전을 분명히 강조한다. 여전히 불명확한 것은 코칭이 피드백에 정확히 무엇을 더해줄 수 있으며, 피드백과는 어떻게 다른지, 과연 다르긴 한지 여부이다.

Medicine’s chronic frustration with the state of its feedback practices may contribute to coaching’s new-found popularity. But in fact, some contemporary models of feedback explicitly emphasise relationship, honest conversation and learner safety in efforts to overcome some of the feedback challenges and render it more useful to learners.15,16 What remains unclear is exactly what coaching approaches are expected to add, and how – or whether – coaching differs from feedback performed with care.


다른 분야에서 수입된 코칭 아이디어가 (의학교육 분야의 코칭에 영감을 줄 수 있지만) 강력한 이론적 토대를 제공하지는 못할 수 있다. 스포츠 코칭 문헌은 광범위하지만, 접근방식을 지지하는 경험적 근거는 제한적으로만 제공한다는 점에서 비판을 받았다.17,18

Ideas of coaching imported from other domains may offer inspiration for medicine’s use of coaching, but may not provide a strong theoretical foundation. Although the sports coaching literature is extensive, it has been critiqued for offering a limited empirical basis for the approaches it endorses.17,18


이러한 한계에도 불구하고, 몇 가지 중요한 코칭 원칙이 나타났다. 코칭은 운동선수든 임원이든 학습, 성과 향상, 개인적 성장에 초점을 맞추고 있다.20 그러나 연구자들은 코칭이란, 그것이 벌어지는 문화와 환경을 고려하지 않고는 충분히 이해할 수 없는 사회적 관행으로 보고 있다.

Despite these limitations, a few overarching principles of coaching have emerged. Coaching, whether for athletes or executives, focuses on learning, performance improvement and personal growth.20 Increasingly, however, researchers are viewing coaching as a social practice that cannot be understood adequately without reference to the culture and environment in which it is situated.20,21


현재의 의학교육 방식에 코칭을 접목시키는데에는 다음의 어려움이 있다. 예를 들어, 의료계에는 역량을 발휘해야 한다는 압박이 있고, 자율성을 가치있게 여기는 것 때문에 코칭 전략을 채택하는 데 있어 문화적 장벽이 존재한다.22

the challenges of grafting coaching onto current medical education approaches. Mutabdzic identified cultural barriers to adopting coaching strategies in medicine, including the pressure for doctors to portray competency and the value doctors place on autonomy.22


또한 직접관찰은 학습자들에 의해 가치 있는 것으로 받아들여지지만, 직접관찰을 사용한다는 것은 의학의 두 가지 핵심 문화적 가치, 즉 효율성과 자율성에 반하는 것이다.그 결과, 교사들과 학습자들 모두 직접적인 관찰에 대해 양가감정을 가질 수 있다.

Direct observation is embraced as valuable by learners, but its use challenges two core cultural values in medicine: efficiency and autonomy.23 As a result, teachers and learners alike may harbour ambivalence about direct observation


피드백이 학습자들에 의해 유용하게 여겨지기 위해서는 신뢰할 수 있고 건설적일 필요가 있지만, "신뢰할 수 있고 건설적인 것"에 대한 정의조차 문화적으로 영향을 받는다; 음악에 대한 신뢰할 수 있는 피드백과 의학에서의 신뢰할 수 있는 피드백은 동일하지 않을 수 있다.27

although feedback needs to be credible and constructive to be viewed as useful by learners, the very definitions of credible and constructive are culturally influenced; credible feedback in music and credible feedback in medicine may not be the same.27


의학교육은 다른 영역(예: 음악과 스포츠)에서 코칭의 개념을 수입하고 있기 때문에, 우리는 코칭을 특징짓는 철학과 관행에 대한 우리의 문화적 수용성을 고려할 필요가 있다. 교육적 관행이 전개되는 방식은 문화와 맥락에 의해 강하게 영향을 받는다.

Because medical education is importing notions of coaching from other domains (e.g. music and sports) we need to consider our cultural receptivity to the philosophies and practices that characterise coaching. The way that pedagogical practices unfold is strongly influenced by culture and context.


슐먼이 지적했듯이, 전문직마다 '시그니처 교수법'을 진화시켜간다.28 시그니쳐 교수법은 때때로 교육 및 학습에 대한 특이한 접근은 문화에 깊이 뿌리박고 있기 때문에 변화에 저항하는 경향이 있다.28

As Shulman has pointed out, ‘signature pedagogies’ have evolved within professions.28 These characteristic and sometimes singular approaches to teaching and learning tend to resist change because they are deeply rooted in culture.28



방법

METHODS


코칭은 의학 교육에서 불투명하게 정의되고 이론화되지 않았기 때문에, 우리는 잘 이해되지 않는 개념이나 과정에 대한 강력한 이론적 통찰력을 생산하는데 유용한 정성적 연구 방법론인 구성주의적 근거이론을 사용했다. 이로서 의학에서 코칭이 의미하는 것과 훈련에서 코칭이 습득하는 데 영향을 미칠 수 있는 문화적 영향을 모두 더 잘 이해할 수 있을 것이다.

Because coaching is opaquely defined and under- theorised in medical education, we used constructivist grounded theory,29a qualitative research methodology useful for producing robust theoretical insights about poorly understood concepts or processes, to better understand both what coaching means in medicine and the cultural influences that may impact its uptake in training.


총 24명이 동의했고(우리 기관 21명, 외부 3명), 표본에는 

  • 의료 바깥에서 코치로 확인된 임상의사 교사 10명, 

  • 코칭에 관심이 있지만 의료 이외의 직접적인 코칭 경험이 없는 임상의사 교사 8명, 

  • 대학 스포츠 코치 6명 등이 포함됐다.

A total of 24 individuals consented (21 from our institution and three from outside); the sample included 10 clinician teachers who identified as coaches outside of medicine, eight clinician teachers with an interest in coaching but no direct coaching experience outside of medicine, and six university sports coaches.


코칭 역할에 내재된 개념적 모호성을 지우기 보다는 탐구하는 것을 목표로 했기 때문에, 연구참여자 중 의사들이 의료 '코치'로서 명시적으로 명명된 역할을 해야한다고 요구하지는 않았다.

As we aimed to explore rather than erase the conceptual ambiguity inherent in the coaching role, we allowed clinician participants to self-identify as coaches; clinicians did not require explicit named roles as medical ‘coaches’ to participate.


자료 수집과 분석

Data collection and analysis


데이터 수집과 분석은 초기, 집중 및 이론적 탐사의 세 단계에서 반복적으로 일어났다. 우리는 참가자들과 30~75분 동안 반구조적 인터뷰를 진행했다. 모든 인터뷰는 녹음되었고 구어체로 필사되었다.

Data collection and analysis occurred iteratively during three stages of initial, focused and theoretical exploration. We conducted semi-structured interviews lasting 30–75 minutes with participants; all interviews were audiorecorded and transcribed verbatim.


근거이론 연구에서 관례처럼, 인터뷰에 대한 접근은 연구가 진행됨에 따라 진화했고, 동시 데이터 분석에 대응했다.29

As is customary in grounded theory research, our approach to interviews evolved as the study progressed, responsive to our concurrent data analysis.29


CJW와 KALD는 참가자의 설명과 경험을 바탕으로 최초 코드를 개발하기 위해 처음 두 개의 녹취록을 독자적으로 읽는다. 이 초기 코드들은 그 후 집중 코드로 통합되었는데, KALD와 한 연구원이 녹취록의 일부를 코드화하는 데 사용했다. 분석이 진행됨에 따라 CJW와 KALD는 해석에 관한 메모의 초안을 작성하고 정기적으로 만나 분석 내용을 서술적 자료에서 이론적 자료로 옮겼다. 즉, 코칭의 정의, 코칭 철학, 의료에서의 코칭에 대한 장벽과 촉진자, 의료와 다른 영역의 코칭 사이의 유사성과 차이와 같은 예비 주제들 사이의 패턴을 확인하기 위해 지속적인 비교 분석을 사용하였다. 또한 이러한 주제를 바탕으로 이론적 범주나 '스토리라인'을 개발하기 위해서, 그리고 우리 탐구inquiry의 엄격함과 신뢰도를 확보하기 위해 반사성reflexivity을 발휘했다.31

CJW and KALD independently read the first two transcripts to develop initial codes based on participants’ descriptions and experiences. These initial codes were then consolidated into focused codes, which KALD and a research associate used to code a subset of transcripts. As analysis progressed, CJW and KALD drafted memos about their interpretations and met regularly to move their analysis from the descriptive to the theoretical. That is, we used constant comparative analysis to identify patterns amongst preliminary themes such as definitions of coaching, coaching philosophies, barriers and facilitators to coaching in medicine, and similarities and differences between coaching in medicine and other domains. We also engaged in reflexivity,30 not only to develop theoretical categories or ‘storylines’ based on these themes, but also to ensure the rigour and trustworthiness of our inquiry.31


구성주의자로서, 우리는 우리의 경험과 관심사가 우리가 묻는 질문들뿐만 아니라 참여자들의 경험을 어떻게 해석하는지도 형성한다는 것을 인정한다.

As constructivists, we recognise that our experiences and interests shape not only the questions we ask, but also how we interpret participants’ experiences.


우리는 우리의 자료가 코칭에 대한 우리의 가정을 강화하였는지 혹은 뒤집었는지를 고려했다. 또한 진화해가는 관점에 대해 서로 정기적으로 토론하하였다. 예를 들어, 우리는 스포츠 코치들이 그들의 스포츠 성공보다 그들의 운동선수가 사람으로서 발전하는 것을 더 강하게 강조하는 것에 놀랐다. 이러한 데이터가 스포츠 코칭에 대한 우리의 가정에 어떻게 도전했는지를 반영하여 우리는 그러한 총체적이고 학습자 중심적인 접근방식이 환자 중심의 임상 환경에 어떻게 맞을지 궁금하게 만들었다.

We considered whether our data reinforced or upended our assumptions about coaching and engaged in regular discussions with each other about our evolving perspectives. For example, we were surprised by sports coaches’ stronger emphasis on the development of their athletes as people than on their athletic success. Reflecting on how these data challenged our assumptions about sports coaching led us to wonder how such a holistic, learner-centred approach would fit in a patient-centred clinical environment.


결과

RESULTS


우리는 컨텍스트와 상관없이 참가자들의 코칭 접근 방식을 좌우하는 것으로 보이는 여러 핵심 요소들로 구성된 공유된 코칭 철학을 확인했다. 그러나 실제 상황에서 이러한 철학의 집행enactment은 전문직 문화에 영향을 받는 듯 보인다. 전문직 문화는 다음에 영향을 준다.

  • 어떻게 코칭이 정의되고 개발되며, 

  • 코칭 역할이 학습 맥락 안에서 어떻게 배치되며, 

  • 얼마나 편안하게 취약성을 표현할 수 있는지

We identified a shared philosophy of coaching, comprising a number of core elements that seemed to anchor participants’ approaches to coaching, regardless of context. But the enactment of this philosophy appeared to be influenced by professional culture, which affected 

  • how coaching was defined and developed, 

  • how the coaching role was positioned within the learning context, and 

  • how comfortably vulnerability could be expressed.


철학적 공통점: 성장과 발전을 향한 상호 지향

Shared philosophy: mutual orientation towards growth and development


스포츠 코칭이든 의료 코칭이든 둘 다든, 참가자들은 코칭의 의도된 목적을 인간 잠재력의 언락킹로 묘사하는 경향이 있었다. 한 참가자가 언급했듯이, '나에게는, 그 사람이 자신의 개인적인 목표에 도달하고, 그들이 할 수 있는 최고의 사람이 되는 것이 전부다' (P5, 의사 및 음악 코치) 참가자는 자신의 지식과 기술보다는 [학습자가 최고의 실력을 발휘하게 이끌어내는 능력]을 통해 훌륭한 코치를 구별했다.

Whether experienced in sports coaching or medical coaching or both, participants tended to describe the intended purpose of coaching as the unlocking of human potential. As one participant noted, ‘For me, it’s all about that person reaching their own personal goals, and being the best they can be’ (P5, physician and music coach). Participants distinguished good coaches, in fact, less by their own knowledge and skill, and more by their ability to bring out the best in their learners:


코칭 역할은 가르치고 지도할 뿐만 아니라 동기부여영감부여inspiring도 수반하는 것으로 인식되었다. 예를 들어, 한 참가자는 코칭이 학습자들에게 '당신이 원하는 것을 넘어서서 자신의 한계를 밀고 나아가서, 근본적으로 더 많은 것을 하도록 장려한다고 언급했다(P1, 의사). 한 참가자가 언급했듯이, '동기와 흥분'을 더하여, 학습자에게 자신감을 불어넣고 자기신념을 갖게하능력도 학습자에게 가장 좋은 결과를 가져다 주었다. "그들은 사람들로 하여금 자신을 믿게 만들며 그것은 엄청나다". 코칭으로 간주되는 근본적으로 학습자 중심의 접근방식은 이타적인 용어로 설명하는 경향에 반영되었다. 한 참가자는 코칭에 내재된 '겸손함'을 확인하면서 '내가 봤거나 관여해 본 최고의 코치는 더 사심이 없었다'(P22, 의사 및 스포츠 코치)

The coaching role was perceived to involve not only instructing and guiding, but also motivating and inspiring. One participant noted, for example, that coaching encourages learners ‘to go beyond what you want and push your own limits – and do more, basically’ (P1, physician). Part of drawing out the best in learners was the ability to inspire confidence and self-belief; as one participant noted, coaches ‘add motivation and excitement ... they make people believe in themselves and that’s huge’ (P9, sports coach). The fundamentally learner-centred approach ascribed to coaching was reflected in the tendency to describe it in altruistic terms. One participant identified ‘a humility’ inherent in coaching, noting that ‘the best coaches I’ve seen or ever been involved with were more selfless’ (P22, physician and sports coach).


참가자들이 선생의 아젠다에 의해 추진되는 것으로 인식하는 경향이 있는 티칭이나 멘토링과 달리, 코칭은 평등주의적인 것으로 보였다. 참가자들은 코칭 다이내믹을 상호 참여의 하나로 묘사하는 경향이 있었다. 한 사람이 언급했듯이, '코칭의 큰 점은 양측이 코칭에 참여하기를 원한다는 것이다.' (P5, 의사, 음악 코치)

Unlike teaching, which participants tended to perceive as being driven by the teacher’s agenda, and mentoring, which they perceived as being driven by the mentee’s needs, coaching was seen as egalitarian. Participants tended to portray the coaching dynamic as one of mutual engagement; as one noted, ‘the big thing with coaching is that both sides want to be there’ (P5, physician and music coach).


개인 스포츠 코치로서의 그들의 경험에 대해 말하자면, 한 의사는 코칭에서 '우리에 관한 언어를 쓴다. '우리'는 이 일을 하고 있다. '당신'이 그렇게 했다거나 그렇게 했다거나 한 것이 아니다'라고 말했다. (P6, 의사 및 스포츠 코치). 이 논평은 코칭이 파트너쉽을 내포하고 있다는 널리 알려진 정서를 요약한다.

Speaking of their experiences as a coach of an individual sport, one physician commented that in coaching, ‘the language is about we - we’re working on this, not that you’ve done that or you’ve done that’ (P6, physician and sports coach); this comment encapsulates the widely held sentiment that coaching implies a partnership.


어떤 사람들에게는, 코칭 과정에 대한 상호간의 헌신이 관여를 넘어, 열정과 관심의 함축으로, '두 사람 사이의 책임감-나는 당신에게 시간과 조언을 제공하고, 당신은 나에게 어떤 결과를 제공해야 한다'를 포함시켰다. (P2, 의사) 사람들은 코칭을 개인의 성장과 발달을 우선 목표로 두는 신중한 선택으로 묘사했다.

For some, the mutual commitment to the coaching process went beyond engagement, with its connotations of enthusiasmand interest, and included ‘a sense of accountability between the two of them– I provide you with time and advice, you should provide me with some results’ (P2, physician) Coaching was described by some as a deliberate choice to foreground goals of personal growth and development:


철학적 공통점: 성찰을 중시함

Shared philosophy: valuing reflection


참가자들 사이에서 성찰은 효과적인 코칭을 지원하기 위한 기본적인 전략으로 거듭 확인되었다. 첫째로, 코치들은 그들의 직업의 핵심 부분은 학습자들이 운동선수든 의사든 간에 그들의 학습자들에게 성찰의 습관을 심어주는 것이라고 생각했다. 한 참가자는 임원 코치로서의 그들의 역할을 "그들이 그들의 상황을 성찰하도록 돕는 것. 다시 말하지만, 그들에게 무엇을 해야 하는지 말해주는 것이 아니다. 이는 그들이 무엇이 잘 되었는지, 무엇이 잘 되지 않았는지 되돌아보고, 미래에 대한 옵션(복수의 옵션)을 구체화하는 데 도움을 주는 것이다(P13, 의사 및 경영진 코치)"라고 말했다.

Across participants, reflection was repeatedly identified as a fundamental strategy to support effective coaching. First, coaches felt that a core part of their job was to instill habits of reflection in their learners, whether those learners were athletes or physicians. One participant described their role as an executive coach as ‘... helping them to reflect on their situation. Again, it’s not telling them what they should do. It’s helping them to reflect on what went well, what didn’t, and elaborating some options – options in the plural – for the future’ (P13, physician and executive coach).


스포츠에서도 코치들은 성찰을 길러줄 질문을 지속적으로 한다고 말했다. 노 젓는 사람들에게 접근법을 설명하면서, 한 코치는 선수들에게 '어떻게 느꼈는가?'라고 묻는 것에 대해 논평했다.어디서 가장 높은 힘을 느꼈나? 칼날이 물 밖으로 나오지 않을 것 같은 느낌이 든 곳은 어디인가? 코치는 그들이 영원히 학습자들과 함께 할 수는 없음을 것을 인식했고, 성찰적 습관은 '끊임없이 일하는 과정에서 적용할 수 있는 유용한 것을 줄 수 있다'고 말했다(P6, 의사 및 스포츠 코치). 따라서 성찰은 학습자가 코칭이 진행되는 과정에서 발전progress 상태를 유지할 수 있도록 하는 것으로 간주되었다.

In sports, too, coaches spoke of asking questions that would nurture continuous reflection. Describing an approach to rowers, one coach commented on asking the athletes ‘How did you feel? ...Where did you feel the highest force? Where did you feel that your blade would not come out of the water?’ (P12, sports coach). Coaches recognised that they could not be with their learners continually, and reflective habits could ‘give somebody something useful that they can apply as they continue to work’ (P6, physician and sports coach). Reflection, thus, was viewed as empowering learners to maintain their progress between coaching sessions.


그러나 우리의 자료에서 코치들이 자신의 역할에 대한 자기 성찰을 수행하는 것은 학습자들 사이의 성찰을 지원하려는 그들의 표현된 노력보다 훨씬 더 두드러졌다. 스포츠 코칭 경험이 있는 이들은 선수들의 발전과 성패에 대한 자신의 역할을 되새기는데 상당한 노력을 기울였다.

In our data, however, coaches’ engagement in self- reflection on their own roles was even more striking than their expressed efforts to support reflection amongst their learners. Those with experience coaching in sports devoted considerable effort to reflecting on their role in the development, success or failure of their athletes.


이러한 성찰은 코치들로 하여금 자신의 역할에 어떻게 더 효과적일 수 있는지에 대해 일상적으로 의문을 제기하는 과정을 포함했다; 이러한 자기 성찰을 꾸준히 실천하는 것은 지속적인 자기 계발을 하는 것이었다.

Reflection thus involved a process of coaches routinely questioning how they could be more effective in their role; growing out of this routine of self-reflection was an openness to continuous self-improvement:


자기 계발에 대한 이러한 헌신은 위에서 설명한 상호 참여라는 주제를 되풀이하는 선수들에 대한 책임이라는 관점에서 표현되었다. 예를 들어, 한 수영 코치는 자신이 감독으로서 현재 상태를 유지하기 위해 한 일에 대해 말했다: '새로운 경향과 새로운 연구와 더 나은 방법들에 자신을 적용하지 않는 것은 내가 가르치는 사람들에게 해를 끼치는 것이다. 그 사람들을 실망시키고 싶지 않다.... 나는 내가 어떻게 더 잘할 수 있는지 알고 싶다.(P9, 스포츠 코치) 

This commitment to self-improvement was, for some, expressed in terms of an accountability to athletes that echoes the theme of mutual engagement described above. One swim coach, for example, spoke of the work he or she did to remain current as a coach as a responsibility: ‘You’re doing a disservice to your swimmers by not applying yourself to new trends and new research and better ways of doing things. I don’t want to let my swimmers down so that’s why ... I want to know how I can be better’ (P9, sports coach). 


그러나 우리는 참가자들의 의료 코칭 경험에 대해서는 이러한 자기 성찰에 대한 강조를 확인하지는 못했다.

We did not, however, identify a similar emphasis on self- reflection in participants’ descriptions of their medical coaching experiences.


철학적 공통점: 실패를 포용함

Shared philosophy: embracing failure


그들이 생각하는 가치에 맞추어, 스포츠 코치는 실패의 학습적 가치를 인정했다. 많은 스포츠 감독들은 '실수에서 배울 수 있다'(P12, 스포츠 코치)라는 철학을 내세우며 실패를 운동선수 발전의 촉매제로 여겼는데, 실패는 사려 깊게 다루면 실제로 진보를 가속화할 수 있는 경험이다.

Aligned with the value they placed on reflection, sports coaches endorsed the learning value of failure. Espousing the philosophy that ‘you can learn out of mistakes’ (P12, sports coach), many sports coaches viewed failure as a catalyst for athlete development, an experience that, if handled thoughtfully, could actually accelerate progress.


스포츠와 유사점을 그린 한 임원 코치는 '이것은 모든 스포츠와 같다'고 말했다. 더 빠르게 실패를 경험할수록, 이기는 것보다 더 많은 것을 배우게 된다. 일부 코치는 자신의 핵심 기능이 선수들이 '잘 실패하도록 돕는 것'이라고 보았다. 한 코치는 자신의 발전을 촉진하기 위해 실패에 대한 선수들의 관점을 바꿀 필요가 있음을 분명히 했다.

Drawing a parallel with sports, one executive coach noted ‘It’s like any sport. The faster ... if you fail, you’ll learn something more than if you win’ (P13, physician and executive coach). Some coaches viewed it as one of their key functions to help athletes to ‘fail well’ (P10, physician and sports coach); one articulated the need to shift the athletes’ perspective about failure in order to stimulate their development:


그들에게 실패를 다른 관점에서 볼 필요가 있다. 그건 정말, 정말 중요하다. 그들은 실패해도 괜찮고 실패가 일어날 것이라는 것을 알아야 하고, 결국 실패를 다룰 것이고, 그것으로부터 배울 것이고, 당신을 더 높이 데려갈 것임을 알아야 한다. 만약 그들이 실패하지 않는다면, 그들은 충분히 노력하지 않은 것이다. (P14, 스포츠 코치) 

It’s good for themto see failure in a different perspective. That’s really, really important. They have to know that it’s okay and it will happen andyou’ll deal with it and you’ll learn fromit and it will take you higher. If they are not failing, they are not trying hard enough. (P14, sports coach) 


참가자들이 의료 환경에서 코칭을 설명할 때에도 실패의 잠재적 가치는 높이 평가되었다. 예를 들어, 일반적인 절차에 대한 코칭을 설명하면서, 한 참가자는 '가끔, 나는 실제로 그들이 실수를 하도록 강요하거나 잘못하게 한다, 그들이 그들의 실수가 무엇이고 그것이 그들이 배우는 방법이다'라고 말했다. 의료 학습자에게 실수를 허용한다는 것은 어떤 경우에는 코치의 핵심 기능으로 간주되었다.

The potential value of failure was appreciated even when participants described coaching in the medical setting. Describing coaching around a common procedure, for example, one participant commented ‘Sometimes, I actually just push them to do the mistake or let them do it wrong, until they figure out what their mistake is and that’s how they learn’ (P2, physician). Giving a medical learner permission to make a mistake, in fact, was seen as a key function of a coach in some situations:


문화적 영향: 취약성에 대한 편안함

Cultural influences: comfort with vulnerability


성찰의 철학과 실패로부터 배우는 것을 수용하는 것은 취약성에 대한 높은 편안함 수준을 요구하지만, 이는 의학에서 달성하기 어려워 보인다. 비록 운동선수들이 코치와의 훈련에서 취약성을 보이는 것이 편할 수 있지만, 의학에서는 그렇지 않을 수도 있다. '나의 경험에 따르면 가장 민감하고 신뢰할 수 있는 교수 앞에서도 쉽지 않다. 그렇게 되지 않도록 너무 조건화되어 있기 때문에, 그런 사람 앞에서조차 취약해도 된다는 것을 학습자가 믿기란 정말 어렵다.', 의사 및 경영진 코치).

Embracing a philosophy of reflection and of learning from failure demands a level of comfort with vulnerability that appeared difficult to achieve in medicine. Although athletes might be comfortable displaying vulnerability in training with their coaches, the same might not be true in medicine: ‘My experience has been that even in the most sensitive, trustworthy faculty, it’s really hard for the learner to believe that they can be vulnerable with that person because they’re so conditioned not to be’ (P13, physician and executive coach).


취약성에 대한 불편함은 학습자에게만 있는 것이 아니다. 한 참가자는 '사람들은 임상 환경에서 관찰되는 것에 대해 믿을 수 없을 정도로 수줍어하고 불안해한다. 사람들은 그것을 원하지 않는 것 같다.'

the discomfort with vulnerability lies not only with learners. One participant, noted ‘People are incredibly shy and nervous about being observed in the clinical setting. People don’t seem to want it’(P17, physician).


따라서 우리의 참가자들은 [코칭의 성공으로 이끄는 데 필요한 취약성에 대한 불편함]을 드러냈다. 그들은 코칭의 수용과 수용을 모델화하는데 어려움을 겪을 지도 모르는 학습자들에게는 잠재적으로 어려울 뿐만 아니라 코치 자신에게도 어렵다는 것을 인정했다.

Our participants thus revealed a discomfort with the vulnerability required to drive coaching success; they acknowledged that it was not only potentially difficult for learners, but also challenging for coaches themselves, who may struggle to role model the acceptance and embrace of coaching for their own development.


문화적 영향: 코칭의 정의와 개발

Cultural influences: defining and developing coaching


스포츠에서, 코칭 역할에 대한 명확한 정의가 있는 것처럼 보였다. 코치는 스포츠에 대한 지식에 의해 알려졌지만, 그들의 역할은 선수나 연주자의 역할과 매우 구별되는 것으로 보였다.

In sports, there appeared to be a clear definition of the coaching role; although coaches were informed by knowledge of the sport, their roles were seen as very distinct from the roles of players or performers.


스포츠 코칭은 종종 코치의 역할을 준비하기 위한 잘 정의된 경로가 존재하며, 코칭 책임을 맡도록 허용되기 전에 적절히 훈련받을 것을 요구한다.

Sports coaching often involves well-defined pathways for preparing coaches for their roles, and demands that coaches be properly trained before they are allowed to take on coaching responsibilities.


그러나 의학에서 코칭 역할은 명확하지 않은 경향이 있다. 그 역할과 책임은 종종 임상 감독 안에 포함된다. 한 참가자는 스키 강사의 엄격한 준비와 의료 교육 분야를 비교했다.

In medicine, however, the coaching role tends to be ill-defined; the role and its responsibilities are often embedded within clinical supervision. One participant instructively contrasted the regimented preparation required of ski instructors with the world of medical education:


의과대학에서, 어떤 결정에 대한 책임을 지는 사람은 대개 principal supervisor이고, 이것은 그 사람이 누구인지와는 무관하다. (P6, 의사 및 스포츠 코치)

in medical faculties, it’s usually the principal supervisor, whoever they happen to be, who really carries the brunt of responsibility for that decision. (P6, physician and sports coach)


코칭 전문성과 그 경계에 대한 보다 날카로운 정의에도 불구하고, 또는 아마도 스포츠 코치는 다른 사람들에게 코칭 작업의 요소를 편안하게 위임하는 것처럼 보였다. 참가자들은 [어떤 코치도 각 선수가 요구할 수 있는 모든 지원과 지도를 제공하는 데 필요한 모든 기술을 보유할 수 없다]는 것을 거듭 인정했다.

Despite, or perhaps because of, the sharper definition of coaching expertise and its boundaries, sports coaches appeared comfortable delegating elements of the coaching work to others. Participants repeatedly acknowledged that no coach could possess all the skills necessary to offer all the support and guidance that each player might require.


그 결과 '(내가 직접) 무엇을 할 수 없다면 할 수 있는 사람을 찾는다'(P11, 스포츠 코치)는 태도에 상당한 편안함이 있었다. 코칭 역할에 대한 보다 명확한 정의는 아마도 한 코치가 언급했듯이, '나와는 다른 것을 가르치는 코치들이 있다. 기술을 더 담당하는 코치를 가지고 있다. 전술 더 담당하는 코치들을 보유하고 있다. 사실, 스포츠 코치들은 모든 것을 스스로 할 것이라고 기대하지 않았고, 오히려 그들의 선수들에 대한 책임의 일부로 특정한 필요를 위해 코치 팀을 구성하는 것을 보았다.

As a result, there was considerable comfort with the attitude that ‘If I can’t do something, I find someone who can’ (P11, sports coach). The clearer definition of the coaching role perhaps led to a readier acceptance of the need to bring in others to support an athlete’s development; as one coach noted, ‘We have coaches that teach different things. We have coaches that are more technical. We have coaches that are more tactical’ (P14, sports coach). Sports coaches, in fact, did not expect to do it all themselves, and rather viewed assembling a team of coaches for specific needs as part of their responsibility to their athletes.


사실, 의료 코치는 코칭 역할을 단독으로 맡는 경향이 있었다. 일부 사람들은 이러한 경향이 의사들이 '신경증적으로 자급자족하다; 그들은 모든 것을 할 수 있다고 생각하지만 실제로는 그렇지 못하다'는 문화를 반영한다고 느꼈다.

Medical coaches, in fact, tended to assume the coaching role solo; some felt this tendency reflected a culture in which physicians are ‘neurotically self-sufficient; they think they can do it all, but actually they don’t’ (P23, physician).


문화적 영향: 애매한 구분선

Cultural influences: blurring the lines


의학에서 코칭 역할과 다른 교육적 역할 사이의 경계가 모호해지는 경우가 많다. 예를 들어, 코칭으로 시작했지만, 상황에 따라 감독, 평가 또는 심지어 처벌discipline로 흐를 수 있다.

In medicine, the lines between the coaching role and other pedagogic roles are often blurred. For example, what might begin as coaching might bleed into supervision, assessment, or even discipline depending on the circumstances.



코치와 선수 사이에서도 선이 흐려졌다. 의료 코칭을 스포츠와 비교하면서 한 참가자는 '스포츠에서는...코치는 갑자기 빙판이나 테니스 코트에 발을 들여놓고 실제로 경기를 시작할 수 없다. 반면에 의학에서는 코치가 갑자기 선수가 될 수 있다.

Lines also blurred in medicine between coaching and playing. Comparing medical coaching to sports, one participant noted ‘In sports ...the coach can’t all of a sudden set foot on the ice or the tennis court and actually start playing. Whereas in medicine a coach can very quickly become a player’ (P24, physician).


그러므로 의학에서 코치의 신원은 스포츠에서 일어나지 않을 수 있는 방식으로 '선수'라는 것에 강하게 얽매여 있을 수 있다.

In medicine, therefore, a coach’s identity may remain strongly tied to being a ‘player’ in a way that may not occur in sports.


고찰

DISCUSSION


의학과 비교했을 때 스포츠에서의 성질의 차이와 경기에서의 이해 관계에도 불구하고, 우리 참가자들은 코치로서의 그들의 역할에 대해 반성하면서 공유된 철학적 기반을 지지하는 경향이 있었다. 이러한 공유된 원칙은 의료 교육에서 코칭이 채택될 수 있는 방법에 대한 대화의 출발점을 제공한다.

Despite differences in the nature and stakes of performance in sports compared with medicine, our participants tended to endorse shared philosophical underpinnings as they reflected on their roles as coaches. These shared principles offer a starting point for conversations about how coaching might be adopted in medical education.


코칭이 코칭이 되려면,

  • 학습자의 성장과 발전을 향한 명확하고 양방향 지향성을 가진 상호간의 참여를 포함한다.

  • 성찰의 임베딩: 이 역시 양방향적이어야 한다. 코치는 학습자에게 반성의 습관을 심어줄 뿐만 아니라 그러한 습관 자체를 실천한다.

  • 실패를 필연적이고 가치 있는 것으로 인정하여, 지속적인 개선을 촉진할 수 있는 잠재력을 포용한다.

for coaching to be coaching,

  • involve mutual engagement, with a clear and bilateral orientation towards learner growth and development.

  • embed reflection, which is also bidirectional: coaches not only instil habits of reflection in learners, but also practise those habits themselves.

  • acknowledge failure as inevitable and invaluable, embracing its potential to stimulate continued improvement.


다만 주의할 것은 그러한 원칙들을 의학교육 환경으로 translation하는 것이다. 예를 들어, 의학이 코칭 역할에 대한 명확한 정의가 결여되어 있다는 것은 교수진을 그 직책에 적합하게 개발할 수 있는 능력을 손상시킨다. 이전의 연구는 임상 학습자는 "가장 뛰어난 임상의사"를 "가장 신뢰할 수 있는 피드백 원천"으로 보는 경향이 있다는 것을 보여주었다.33 스포츠, 음악 등의 분야와 달리 의학은 [expert performer과 구분되는 전문 강사나 코치]의 원형을 가지고 있지 않다. 하지만 참가자들이 우리에게 상기시켜 주듯이, 우수한 수행 능력이 항상 우수한 코칭으로 바뀌는 것은 아니다.

The notes of caution relate to the translation of those principles into the medical education setting. Medicine’s lack of a clear definition of the coaching role, for example, compromises its ability to develop faculty members adequately for the job. Earlier work has shown that clinical learners tend to look to the strongest clinicians as their most credible sources of feedback.33 Unlike fields such as sports and music, medicine does not have an archetype of the expert instructor or coach that is distinct from its notion of an expert performer. However, as our participants reminded us, performance ability does not always translate into coaching ability.


우리는 코칭이 무엇인지, 어떻게 그것이 수행과 역할 모델링과 구별되는지, 그리고 어떻게 그것을 교직원들에게 훈련시키고 개발할 수 있는지에 대한 더 명확한 설명을 필요로 할 것이다.

we will require a clearer articulation of what coaching is, how it is distinct from performing and role modelling, and how we can train and develop it in our faculty members.


의학이 코칭을 명확하게 정의하지 못하는 것과 맞물려 있는 것은 교육적 역할을 흐리게하는blur 습관적인 경향이다. 의학교육에서 한 명의 교수자는 코칭, 멘토링, 감독, 교육, 평가를 모두 수행하는데, 때로는 이것들을 동시에 수행하기도 한다. 일부에서는 의학교육에서 코치와 평가자의 역할을 분리해야 한다고 제안했지만, 특히 소수의 교직원이 교육 의무를 부담하는 경우, 완전한 분리를 달성하는 것은 쉬운 일이 아니다.

Intertwined with medicine’s failure to clearly define coaching is its habitual tendency to blur educational roles; coaching, mentoring, supervising, teaching and assessing are all roles that may be assigned to the same individual faculty member, sometimes simultaneously. Although some have suggested that the roles of coach and assessor should be divorced in medical education,3,15,34 achieving a clean separation is no easy task, particularly when educational duties are shouldered by a small number of faculty members.


문제를 더 복잡하게 만드는 것은 코치들이 갑자기 선수로 변할 가능성이 있기 때문이다. 그 이유는 환자의 안전문제일 수도 있고, 코치에게 여전히 선수로서의 정체성을 위한 투자를 하기 위함일 수도 있다.

Our work adds an additional layer of complexity , highlighting the potential for coaches to rapidly become players, either because the safety of the patient demands or because they remain invested in their identities as players.


그러나 코칭은 코치와 학습자가 공유된 목적의식을 달성하는 데 크게 의존한다. 두 사람 모두 학습자가 자신의 잠재력을 실현할 수 있도록 지원하는 것이 상호 작용의 목적임을 확신해야 한다. 역할이 유동적인 상황에서는 이런 자신감이 흔들릴 수 있고, 코칭의 잠재적 가치도 훼손될 수 있다.

Coaching, however, relies heavily on coach and learner achieving a shared sense of purpose; both must be confident that the intent of their interaction is to support learners in realising their potential. When roles are fluid, this confidence may be shaken, and the potential value of coaching may be undermined.


[학습자가 [자신의 약점과 불확실성의 영역]을 [효과적인 코칭을 받을 scrutiny에 노출시킬 정도로 충분히 취약할 것]을 기대]한다면 으로 코칭이 갖는 발달적 의도에 대한 자신감 역시 중요하다

Confidence in the developmental intent of coaching is also critical if we expect learners to be vulnerable enough to expose their areas of weakness and uncertainty to the scrutiny that begets effective coaching.


의학의 전문직 문화는 학습자나 의사들 사이에서 [취약성을 위한 공간]을 마련하기 위해 애쓰고 있다. Molloy와 Bearman은 "임상에서는 약점을 숨김으로써 자신을 믿음직스럽게 만들어야 하는 고질적 요구"가 존재한다고 지적한다.

Medicine’s professional culture struggles to make room for vulnerability amongst either its learners or its practitioners. Molloy and Bearman describe ‘an endemic need to promote credibility and hide vulnerabilities’ in clinical medicine;


또한, 의학의 전문직 문화는 자율성과 독립을 중시한다. 사실, 학습자들은 더 이상 관찰되고 지도되어야 할 수준에 도달하여 졸업해야 한다. 이러한 배경에서 학습자들은 자신의 능력을 보여줘야 한다는 끈질긴 압력을 느낄 수 있다. 그러나 의미 있는 코칭을 받기 위해서는, (유지하고자 열렬히 노력하는) 역량의 균열을 노출시킬 준비가 되어 있어야 한다.

Additionally, medicine’s professional culture values autonomy and independence;22 learners, in fact, aim to graduate from the need to be observed and coached.23 Against this backdrop, learners may feel a persistent pressure to portray competence. But for individuals to receive meaningful coaching, they must be prepared to expose the cracks in a veneer of competence that they may work hard to maintain.


물론, 의료에서 실패를 불편해하는 것이 단지 문화 때문만은 아니다; 그것은 또한 환자의 안전 때문이기도 하다. 따라서 우리는 [학습자가 코칭을 받을 수 있는 안전한 공간]과 [환자가 치료를 받을 수 있는 안전한 공간] 사이에서 올바른 균형을 이룰 수 있는 방법을 고민할 필요가 있다.

Of course, medicine’s discomfort with failure is not solely grounded in culture; it is also grounded in patient safety. We therefore need to grapple with how to achieve the right balance between safe spaces in which learners can receive coaching and safe spaces in which patients can receive care.


코칭의 핵심 개념을 유지하면서, 어디까지 얼마나 많은 타협을 할 수 있을까?

how much compromise can we make before this thing we are calling coaching is no longer coaching?


Varpio는 새로운 교육학적 접근법이 새로운 맥락으로 통합될 때, 근본적인 원칙 중 일부는 보존되고, 일부는 변형되고, 일부는 버려진다고 경고했다.36 Varpio 외 연구진들은 교육적 변화educational movement는 기술보다는 철학에 좌우된다고 결론지었다. 기술이나 원칙이 새로운 맥락으로 옮겨갈 때, 우리는 [접근방식의 철학]이 반드시 유지되도록 할 필요가 있다. 그렇지 않으면, 그 새로운 접근방식의 약속된 이익은 번역에서 상실될 수도 있다.

Varpio cautioned that when a novel pedagogical approach is integrated into a new context, some of its fundamental principles are preserved, some are transformed and some are abandoned.36 Varpio et al.36 concluded that educational movements hinge on the philosophical rather than on the technical; when techniques or principles are modified to a new context, we need to ensure that the philosophy of the approach is upheld. Otherwise, the promised benefits of that novel approach may be lost in translation.


의학교육에 있어서 어떤 형태 코칭을 도입하든, 우리는 [학습자 발전 지향성, 성찰의 습관, 학습자의 실패로부터 성장하는 능력]을 보존할 필요가 있다.

Whatever form coaching takes in medical education, we need to preserve its orientation towards learner development, its habits of reflection and its capacity to enable learners to grow from failure.


아마도 모든 피드백이 그것의 효과를 보장하기 위해 코칭을 요구하는 것은 아니다.

Not all feedback, perhaps, demands coaching to ensure its effectiveness.


결론

CONCLUSIONS


의료 훈련에서 코칭 접근법을 채택하는 것은 우리가 코칭을 어떻게 정의하고, 코칭의 핵심 원리를 어떻게 이해하며, 의료의 전문적 문화에 어떻게 그 철학을 적응시키는지에 대한 세심한 주의를 필요로 한다. 코칭은 단순히 오래된 접근방식의 새로운 라벨이 아니어야 한다. 예를 들어, '피드백'을 '코칭'으로 바꾸는 것은 학습자에게 더 생산적인 결과를 산출할 수 없을 것이다. 오히려 코칭이 접근하는 철학적 기둥을 이해하고 보존할 수 있도록 해야 한다.

Adopting coaching approaches in medical training demands careful attention to how we define coaching, how we understand its key principles, and how we adapt its philosophies to medicine’s professional culture. Coaching must not simply be a new label for old approaches; merely rebranding ‘feedback’ as ‘coaching’, for example, is unlikely to yield more productive outcomes for learners. Rather, we should ensure that we understand and preserve the philosophical pillars on which our coaching approaches must be built.



2 Lovell B. What do we know about coaching in medical education? A literature review. Med Educ 2018;52 (4):376–90.





 2019 May;53(5):467-476. doi: 10.1111/medu.13799. Epub 2019 Jan 23.

Where philosophy meets cultureexploring how coaches conceptualise their roles.

Author information

1
Department of Clinical Neurological Sciences, Schulich School of Medicine and Dentistry, The University of Western Ontario, London, Ontario, Canada.
2
Department of Innovation in Medical Education and Department of Medicine, University of Ottawa, Ottawa, Ontario, Canada.

Abstract

CONTEXT:

Although conceptually attractive, coaching in medicine remains ill-defined, with little examination of the transferability of coaching principles from other fields. Here we explore how coaching is enacted both within and outside of medicine; we aim to understand both the elements required for coaching to be useful and the factors that may influence its translation to the medical education context.

METHODS:

In this constructivist grounded theory study, we interviewed 24 individuals across three groups: physicians who consider themselves coaches in clinical learning settings (n = 8), physicians with experience as sports, arts or business coaches (n = 10), and sports coaches without medical backgrounds (n = 6). Data collection and analysis were conducted iteratively using constant comparison to identify themes and explore their relationships.

RESULTS:

We identified a shared philosophy of coaching, comprising three core elements that our participants endorsed regardless of the coaching context: (i) mutual engagement, with a shared orientation towards growth and development; (ii) ongoing reflection involving both learners and coaches, and (iii) an embrace of failure as a catalyst for learning. Enacting these features appeared to be influenced by culture, which affected how coaching was defined and developed, how the coaching role was positioned within the learning context, and how comfortably vulnerability could be expressed. Participants struggled to clearly define the coaching role in medicine, instead acknowledging that the lines between educational roles were often blurred. Further, the embrace of failure appeared challenging in medicine, where showing vulnerability was perceived as difficult for both learners and teachers.

CONCLUSIONS:

Medical education's embrace of coaching should be informed by an understanding of both coach and learner behaviours that need to be encouraged and trained, and the cultural and organisational supports that are required to foster success.


십대 학습자의 저주(Med Educ, 2014)

The curse of the teenage learner

Liz Mossop





임상 교육자로서, 우리는 학습자들에게 가장 좋은 것을 하려고 한다. 만약 우리가 시간과 에너지, 돈을 투자하여, 상호작용 중심의 참여적 커리큘럼 개발에 투자한다면, 학생들은 참석할 것이라고 가정한다. 이들은 참석뿐만 아니라 우리가 정해놓은 사전 업무도 완수하고, 소셜미디어와 동료들의 산만함을 버리고 세션 중 학습에 주력할 것이라고 생각한다. 그런데 왜 실제로는 이러지 않는 거지?

As clinical educators, We want to do what is best for our learners. We assume that if we invest time, energy and money into developing an interactive and engaging curriculum, students will attend. Not only will they attend, but they will also have completed the pre-sessional tasks we have set them, and will focus on their learning during the session, abandoning the distractions of social media and their peers. Why wouldn’t they do this?


이 FC 접근법의 예에서, 학생들의 출석률은 25%까지 떨어졌다. 이 기관의 교수 회의모습이 쉽게 상상된다. 좌절된 목소리가 자원 낭비와 새로운 전달 방법의 무의미한 실행을 지적할 것이다. 학생들이 학습자로서의 의무를 다하지 않는데, 우리가 왜 새로운 접근법을 택해야 하는가?

In this example of a flipped classroom approach, student attendance dropped as low as 25%.1 It is easy to picture a faculty meeting at this institution, at which frustrated voices cite the wasting of resources and the pointless implementation of new methods of delivery. Why should we bother taking new approaches when students fail to fulfil their obligations as learners?


교육문화는 커리큘럼의 변경이 이루어지는 시기에 대해 많은 것을 해답해야 한다.

The learning culture has much to answer for when curricular changes are made.


아직 성인 학습의 기술을 배우지 못한, 감성적인 십대 학습자들은 우리가 전달하는 공식적인 커리큘럼뿐만 아니라 잠재 커리큘럼에 의해 [동료 롤모델과 의식ritual의 보이지 않는 영향]에도 노출된다.2 이러한 후자의 영향은 그들을 연구와 학습의 핵심 활동에서 멀어지게 하여, 그들을 임상 의사가 되기 위한 학습의 중심적인 초점을 방해할 수 있다.

Impressionable teenage learners, who have not yet learned the skills of adult learning, are not only exposed to the formal curriculum we deliver, but also to the unseen influences of peer role models and rituals within the hidden curriculum.2 These latter influences may pull them away from their core activities of study and learning, distracting them from their central focus of learning to be a clinician.


그러므로 뒤집힌 교실에 관련된 사람들과 같은 새로운 교육 전략을 실행하기 위해서는 학습 문화의 변화가 필요하다3

A change in learning culture is therefore necessary in order to implement new teaching strategies such as those involved in the flipped classroom3


능동적인 학습 기법에 대해 학생들을 훈련해야 할 뿐만 아니라, 우리는 숨겨진 커리큘럼의 영향에 대한 명시적인 논의를 촉진해야 한다. 이것은 학생들이 부정적인 롤모델과 같은 요소들이 그들의 학습 행동에 어떻게 영향을 미칠 수 있는지를 이해하는데 도움을 줄 것이다.5 학생들은 또한 그들이 고려하고 있는 정보에 초점을 맞추면서 서로를 계속적으로 돕는 학습자로 발전하기 위해 자신의 학습 스타일과 능력에 대해 성찰하도록 권장되어야 한다. 능동적인 학습 접근법을 수용하는 커리큘럼에서 학생으로서 성공을 위해 필요한 속성이기 때문에, 그들은 성찰의 과정을 분명히 가르쳐야 한다.6

As well as training students in active learning techniques, we should facilitate explicit discussion of the influence of the hidden curriculum as this will help students to understand how elements such as negative role models can influence their learning behaviours.5 Students should also be encouraged to reflect on their learning styles and abilities in order to develop as learners who continue to help each other whilst maintaining their focus on the information they are considering. They should be explicitly taught the process of reflection because this is a required attribute for success as a student in a curriculum that embraces active learning approaches.6


커리큘럼이 계획될 때마다 교수진들은 철저한 니즈 평가NA를 수행해야 하며, NA는 학습자의 능력과 교수진의 참여에 정렬된aligned 교육 전달방식의 분석도 포함되어야 한다. 학습자 준비 및 제도적 문화와 같은 문제를 고려하지 않는다면, FC와 같은 이니셔티브에 대해 조금 읽어보고, 이를 커리큘럼 재설계 시에 넣는 것은 쉬운 일이다

Whenever curricula are planned, faculty staff must perform a thorough needs assessment,9 which should include an analysis of teaching delivery aligned to learners’ ability and faculty staff engagement. It is easy to read about initiatives such as that of flipping the classroom and start to embed them in a redesigned curriculum without considering issues such as learner readiness and institutional culture.


문제 기반 학습 커리큘럼은 이미 잘 연구되었고, 연구에 따르면 PBL의 실패는 종종 교수진의 준비 부족, 학생들의 해산 그리고 숨겨진 커리큘럼의 강력한 영향에서 비롯된다는 것을 보여줄 많은 증거가 있다.

Problem-based learning curricula have certainly been well researched and there is plenty of evidence to show that their failure often derives from a lack of faculty readiness, student disengagement and the powerful influence of the hidden curriculum.10,11


한 환경에서 잘 작동하는 것이 다른 맥락에서는 완전히 비효율적일 수 있다.

What works in one environment may be entirely ineffective in another context.




 2014 Mar;48(3):231-3. doi: 10.1111/medu.12405.

The curse of the teenage learner.

Author information

1
Nottingham, UK.
PMID:
 
24528456
 
PMCID:
 
PMC4016749
 
DOI:
 
10.1111/medu.12405


의과대학생을 미래학습에 준비시키는 용도로 기초의학교육 (Med Educ, 2014)

Preparing medical students for future learning using basic science instruction

Maria Mylopoulos1 & Nicole Woods2




도입

INTRODUCTION


'미래 학습을 위한 준비(PFL)'의 구인는 

  • 가용 자원으로부터 새로운 정보를 배우고, 

  • 새로운 학습을 과거의 경험과 연관시키며, 

  • 문제 해결에 있어 혁신과 유연성을 증명하는 능력으로 이해된다.1

The construct of ‘preparation for future learning’ (PFL) is understood as the ability to learn new information from available resources, relate new learning to past experiences and demonstrate innovation and flexibility in problem solving.1


적응적 전문지식Adaptive expertise 은 임상 실무에서 탁월함을 나타내는 것으로 이해되며, 비판적으로, 훈련 내내 개발되어야 하는 학습 기술이다.4,5

Adaptive expertise is understood to represent excellence in clinical practice2,3 and, critically, to be the product of a learned skill set that must be developed throughout training.4,5


PFL은 학생들이 [하나의 문제 해결 상황에서 다음 문제 해결 시점으로 자신의 지식을 전달할 수 있는 정도]를 탐구하는 학습 전이에 관한 연구를 통해 교육 문헌에 나타났다.1

Preparation for future learning has emerged in the education literature through studies in the area of learning transfer, which explore the extent to which students are able to transfer their knowledge from one problem-solving situation to the next.1


최근 연구원들은 가장 훌륭한 문제 해결 교육일지라도 학생들이 실제로 마주칠 수 있는 모든 상황에 대비할 가능성은 낮다는 사실에 초점을 맞추고 있다. 예를 들어, 학생들이 가능한 한 많은 문제 해결 컨텍스트를 경험할 수 있도록 하기 위한 의료 교육자들의 노력에도 불구하고, 의사는 이전에 본 적이 없는 새로운 문제와 컨텍스트에 직면할 수밖에 없다.

recently researchers have focused on the fact that even the finest problem-solving instruction is unlikely to prepare students for every situation they might come across in practice. For example, despite the efforts of medical educators to ensure that students experience as many problem-solving contexts as possible,6 it is inevitable that physicians will encounter new problems and contexts they have never seen before.


습득한 지식을 적용하고 복제하는 능력은 의사들이 효과적으로 의료행위를 수행할 준비로는 충분하지 않다. 더욱이 적응적 전문가는 습득한 지식을 효과적이고 효율적으로 사용할 수 있을 뿐만 아니라, 새로운 문제에 직면했을 때 새로운 해결책을 세울 수 있을 것으로 기대된다.7

the ability to apply and replicate acquired knowledge is insufficient to enable physicians to perform effectively. Moreover, adaptive experts are expected to be both able to use their acquired knowledge effectively and efficiently, and able to construct new solutions when faced with novel problems.7


교과과정에 기초과학을 포함시키는 가장 강력한 주장은 그것이 미래의 학습을 위한 준비도구 역할을 한다는 것이다. 우즈12는 기초과학 훈련의 가치는 학생들에게 [임상 지식의 이해를 위한 일관된 프레임워크의 개발]을 돕는 것이라고 주장한다.

the most powerful argument for the inclusion of basic science in the curriculum is that it serves as a preparatory tool for future learning. Woods12 argues that the value of basic science training is its ability to assist students with the development of a coherent framework for the understanding of clinical knowledge.


따라서 기초 과학 교육을 사용하여 훈련된 학생들은 그들의 현존하는 정신적 표현에 새로운 임상 정보를 더 잘 통합할 수 있을 것이고 새로운 진단 문제를 더 빨리 해결할 수 있을 것이다. 따라서 기초과학교육은 PFL지침의 일종으로 볼 수 있다.

Students trained using basic science instruction might therefore be better able to incorporate novel clinical information into their existing mental representations and more quickly solve new diagnostic problems . Therefore, basic science instruction can be considered to be a form of PFL instruction


본 연구의 목적은 PFL 평가(PFLA)에 대한 임상적으로 성능에 초점을 맞춘 지침과 기초 과학 지침의 영향을 비교하는 것이었다.

The objective of this research was to compare the impact of basic science instruction with that of clinically focused instruction on performance on a PFL assessment (PFLA).


방법

METHODS


설계

Design


본 연구는 PFLA가 두 가지 교육 방법 중 하나를 사용하여 훈련된 학생들 사이의 성능 차이를 드러낼 수 있는지 여부를 결정하기 위해 Bransford와 Schwartz,1,8에서 채택된 '이중 전이' 설계를 채택했다.

This study employed a ‘double transfer’ design, adapted from Bransford and Schwartz,1,8 to determine whether a PFLA can reveal differences in performance between students trained using one of two instructional methods (clinically focused instruction and basic science instruction, respectively)


(Fig. 1).


자료 개발

Material development


교육 자료

Instruction materials


임상 컨설턴트와 함께, 연구팀의 구성원들은 이전의 실험에서 적응한 8개 범주의 신경 질환에 대한 기초적인 과학 메커니즘을 기술하는 징후와 증상 및 구절 목록을 작성했다.13,14 4개 범주의 범주는 신경 질환의 일반적인 분류(상기 운동 니어)를 나타낸다.유론병변, 운동신경병변, 신경근육 접합 장애 및 근육 질환). 다른 네 가지 범주는 이러한 분류(뇌졸중, 골수중력, 척수압축, 다낭성)의 보다 구체적인 예였다.

Along with the clinical consultants, members of the research team created a list of signs and symptoms and passages describing the underlying basic science mechanisms for eight categories of neurological disease adapted from previous experiments.13,14 

    • Four of the categories represented general classifications of neurological disease (upper motor neuron lesions, lower motor neuron lesions, neuromuscular junction disorders and muscle disease). 

    • The other four categories were more specific examples of these classifications (brainstem stroke, myasthenia gravis, spinal cord compression and polyneuropathy).


연구의 초기 지침 단계에서는 각 일반 분류를 설명하는 4개의 개별 서술 구절로 구성된 2개의 학습 자료를 작성했다. 

    • 임상적 징후와 증상만 기술한 한 세트의 교육 자료, 

    • 두 번째 세트는 징후와 증상 및 기본적인 과학 메커니즘을 포함했다.

For the initial instruction phase of the study, we created two sets of learning materials each consisting of four separate narrative passages describing each of the general classifications. One set of instructional materials described only the clinical signs and symptoms; the second set included the signs and symptoms plus underlying basic science mechanisms.


평가 자료

Assessment materials


암기 퀴즈

Memory quizzes


초기 학습 후 평가와 PFL 명령 단계 후 평가는 메모리 쿼리 진단 평가의 두 가지 구성 요소로 이루어졌다. 첫째, 참가자의 학습 자료에 대한 기초적 획득을 독립적으로 평가하기 위해 기억에서 대답할 일련의 객관식 질문(MCQ)을 사용하였다. 이러한 짧은 퀴즈는 문제 해결보다는 사실의 인식을 평가하도록 설계되었기 때문에 초기 평가 및 PFLA와 구별되는 것으로 간주되었다.

Assessment after initial learning and after the PFL instruction phase consisted of two components: memory quizzes, and diagnostic assessments. First, a series of multiple-choice questions (MCQs) to be answered from memory were used to independently assess participants’ basic acquisition of the learning material. These short quizzes were considered as distinct from the initial assessment and the PFLA as they were designed to assess the recognition of facts rather than problem solving.


초기 평가와 PFLA 진단검사

Initial assessment and PFLA diagnostic tests


기억력 퀴즈와 대조적으로, 초기 평가와 PFLA는 학생들이 단순히 임상 사실을 상기하지 않고 임상 사례를 해결하도록 요구하였다.

By contrast with the memory quizzes, the initial assessment and the PFLA required that students solve clinical cases and not simply recall clinical facts.


이 과정의 최종 결과는 4개의 광범위한 범주에서 도출한 16개의 사례로 구성된 초기 평가와 4개의 특정 질병을 언급하는 16개의 사례로 구성된 PFLA이다.

The end result of this process was the initial assessment, which consisted of 16 cases drawn from the four broader categories, and the PFLA, which consisted of 16 cases referring to the four specific diseases.


참여자와 자료수집

Participants and data collection


참가자들은 먼저 맞춤형 컴퓨터 프로그램을 통해 네 가지 광범위한 분류 범주(상부 운동 뉴런 병변, 하부 운동 뉴런 병변, 신경근육 접합 장애 및 근육 질환)를 배우도록 요청받은 초기 학습 단계를 완료했다.

Participants first completed the initial instruction phase in which they were asked to learn four broad classification categories (upper motor neuron lesions, lower motor neuron lesions, neuromuscular junction disorders and muscle disease) via a custom designed computer program.


BS 조건의 참가자들은 각 질병의 임상적 특징과 각 특징이 발생하는 이유를 설명하는 기본 과학 메커니즘을 포함하는 네 가지 기초 과학 질병 설명을 검토했다.

Participants in the BS condition reviewed four basic science disease descriptions, which included the clinical features of the respective diseases and basic science mechanisms explaining why each feature occurred.


CL 조건의 참가자들은 4개의 해당 임상 질병 설명을 검토했으며, 여기에는 수반되는 역학 요인이 포함된 임상 특징이 포함되었다.

Participants in the CL condition reviewed four corresponding clinical disease descriptions, which included clinical features with accompanying epidemiological factors.


초기 지침 단계에 따라, 두 조건의 참가자들에게 초기 학습 자료의 임상적 측면에 대한 기억 시험을 완료하도록 요청하였다. 그런 다음 학생들은 진단 기술을 평가하기 위해 설계된 초기 평가를 완료했다. 참가자들은 총 16개의 환자 사례를 읽고 가장 적절한 진단을 선택해야 했다.

Following the initial instruction phase, participants in both conditions were asked to complete a memory test of the clinical aspects of the initial learning material. Students then completed the initial assessment designed to assess diagnostic skill. Participants were required to read a total of 16 patient cases and to select the most appropriate diagnosis.


그런 다음 참가자들에게 PFL 학습 단계를 완료하도록 요청하였다. 이미 신경 질환의 일반적인 분류를 배운 학생들은 이제 그러한 범주의 네 가지 구체적인 예를 배워야 했다. 학생들은 초기 교육 단계에서 제시되지 않았던 4개의 새로운 질병 설명을 읽는다.

Participants were then asked to complete the PFL instruction phase. Having already learned the general classifications of neurological diseases, students were now required to learn four specific examples of those categories. Students read four novel disease passages that had not been presented in the initial instruction phase.


학생들에게 20개의 새로운 기억력 테스트 항목이 제시되었다. 새로운 질병의 임상적 특징에 대한 그들의 지식을 평가하기 위해서. 마지막으로 참가자들은 새로운 질병 상태를 바탕으로 16개의 새로운 환자 사례를 진단해야 하는 PFLA를 완료했다.

Students were then presented with 20 new memory test items assessing their knowledge of the clinical features of the novel diseases. Finally, participants completed the PFLA, which required them to diagnose 16 new patient cases based on the novel disease conditions.


자료 분석

Data analysis



결과

RESULTS


(Fig. 2).


고찰

DISCUSSION


우리의 결과는 기초과학 교육을 함께 받은 참가자들이, 임상적으로만 초점을 둔 교육을 받은 참가자들보다 새로운 관련 내용을 더 잘 배운다는 것을 보여준다.

our results indicate that participants who received basic science instruction demonstrated better learning of novel related content than did those who received only clinically focused instruction.


적응적 전문성을 탐구하는 연구자들은 적응 전문가가 새로운 문제 해결 맥락에서 지식을 유연하게 사용하는 능력은 해석적 지식(knowing with)에 초점을 맞춘 교육과 평가를 통해 학습되며, 복제적(knowing that)이나 적용적(knowing how)를 통해 학습되는 것이 아니라고 말한다.

researchers exploring adaptive expertise have suggested that the ability of adaptive experts to flexibly use knowledge in new problem-solving contexts is learned through instruction and assessment that focus on interpretive knowing (knowing with), rather than replicative (knowing that) or applicative (knowing how) knowing.1,15 


기초과학 교육을 포함하는 것은 이러한 형태의 학습을 지원하고 따라서 적응적 전문지식의 개발을 더욱 광범위하게 지원하는 것으로 보인다.

The inclusion of basic science instruction appears to support this form of learning and thus the development of adaptive expertise more broadly.


일부 연구자들은 기초과학의 긍정적인 영향을 밝혀내기 위한 이것과 다른 유사한 연구들의 실패가 그러한 영향은 존재하지 않는다는 주장을 뒷받침한다고 결론지었다.911 더구나 임상교사와 학생들은 종종 기초과학 연수의 가치를 보기 위해 애쓴다.16 우리의 결과는 기존 연구에 대한 다른 해석을 제안한다.

Some researchers have concluded that the failure of this and other similar studies to uncover the positive impact of basic science supports the claim that no such impact exists.9–11 Moreover, clinical teachers and students often struggle to see the value of basic science training.16 Our results imply a different interpretation of this body of research


전통적으로 의학교육에서의 평가는 학습 자료를 복제replicate하고 적용apply하는 학생들의 능력을 평가하는 반면, PFL 평가는 학생들이 새로운 내용을 학습하는 데 지식을 사용할 수 있는 정도를 평가한다. 따라서 PFL 평가는 어떤 의학교육이 [학생들이 미래 문제 해결에서 혁신적이고 유연하게 대처할 수 있도록 가장 잘 준비시키는지]를 판단하는 데 사용될 수 있다.

whereas assessment in medical education traditionally evaluates students’ ability to replicate and apply learning material, PFL assessments that explore the extent to which students are able to use their knowledge to learn new content can be used to determine which educational practices in medicine (e.g. basic science instruction, test-enhanced learning17,18) are best able to prepare students to be innovative and flexible in their future problem solving.



특히, 지식의 응용이나 복제의 입증보다 [새롭게 내용을 배우는 것에 중점을 두는 것]은 평가에 대한 우리의 이해에 중요한 의미를 가진다.

In particular, an emphasis on learning novel content rather than demonstrating the application or replication of prior knowledge has significant implications for our understanding of assessment


적응형 전문가들은 'optimal adaptability corridor'라는 것을 발휘한다. 즉, 이는 직무수행 중 효율적 문제해결과 혁신적 문제해결의 균형을 맞추는 것이다. 적응형 전문지식을 개발하려면 발달과정 내내 동일한 균형을 유지해야 하므로, [효율적 실천]과 [혁신적 실천] 모두에 초점을 맞춘 교육이 필요하다.7 따라서 적응형 전문성 개발을 위해서는 전통적인 방식과 PFL 지침과 평가를 모두 할 수 있는 여지가 있다고 주장할 수 있다.19,20

Adaptive experts work within what has been described as the ‘optimal adaptability corridor’, balancing efficient and innovative problem solving in their work. Developing adaptive expertise requires that the same balance be maintained throughout development, thus requiring instruction that is focused on both the efficiencyand the innovation-related dimensions of practice.7 It can be argued therefore that there is a place for both traditional and PFL instruction and assessments in the development of adaptive expertise.19,20


유사한 조건에서 자발적인 지식 전이의 사례를 찾으려는 많은 연구에서, 성공적인 전이는 관찰하기 어렵다는 결론을 내려왔다.21,22 한 문제의 해결책을 새로운 문제의 해결에 직접 적용해야 하는 기존의 transfer에 연구와 달리, PFL은 성공적인 전이를 [새로운 문제의 해결을 위한 플랫폼으로서 기존 경험의 사용]으로 정의한다. 학습자가 특정 해결책을 어떻게 전이하는지를 평가하는 것에서, 새로운 지식의 습득을 위한 기초를 제공하는 것으로 초점을 옮겼다는 사실은 아마도 다른 사람들이 실패한 곳에서 성공적인 전이가 발견되었다는 것을 설명해 줄 것이다.

it is important to note that a large body of research attempting to find instances of spontaneous transfer of knowledge under similar conditions has revealed that successful transfer is notoriously difficult to observe.21,22 Unlike traditional investigations of transfer, which require the direct application of the solution of one problem to a novel problem, PFL defines successful transfer as the use of the prior experience as a platform for the solution of novel problems. The fact that we shifted our focus from assessing how learners’ transfer a specific solution to giving them a foundation for the acquisition of new knowledge probably explains our finding of successful transfer where others have failed.








 2014 Jul;48(7):667-73. doi: 10.1111/medu.12426.

Preparing medical students for future learning using basic science instruction.

Author information

1
SickKids Learning Institute, The Hospital for Sick Children and The Wilson Centre University of Toronto, Toronto, Ontario, Canada.

Abstract

OBJECTIVES:

The construct of 'preparation for future learning' (PFL) is understood as the ability to learn new information from available resources, relate new learning to past experiences and demonstrate innovation and flexibility in problem solving. Preparation for futurelearning has been proposed as a key competence of adaptive expertise. There is a need for educators to ensure that opportunities are provided for students to develop PFL ability and that assessments accurately measure the development of this form of competence. The objective of this research was to compare the relative impacts of basic science instruction and clinically focused instruction on performance on a PFL assessment (PFLA).

METHODS:

This study employed a 'double transfer' design. Fifty-one pre-clerkship students were randomly assigned to either basic scienceinstruction or clinically focused instruction to learn four categories of disease. After completing an initial assessment on the learned material, all participants received clinically focused instruction for four novel diseases and completed a PFLA. The data from the initial assessment and the PFLA were submitted to independent-sample t-tests.

RESULTS:

Mean ± standard deviation [SD] scores on the diagnostic cases in the initial assessment were similar for participants in the basicscience (0.65 ± 0.11) and clinical learning (0.62 ± 0.11) conditions. The difference was not significant (t[42] = 0.90, p = 0.37, d = 0.27). Analysis of the diagnostic cases on the PFLA revealed significantly higher mean ± SD scores for participants in the basic science learningcondition (0.72 ± 0.14) compared with those in the clinical learning condition (0.63 ± 0.15) (t[42] = 2.02, p = 0.05, d = 0.62).

CONCLUSIONS:

Our results show that the inclusion of basic science instruction enhanced the learning of novel related content. We discuss this finding within the broader context of research on basic science instruction, development of adaptive expertise and assessment in medicaleducation.

PMID:
 
24909528
 
DOI:
 
10.1111/medu.12426


+ Recent posts